Sunteți pe pagina 1din 286

Societatea de Ştiinţe Matematice Filiala Ialomiţa

Colegiul Naţional „Mihai Viteazul” Slobozia

PAPACU NICOLAE DUMITRU COSTICĂ

TUDOR MIOARA BĂNICĂ ANCA

MĂRGĂRIT MARIAN POPESCU MARCEL TONȚ ADRIANA

CONCURSUL REGIONAL
DE MATEMATICĂ
„GHEORGHE MIHOC”
EDIȚIILE 1995-2018
REZULTATE ȘI SUBIECTE
‐ENUNȚURI ŞI REZOLVĂRI-

SLOBOZIA 2019

1
Lucrare editată de Colegiul Naţional „Mihai Viteazul” Slobozia

B-dul Unirii, 10, Slobozia, Ialomița

www.cnvmslobozia.com

cn_mv_slobozia@yahoo.com

tel 0243 236 322

editori: Vitalie Buzu, Nicolae Papacu

Tehnoredactare: Tudor Mioara, Anca Bănică, Bratu Oana, Papacu Nicolae

CONCURSUL REGIONAL DE MATEMATICĂ ”GHEORGHE MIHOC”:


Edițiile 1995-2018 Rezultate și subiecte – enunțuri și rezolvări- /Papacu Nicolae,
Dumitru Costică, Tudor Mioara, Bănică Anca, Mărgărit Marian, Popescu Marcel,
Tonț Adriana

2
Concursul de Matematică „Gheorghe Mihoc“, Slobozia, la a 25-a ediţie

Prof. Univ. Dr. Ion Chiţescu

Facultatea de Matematică şi Informatică, Universitatea din Bucureşti

Concursul de Matematică „Gheorghe Mihoc“, care se desfăşoară la Slobozia, a ajuns în


anul 2014 la a 20-a ediţie. Sunt onorat de invitaţia pe care mi-au adresat-o organizatorii ca, în
calitatea mea de preşedinte al concursului, să scriu câteva rânduri cu aceasta ocazie aniversară.

1. Câte ceva despre începuturi

Iniţiativa demarării acestui concurs au avut-o domnii Costică Dumitru (profesor de mare
valoare şi prestigiu din municipiul Slobozia, preşedinte al Filialei Ialomiţa a Societăţii de Ştiinţe
Matematice din România timp de mulţi ani) şi Inocenţiu Drăghicescu (Conf. Univ. Dr. la
Universitatea Politehnică din Bucureşti, fiu al zonei geografice – născut la Amara).
M-am alăturat imediat domniilor lor cu mare entuziasm, ţinând cont şi de vechea prietenie care
mă leagă de colegul Inocenţiu Drăghicescu. Motivarea alegerii numelui lui Gheorghe Mihoc
pentru denumirea concursului a fost triplă: Gheorghe Mihoc a fost un mare matematician român,
Gheorghe Mihoc a iubit mult Slobozia unde a venit de multe ori cu multă plăcere, iar noi, cei trei
“fondatori” ai concursului, i-am fost studenţi şi l-am iubit.

Mi s-a făcut onoarea (meritată?) să fiu preşedinte al concursului şi, de atunci, am rămas în
această funcţie de care am căutat să mă achit cât mai bine. Subliniez că, de la început şi până
acum, colaborarea dintre noi, cei trei “fondatori” a continuat neîncetat, pentru bunul mers al
concursului.

2. Câte ceva despre Gheorghe Mihoc

Gheorghe Mihoc (1906-1981) a fost unul din cei mai importanţi matematicieni români. El
rămâne cunoscut mai ales pentru contribuţiile sale în domeniul teoriei probabilităţilor. Trebuie
subliniată şi activitatea sa de mare importante în statistica matematică, teoria asigurărilor şi
matematica actuarială (tratatul său intitulat “Tratat de matematici actuariale”, Universitatea
Bucureşti, Institutul de statistică, actuariat şi calcul, Bucureşti, 1943, este şi astăzi o lucrare greu
de egalat).

Vom schiţa câteva aspecte privind viaţa şi activitatea lui Gheorghe Mihoc. După absolvirea
Facultăţii de Ştiinţe a Universităţii din Bucureşti, în 1928, a plecat în Italia, unde, în 1930 a
obţinut titlul de doctor în ştiinţe statistice şi actuariale la Universitatea din Roma, sub îndrumarea
lui Guido Castelnouvo. Întors în ţară, şi-a început îndelungata colaborare cu Octav Onicescu (al
cărui discipol a fost) la Institul de statistică, actuariat şi calcul (înfiinţat şi condus de Octav
Onicescu).

3
În 1934 şi-a susţinut teza de doctorat la Universitatea din Bucureşti (titlul tezei: “Asupra
proprietăţilor generale ale variabilelor statistice interdependente”) sub conducerea lui Octav
Onicescu.
Începând cu anul 1937, Gheorghe Mihoc trece cu totul la Universitatea din Bucureşti, unde a
funcţionat până la sfârşitul carierei, având pe parcurs colaborări cu Institutul de statistică,
actuariat şi calcul, Politehnica din Bucureşti şi Academia Comercială din Bucureşti.
A fost şef de catedră la Calculul Probabilităţilor şi Statistică Matematică, apoi la Matematici
Aplicate şi, apoi, din nou, la Calculul Probabilităţilor şi Statistică Matematică (ca succesor al lui
Octav Onicescu).
Menţionăm că Gheorghe Mihoc a avut o îndelungată activitate la Casa Centrală a Asigurărilor
Sociale, începând cu 1929, unde, între anii 1939-1945, a funcţionat ca director al mai multor
direcţii.
Opera de probabilist a lui Gheorghe Mihoc îl situează în primele rânduri ale matematicii
româneşti. Teoria lanţurilor cu legături complete, elaborată împreună cu Octav Onicescu, este
una din cele mai mari realizări matematice româneşti. Citări legate de această teorie apar în
operele unor mari matematicieni, ca M. Frechet, A. Blanc-Lapierre, R. Fortet şi colateral, P.
Billingsley. Şcoala românească, prin C. T. Ionescu Tulcea, Gh. Marinescu, R. Theodorescu, M.
Iosifescu şi Ş. Grigorescu, a continuat studiul şi aprofundarea teoriei lanţurilor cu legături
complete.
Gh. Mihoc mai are contribuţii importante în teoria asigurărilor (după cum am mai amintit
deja), statistică matematică, teoria generală a proceselor stohastice (în particular, teoria generală
a lanţurilor probabilistice), extinderea legii lui Poisson şi analiză matematică (formule de medie
pentru polinoame, ecuaţii diferenţiale şi ecuaţii cu derivate parţiale).

Activitatea de matematician cercetător a lui Gheorghe Mihoc nu poate fi despărţită de


activitatea sa de matematician profesor. A scris, singur sau în colaborare, un mare număr de
monografii şi cursuri, după care au învăţat multe generaţii de studenţi şi matematicieni formaţi.
Cursurile sale erau impecabile, atât prin claritate (îmi amintesc scrisul său frumos pe tablă, cu
caractere îngrijite şi mari…) cât şi prin magia apropierii umane care le caracteriza.
În cursul vieţii sale, Gheorghe Mihoc a deţinut multe funcţii importante, dintre care
menţionăm următoarele: decan al Facultăţii de Matematică şi Fizică a Universităţii din Bucureşti,
prorector al Universităţii din Bucureşti, rector al Universităţii din Bucureşti, director al Centrului
de Statistică al Academiei Române, membru corespondent al Academiei Române, membru al
Academiei Române, preşedinte al Academiei Române.

Încheiem trecerea în revistă a vieţii şi activităţii lui Gheorghe Mihoc cu menţiunea obligatorie
că el, împreună cu Octav Onicescu, au pus bazele şcolii româneşti de Teoria Probabilităţilor şi
Statistică Matematică.
Moartea l-a surprins pe Gheorghe Mihoc, pe atunci preşedinte al Academiei Române, într-o
iarnă friguroasă care nu l-a iertat şi care a adus mult frig şi în sufletele tuturor celor care l-au
cunoscut şi iubit.
Ca unul care a avut onoarea şi şansa de a-i fi student, ţin să pun în evidenţă ceva ce am simţit
şi eu şi mulţi colegi: imensa bunătate care îl caracteriza pe omul Gheorghe Mihoc. Era convins
(şi această convingere o împărtăşea tuturor) că niciodată nu trebuie ratată ocazia de a face bine.
Şi a făcut mult bine.....

4
3. Concursul “Gheorghe Mihoc”: evoluţie şi perspective

Început ca un concurs de matematică local (judeţean), concursul „Gheorghe Mihoc” a


crescut în amploare şi nivel, devenind treptat concurs interjudeţean şi, acum, naţional.
Această evoluţie a fost posibilă, în primul rând, datorită sprijinului acordat de organele de
conducere locale şi sponsorilor, care s-au implicat cu entuziasm şi dăruire şi cărora le mulţumim
din inimă. Credem, de asemenea, că nivelul matematic ridicat al ediţiilor succesive a făcut să
sporească interesul pentru concursul nostru, făcându-l din ce în ce mai „popular” pe plan
naţional. Legat de acest al doilea aspect, ţinem să aducem mulţumiri colegiale autorităţilor
şcolare locale, care au asigurat o desfăşurare perfectă a ediţiilor succesive ale concursului,
autorilor de probleme şi membrilor comisiilor de corectare.
Ne dorim ca şi în viitor, Concursul de Matematică „Gheorghe Mihoc” să rămână unul din
punctele de reper în peisajul matematicii şcolare din România.

5
Concursul de Matematică „Gheorghe Mihoc” – 25 de ani de tradiție și
excelență
În octombrie 1894, cinci tineri ingineri, absolvenţi ai Şcolii de Poduri şi Şosele din
Bucureşti (azi Universitatea "Politehnica" Bucureşti), au luat în discuţie rezultatele slabe obţinute
de candidaţi la examenul de admitere din acel an. În concluzie, s-a propus înfiinţarea unei reviste
româneşti de matematică pentru "elevii liceelor noastre", revistă care va apare începând cu anul
1895 și având denumirea Gazeta Matematică.
În anul 1901 revista inaugurează colecţia Biblioteca Gazetei Matematice cu publicarea
volumului „Culegere de probleme de aritmetică, algebră, geometrie şi trigonometrie“, autori I.
Ionescu, A. Ioachimescu, Gh. Ţiţeica, V. Cristescu, care va cunoaşte mai multe ediţii, iar în anul
următor se organizează primul concurs destinat cititorilor revistei.
În anul 1910 se constituie Societatea Gazeta Matematică care a asigurat în continuare
administrarea ”Gazetei Matematice”. Indiferent de numele pe care l-a avut (Societatea Gazeta
Matematică 1910-1949, Societatea de Științe Matematice și Fizice 1949-1964, Societatea de
Științe Matematice din România, (S.S.M.R) din 1964 și în prezent) asociația profesorilor de
matematică, a matematicienilor și a celor care îndrăgesc matematica a avut un rol primordial în
dezvoltarea matematicii în România. Societatea este cea care a organizat toate concursurile
importante de matematică din țară. La nivel central exista o activitate deosebită a societății, dar
la nivel local erau zone în care societatea nu avea niciun membru. Începând cu anul 1940 au
apărut filiale ale societății, în special în marile orașe. Odată cu reforma administrativ-teritorială
din 1968, s-au înființat în multe județe filiale ale societății.
Unul din primele județe, în care ia ființă o filială, este și județul Ialomița. Cel care are
inițiativa, este profesorul Dumitru Costică, dânsul fiind și cel care a fost președintele filialei
vreme de 45 de ani (1968-2013) și care este în prezent președinte de onoare la filialei Ialomița.
Filiala Ialomița a S.S.M.R. a desfășurat o activitate bogată încă de la înființare. Începând cu
anul 1975, filiala, împreună cu alte instituții de învățământ, a organizat ”Sesiuni de referate și
comunicări științifice” pentru elevi și profesori, la nivel județean. Aceste acțiuni ale filialei erau
patronate de profesori universitari de renume precum. Este de amintit numele academicienilor
Gheorghe Mihoc, Octav Onicescu, Nicolae Teodorescu, Solomon Marcus, etc.

In anul 1995 s-au aniversat 100 de ani de apariţie neîntreruptă a revistei „Gazeta
Matematică”. Societatea de Ştiinţe Matematice din România a celebrat momentul prin diferite
manifestări la nivel naţional. Filiala Ialomiţa a S.S.M.R. prin preşedintele său, profesor Dumitru
Costică a avut ideea organizării unui concurs interjudeţean de matematică. Necesitatea
concursului a apărut şi datorită faptului că a fost o perioadă în care existau foarte puţine
concursuri de matematică iar dorinţa elevilor de a participa la competiţii nu putea fi satisfăcută
doar de Olimpiada Naţională de Matematică. Concursurile, de orice tip, sunt cele prin care se
poate „măsura” progresul şi performanţa. Spiritul de competiţie şi de performanţă reprezintă un
element esenţial în ierarhia valorilor, ierarhie care se realizează şi prin concursurile şcolare. Un
astfel de concurs este şi Concursul de Matematică „Gheorghe Mihoc” numit astfel în memoria
Academicianului Gheorghe Mihoc în semn de respect şi omagiu adus celui care mulţi ani a
patronat activitatea matematică din judeţ. Înainte de a obţine rezultate remarcabile în domeniul
matematicii superioare, profesori universitari români au parcurs fiecare etapă a însuşirii
matematicii de nivel gimnazial sau liceal. Au reuşit aceasta şi prin participarea la diferite
concursuri şcolare de matematică sau sesiuni de comunicări şi referate ştiinţifice. După ce au
obţinut recunoaşterea internaţională a valorilor lor în domeniu, mulţi dintre ei nu au uitat că baza
pregătirii lor în domeniul matematicii s-a realizat în şcoala românească. Un astfel de exemplu a

6
fost şi Academicianul Gheorghe Mihoc. În calitatea sa de Preşedinte al Academiei Române găsea
timp pentru a se deplasa la Slobozia pentru a coordona sesiunile de comunicări şi referate ale
elevilor şi profesorilor.

Concursul de Matematică „Gheorghe Mihoc” este printre primele concursuri interjudețene din
România. El s-a născut la ideea profesorului Dumitru Costică și realizat cu contribuția d-lor conf.
dr. Ion Chițescu și Inocențiu Drăghicescu. Pe bună dreptate putem considera că domniile lor sunt
fondatorii acestui concurs.
Concursul a fost și este organizat de filiala Ialomița a S.S.M.R. și de Colegiul Național ”Mihai
Viteazul” din Slobozia, școală de referință în învățământul ialomițean și cel românesc, iar
Inspectoratul Școlar Județean partener la toate edițiile desfășurate. Filiala Ialomița a S.S.M.R.
este una din filialele cu activitatea deosebită, la ora actuală în colaborare cu diferite instituții de
învățământ organizează anual cinci concursuri regionale sau interjudețene, o sesiune de referate
și comunicări științifice și editează o revistă de matematică.
La prima ediţie au participat elevi din judeţele Brăila, Călăraşi, Constanţa, Prahova şi Ialomiţa.
Ediţia inaugurală a avut parte de un succes şi interes deosebit, fapt ce a dus la continuarea acestei
idei. S-a ajuns actualmente la ediţia a XXIV-a, iar numărul judeţelor participante a crescut
adăugându-se pe parcurs judeţele Buzău, Tulcea, Galaţi, Dâmboviţa, Vâlcea, Vrancea, Vaslui,
Giurgiu şi municipiul Bucureşti).
De la prima ediție, și în prezent, președintele concursului este prof. univ. dr. Ion Chițescu, fost
decan al Facultății de Matematică Informatică, Universitatea din București.
Din Comisia Ştiinţifică în afară de preşedintele concursului au făcut parte şi fac parte
personalităţi recunoscute la nivel naţional: prof. univ. dr. Dumitru Popa, Universitatea „Ovidius”
Constanţa, prof. univ. dr. Stere Ianoş (†), Universitatea Bucureşti, conf. univ. dr. I.C.
Drăghicescu, Universitatea Politehnică Bucureşti, prof. dr. Marcel Ţena, redactor şef la Gazeta
Matematică, prof. Mircea Trifu secretar general al S.S.M.R., etc.

Concursul precede faza naţională a Olimpiadei de Matematică şi reprezintă un excelent prilej


de evaluare a nivelului de pregătire. Concursul este unul de importanţă naţională prin calitatea şi
valoarea profesorilor şi elevilor care participă la acest concurs. Anual, laureații concursului se
regăsesc în listele cu premianții Olimpiadei Naționale de Matematică. De asemenea subiectele şi
lista premianţilor apar anual în Gazeta Matematică și în Revista de Matematică din Ialomița.
Concursul se regăsește anual în Calendarul Activităților Naționale sau Regionale ale Ministerului
Educației Naționale și în Calendarul Activităților Societății de Științe Matematice din România.

Concursul este unul fără taxă de participare şi fără alte cheltuieli băneşti pentru elevi. Acest
lucru este posibil prin contribuţia financiară a diferiţilor sponsori locali, a Consiliul Judeţean
Ialomiţa și mai ales prin susținerea materială și morală din partea autorităţilor locale: Primăria
Municipiului Slobozia şi Consiliul Local. Încă de la prima ediție Primăria Municipiului Slobozia
și Consiliul Local au asigurat suportul material pentru o bună desfășurare a concursului. Primăria
este cea care a asigurat fondul de premiere, unul din cele mai mari din țară, pentru fiecare ediție.
Începând cu ediția a XIX-a Primăria Slobozia a instituit un premiu special, recompensat material,
pentru cel mai bine clasat slobozean la nivelul fiecărui an de studiu.
Sprijinul autorităților locale în organizarea concursului, implicarea organizatorilor și rezultatele
de excepție obținute de elevii orașului Slobozia și județului Ialomița promovează imaginea și
contribuie la mărirea prestigiului municipiului Slobozia și județului Ialomița.

Prof. Nicolae Papacu,


Președinte filiala Ialomița a S.S.M.R.

7
Partea I

Enunțuri și rezultate

EDIȚIA I, 19-20 mai 1995

În cadrul manifestărilor prilejuite de centenarul revistei ”Gazeta Matematică”, filiala


Ialomița a Societății de Științe Matematice, în colaborare cu Direcția județeană pentru tioneret și
sport șu cu Liceul teoretic ”Mihai Viteazul” din Slobozia, a organizat în zilele de 19-20 mai
1995, la liceul teoretic ”Mihai Viteazul”, Concursul interjudețean de matematică ”Gheorghe
Mihoc”.
Concursul, dedicat în același timp, memoriei ilustrului dispărut al cărui nume îl poartă, a
readus în mintea celor prezenți imaginea marelui savant, profesor și om care, cu ani în urmă, în
mai multe rânduri, a patronat, în inegalabila sa generozitate diferitele manifestări cu caracter
matematic organizate pe aceste meleaguri ialomițene.
Au participat concurenți din clasele VI-XII reprezentând județele limitrofe Brăila, Călărași,
Constanța, Prahova, precum și – bineînțeles - din județul Ialomița.
Din partea comitetului de redacție al G.M. au fost d-nii conf. dr. Ion Chițescu, de la
facultatea de Matematică a Universității din București și conf. dr. Inocențiu Drăghicescu. De la
catedra de matematică 3 a Universității Politehnice din București. A participat, de asemenea, ca
invitat, prof. Armand Martinov, de la Liceul de informatică din București.
Comisia științifică a concursului a avut următoarea alcătuire:
Conf. dr. Ion Chițescu – președinte, Conf. dr. Inocențiu Drăghicescu, Prof. Armand Martinov,
Prof. Costică Dumitru – președintele Filialei Ialomița a S.S.M.R., Prof. Teodor Popescu –
vicepreședinte al Filialei Ialomița a S.S.M.R., inspector școlar.
Dintru început trebuie menționate, în termeni superlativi, atât organizarea concursului, cât și
realizarea în sine a acestuia, toți cei implicați și îndeosebi conducerea liceului la care s-au
desfășurat lucrările meritând cele mai călduroase felicitări pentru reușita deplină a acțiunii.
Ținând seama că aceasta a fost ediția inaugurală a concursului, succesul său reprezintă,
conform asigurărilor organizatorilor, cea mai bună încurajare a proiectelor de a-l reedita anual. O
garanție în acest sens o constituie și modul în care acțiunea a fost sprijinită de către autoritățile
locale, precum și de sponsorii AMONIL S.A., CARTEXIM S.R.L. și editura HUMANITAS, a
căror generozitate trebuie relevată cu tot respectul și toată prețuirea ce se cuvin celor ce au
considerat că o astfel de întreprindere, departe de a fi rentabilă în termeni economici, aduce după
sine, în timp, o astfel de rentabilitate, de o calitate cu totul aparte și atât de necesară viitorului
societății.
Concursul propriu zis a fost precedat de o deschidere festivă în cadrul căreia au adresat
cuvinte de salut participanților d-l ing. Iulian Zaharia, președintele Consiliului Județean Ialomița
și d-l prof. Alex. Cobușceanu, directorul Liceului ”Mihai Viteazul”.
Au luat cuvântul, de asemenea, d-l conf. dr. Ion Chițescu, care le-a evocat participanților
câteva pagini ale începuturilor Gazetei Matematice și d-l conf. dr. Inocențiu Drăghicescu, care a
prezentat cartea ”Exerciții și probleme elementare de teoria probabilităților”, al cărei autor este
împreună cu conf. dr. Gh. Budianu. Lucrarea, recent apărută la Casa de editură și presă ”ANSA”
8
SRL din București, constituie și un omagiu postum adus academicienilor Octav Onicescu și
Gheorghe Mihoc.
Concursul a constat din probe scrise desfășurate pe parcursul a trei ore, la fiecare clasă dându-
se câte trei probleme, seturile acestora fiind alcătuită de cei trei oaspeți din comisia științifică.
Corectarea lucrărilor s-a făcut de către profesori din Slobozia împreună cu profesorii oaspeți care
au însoțit concurenții.
Conform celor prevăzute în organizarea concursului urma să fie acordate, la fiecare clasă, câte
un premiu întâi în valoare de 50.000 lei, câte un premiu doi în valoare de 35.000 lei, câte un
premiu trei în valoare de 25.000 lei și câte o mențiune în valoare de 10.000 lei.
În urma desfășurării concursului s-au acordat premiile și mențiunile următoare:

Premianții
Clasa a VI-a
Premiul I: Cioacă Florin, Lic. “M. Viteazul”, Slobozia
Premiul II: Stan Bogdan, Şc. gen. nr. 2, Câmpina
Premiul III: Pol Ana-Maria, Lic. “Al. I. Cuza” Ploieşti
Menţiune i: Gonciar Ștefania, Şc. gen. nr. 7, Brăila

Clasa a VII-a

Premiul I: Nicolau Ioana, Șc. gen. nr. 24, Constanța


Premiul II: Zagarin Mădălina, Șc. gen. nr. 2, Borcea
Premiul III: Fulger Elena, Șc. gen. nr. 27, Constanța
Menţiune I: Sava Andrei, Șc. Gen. nr. 1. Urziceni

Clasa a VIII-a

Premiul I: Marin Bogdan, Lic. “I. L. Caragiale” Ploieşti


Premiul II: Zaharia Mihai, Şc. Normală Slobozia
Premiul III: Alexe Bogdan, Lic. “Al. I. Cuza” Ploieşti
Menţiune I: Mihalache Dan, Şc. gen. nr. 31, Brăila

Clasa a IX-a

Premiul I: Cornățeanu Remus, Lic. de Artă Slobozia


Premiul III: și
Menţiune Specială: Pintilie Adina Lic. Teoretic Urziceni
Menţiune I: Nedelcu Adina, Lic. “G.M. Murgoci” Brăila

Clasa a X-a

Premiul I: Anton Ramona, Lic. “M. cel Bătrân” Constanța


Secăreanu Ioana, Lic. “M. cel Bătrân” Constanța
Premiul III: Berbec Ioana, Lic. “M. cel Bătrân” Constanța
Menţiune I: Stejar Cristian, Lic. “N. Bălcescu” Brăila

9
Clasa a XI-a

Premiul I: Mihai Stere, Lic. “Ovidius” Constanța


Premiul II: Popescu Julian, Coleg. “Gh. Murgoci” Brăila
Premiul III: Toma Mădălina, Lic. Teoretic Feteşti
Menţiune I: și
Menţiune specială : Răducanu Cristian, Lic. “M. Viteazul” Slobozia

Clasa a XII-a

Premiul I: Iorga Bogdan, Lic. “M. Viteazul” Slobozia


Premiul II: Spânu Daniela, Lic. “I.L. Caragiale” Ploieşti
Premiul III: Limona Lucian, Lic. “M. cel Bătrân” Constanța
Menţiune I: Apfelbaum Marcel, Lic. “Gh. Murgoci” Brăila

Pe lângă aceștia, la fiecare clasă, au fost recompensați, în ordinea punctajelor obținute, și câte
doi concurenți prin mențiuni a câte 10.000 lei fiecare constând în cărți.
Tuturor participanților li s-au decernat diplome de participare. Cazarea, masa și cheltuielile
celelalte au fost asigurate de organizatorii concursului.
În aceeași notă de ospitalitate și generozitate autoritățile locale au acordat distincții oaspeților din
comisia științifică după cum urmează: d-lui conf. dr. Ion Chițescu, președintele comisiei – cheia
Municipiului Slobozia, iar d-lor conf. dr. Inocențiu Drăghicescu și prof. Armand Martinov –
medalia jubiliară ”400 de ani de atestare documentară a orașului Slobozia”. De asemenea, li s-a
oferit câte un exemplar din volumul jubiliar ”SLOBOZIA – 400, editat de Consiliul local al
Municipiului Slobozia.
Onorați și – deopotrivă – emoționați, cei distinși și-au exprimat gratitudinea considerând
aceasta și ca un legământ pentru sprijinirea viitoarelor concursuri sau altor activități organizate
de filiala Ialomița a S.S.M.R.
O cât se poate de agreabilă reuniune a încheiat întreaga acțiune printr-o cină amicală la care
au fost prezenți toți profesorii participanți la concurs, organizatori sau oaspeți, precum și d-nii
ing. Gabi Ionașcu, viceprimar al Municipiului Slobozia și prof. dr. Nicolae Stancu, inspector de
general al Inspectoratului școlar județean Ialomița. Spiritul prietenesc și cordial care a dominat
această reuniune reprezintă, suntem convinși, o certitudine a adeziunii celor prezenți față de
ideile nobile care i-au călăuzit pe organizatorii concursului, precum și fașă de proiectele lor
similare pentru anii următori.
Notă. Prezentarea primei ediții a concursului aparținând de distinșilor domni profesori a apărut
cu titlul Concursul Interjudețean de Matematică ”Gheorghe Mihoc” – Ediția I – Slobozia 1995
în Gazeta Matematică nr. 7/1995

EDIŢIA a II-a, 8-10 noiembrie 1996


SUBIECTE

Clasa a VII-a

I. Fie 𝑎 și 𝑏 două numere raționale. Să se rezolve în numere raționale ecuația 𝑎√2 + 𝑏 = 𝑥√2..
G.M.

10
II. Fie ABCD un patrulater convex cu AB CD , AD neparalel cu CB, iar (AE şi (DF
bisectoarele unghiurilor DAB, respectiv ADC, astfel încât F   AB,E  DC  . Dacă AE = DF,
demonstraţi că AD  AB .
Emil Vasile
2 2
III. Să se rezolve în numere întregi ecuaţia: (𝑥 − 𝑦) = 2(𝑥 − 𝑦 )

Clasa a VIII-a

I. Fie a,b,c>0. Să se demonstreze echivalenţa:


1 1 1 1 1 1 1
abc        .
b c c a a b 2a b c 
II. Să se demonstreze că dacă x2  y 2  z2  2xyz  1, atunci:
 x  yz y  xz z  xy   1 x2 1 y 2 1 z2  .
M. Chiriţă
III. Fie tetraedrul OABC în care muchiile OA, OB, OC sunt egale. Să se arate că dacă medianele
care pornesc din O a două feţe sunt perpendiculare între ele, atunci a treia mediană ce porneşte
din O este perpendiculară pe planul determinat de acestea două.
G.M.
Clasa a IX-a

I. Să se determine numerele a, b  *
 
, cu proprietatea , 8a  b   0,1,2,...,a  b .
Emil Vasile
 
II. Fie M mijlocul laturii (AB) a triunghiului ABC m A  90o , m B  90o   şi N un punct
astfel încât NM  AC şi NB  BC . Să se arate că NP=R, unde P este mijlocul laturii (BC) şi
R este raza cercului circumscris triunghiului ABC.
Apostol Constantin
III. Fie ABCD un pătrat şi M un punct interior pătratului pentru care MA=1, MB= 2 şi MC=
3.
a) Să se arate că MB=MD.
b) Să se calculeze lungimea laturii pătratului.
M. Chiriţă

Clasa a X-a
I. Să se rezolve ecuaţia:
5x  12x  13x
G.M.
II. Să se arate că într-un triunghi ascuţitunghic ABC, în care O este centrul cercului circumscris,
au loc relaţiile:
  
tg BAO  tg CAO   tg CBO  tg  ABO  tg  ACO  tg BCO .
sin  A
2
sin B 
2
sin C 
2

Constantin Apostol
III. Să se determine trei numere complexe a, b, c distincte astfel încât:

11
a) |a|=|b|=|c|=1.
b) z  a  z  b  z  c  6, z  , z  1.
2 2 2

M. Chiriţă

Clasa a XI-a
nxn
I. Fie x1  0. Definim şirul  xn n prin relaţia de recurenţă: xn1  ,n  .
n  xn
x1
1) Să se arate că, pentru orice n  2 este adevărată relaţia: xn  .
 1 1 
1 2  ...  n  1  x1  1
 
2) Să se calculeze lim xn .
n 
M. Chiriţă
II. Pentru fiecare matrice A  M2   se defineşte funcţia :
A : M2   ,A  X   det  A  X   det  A  X  .
Să se arate că :
1)  A    A ,  ;
2) AB  A  B , A,B  M2  ;
3) Pentru orice n  *
există şirurile  xn n , yn n astfel încât An  xn  A  yn  I2 ,
unde I2 este matricea unitate de ordinul al doilea.
Nicolae Papacu
1 1 0 0
0 1 1 0 
III. Se consideră matricea A   .
0 0 1 1
 
0 0 0 1
Să se determine An , n  *
.
G.M.

Clasa a XII-a

I. Fie a  şi I  un interval cu proprietatea că x 2  x  a  0 pentru orice x I . Să se


dx
calculeze : x 2
 x a
(discuţie după valorile lui a).
G.M.
 1 1 1
sin x sin x 2 ...sin x n , x  0
II. Fie n  *
şi f :  , f x  
 , x  0.

Să se arate că f admite primitive dacă şi numai dacă  =0.
Nicolae Papacu

12
III. Fie  X, un monoid. Să se arate că există o mulţime T şi o funcţie injectivă  : X  F(T )
cu proprietatea că  ab   a   b pentru orice a,b  X . Aici F(T)  f : T  T  .
Ion Chiţescu

Concursul pentru cea mai frumoasă rezolvare a unei probleme a profesorului


Gheorghe Mihoc

Problemă propusă în Gazeta Matematică în anul 1928:

Fie I centrul cercului înscris al triunghiului ABC şi d o dreaptă ce trece prin I şi taie
laturile triunghiului în M, N şi P; simetricele acestor puncte, respectiv faţă de dreptele AI, BI, CI
se regăsesc pe o dreaptă d’ tangentă cercului înscris.
Ducând dreptele da,db,dc, respectiv prin Ia, Ib, Ic, centrele cercurilor exînscrise triunghiului
şi efectuând construcţii analoage obţinem dreptele d’a,d’b,d’c ce sunt tangente cercurilor
exînscrise Ia, Ib, Ic.
Să se arate că dacă dreptele d, da,db,dc sunt paralele, atunci dreptele d’, d’a,d’b,d’c sunt de
asemenea paralele.
Notă: Se va premia cea mai frumoasă soluţie ce se va prezenta la juriu, până la
festivitatea de premiere.

Premiile concursului

Clasa a VII-a
Premiul I: Ştefan Cristian, Lic. “M. Viteazul” Ploieşti
Premiul II: Avram Sorin Alexandru, Lic. “M. Viteazul” Ploieşti
Premiul III: Laza Andrei, Ş.g. nr. 7 Feteşti
Menţiune: Mărgărit Alexandru, Ş.g. nr. 2 Slobozia
Menţiune: Ilie Mădălin, Ş.g. nr. 15 Buzău

Clasa a VIII-a
Premiul I: Cioacă Florin, Lic. “M. Viteazul” Slobozia
Premiul II: Ene Ciprian, Şc. Norm. Buzău
Premiul III: Stan Gheorghe, Ş.g. nr. 2 Câmpina
Menţiune: Oprea George, Ş.g. Măgurele
Menţiune: Buzoiu Mădălina, Ş.g. nr. 5 Buzău

Clasa a IX-a

Premiul I: Apostol Costin, Lic. “Iulia Haşdeu” Buzău


Premiul II: Iancu Mihai, Lic. “Al. Vlahuţă” Rm. Sărat
Premiul III: Porumbel Daniel, Lic. “Iulia Haşdeu” Buzău
Menţiune: Radu Mihaela, Lic. “M. Viteazul” Slobozia
Menţiune: Bădicioiu Alexandra, Lic. “I.L. Caragiale” Ploieşti
Menţiune specială : Sava Andrei, Lic. Teoretic Urziceni
Menţiune specială: Savu Iulia, Lic. “I.L. Caragiale” Ploieşti

13
Clasa a X-a

Premiul I: Grigore Bogdan, Lic. “M. Viteazul” Ploieşti


Premiul II: Minescu Ana Maria, Lic. “M. Viteazul” Ploieşti
Premiul III: Nicolae Mihai, Şc. Norm. Slobozia
Menţiune: Zotic Marius Adrian, Lic. Teoretic Feteşti
Menţiune: Anghelescu Leon, Lic. “P. Georgescu” Ţăndărei
Menţiune: Ştefan George, Lic. “M. Viteazul” Slobozia
Menţiune specială: Ghiţă Ştefania, Coleg. “B.P. Haşdeu” Buzău
Menţiune specială: Tache Silvia, Lic. “M. Viteazul” Slobozia

Clasa a XI-a

Premiul I: Epure Mihai, Lic. de Artă Slobozia


Premiul II: Badea Dorin Alexandru, Lic. “G.M. Murgoci” Brăila
Premiul III: Nedelcu Adina, Lic. “G.M. Murgoci” Brăila
Menţiune: Cornăţeanu Marian, Lic. de Artă Slobozia
Menţiune: Popescu Sabin Adrian, Lic. “I.L. Caragiale” Ploieşti
Premiu Special: Băbuş Ana-Maria, Lic. “N. Bălcescu” Brăila
Menţiune specială: Pâslaru Liviu, Coleg. “B.P. Haşdeu” Buzău
Menţiune specială: Bociu Lorena Viorica, Lic. “N. Grigorescu” Câmpina

Clasa a XII-a

Premiul I: Stejar Cristian, Lic. “N. Bălcescu” Brăila


Premiul II: Necula Ciprian, Lic. “I.L. Caragiale” Ploieşti
Premiul III: Negroiu Iulian Alexandru, Lic. “I.L. Caragiale” Ploieşti
Menţiune: Brătulescu Carmen, Lic. Teoretic Feteşti
Menţiune: Voicu Ramiro, Lic. “M. Viteazul” Slobozia
Menţiune specială : Budariu Gigi Daniel, Coleg. “Gh. Murgoci” Brăila
Menţiune specială : Mihăescu Marga, Lic. “M. Viteazul” Slobozia

EDIŢIA a III-a, 31octombrie-2noiembrie 1997


SUBIECTE

Clasa a VII-a

I. Fie Două semidrepte [OX, [OY şi pe ele punctele A, B (pe [OX) şi C, D (pe [OY). Se mai
presupune că OC=a, CD=b, a>b>0.
Să se determine lungimea segmentului AB astfel încât să fie îndeplinite simultan
proprietăţile:
a) CB este mediană în triunghiul OAC.
b) Unghiurile OAD şi ODA au aceeaşi măsură.
II. Se consideră un triunghi ABC cu următoarea proprietate: există un număr natural nenul n
astfel încât măsurile unghiurilor A, B, C să fie proporţionale cu numerele n, n+1, n+2. Fie I
punctul de intersecţie a bisectoarelor triunghiului ABC. Se cere măsura AIC.

14
III. Fie x şi y două numere naturale nenule. Să se demonstreze echivalenţa:
4x  y 1 x 5
   .
2x  3y 2 y 6

Clasa a VIII-a
I. Fie ABC un triunghi astfel încât m(BAC)  90o. Construim în exteriorul lui ABC
triunghiurile dreptunghice isoscele A’BC (A’B=A’C), B’CA (B’C=B’A), C’AB(C’A=C’B).
Să se arate că dreptele B’C’ şi AA’ sunt perpendiculare.
II. Fie ABCD un pătrat. Notăm cu M şi N punctele de intersecţie ale cercului de diametru AB cu
cercul înscris în pătratul ABCD. Fie P mijlocul laturii CD. Să se arate că triunghiul MNP este
echilateral.
III. a) Să se arate că 5 este număr iraţional.
b) Să se construiască grafic un segment de lungime 5 , fiind dat un segment de lungime 1.

Clasa a IX-a
I. Fie ABCD un tetraedru regulat şi fie M un punct interior segmentului AC. Se duce prin M un
plan paralel cu muchiile AB şi CD care taie muchiile BC, BD şi AD respectiv în N, P, Q.
a) Să se arate că patrulaterul MNPQ este dreptunghi.
b) Să se determine poziţia lui M, pentru care aria lui MNPQ este maximă.
II Fie A şi B două puncte fixe în spaţiu şi a un număr real. Să se determine locul geometric al
punctelor M din spaţiu pentru care:
MA2-MB2=a.
III. Să se rezolve şi să se discute sistemul de ecuaţii:
 xy 2

ax  y  0
unde a este un număr real fixat.

Clasa a X-a
I. Se consideră dreptunghiul ABCD şi mijloacele P, Q ale laturilor[AD], respectiv [BC]. Pe
prelungirea laturii DC se consideră punctul M (MC<MD). Dreapta MQ intersectează diagonala
[BD] în N. Să se demonstreze (PQ este bisectoarea unghiului MPN.
II. Să se rezolve ecuaţia:
cosm x  sinn x  1,
unde m,n * .
III. Să se rezolve ecuaţia:
3x 2  2  x   x   5,
unde [x] este partea întreagă a numărului real x.

Clasa a XI-a
I. Fie ABC un triunghi şi A’, B’, C’ respectiv mijloacele laturilor BC, CA, AB. Se proiectează
A’ pe AB în A1 şi pe AC în A2. Similar, B’ se proiectează pe BC în B1 şi pe BA în B2, iar C’ se
proiectează pe CA în C1 şi pe CB în C2. Să se arate că:
9R 3
A1A2  B1B2  C1C2 
4
unde R este raza cercului circumscris triunghiului ABC.

15
II. Să se afle rădăcinile reale ale ecuaţiei
5  x  2 x  32  4  x  1 x  33  1.
III. Să se demonstreze că punctele de contact cu o suprafaţă sferică ale laturilor unui patrulater
strâmb sunt coplanare.

Clasa a XII-a
I. Fie a,b numere reale şi n  2 un număr natural. Se consideră matricea pătrată de ordinul n
a b b b
b a b b 

A  b b a b .
 
 
b a 
 b b

Să se calculeze Ak pentru orice k  * .


II. Se dă un punct fix A  O pe axa OX şi se consideră două puncte distincte M şi N pe Oy. Fie
m perpendiculara dusă prin M pe AM şi n perpendiculara dusă prin N pe AN.
a) Să se arate că pentru orice poziţii ale lui M şi N dreptele m şi n sunt concurente.
b) Să se determine locul geometric al punctului de intersecţie al dreptelor m şi n atunci
când M şi N se deplasează pe Oy, astfel încât segmentul MN să aibă lungime constantă.
III. Să se reprezinte grafic funcţia:
cos3x
f: \ k | k   , f x  .
sin x

Premiile Concursului

Clasa a VII-a

Premiul I: Costache Ionuţ, Lic. „Mihai Viteazul”, Slobozia, Ialomiţa


Badea Cătălin, Şc. Gen. Nr. 1, Câmpina, Prahova
Premiul II: Struţu Mircea, C.N. „B.P. Haşdeu”, Buzău
Carpen Alexandra, Şc. Gen. Nr. 1, Urziceni, Ialomiţa
Premiul III: Predescu Alexandru, Şc. Gen. Nr. 1, Urziceni, Ialomiţa
Dumitru Marius, Şc. Normală, Slobozia, Ialomiţa.

Clasa a VIII-a

Premiul I: Petrişan Daniela, Şc. Gen. Nr. 1, Buzău


Premiul II: Dinu Roxana, Lic. „Mihai Viteazul”, Slobozia, Ialomiţa
Premiul III: Purice Andrei, Şc. Gen. Nr. 2, Slobozia, Ialomiţa
Menţiune : Laza Dorin, Şc. Gen. Nr. 7, Feteşti, Ialomiţa
Menţiune : Dragomir Irina, Lic. „I.L. Caragiale”, Ploieşti, Prahova

16
Clasa a IX-a

Premiul I: Cioacă Florin, Lic. „Mihai Viteazul”, Slobozia, Ialomiţa


Premiul II: Pârvu Ionuţ Cătălin, Şc. Normală, Slobozia, Ialomiţa
Premiul III: Puia Vasile Gabriel, C.N. „B.P. Haşdeu”, Buzău
Menţiune : Oprea George Adrian, Lic. „Mihai Viteazul”, Ploieşti, Prahova
Menţiune: Nistor Cristina, Lic. „Gh. Murgoci”, Brăila

Clasa a X-a

Premiul I: Iancu Mihai, Lic. „Al. Vlahuţă”, Rm. Sărat, Buzău


Premiul II: Apostol Costin, C.N. „B.P. Haşdeu”, Buzău
Premiul III: Mocanu Irina, Lic. „Mihai Viteazul”, Slobozia, Ialomiţa
Menţiune : Radu Mihaela, Lic. „Mihai Viteazul”, Slobozia, Ialomiţa
Menţiune : Porumbel Cosmin, C.N. „B.P. Haşdeu”, Buzău

Clasa a XI-a

Premiul I: Minescu Ana-Maria, Lic. „Mihai Viteazul”, Ploieşti, Prahova


Premiul II: Brie Alexandru-Horia, Lic. Teoretic, Sinaia, Prahova
Premiul III: Grigore Bogdan Lic. „Mihai Viteazul”, Ploieşti, Prahova
Menţiune : Surdeanu Valeriu Viorel, Lic. „Gh. Murgoci”, Brăila
Menţiune : Ghiţă Ştefania, C.N. „B.P. Haşdeu”, Buzău
Munteanu Valentin, Lic. „Mihai Viteazul”, Slobozia, Ialomiţa

Clasa a XII-a

Premiul I: Stan Florin, Lic. „Mihai Viteazul”, Ploieşti, Prahova


Premiul II: Neacşu Adrian, C.N. „B.P. Haşdeu”, Buzău
Premiul III: Ghido Florin, Lic. „Gh. Murgoci”, Brăila
Menţiune: Cornăţeanu Marian, Liceul de Artă, Slobozia, Ialomiţa
Menţiune: Popescu Sabin, Lic. „I.L. Caragiale”, Ploieşti, Prahova.

EDIŢIA a IV-a, 30 octombrie-1 noiembrie 1998


SUBIECTE

Clasa a VII-a
I. a) Fie a, b, c  . Să se arate că expresia
E(a, b,c)  a2  b2  c 2  2 a  b  c   3
se poate scrie ca sumă de trei pătrate.
b) Să se determine a, b, c  astfel încât
a  b  c 2  2 a  b  c   6 .
2 2

II. Pentru k  *
şi n  *
\ 1 notăm cu

17
 1  1  1 
A(n, k )  1 k 1 k ...1 k .
 2  3   n 
Să se determine valorile lui k  pentru care:
*

1
A(n, k )  ,  n  * \ 1.
2
III. Fie ABCD un patrulater convex în care BC=CD şi m  BCD  90o . Dacă AC este
bisectoarea unghiului BAD, atunci AB=AD sau m  BAD  90o

Clasa a VIII-a
 1
 x  1  a  y  2  1
I. Rezolvaţi sistemul:  unde a  . Discuţie.
 a  y  1  2a
 x  1
II. Să se arate că ecuaţia x3  y 4  z2 are o infinitate de soluţii în numere întregi.
III. Fie triunghiul dreptunghic isoscel ABC cu AB=AC=a. Pe planul (ABC) se ridică
perpendicularele AA’ şi BB’. Dacă BB’=a, să se determine AA’, astfel încât:
a) triunghiul A’B’C să fie dreptunghic in B’;
b) triunghiul A’B’C să fie isoscel cu vârful în C.

Clasa a IX-a
I. Fie a, b  , A  {x  | x  ax  b  0} și
2

B  {x  | ax 2  bx  8  0}.
a) Dacă A  B  , atunci A  B  {2};
b) Se consideră în plan punctele M(a,b). Să se reprezinte aceste puncte dacă A\B are un
singur element.
II. Să se arate că dacă a>0, b>0, c>0 şi a+b+c=k, atunci :
k2
ab  bc  ca   a2  b2  c 2 .
3
III. Fie cubul ABCDA’B’C’D’ şi B1 simetricul lui A faţă de B, C1 simetricul lui C’ faţă de C, D1
simetricul lui C’ faţă de D’, A1 simetricul lui A faţă de A’. Să se arate că:
a) A1D1 şi B1C1 sunt coplanare.
b) Notăm (A1D1,B1C1)  A’C’  {M } . Să se arate că
d 2 M, ABC   d 2 M,BCC '   d 2 M, AB 'C '   d 2 M, ADD '   a2 , unde a este latura
cubului.
c) Să se afle m  B1C1, AC  .

Clasa a X-a
I. Rezolvaţi ecuaţia:
3x  1 2x  1
 x 1  1.
2x 1 3
II. Să se demonstreze că nu există numere complexe z de modul număr întreg pentru care să fie
adevărată egalitatea:

18
4z2  4z  3.
III. Într-un triunghi se duc trei ceviene concurente. Pentru fiecare dintre ele se consideră dreapta
determinată de mijlocul său şi de mijlocul laturii corespunzătoare a triunghiului. Să se arate că
cele trei drepte sunt concurente.

Clasa a XI-a
I. Să se determine matricele A  M2   pentru care este adevărată egalitatea:
4 3
A3   .
 3 2 
II. Fie şirul  xn n0 , unde x0  0, x1, x2  * şi
xn xn1  2xn1xn2  xn2 xn3  0, pentru n  3.
Să se determine termenul general şi să se studieze convergenţa şirului.
III. În piramida ABCD un plan paralel cu (BCD) taie muchiile AB, AC şi AD în E, F respectiv
în G. Un plan paralel cu (ACD) care trece prin E taie muchiile BC şi BD în H, respectiv în I.
a) Demonstraţi că, indiferent de poziţia punctului E   AB ,V EFGHCDI   V  ABCD.
3
4
b) Calculaţi V EFGHCDI  , cunoscând că: V  AEFG  27cm3 şi V BEHI   125cm3 .

Clasa a XII-a
arctgx
I. Să se calculeze primitivele funcţiei: f  x   ,a  0, pe un interval I inclus în
ax  b2
 b 
  a ,  .
 
II. Fie a,b  ,a  b şi o funcţie f :  a, b bijectivă. Atunci f nu este o funcţie raţională.
III. a) Să se arate că dacă a, b,c  0,  atunci

a   
3 2
2
 b2  c2  3 a3  b3  c3 .
b) Dacă z1, z2 , z3  cu z1  z2  z3  1 şi dacă z1  z2  z2  z3  z1  z3  3 ,
2 2 2

atunci z1, z2 , z3 sunt afixele vârfurilor unui triunghi echilateral.

EDIŢIA a V-a, 20 - 21 noiembrie 1999


SUBIECTE

Clasa a VII-a
I. Determinaţi x  astfel încât  x  1999   x  2001
2000 2000
.
x 1 2
II. Aflaţi x, y  *
din proporţia:  .
4 y 2

19
III. Fie S un punct situat în planul triunghiului oarecare ABC şi fie A’, B’, C’ simetricele
punctului S faţă de mijloacele laturilor [BC], [CA] şi respectiv [AB].
Arătaţi că:
a) Triunghiurile ABC şi A’B’C’ sunt congruente.
b) Dreptele AA’, BB’, CC’ sunt concurente.

Clasa a VIII-a

I. Fie triunghiul ABC în care AB=2-x, unde x  . Ducem MN BC,


M (AB, N (AC, astfel încât AM=1 şi AN= 1 x  .
2

Ordonaţi crescător lungimile segmentelor AM, MB, AN şi NC.


II. Fie fracţia raţională
3x2  12
F x  2 , x  \ 4;2.
x  2x  8
a) Simplificaţi fracţia F(x).
b) Aflaţi x  astfel încât F  x   * .
III. Fie pătratele ABCD şi ABEF situate în plane perpendiculare, cu AB=a. Calculaţi:
a) Măsura unghiului dintre dreptele AC şi AE.
b) Distanţa de la E la dreapta AC.
c) Distanţa de la F la dreapta CD.
d) Distanţa de la A la planul (ECD).
e) Tangenta trigonometrică a unghiului diedru format de planele (AEC) şi (ABC).

Clasa a IX-a

I. Fie n  ,n  1. Stabiliţi semnul numărului


a  n  2 2n  4  n  2 2n  4  2 2.
II. Să se arate că dacă A, B, C sunt trei mulţimi oarecare, avem incluziunea
A \ C   A \ B   B \ C .
Incluziunea de mai sus poate fi strictă sau nestrictă? Argumentaţi răspunsul.
 1 3 
III. Fie f :  .     ,   definită prin f  x   x 2  x  1.
 2 4 
Să se arate că f este bijectivă şi să se calculeze f 1 (adică inversa lui f).

Clasa a X-a

I. Să se rezolve ecuaţia
x  1  x  4  x  9  x  16  x  25  15.
II. Se dau numerele a>0, b>0 şi   0, .
Să se dea condiţiile necesare şi suficiente pentru ca să avem simultan:
a) Există un triunghi ABC cu proprietăţile: BC=a, AC=b şi măsura unghiului B să fie
egală cu  .
b) Toate triunghiurile cu proprietăţile de la a) sunt congruente.

20
III. Fie z \ . Să se arate că pentru orice n  *
există şi sunt unic determinate numerele an,
bn reale, astfel încât
zn  an z  bn .

Clasa a XI-a

I. Reamintim că un polinom
P  X   a0  a1 X  ...  an X n  X
Se numeşte reciproc dacă ai=an-i, pentru i=1, 2, …, n.
Să se demonstreze că produsul a două polinoame reciproce este un polinom reciproc.
II. Fie n  , n  2. Fie a  ,a  1,a  1 n.
Să se afle inversa matricei pătratice de ordinul n:
a 1 1 1
1 a 1 1 

M  1 1 a 1.
 
 
1 a 
 1 1
III. Fie 0  a  b   . Să se calculeze
a(a  1)(a  2)...(a  n)
lim .
n b(b  1)(b  2)...(b  n)

Clasa a XII-a

I. Două puncte mobile pornesc în acelaşi moment din originea axelor de coordonate O, primul
punct mobil M1 parcurgând axa Ox pozitivă cu viteza constantă v1  0 , iar al doilea punct mobil
M2 parcurgând axa Oy pozitivă cu viteza constantă v 2  0 .
Să se determine locul geometric al punctului M care împarte în fiecare moment
segmentul M1M2 într-un raport constant k>0, adică
MM1
 k.
MM2
II. Fie G, un grup abelian. Pentru fiecare n  *
notăm

Gn  x G | x n  e 
şi presupunem că mulţimea Gn are cel mult n elemente.
Să se arate că:
a) Gn este subgrup al grupului G, pentru orice n  * .
b)Singurele subgrupuri finite ale grupului G sunt grupurile Gn, unde n  * .
III. Fie a  şi I  un interval nedegenerat astfel încât x 2  ax  1  0,  x  I.
1
Să se calculeze primitivele funcţiei f :  , f x  (discuţie după
x  ax  1
2

valorile lui a).

21
Premiile Concursului

Clasa a VII-a

Premiul I: Voiteanu Mircea, Şcoala Nr. 2, Slobozia, Ialomiţa


Premiul II: Manea Flavius, Şcoala Nr. 11, Călăraşi
Premiul III: Antone Ionuţ, Şcoala „Tudor Vladimirescu”, Drăgăşani
Menţiune : Dorin Radu, C.N. „M. Viteazul”, Ploieşti
Menţiune : Decianu Florentina, Şcoala Nr. 1, Năvodari - Constanţa

Clasa a VIII-a

Premiul I: Pantelimon Ioana, Şcoala Nr. 2, Slobozia


Premiul II: Tocan Ioana, C.N. „I.L. Caragiale”, Ploieşti
Premiul III: Roşca Ştefan, Şcoala Nr. 1, Rm. Sărat
Menţiune: Alexe Alexandru, Liceul „M. Viteazul”, Slobozia
Menţiune: Truţoiu Laura, Şcoala „Tudor Vladimirescu”, Drăgăşani

Clasa a IX-a

Premiul I: Spirea Andreea, C.N. “Mircea cel Bătrân”, Constanţa


Premiul II: Martin Liviu, Liceul „M. Viteazul”, Slobozia
Premiul III: Costache Sebastian, Liceul „M. Viteazul”, Slobozia
Menţiune: Vişan Răzvan, C.N. „B.P. Haşdeu”, Buzău
Menţiune: Căpăţână Adrian, Liceul „M. Viteazul”, Slobozia

Clasa a X-a

Premiul I: Purice Radu Andrei, Liceul „M. Viteazul”, Slobozia


Premiul II: Fulger Stan, C.N. “Mircea cel Bătrân”, Constanţa
Premiul III: Tănase Raluca, C.N. „B.P. Haşdeu”, Buzău
Menţiune: Petrişan Daniela, C.N. „B.P. Haşdeu”, Buzău
Menţiune: Dragomir Irina, C.N. „I.L. Caragiale”, Ploieşti

Clasa a XI-a

Premiul I: Petre Liviu, C.N. „Barbu Ştirbei”, Călăraşi


Premiul II: Puia Gabriel, C.N. „B.P. Haşdeu”, Buzău
Premiul III: Lungu Alexandra, C.N. „N. Grigorescu”, Câmpina
Menţiune: Cioacă Florin, Liceul „M. Viteazul”, Slobozia
Menţiune: Rusu Valentin, C.N. „I.L. Caragiale”, Ploieşti
Menţiune: Nistor Teodor-Cristian, Liceul Feteşti

Clasa a XII-a

Premiul I: Ene Ionuţ, Liceul Urziceni


Premiul II: Apostol Bogdan, C.N. „B.P. Haşdeu”, Buzău
Premiul III: Fulger Elena, C.N. “Mircea cel Bătrân”, Constanţa
22
Menţiune: Florea Florentina, Liceul „M. Viteazul”, Slobozia
Menţiune: Porumbel Daniel, C.N. „B.P. Haşdeu”, Buzău
Menţiune: Pătrănescu Oana, C.N. „B.P. Haşdeu”, Buzău

EDIŢIA a VI-a, noiembrie 2000


SUBIECTE

Clasa a VII-a
I.. Să se construiască un triunghi ABC cu următoarele proprietăţi:

a) m A  45o ;
b) AB=4cm şi BC=6cm.
Câte soluţii are problema?
II. Să se determine toate numerele naturale scrise cu două cifre care se divid cu suma cifrelor lor.
III. Fie ABC un triunghi cu următoarea proprietate: unghiurile exterioare unghiurilor
A, B, C sunt direct proporţionale cu numerele 3, 4 şi 5. Să se determine măsurile
unghiurilor A, B, C.

Clasa a VIII-a

I. Fie x>0 un număr real. Se presupune că x, x+5, x+10 sunt lungimile laturilor unui triunghi. Să
se demonstreze că pentru orice număr k>0, numerele x  k , x  5  k , x  10  k sunt
lungimile laturilor unui triunghi.
II. Să se arate că 2000 nu este număr raţional.
III. Pentru ce valori ale lui a  egalitatea
x 1 x  2

a 1 a  3
este proporţie, cu x>0?

Clasa a IX-a

I. Se consideră tetraedrul ABCD în care O este centrul sferei circumscrise, iar L, M, N sunt
respectiv mijloacele muchiilor [BC], [CD], [BD]. Să se demonstreze că
LOM  LON  NOM dacă şi numai dacă triunghiul ABC este echilateral.
II. Fie a  . Să se studieze sistemul :
a  3 x  2y  4

2x  a  3 y  4
În funcţie de valorile lui a.
ecuaţia: x2    x .
2
III. Să se rezolve în
Clasa a X-a

I. Să se determine numerele strict pozitive p cu proprietatea că, pentru orice z1, z2  , este
adevărată egalitatea:
23
z1  z2  z1  z2  2 z1  z2
p p
 p p
.
II. Să se arate că într-un triunghi oarecare ABC este adevărată inegalitatea:
9
sin A sinB  sinB sinC  sinC sin A  .
4
III. Să se arate că oricare ar fi numerele a1,a2 ,a3 ,...,an  0, n  1, este adevărată inegalitatea:
1
a1  a2  a3  ...  an   n  1.
a1a2a3 ...an
În ce caz avem egalitate?

Clasa a XI-a

I. Fie un tetraedru OABC în care OA  OB, OB  OC, OC  OA,


OA  a, OB  b, OC  c şi fie h lungimea perpendicularei dusă din O pe planul (ABC). Să se
arate că:
a2  b2  c2
h .
3
II. Fie n  *
şi polinomul P  X   n3 X n   n2  2 X  n2  n  1.
Să se arate că ecuaţia P(X)=0 nu are rădăcini întregi.
III. Fie a,b astfel încât a  b  1 şi fie an n1 ,  bn n1 două şiruri de numere reale
convergente către a, respectiv b. Studiaţi convergenţa şirului  cn n1 dat de egalitatea:
cn  an  b1 an1  b2 ...a2  bn1 a1  bn .

Clasa a XII-a

I. Fie a  . Construim şirul an n0 definit astfel:


a0  a şi an1  an2  an  1 , n  .
2

Să se determine valorile lui a pentru care şirul an n0 este convergent şi să se calculeze
limita sa pentru aceste valori.

II. Să se calculeze:


ln x  x 2  1 dx.
 x 1
2

III. Fie n  ,n  2. Să se arate că următoarele afirmaţii sunt echivalente:


a) Există A  Mn   astfel încât A2  2A  5In  On .
b) Numărul n este par.

24
Premiile Concursului

Clasa a VII-a

Premiul I: Chiuru Andrei, Lic. . „Radu Vlădescu”, Ploieşti


Premiul II: Mosescu Dragoş, Şcoala Gen. Nr. 23, Brăila
Premiul III: Popescu Gabriela, Şcoala Gen. „Sf. Vasile”, Ploieşti
Menţiune : Olteanu Andreea, Şcoala Gen. Nr. 2, Slobozia
Menţiune : Popa Irina, Liceul de Artă, Slobozia

Clasa a VIII-a

Premiul I: Voiteanu Victor, Şcoala Gen.Nr. 2, Slobozia


Chirbocea Sterian, Şcoala Gen. Măgurele, Prahova
Premiul II: Toma Ana Maria, Lic. „Iorga”, Brăila
Moroianu Raluca, Şcoala Gen. Nr. 10, Călăraşi
Premiul III: Corea Mihai, Clegiul Naţional, Ploieşti

Clasa a IX-a

Premiul I: Dinu Răzvan, C.N. “Mihai Viteazul”, Ploieşti


Premiul II: Sulhan Alexandru, C.N. “Mircea cel Bătrân”, Constanţa
Premiul III: Ganea Roxana, Liceul „Al. I. Cuza”, Slobozia
Aron Bogdan, Liceul „M. Viteazul”, Slobozia
Menţiune: Vasilescu Andra, Liceul „M. Viteazul”, Slobozia

Clasa a X-a

Premiul I: Spirea Andreea, C.N. “Mircea cel Bătrân”, Constanţa


Bratu Florin, C.N. “Nicolae Bălcescu”, Brăila
Curtasan Eden, C.N. “Mircea cel Bătrân”, Constanţa
Pascu Iuliana, C.N. „B.P. Haşdeu”, Buzău
Premiul II: Vădeanu Vladimir, C.N. “Nicolae Bălcescu”, Brăila

Clasa a XI-a

Premiul I: Păunescu Liviu, C.N. “Mircea cel Bătrân”, Constanţa


Premiul II: Laza Dorin, C.N. “Mircea cel Bătrân”, Constanţa
Premiul III: Tănase Raluca, C.N. „B.P. Haşdeu”, Buzău
Menţiune: Petrişan Daniela, C.N. „B.P. Haşdeu”, Buzău
Menţiune: Fulger Aurel, C.N. “Mircea cel Bătrân”, Constanţa

Clasa a XII-a

Premiul I: Cioacă Florin Daniel, Liceul Teoretic, Slobozia


Premiul II: Dincă Petre Liviu, C.N. „Barbu Ştirbei”, Călăraşi
Premiul III: Dumitrache Petre-Nicu, C.N. “Nicolae Bălcescu”, Brăila
Menţiune: Crăciun Marian, Liceul Teoretic, Slobozia
Menţiune: Iliescu Oana Adina, Liceul Teoretic, Sinaia.
25
EDIŢIA a VII-a, noiembrie 2001
SUBIECTE

Clasa a VII-a
I. Numerele a,b,c  *
 sunt direct proporţionale cu 2, 3 şi respectiv 6.
Demonstraţi că 2a+3b+6c=7 dacă şi numai dacă a2  b2  c 2  1.
34
II. Aflaţi numărul xyz astfel încât  .
x  y 2  z2
2

III. Pe laturile (AB) şi (AC) ale unui triunghi oarecare se construiesc în afară pătratele ABDE şi
ACFG. Arătaţi că:
a) EC   BG ; b) EC  BG.

Clasa a VIII-a
I. Fie E  2a  5b  19c  6ab  12bc  2c  8 . Determinaţi valorile lui a, b, c 
2 2 2
pentru
care expresia este minimă.
II. Să se rezolve în  sistemul
 x 3  y 3  35
 5
x  y  211.
5

III. Fie ABCDA’B’C’D’ un cub şi M   AB arbitrar. Considerăm N  DD ' , astfel încât
AM=DN, iar P este mijlocul lui [MN]. Să se afle ce descrie P atunci când M parcurge [AB].
Justificaţi răspunsul.

Clasa a IX-a

𝑋 2000 +𝑋 1000 +𝑋 998 +𝑋 997 +𝑋 996 +⋯+𝑋+1


I. a) Simplificaţi fracţia 𝐹 = .
𝑋 2001 +𝑋 999 +𝑋 998 +𝑋 997 +𝑋 996 +⋯+𝑋+1

b)Fie a, b,c  * 1 1 1

   abc, iar A  a2b2  1 b2c 2  1 c 2a2  1 .
astfel încâ
a b c
  
Demonstraţi că există B  cu proprietatea B2  A.
II. Se consideră ecuaţia ax 2  b  x   c x  d , unde a,b,c,d  ,a  0 ; [x] reprezintă partea
întreagă a lui a, iar {x} reprezintă partea fracţionară a lui x. Ce condiţie trebuie să satisfacă
respectivii coeficienţi a, b, c, d pentru ca ecuaţia să aibă soluţii reale? Care sunt acestea?
III. Fie paralelipipedul dreptunghic ABCDA’B’C’D’, astfel încât:
AA '  min  AB,BC .
Notăm cu O intersecţia diagonalelor dreptunghiului ABCD. Să se demonstreze că
paralelipipedul este cub dacă şi numai dacă CO '  A'C.

Clasa a X-a

I. În triunghiul ABC, înălţimea AA’, mediana BB’ şi bisectoarea CC’ sunt concurente
 A'  BC,B '  AC,C '  AB .
a)Arătaţi că BB’ intersectează segmentul [A’C’] în mijlocul acestuia.

26
b)Dacă s este aria triunghiului A’B’C’, iar S este aria triunghiului ABC, să se arate că:
S
s .
4
II. Fie a,b,c>0, astfel încât a2  b2  c 2  abc  4 . Să se arate că:
 2  a 2  b   2  b2  c   2  c 2  a  1.
 2  a 2  b  2  b2  c  2  c 2  a
III. Fie M  o mulţime nevidă cu proprietăţile:
a)  z  cu z  1 z  M ;
b)  z1, z2  M  z1  z2  M.
Să se arate că M= .

Clasa a XI-a
I. Fie z1, z2 , z3  *
.
a) Dacă z1  z2  z3 , atunci z1  z2  z3 =0 dacă şi numai dacă
z1z2  z1z3  z2 z3  0 .
b) Dacă z1  z2  z3 = z1z2  z1z3  z2 z3  0 , atunci z1  z2  z3
II. Să se arate că:
a) şirul an  2  3   , n 
n
*
, are limita 1.

 1 2  , n 
n
b) şirul bn *
, nu are limită.
(Acoladele semnifică partea fracţionară.)
III. Fie numerele reale x1, x2 ,..., xn ,a, astfel încât pentru orice k=1,2,3,…,n este adevărată
inegalitatea 1 a xk  S, unde S  x1  x2  ...  xn . Să se arate că:
x12 x22 xn2 S
  ...   .
ax1  x2  ...  xn x1  ax2  ...  xn x1  x2  ...  axn a  n  1

Clasa a XII-a

I. Să se calculeze:
n
 x 2y x 
 y 2x y  , unde x, y  , n  * .
 
 x 2y x 
 
II. Să se determine funcţiile f :  care verifică condiţiile:
a) f este derivabilă în x0  0 şi f’(0)=1+f(0)=2.
b) f (2x )  f 2 ( x )  x 2  2x(1 f ( x )),()x  .
III. Fie (G,) un grup cu n elemente. Pentru fiecare k  cu proprietatea (k,n)=1, definim
aplicaţia fk : G  G,fk ( x)  xk , pentru orice x G. Să se arate că mulţimea  formată din
aceste aplicaţii este un grup abelian finit relativ la operaţia de compunere a funcţiilor.

27
Premianţii concursului

Clasa a VII-a

Premul I: Iancu Teodor Daniel, Şcoala Generala 2, Slobozia


Tănase Gheorghe Narcis, Şcoala Generala 7, Feteşti
Andrei Alexandru, Şcoala „Mihai Eminscu”, Brăila
Menţiuni: Iancu Bogdan, C.N. „Mihai Viteazul”, Ploieşti
Petre Camelia Maria, C.N. „Mihai Eminscu”, Călăraşi
Presa Irina, C.N. „Mircea cel Bătrân”, Constanţa
Binder Elena Sabina, C.N. „Mircea cel Bătrân”, Constanţa
Petrea Carmen Daniela, C.N. „Mircea cel Bătrân”, Constanţa

Clasa a VIII-a

Premiul I: Popa Irina, Liceul de Artă, Slobozia


Premiul II: Ibram Remzi, Şcoala Generala 12, Constanţa
Premiul III: Butnaru Andrada, C.N. „Mihai Viteazul”, Ploieşti
Menţiuni: Bănceanu Alexandru, C.N. „Nicolae Bălaşa”, Brăila
Pupezeanu Otilia, C.N. „Nicolae Bălaşa”, Brăila

Clasa a IX-a

Premiul I: Voiteanu Mircea, C.N. „Mihai Viteazul”, Slobozia


Premiul II: Fulger Andreea, C.N. „Mircea cel Bătrân”, Constanţa
Premiul III: Delianu Florentina, C.N. „Mircea cel Bătrân”, Constanţa
Menţiuni: Voicu Ciprian, Liceul Teoretic, Urziceni
Rumalschi Radu, C.N. „I.L. Caragiale”, Ploieşti

Clasa a X-a

Premiul I: Pantelimon Ioana, Liceul Internaţional de Informatică, Constanţa


Premiul II: Nicolae Iulian, C.N. „I.L. Caragiale”, Ploieşti
Premiul III: Mirici Irina, C.N. „Mihai Eminscu”, Călăraşi
Menţiuni: Petre Vlad, C.N. „I.L. Caragiale”, Ploieşti
Amărioarei Alexandru, C.N. „Nicolae Bălcescu”, Brăila

Clasa a XI-a

Premiul I: Nicorici Ileana, C.N. „Mircea cel Bătrân”, Constanţa


Premiul II: Cuciumita Cleopatra, Liceul Teoretic „Anghel Saligny”, Cernavodă
Premiul III: Săndulescu Vlad, Nicolae Bălcescu”, Brăila
Menţiuni: Duţă Alexandru Andrei, C.N. „Mihai Viteazul”, Slobozia
Dochiu Marian, C.N. „Mihai Viteazul”, Slobozia

28
Clasa a XII-a

Premiul I: Păunescu Liviu, C.N. „Mircea cel Bătrân”, Constanţa


Premiul II: Laza Dorin Andrei, C.N. „Mircea cel Bătrân”, Constanţa
Premiul III: Fulger Aurel Mihai, C.N. „Mircea cel Bătrân”, Constanţa
Menţiuni: Ştefănescu Valentin, C.N. „Mihai Viteazul”, Ploieşti
Floreanu Cristian, C.N. „Barbu Ştirbei“, Călăraşi.

EDIȚIA a VIII-a, 15-16 noiembrie 2002


SUBIECTE

Clasa a V-a
I. Calculaţi:
1. 20023 − 20022 · 2001 − 2002 · 2001 − 2002;
0 2 5 20 0
2.[52002 + 55 ∙ 52 : (53 )19 + (52 ∙ 25 )4 : (57 ∙ 219 )] − 22 ∙ 33 (𝑎 − 𝑏) ∙ 20022002 : 20012001 ,
unde 𝑎 = 3 ∙ 42524 , iar 𝑏 = 21 ∙ 22 ∙ 23 ∙ ⋯ ∙ 2100 − (16631 )2;
3. Produsul tuturor numerelor de forma (𝑥 𝑦 − 𝑦 𝑥 ), ştiind că 𝑥 şi 𝑦 sunt numere naturale nenule
mai mici decât 10; 𝑥 ≠ 𝑦.

II. 1) Aflaţi necunoscuta y din ecuaţia: 1000𝑦 + 2 + 4 + 6 + ⋯ + 2002 = 1001 ∙ 2002 .


2) Stabiliţi numerele naturale 𝑎, 𝑏, 𝑥, 𝑦, 𝑧 ştiind că :4 ∙ (22𝑎 ∙ 5𝑎 + 𝑥𝑦𝑦
̅̅̅̅̅) + 2𝑏 = 2001 .
3) Suma a şapte numere naturale consecutive este un număr natural de forma 1𝑎000 ̅̅̅̅̅̅̅̅̅; aflaţi
aceste numere.
III. Un număr de trei cifre împărţit la răsturnatul său dă câtul 2 şi restul 100, iar diferenţa
dintre cifra sutelor şi cea a unităţilor numărului este egală cu 4. Aflaţi numărul.

Clasa a VI-a
2
2660∙10𝑛−1 168 12,(2)∙(0,2+ ∙𝑏)
15
I. 1) Se consideră: 𝑎 = 2𝑛+1 ∙5𝑛+3∙2𝑛∙5𝑛+2 +7∙2𝑛+3 ∙5𝑛 şi 810 = .
2,1(3)
a)Calculaţi 𝑎 şi 𝑏. b) Comparaţi 𝑎39 şi 𝑏 .
26

2) Fie a, b, c cifre în baza 10 şi diferite două câte două, 𝑛 = ̅̅̅


𝑎𝑏 + ̅̅̅
𝑏𝑎 + ̅̅̅
𝑏𝑐 + ̅̅̅
𝑐𝑏 + 𝑎𝑐
̅̅̅ + 𝑐𝑎
̅̅̅ .
a) Arătaţi că 11|𝑛.
b) Arătaţi că dacă 9|𝑛, atunci 9|𝑎𝑏𝑐̅̅̅̅̅.
c) Determinaţi ̅̅̅̅̅
𝑎𝑏𝑐 dacă 𝑛 = pătrat perfect.
̂ şi 𝐵𝑂𝐶
II. Fie unghiurile 𝐴𝑂𝐵 ̂ adiacente şi suplementare. Ştiind că suplementul complementului
̂ este cu 1000 mai mare decât complementul suplementului 𝐵𝑂𝐶
unghiului 𝐴𝑂𝐵 ̂ , să se determine
̂ ̂
măsurile unghiurilor 𝐴𝑂𝐵 şi 𝐵𝑂𝐶 .
III. Punctele 𝑀0 , 𝑀1 , 𝑀2 , ⋯ , 𝑀49 , 𝑀50 se află pe dreapte d (în această ordine), astfel încât
𝑀0 𝑀1 = 1cm , 𝑀1 𝑀2 = 2cm,..., 𝑀49 𝑀50 = 50cm. Să se afle lungimea segmentului 𝑀40 𝑀50 .

29
Clasa a VII-a

I. Să se determine toate numerele raţionale 𝑎, 𝑏, 𝑐 care au proprietatea că:


𝑎√𝑛 + 𝑏√𝑛 + 1 + 𝑐 √𝑛 + 2 = 0 , pentru orice 𝑛 ∈ ℕ.

II. În orice triunghi 𝐴𝐵𝐶, înălţimea 𝐴𝐷 şi mediana 𝐵𝑀 sunt congruente . Să se calculeze masura
̂.
𝑀𝐵𝐶
1 ̅̅̅̅̅̅̅̅̅
III. Să se determine cifrele nenule 𝑥, 𝑦 în baza 10 care au proprietatea că 𝑥+𝑦−1 = 0, 𝑥(𝑦).

Clasa a VIII-a

I. Pentru ce valori ale numărului real 𝑎 mulţimea [0,𝑎) ∪ [1,2] este interval?

II. Fie 𝐴𝐵 un diametru al cercului (𝐶) şi 𝑃 un punct din planul cercului nesituat pe cerc sau pe
𝐴𝐵. Să se construiască cu rigla o perpendiculară pe 𝐴𝐵, care trece prin 𝑃.

III. Să se determine toate numerele raţionale 𝑎, 𝑏, 𝑐 care au proprietatea că:


𝑎√𝑛 + 𝑏√𝑛 + 1 + 𝑐 √𝑛 + 2 = 0 , pentru orice 𝑛 ∈ ℕ.

Clasa a IX-a

I. Fie 𝑎, 𝑏, 𝑐 ∈ ℝ cu proprietatea că 𝑎 ≠ 0 şi 2𝑎 + 3𝑏 + 6𝑐 = 0. Să se arate că 𝑥 2 + 𝑏𝑥 + 𝑐 = 0


are rădăcini reale distincte şi cel puţin una din rădăcini este în [0,1].

II. Se dau punctele 𝐴 şi 𝐵 situate de aceeaşi parte a planului dat 𝛼. Să sa determine în planul 𝛼
un punct 𝑀, astfel încât suma 𝐴𝑀2 + 𝐵𝑀2 să fie minimă.

III. Pentru ce valori ale numărului real 𝑎, mulţimea [0,𝑎) ∪ [1,2] este interval?

Clasa a X-a

I. Fie 𝑝 > 0 un număr prim şi 𝑎 ∈ ℝ astfel încât 𝑝𝑎 ∈ ℚ.


1) Arătaţi că 𝑎 ∈ ℝ sau 𝑎 ∈ ℝ\ℚ.
2) Daţi un exemplu de număr 𝑎 ∈ ℝ\ℚ pentru care 𝑝𝑎 ∈ ℚ.

II. Fie 𝑎, 𝑏, 𝑐 ∈ ℝ cu proprietatea că 𝑎 ≠ 0 şi 2𝑎 + 3𝑏 + 6𝑐 = 0. Să se arate că 𝑥 2 + 𝑏𝑥 + 𝑐 = 0


are rădăcini reale distincte şi cel puţin una din rădăcini este în [0,1].

III. Fie un triunghi 𝐴𝐵𝐶 cu bisectoarele interioare 𝐴𝐴′ , 𝐵𝐵 ′, 𝐶𝐶 ′ , 𝐴′ ∈ (𝐵𝐶), 𝐵 ′ ∈ (𝐶𝐴),


𝐶 ′ ∈ (𝐴𝐵). În plus, 𝐴𝐴′ intersectează cercul circumscris lui 𝐴𝐵𝐶 în 𝐴′′ şi în mod similar, 𝐵𝐵 ′ în
𝐵′′ şi 𝐶𝐶 ′ în 𝐶′′. Să se demonstreze că :
𝐴𝐴′ 𝐵𝐵′ 𝐶𝐶 ′ 9
+ 𝐵𝐵′′ + 𝐶𝐶 ′′ ≤ 4.
𝐴𝐴′′

30
Clasa a XI-a
I. Fie 𝑎 ∈ ℕ, 𝑎 > 0 şi 𝑏 > 0. Se defineşte şirul (𝑥𝑛 )𝑛 prin relaţia 𝑥𝑛 = {√𝑎2 𝑛2 + 𝑏𝑛} , unde {𝑡}
este partea fracţionară a numărului real 𝑡. Să se arate că şirul (𝑥𝑛 )𝑛 este convergent şi să se
calculeze lim𝑛 𝑥𝑛 .
II. Fie 𝑧 ∈ ℂ cu |𝑧| = 1 şi 𝑛 ∈ ℕ∗ astfel încât 1 + 𝑧 + 𝑧 𝑛 = 0. Arătaţi că :
1) avem relaţia 𝑧 3 = 1;
2) numărul 𝑛 are forma 𝑛 = 3𝑘 + 2 , cu 𝑘 ∈ ℕ.

III. Fie 𝑝 > 0 un număr prim şi 𝑎 ∈ ℝ astfel încât 𝑝𝑎 ∈ ℚ.


1)Arătaţi că 𝑎 ∈ ℝ sau 𝑎 ∈ ℝ\ℚ.
2) Daţi un exemplu de număr 𝑎 ∈ ℝ\ℚ pentru care 𝑝𝑎 ∈ ℚ.

Clasa a XII-a

I. Pe o mulţime nevidă 𝑀 definim operaţiile ∗ şi ∘ prin 𝑥 ∗ 𝑦 = 𝑥 şi 𝑥 ∘ 𝑦 = 𝑦, pentru orice


𝑥, 𝑦 ∈ 𝑀. Arătaţi că:
1) Operaţiile ∗ şi ∘ sunt asociative.
2) Fiecare din operaţiile ∗ şi ∘ are element neutru dacă şi numai dacă 𝑀 conţine un singur
element.
II. Fie numerele 𝑝, 𝑞 reale nenule. Arătaţi că următoarele afirmaţii sunt echivalente:
𝑎𝑝 𝑏𝑞
1) Avem inegalitatea 𝑎𝑏 ≤ + , (∀)𝑎 > 0, 𝑏 > 0.
𝑝 𝑞
1 1
2) Avem simultan 𝑝 > 1 şi 𝑝 + 𝑞 = 1.

𝑥2 𝑦2
III. Se consideră două numere reale 𝑎 > 0, 𝑏 > 0 şi elipsa de ecuaţie + 𝑏2 = 1. Să se scrie
𝑎2
ecuaţiile tangentelor duse din punctul 𝑀(5𝑎, 5𝑏) la elipsă.

Premianţii concursului

Clasa a V– a

1. Bejgu Andreea – C.N. „ Mihai Viteazul” - Slobozia


2. Bengescu Adela – Liceul de Artă – Slobozia
3. Dumitrescu Diana – C.N. „Mihai Viteazul” – Slobozia
4. Gavrilă Amalia – Şc. nr. 2 – Slobozia
5. Mocanu Raluca – Liceul de Artă - Slobozia

Clasa a VI-a

1. Andrei Cristina – Şc. nr. 2 „Sf. Andrei” - Slobozia


2. Iosif George Mădălin – C.N. „Mihai Viteazul” - Slobozia
3. Chioveanu Adrian – C.N. „Mihai Viteazul” - Slobozia
4.Marin Adrian-Bogdan – Şc. nr.2 „Sf. Andrei” – Slobozia
5. Butoi-Varga Ştefania - Şc. nr.2 „Sf. Andrei” – Slobozia

31
Clasa a VII – a

1. Dobrota Valentin – Liceul Teoretic „Ovidius” - Constanţa


2. Cepoi Alexandru – Şc. nr.31 – Brăila
3. Beteringhe Elena – Sc. nr. 2 – Ţăndărei
4. Oprea Mihaela – C.N. „Mihai Viteazul” Slobozia
5. Diaconu Irina – C.N. „Gh. Murgoci” – Brăila

Clasa a VIII – a

1. Urda Ioan Daniel – Liceul Sportiv - Constanţa


2. Istrate Stefan – Alexeni – Ialomiţa
3. Săvescu Cristian – Şc. nr.23 – Ploieşti
4. Andrei Alexandru-Florin – Şc. „Mihai Eminescu” Brăila
5. Pană Emilia – Şc.nr.3 Slobozia

Clasa a IX – a

1. Bratu Ovidiu – C.N. „N. Balcescu” Brăila


2. Ibram Remzi – C.N. „Mircea cel Bătrân” – Constanţa
3. Popa-Balaciu Irina – C.N. „Mihai Viteazul” – Slobozia
4. Stanciu Ioana – Liceul Teoretic „ M. Eminescu” – Călăraşi
5. Moraru Catalin – C.N. „Mihai Vitezul” – Ploieşti

Clasa a X – a

1. Voiteanu Mircea – C.N. „Mihai Viteazul” - Slobozia


2. Puhalschi Radu-Cristian – C.N. „I. L. Caragiale” – Ploieşti
3. Andrei Asiria – C.N. „MihaiViteazul” – Slobozia
4. Fulger Andreea – C.N. „Mircea cel Batran” – Constanţa
5. Brînză Horaţiu-Vlăduţ – C.N. „ Gh. Murgoci” - Brăila

Clasa a XI – a

1. Nicolae Iulian-Mihai – C.N. I.L. Caragiale” - Ploieşti


2. Pantelimon Ioan – Ruxandru – Lic. Internaţional de inf. – Constanţa
3. Chirvasitu Alexandru – C.N. „Mircea cel Bătrân” – Constanţa
4. Andronescu Corina-Raluca – C.N. „Mircea cel Bătrân” – Constanţa
5. Popescu Adrian – C.N. „Mihai Viteazul” – Slobozia

Clasa a XII – a

1. Nicorici Constantin – C.N. „ Mircea cel Bătrân” Constanţa


2. Bratu Eugen-Florin – C.N. „N. Balcescu” – Brăila
3. Vădeanu T.V. Vladimir – C.N. „ N. Balcescu” – Brăila
4. Plăiaşu Tanase – C.N. „ I.L. Caragiale” – Ploieşti
5. Carpen Amarie Gh. Alexandra – Liceul Teoretic - Urziceni
32
EDIŢIA a X-a, 24- IV-2004

SUBIECTE

Clasa a VI-a

I. Să se determine numerele naturale 𝑥 şi 𝑦 astfel încât 𝑁 ⋮ 2004, unde 𝑁 = ̅̅̅̅̅̅̅̅̅̅


334𝑥𝑥𝑦.
1 2 3 2003 1 1 1 1
II. a) Arătaţi că 3|(𝑎 + 𝑏) dacă 𝑎 = 2 + 3 + 4 + ⋯ + 2004 şi 𝑏 = 1 + 2 + 3 + ⋯ + 2004 .
𝑥 𝑦 𝑧 𝑡
b) Să se afle 𝑥, 𝑦, 𝑧, 𝑡, ştiind că 𝑥𝑦𝑧𝑡 = 1756920 şi 2 = 3 = 4 = 5.
III. Fie ∆𝐴𝐵𝐶 în care 2𝐴𝐵 = 𝐵𝐶 şi 𝑚(𝐴𝐵𝐶 ̂ ) = 2𝑚(𝐴𝐶𝐵 ̂)
a) Stabiliţi natura triunghiului 𝐴𝐵𝐶: isoscel, dreptunghic, echilateral sau dreptunghic
isoscel.
b) Dacă 𝐷 este mijlocul lui (𝐵𝐶), 𝐸 ∈ (𝐴𝐶) astfel încât 𝑚(𝐸𝐵𝐶̂ ) = 𝑚(𝐸𝐶𝐵 ̂ ),
{𝐹} = 𝐴𝐷 ∩ 𝐵𝐸 şi aria ∆𝐴𝐵𝐶 este 𝑎, să se calculeze aria ∆𝐴𝐵𝐹.

Clasa a VII-a

I. Fie 𝐴𝐵𝐶𝐷 un paralelogram şi 𝐸, 𝐹 mijloacele laturilor 𝐴𝐵 şi 𝐴𝐷. Arătaţi că dreptele 𝐶𝐸 şi 𝐶𝐹


împart diagonala 𝐵𝐷 în trei segmente de lungimi egale.
II. Arătaţi că 31 divide numărul: 1 + 2 + 22 + 23 + ⋯ + 22004 .
III. Determinaţi numerele întregi 𝑥, 𝑦 care satisfac relaţia 4𝑥𝑦 + 𝑦 − 3𝑥 + 2 = 0.

Clasa a VIII-a

I. Arătaţi că dacă 𝑎 ∈ ℝ şi 𝑏 ∈ ℝ, atunci are loc inegalitatea:


𝑎+𝑏 2
𝑎2 + ( 2 ) ≤ 4(𝑎2 + 𝑏 2 ). Când are loc egalitatea?
II. Considerăm cubul 𝐴𝐵𝐶𝐷𝐴′𝐵′𝐶′𝐷′, având muchia de lungime 1 şi mijloacele 𝑀, 𝑁, 𝑃 ale
muchiilor 𝐴𝐵, 𝐶′𝐷′ şi 𝐴𝐷. Aflaţi:
a) lungimile laturilor triunghiului 𝑀𝑁𝑃 şi deduceţi că acesta este dreptunghic.
b) Măsura unghiului diedru dintre planele (𝑀𝑁𝑃) şi (𝐴𝐵𝐶𝐷).

III. Fie 𝐴𝐵𝐶𝐷 un patrulater convex astfel încât 𝑚(𝐴̂ ) ≠ 𝑚(𝐶̂ ) şi (𝐵̂ ) ≠ 𝑚(𝐷 ̂ ) . Arătaţi că
punctele de intersecţie ale bisectoarelor unghiurilor patrulaterului 𝐴𝐵𝐶𝐷 reprezintă vârfurile unui
patrulater inscriptibil.

Clasa a IX-a

I. Arătaţi că dacă 𝑎 ∈ ℝ şi 𝑏 ∈ ℝ nu sunt simultan nule, atunci are loc inegalitatea


|𝑎+𝑏|+|𝑎−𝑏|
√2 ≤ √𝑎2 2 ≤ 2 .
+𝑏

33
II. Reprezentaţi grafic funcţia 𝑓: ℝ → ℝ care satisface relaţia
(𝑓(𝑥) − 𝑥 2 )(4 + 𝑦 2 − 𝑓(𝑦)) ≥ 4, (∀)𝑥, 𝑦 ∈ ℝ

III. Fie 𝐴𝐵𝐶𝐷 un tetraedru regulat şi în interiorul său un punct 𝑀 astfel încât distanţele
𝑀𝐴, 𝑀𝐵, 𝑀𝐶 sunt egale cu √3 , iar distanţa 𝑀𝐷 este egală cu 1. Calculaţi volumul tetraedrului.

Clasa a X-a
I. Fie 𝑛 ∈ ℕ, 𝑛 ≥ 2, iar 𝑃 = ∑𝑛𝑘=0 𝑎𝑛 𝑧 𝑛un polinom cu coeficienţi complecşi astfel încât 𝑎1 ≠ 0.
Arătaţi că dacă ∑𝑛𝑘=2 𝑘|𝑎𝑘 | ≤ |𝑎1 |, atunci polinomul 𝑃 are cel mult o rădăcină 𝑧 ∈ ℂ cu
proprietatea că |𝑧| < 1.
II. Fie 𝑎, 𝑏, 𝑐 numere reale strict pozitive diferite de 1, iar 𝑥, 𝑦, 𝑧 numere reale astfel încât
𝑎 𝑥 = 𝑏𝑐, 𝑏 𝑦 = 𝑐𝑎, 𝑐 𝑧 = 𝑎𝑏. Arătaţi că 𝑥𝑦𝑧 − 𝑥 − 𝑦 − 𝑧 = 2.
III. Determinaţi valorile parametrului real 𝑚 astfel încât pentru orice numere reale 𝑎, 𝑏, 𝑐 are loc
inegalitatea 𝑎2 + 𝑏 2 + 𝑐 2 ≥ 𝑚(𝑎 + 𝑏 + 𝑐)2 .

Clasa a XI-a

I. Fie 𝐴, 𝐵 ∈ 𝑀𝑛 (ℝ) cu proprietatea că există 𝑋 ∈ 𝑀𝑛 (ℂ), 𝑋 inversabilă astfel încât 𝑋𝐴𝑋 −1 = 𝐵.


Arătaţi că există 𝑌 ∈ 𝑀𝑛 (ℝ), 𝑌 inversabilă astfel încât 𝑌𝐴𝑌 −1 = 𝐵.
1
II. Fie (𝑥𝑛 )𝑛∈ℕ un şir de numere reale care satisface condiţia 0 ≤ 𝑥𝑛+1 − 𝑥𝑛 ≤ , (∀) 𝑛 ∈ ℕ.
𝑛
Arătaţi că există 𝑎 ∈ ℝ astfel încât lim𝑛→∞ (2𝑛 𝑎 − 𝑥𝑛 ) = 0.
III. Calculaţi ∑𝑛𝑘=0(𝑛 − 2𝑘)2 𝐶𝑛𝑘 .

Clasa a XII-a

I. Fie 𝐺 mulţimea tuturor matricelor pătratice de ordinul 3 cu zero deasupra diagonalei


principale şi unu pe diagonala principală .
i) Arătaţi că 𝐺 este un grup în raport cu operaţia de înmulţire a matricelor.
ii) Determinaţi forma matricelor 𝐴 ∈ 𝐺 care au proprietatea că 𝐴𝑋 = 𝑋𝐴 pentru
orice 𝑋 ∈ 𝐺.
1 𝑥 𝑛 𝑒 −𝑥
II. Calculaţi lim𝑛→∞ 𝑛 ∫0 𝑑𝑥
𝑥 2𝑛 +1
III. Demonstraţi că :
𝑡2
a) pentru orice 𝑡 ∈ [0, ∞) are loc inegalitatea 𝑒 𝑡−2 ≥ .
4
b) pentru orice 𝑥 ∈ [0, ∞) şi orice 𝑦 ∈ [0, ∞) are loc inegalitatea
(𝑥+𝑦)2
𝑒 𝑥+𝑦−2 ≥ . Când are loc egalitatea?
4

Premiile Concursului

Clasa a VI – a

Premiul I : Bejgu Andreea – C.N. „ Mihai Viteazul” - Slobozia

34
Premiul I : Ciucă Bogdan – C.N. „Mihai Viteazul” – Slobozia
Premiul II : Udrea Cosmin - Şc. Gen. Nr. 2, Urziceni, Ialomiţa;
Premiul III : Mincu Diana - Liceul de Artă, Slobozia
Menţiune 1 : Mihalcea Şerban – Liceul de Artă, Slobozia, Ialomiţa,
Menţiune 2 : Buriceanu Florin, Școala nr. 1 Urziceni

Clasa a VII – a

Premiul I : Vătășelu Sotina – L.T. “M. Eminescu”, Călărași


Premiul II : Sima Cotizo – C.N. “I.L. Caragiale”, Ploiești
Premiul III : Mocanu Livia – Școala nr.7 Fetești
Menţiune 1 : Gornea Diana – Școala nr.7 Fetești
Menţiune 2 : Ogrezeanu Mihaela - Şc. Nr. 2, ”Sf. Andrei” Slobozia

Clasa a VIII – a

Premiul I : Zlotea Dorin – C.N. “Mihai Viteazul”, Slobozia, Ialomiţa ;


Premiul II : Mirică Ema - C.N. “Mihai Viteazul”, Slobozia, Ialomiţa;
Premiul III : Mocanu Lucian - C.N. “Mircea cel Bătrân”, Constanța;
Menţiune 1 : Radu Iulian – Liceul Teoretic „M. Eminescu”, Călărași
Menţiune 2 : Vasiliu Radu – Școala nr. 1 Buzău.

Clasa a IX – a

Premiul I : Alexandrescu Andreea – C.N. “C. Cantacuzino”, Sinaia


Premiul I : Istrate Ștefan – Liceul Teoretic Urziceni
Premiul I : Tănase Narcis – Liceul Teoretic Fetești;
Premiul II : Florescu Patricia - C.N. ”I.L. Caragiale” Ploiești
Premiul III : Munteanu Ciprian – Liceul Militar ”M. Basarab” Buzău

Clasa a X – a

Premiul I : Bănică Dan - C.N. ”I.L. Caragiale” Ploiești;


Premiul II: Constantinescu Simona - C.N. “Mircea cel Bătrân”, Constanța
Premiul III : Ibram Remzi – C.N. “Mircea cel Bătrân”, Constanța;
Menţiune 1 : Popa Irina – C.N. “Mihai Viteazul”, Slobozia, Ialomiţa
Menţiune 2 : Posea Simona – C.N. „ B.P. Hașdeui”,Buzău.

Clasa a XI – a

Premiul I : Ablai Ainur – C.N. “ Mircea cel Bătrân”, Constanţa;

35
Premiul II : Andrei Asiria – C.N. „ Mihai Viteazul”, Slobozia
Premiul III : Ciuraru Cosmin - Liceul Teoretic ”M. Eminescu” Călărași
Menţiune 1 : Manea Flavius - Liceul Teoretic ”M. Eminescu” Călărași
Menţiune 2 : Șindrilaru Elvin – C.N. „ M. Viteazul”, Ploiești.

Clasa a XII – a

Premiul I : Nicolae Iulian-Mihai – C.N. ”I.L. Caragiale” - Ploieşti


Premiul II: Petre Vlad – C.N. ”I.L. Caragiale” - Ploieşti
Premiul III : Eșanu Liviu – Liceul Internațional de Informatică Constanța
Menţiune 1: Baraitaru Răzvan – Liceul Teoretic ”A. Saligny Cernavodă
Menţiune 2 : Grigorescu Costin – Liceul Teoretic Urziceni

EDIŢIA a XI-a, 20 noiembrie 2004

SUBIECTE

Clasa a V-a

I. a) Aflaţi restul împărţirii numărului 𝑎 = 1 + 4 + 42 + 43 + ⋯ + 42004 la 21.


b)Se dă numărul 𝑥 = 1 + 2 + 3 + ⋯ + 2004. Să se arate că numărul
2𝑥 + 2005 este pătrat perfect.
c) Se dau numerele 𝐴 = 𝑎1 ∙ 𝑎2 ∙ 𝑎3 ∙ ⋯ ∙ 𝑎1998 şi 𝐵 = 𝑎999 ∙ (𝑎999 )1999 , unde 𝑎 ∈
ℕ. Să se compare 𝐴 şi 𝐵.

***

II. Să se determine numerele ̅̅̅̅̅


𝑎𝑏𝑐 ştiind că : 3𝑎+𝑏 + 3𝑏+𝑐 + 3𝑐+𝑎 = 351.

***

III. Să se determine numerele naturale nenule care împărţite la 4 dau câtul 𝑎 şi restul 𝑏, iar
împărţite la 10 dau câtul 𝑏 şi restul 𝑎.
***

Clasa a VI-a

I. Punctele 𝐴1 , 𝐴2 , … , 𝐴𝑛 , aparţin, în această ordine, segmentului (AB). Ştiind că 𝐴𝐴1 = 1𝑐𝑚,


𝐴1 𝐴2 = 2𝑐𝑚, 𝐴2 𝐴3 = 22 𝑐𝑚, ...., 𝐴𝑛−1 𝐴𝑛 = 2𝑛−1 𝑐𝑚 şi 𝐴𝐵 = 1055𝑐𝑚, aflaţi:
a) valoarea maximă a numărului natural 𝑛;
b) lungimea segmentului [𝐴𝑛 𝐵], pentru 𝑛 determinat anterior.

***

36
II. a) Determinaţi ultima cifră a numărului 5 ∙ 6𝑛 , 𝑛 ∈ ℕ.
b)Să se arate că pentru orice număr natural 𝑛, 𝑛 ≥ 1, numărul
36𝑛 ∙5𝑛+1 ∙7−22𝑛+1 ∙32𝑛+1 ∙5𝑛
𝐴= este număr natural.
6𝑛 ∙5𝑛−1 ∙19+2𝑛+1 ∙15𝑛
***
III. Fie a un număr prim astfel încât a+8 este număr prim. Determinaţi numărul a astfel încât
𝑎+4
fracţia 𝑎+10 să fie ireductibilă.
Marian Mărgărit, Slobozia

Clasa a VII-a

I. Arătaţi că pentru orice 𝑛 ∈ ℕ∗ , numărul 9 ∙ 22𝑛 + 12 ∙ 2𝑛 + 4 este pătrat perfect .


***

II. Se dau 61 de unghiuri congruente cu măsurile egale cu 1200 : ∡𝐴1 𝑂𝐴2 ,


∡𝐴3 𝑂𝐴4 ,...., ∡𝐴121 𝑂𝐴122 . Semidreapta [𝑂𝑋 împarte fiecare unghi în câte două unghiuri care au
măsurile exprimate în grade printr-un număr natural.
Demonstraţi că există unghiurile ∡𝐴𝑖 𝑂𝐴𝑖+1 şi ∡𝐴𝑗 𝑂𝐴𝑗+1 , cu 𝑖 ≠ 𝑗, 𝑖, 𝑗 ∈ {1,2, ⋯ ,122} astfel
încât ∡𝐴𝑖 𝑂𝑋 ≡ ∡𝐴𝑗 𝑂𝑋.
***
1 1 1
III. Demonstraţi că 1 + 2 + 22 + ⋯ + 2𝑛 < 2, pentru orice valori ale lui 𝑛 ∈ ℕ.
***

Clasa a VIII-a

I. Să se arate că nu există numere reale 𝑥, 𝑦, 𝑧 astfel încât:


12𝑦 − 4𝑥 2 = 1
{24𝑧 − 9𝑦 2 = 4
4𝑥 − 16𝑧 2 = 9

Aurel Doboşan, Lugoj


II. Fie 𝐴𝐵𝐶𝐷 un patrulater circumscriptiv. Să se arate că diagonalele sale sunt perpendiculare
dacă şi numai dacă una din diagonale este mediatoarea celeilalte.
***

III. Fie 𝑉𝐴𝐵𝐶 un tetraedru regulat.


a) Fie 𝑀 un punct pe muchia 𝑉𝐴 astfel încât 𝑉𝐴 = 4𝑉𝑀 şi fie 𝑁 mijlocul muchiei
𝐵𝐶. Arătaţi că 𝑀𝑁 trece prin mijlocul înălţimii din 𝑉 a tetraedrului 𝑉𝐴𝐵𝐶.
b) Arătaţi că triunghiul 𝐴𝑀𝑁 este isoscel.
***

37
Clasa a IX-a
I. Să se arate că nu există numere reale 𝑥, 𝑦, 𝑧 astfel încât:
12𝑦 − 4𝑥 2 = 1
{24𝑧 − 9𝑦 2 = 4
4𝑥 − 16𝑧 2 = 9
Aurel Doboşan, Lugoj

II. Rezolvaţi în ℕ ecuaţia 5[√𝑛] = 𝑛 + 2.


Aurel Doboşan, Lugoj

III. Fie VABC un tetraedru regulat.


a) Fie 𝑀 un punct pe muchia 𝑉𝐴 astfel încât 𝑉𝐴 = 4𝑉𝑀 şi fie 𝑁 mijlocul muchiei
𝐵𝐶. Arătaţi că 𝑀𝑁 trece prin mijlocul înălţimii din 𝑉 a tetraedrului 𝑉𝐴𝐵𝐶.
b) Arătaţi că triunghiul 𝐴𝑀𝑁 este isoscel.

***
Clasa a X-a

I. Să se determine cel mai mare număr natural 𝑛 ≥ 1 cu proprietatea că următoarea implicaţie


este adevărată:
∀𝑧1 , 𝑧2 , ⋯ , 𝑧𝑛 ∈ ℂ : (∑𝑛𝑘=1 𝑧𝑘 = 0 ) şi (∑𝑛𝑘=1 𝑧𝑘2 = 0)⇒(𝑧1 = 𝑧2 = ⋯ = 𝑧𝑛 = 0).

Radu Miculescu, Bucureşti


II. Să se arate că unica funcţie injectivă 𝑓: ℝ → ℝ care are proprietatea că
𝑓 (𝑥 + 𝑓(𝑦 + 𝑓(𝑧))) = 𝑓(𝑥 + 𝑦 + 𝑧 + 1) + 1, (∀) 𝑥, 𝑦, 𝑧 ∈ ℝ este dată de 𝑓(𝑥) = 𝑥 + 1.

Petruş Alexandrescu, Bucureşti


III. Patrulaterul convex 𝐴𝐵𝐶𝐷 are laturile de lungimi 𝑎, 𝑏, 𝑐 ş𝑖 𝑑. Se notează cu 𝑆 aria sa. Să se
arate că 𝐴𝐵𝐶𝐷 este pătrat dacă şi numai dacă :
4𝑆 = 𝑎2 + 𝑏 2 + 𝑐 2 + 𝑑2 .

***

Clasa a XI-a

I. Fie şirurile (𝑎𝑛 )𝑛 , (𝑏𝑛 )𝑛 , 𝑛 ∈ ℕ, cu 𝑎0 , 𝑏0 > 0, ce îndeplinesc relaţiile:


𝑎𝑛 = 3𝑎𝑛−1 + 2𝑏𝑛−1 ; 𝑏𝑛 = 2𝑎𝑛−1 + 3𝑏𝑛−1 , (∀)𝑛 ≥ 1.
𝑎𝑛
Să se demonstreze că şirul (𝑥𝑛 )𝑛 , cu 𝑥𝑛 = , este convergent şi să se calculeze limita sa.
𝑏𝑛
***

38
4 4
𝑥+ 2 𝑦+ 2
II. Să se rezolve în ℝ∗+ ecuaţia: 2 𝑦 +2 𝑥 = 16.
***
III. Fie 𝜋 un plan şi 𝐴, 𝐵 două puncte în spaţiu care nu aparţin planului 𝜋 şi nu sunt egal
depărtate de planul .
Să se determine un punct 𝑀 în planul 𝜋 astfel încât |𝐴𝑀 − 𝑀𝐵| să fie maxim.
***
Clasa a XII-a

I. Se consideră matricele 𝐴, 𝐵 ∈ ℳ𝑛 (ℂ), 𝐴 inversabilă şi notăm 𝐴−1 inversa ei. Să se


demonstreze că dacă 3𝐴𝐵𝐴−1 + 𝐴 = 2𝐴−1 𝐵𝐴, atunci 𝑑𝑒𝑡(𝐴𝐵𝐴−1 − 𝐴−1 𝐵𝐴) = 0.
Cezar Lupu, student, Bucureşti
II. Fie 𝑎, 𝑏 ∈ ℤ, 𝑎 ≠ 0. Se defineşte legea de compoziţie ∗ pe ℤ dată după cum urmează :
𝑥 ∗ 𝑦 = 𝑎𝑥𝑦 − 𝑎𝑏(𝑥 + 𝑦) + 𝑏(𝑎𝑏 + 1) , pentru orice 𝑥, 𝑦 ∈ ℤ.
1) Să se arate că legea poate fi rescrisă sub forma: 𝑥 ∗ 𝑦 = 𝑎(𝑥 − 𝑏)(𝑦 − 𝑏) + 𝑏
𝑥 ∗ 𝑥 ∗ ⋯ ∗ 𝑥 = 𝑎𝑛−1 (𝑥 − 𝑏)𝑛 + 𝑏 ∈ 𝑀, (∀)𝑛 ∈ ℕ∗ , 𝑥 ∈ ℤ .
2) Să se arate că ⏟
𝑛 𝑜𝑟𝑖
3) Să se determine părţile finite ale lui ℤ, stabile faşă de ∗.

Nicolae Papacu, Slobozia


III. i) Să se enunţe teorema lui Lagrange.
ii)Fie 𝑓: (𝑎, 𝑏) → ℝ o funcţie derivabilă cu derivata mărginită, unde 𝑎, 𝑏 ∈ ℝ. Să se arate că 𝑓
este mărginită.
iii)Fie 𝑓: ℝ → ℝ o funcţie derivabilă pe ℝ\{1} cu proprietatea că există 𝑀 > 0 astfel încât :
|𝑥𝑓’(𝑥) ln 𝑥 − 𝑓(𝑥)| ≤ 𝑀𝑥(ln 𝑥)2 , (∀)𝑥 ∈ (0,1) ∪ (1, ∞).
Să se arate că lim𝑥→1 𝑓(𝑥) = 0.

Radu Miculescu, Bucureşti

Premianţii concursului

Clasa a V – a

Premiul I : Drăgoi Octav – Şc. Gen. Nr.3 Slobozia, Ialomiţa;


Premiul II : Popescu Teodor - Şc. Gen. Nr. 3, Slobozia, Ialomiţa;
Premiul III : Radu Oana – Şc. Gen. Nr.3, Slobozia, Ialomiţa;
Menţiune : Marin Gina – Şc. Gen. Nr.3 Slobozia, Ialomiţa,
Menţiune :Prahoveanu Goerge – Şc. Gen. Nr.3 Slobozia, Ialomiţa.

Clasa a VI – a

Premiul I : Radu Andreea – Şc. Gen. Nr.3 Slobozia, Ialomiţa;


Premiul II : Frăţilă Vlad - Şc. Gen. Nr. 3, Slobozia, Ialomiţa;
Premiul III : Costea Elisa – Şc. Gen. Nr.3, Slobozia, Ialomiţa,
Premiul III : Drăjneanu Alina – Şc. Gen. Nr.3, Slobozia, Ialomiţa;

39
Menţiune : Neghină Cornel – Liceul de Artă, Slobozia, Ialomiţa.

Clasa a VII – a

Premiul I : Cârstoiu Cristina – C.N. “Vlaicu Vodă”, Curtea de Argeş, Argeş;


Premiul II : Udrea Cosmin - Şc. Gen. Nr. 2, Urziceni, Ialomiţa;
Premiul III : Sandu Smaranda – C.N. „ Bălcescu” Brăila;
Menţiune : Mihalcea Şerban – Liceul de Artă, Slobozia, Ialomiţa,
Menţiune : Mocanu Raluca – Liceul de Artă, Slobozia, Ialomiţa.

Clasa a VIII – a

Premiul I : Ciocan Cristina – C.N. “ Bălcescu”, Brăila;


Premiul II : Bulimar Eliza – C.N. “Bălcescu”, Brăila;
Premiul III : Manea Mara - C.N. “Bălcescu”, Brăila;
Menţiune : Goga Steliana – Liceul Teoretic „ Ovidius” Constanţa;
Menţiune : Marin Adrian - Şc. Gen. Nr. 2, Slobozia, Ialomiţa.

Clasa a IX – a

Premiul I : Marin Violeta – C.N. “Mihai Viteazul”, Slobozia, Ialomiţa ;


Premiul II : Spiridon Adrian – Lic. Teoretic Urziceni, Ialomiţa;
Premiul III : Mirică Ema - C.N. “Mihai Viteazul”, Slobozia, Ialomiţa ;
Menţiune : Barzan Ruxandra – Liceul Teoretic „Ovidius”, Constanţa,
Menţiune : Baranga Ionuţ - C.N. “Mihai Viteazul”, Slobozia, Ialomiţa.

Clasa a X – a

Premiul I : Andrei Alexandru – C.N. “Bălcescu”, Brăila;


Premiul II : Caragea Andrei – Lic. Pedagogic, Galaţi;
Premiul III : Burceanu Elena – C.N. „ Mircea cel Bătrân” Constanţa;
Menţiune : Petre Camelia – Lic. „ M. Eminescu”, Călăraşi;
Menţiune : Petrache Lavinia - C.N. “Mihai Viteazul”, Slobozia, Ialomiţa.

Clasa a XI – a

Premiul I : Bratu Ovidiu – C.N. “Bălcescu”, Brăila ;


Premiul II : Gudu Diana – C.N. “Mihai Viteazul”, Slobozia, Ialomiţa;
Premiul III : Cantaragiu Ramona – Lic. „ M. Eminescu”, Călăraşi;
Menţiune : Amza Cristina – Lic. „ Barbu Ştirbei”, Călăraşi;
Menţiune : Drăgan Anca – C.N. „ Bălcescu”, Brăila.

40
Clasa a XII – a

Premiul I : Ablai Ainur – C.N. “ Mircea cel Bătrân”, Constanţa;


Premiul II : Tardea Anca – C.N. “ Bălcescu”, Brăila;
Premiul III : Andrei Asiria – C.N. „ Mihai Viteazul”, Slobozia ;
Menţiune : Buboiu Manuela – C.N. „ Kogălniceanu”, Galaţi,
Menţiune : Bordianu Andreea – C.N. „Bălcescu”, Brăila.

EDIŢIA a XII-a, 25 martie 2006

SUBIECTE

Clasa a V-a

𝑛
I. a) Arătaţi că fracţia 8𝑛+1 este ireductibilă pentru orice 𝑛 ∈ ℕ∗ .
𝑛+1
b)Determinaţi valorile lui 𝑛 ∈ ℕ∗ pentru care fracţia 8𝑛+1 este reductibilă.
Ilie Neacşu, Slobozia
II. Să se găsească toate numerele :
a) de forma ̅̅̅̅̅
𝑎𝑏𝑐 , 𝑎 < 𝑏 < 𝑐, astfel încât ̅̅̅̅̅
𝑎𝑏𝑐 + 11(𝑎 + 𝑏 + 𝑐) = ̅̅̅̅̅
𝑐𝑏𝑎 .
1 1 1
b) de forma 𝑥𝑦
̅̅̅ cu proprietatea : 𝑥 + 𝑦 + 𝑥𝑦 = 1.

Marian Mărgărit, Slobozia


III. Arătaţi că:
a) Numărul 𝐴 = 20022002 + 20032003 + 20042004 + 20052005 + 20062006 nu
este pătrat perfect;
b) 123321 > 321123 .

Marcel Popescu, Slobozia


Clasa a VI-a
I. Să se arate că:
̅̅̅̅̅ dacă şi numai dacă 7|2𝑎 + 3𝑏 + 𝑐;
a) 7|𝑎𝑏𝑐
3371 +1
b) 1 + 2 ∙ 3 + 3 ∙ 32 + 4 ∙ 33 + 5 ∙ 34 + ⋯ + 365 ∙ 3364 = .
4
Ilie Neacşu, Slobozia
5𝑦+4𝑧−𝑥 5𝑧+4𝑥−𝑦 5𝑥+4𝑦−𝑧
II. Se consideră numerele raţionale nenule 𝑥, 𝑦, 𝑧 astfel încât: = = .
3𝑥 3𝑦 3𝑧
(5𝑦+4𝑧)(5𝑧+4𝑥)(5𝑥+4𝑦)
Calculaţi : .
𝑥𝑦𝑧
Ilie Neacşu, Slobozia

III. Se consideră triunghiul 𝐴𝐵𝐶 cu 𝐴𝐵 > 𝐴𝐶. Paralelele duse prin punctele 𝐵 şi 𝐶 la bisectoarea
̂ intersectează dreptele 𝐴𝐶 ş𝑖 𝐴𝐵 în 𝐷, respectiv 𝐸.
unghiului 𝐵𝐴𝐶

41
a) Să se arate că dreptele 𝐷𝐸 ş𝑖 𝐵𝐶 nu sunt paralele.
b) Dacă 𝐷𝐸 ∩ 𝐵𝐶 = {𝐹}, să se arate că triunghiul 𝐹𝐸𝐶 este isoscel.
Marian Mărgărit, Slobozia

Clasa a VII-a
1 1 1 1 1 1 1 1
I. Fie numerele : 𝑛1 = 1∙2 + 3∙4 + 5∙6 + ⋯ + 2003∙2004 și 𝑛2 = 2∙3 + 4∙5 + 6∙7 + ⋯ + 2004∙2005 .
a) Calculaţi media lor aritmetică.
1002
b) Arătaţi că 𝑛2 < 2005 < 𝑛1 .
Vasile Tarciniu, Odobeşti

II. a) Să se demonstreze că dacă 𝑛 ∈ ℕ, 𝑛 ≥ 2, atunci are loc următorul criteriu de divizibilitate:


𝑎1 𝑎2 ⋯ 𝑎𝑛−1 𝑎𝑛 ⋮ 17 ⇔ (𝑎
̅̅̅̅̅̅̅̅̅̅̅̅̅̅̅̅̅̅̅ 1 𝑎2 ⋯ 𝑎𝑛−1 − 5𝑎𝑛 ) ⋮ 17 .
̅̅̅̅̅̅̅̅̅̅̅̅̅̅̅̅
b) Să se aplice acest criteriu pentru a stabili dacă 17 este divizor al lui 2006.
III. Se consideră triunghiul ascuţitunghic 𝐴𝐵𝐶. Înălţimea din 𝐴 intersectează bisectoarea din 𝐵
în 𝑃; bisectoarea din 𝐵 intersectează mediana din 𝐶 în 𝑄; mediana din 𝐶 intersectează înălţimea
din 𝐴 în 𝑅. Să se arate că punctele 𝑃, 𝑄, 𝑅 nu pot fi vârfurile unui triunghi echilateral.
I.C. Drăghicescu, Bucureşti

Clasa a VIII-a

I. Dacă 𝑥, 𝑦 ∈ ℕ∗ astfel că 4𝑥−2 + 4𝑦+2 ≤ 2𝑥+𝑦+1 , să se arate că numărul 2𝑥 + 2𝑦 este divizibil


prin 34.
Dan Nedeianu, Drobeta Tr. Severin

𝑥+1 𝑥+𝑛+1
II. Determinaţi 𝑛 natural astfel încât ecuaţia: [𝑛+1] + 𝑛 = să admită 2006 soluţii.
𝑛
Gh. Ghiţă, Buzău

III. Fie 𝑉𝐴𝑀𝑁 ş𝑖 𝑉𝐴𝐵𝐶𝐷𝐸𝐹 două piramide regulate cu bazele în acelaşi plan. Demonstraţi că:
𝐴𝑙 (𝑉𝐴𝐵𝐶𝐷𝐸𝐹) 2√2
a) ∈( , 2).
𝐴𝑙 (𝑉𝐴𝑀𝑁) 3
3√2
b) 𝑑( 𝑐𝑒𝑛𝑡𝑟𝑢𝑙 𝑏𝑎𝑧𝑒𝑖 𝐴𝑀𝑁; (𝑉𝑀𝑁)) < 𝐴𝐵 , dacă 𝑉𝑂 < 𝐴𝐵, unde 𝑂 este
8
proiecţia lui 𝑉 pe planul bazelor celor două piramide.
Gheorghe F. Molea, Curtea de Argeş

Clasa a IX-a

I. Să se demonstreze că:
1
∑𝑛𝑘=1 3 3 3 < 1 , 𝑛 ∈ ℕ∗ .
√𝑘(𝑘+1)+ √𝑘 2 (𝑘+1)2 +𝑘 √𝑘+1
Irina Cunţan, Sibiu

42
II. Se consideră ecuaţia: 𝑥 2 + 𝑏|𝑥| + 𝑐 = 0 ; 𝑎 ∈ ℝ∗ , 𝑏, 𝑐 ∈ ℝ
Demonstraţi că următoarele afirmaţii sunt echivalente:
i) ecuaţia are soluţie unică;
ii) avem 𝑐 = 0 şi 𝑎𝑏 ≥ 0.
Dan Nedeianu, Drobeta Tr. Severin

III. Fie triunghiul ABC şi punctul 𝑀 ∈ 𝐼𝑛𝑡(𝐴𝐵𝐶). Dacă 𝐺, 𝐺𝐴 , 𝐺𝐵 respectiv 𝐺𝐶 sunt centrele de
greutate ale triunghiurilor 𝐴𝐵𝐶, 𝑀𝐵𝐶, 𝑀𝐴𝐶, respectiv 𝑀𝐴𝐵, să se arate că: ⃗⃗⃗⃗⃗⃗⃗
𝐴𝐺𝐴 + ⃗⃗⃗⃗⃗⃗⃗⃗
𝐵𝐺𝐵 + ⃗⃗⃗⃗⃗⃗⃗⃗
𝐶𝐺𝐶 =
⃗0 dacă şi numai dacă 𝑀 = 𝐺.
Nicolae Papacu, Slobozia

Clasa a X-a

I. Fie numerele reale 𝑎 > 1, 𝑏 > 0 astfel încât 2𝑎 + 𝑏 = 10. Rezolvaţi în ℝ ecuaţia:
1 log1 (𝑥−𝑎)
(10) 𝑎 − 𝑎𝑙𝑔(𝑥+𝑏) = 𝑎 + 𝑏 .
Marin Chirciu, Piteşti

II. Fie tetraedrul [𝐴𝐵𝐶𝐷] şi punctul M în spaţiu. Dacă 𝐺, 𝐺𝐴 , 𝐺𝐵 , 𝐺𝐶 respectiv 𝐺𝐷 sunt centrele de
greutate ale tetraedrelor [𝐴𝐵𝐶𝐷], [𝑀𝐵𝐶𝐷], [𝑀𝐴𝐶𝐷], [𝑀𝐴𝐵𝐷], [𝑀𝐴𝐵𝐶] , să se arate că ⃗⃗⃗⃗⃗⃗⃗ 𝐴𝐺𝐴 +
⃗⃗⃗⃗⃗⃗⃗⃗ ⃗⃗⃗⃗⃗⃗⃗⃗ ⃗⃗⃗⃗⃗⃗⃗⃗ ⃗
𝐵𝐺𝐵 + 𝐶𝐺𝐶 + 𝐷𝐺𝐷 = 0 dacă şi numai dacă 𝑀 = 𝐺.
Marius Olteanu, Rm. Vâlcea

III. Dacă 𝑧1 , 𝑧2 , 𝑧3 ∈ ℂ sunt distincte două câte două, să se demonstreze echivalenţa:


𝑧 |𝑧 −𝑧 |+𝑧 |𝑧 −𝑧 | 𝑧 +𝑧
|𝑧1 − 𝑧2 | = |𝑧1 − 𝑧3 | ⇔ 2 1 3 3 1 2 = 2 3 .
|𝑧1 −𝑧3 |+|𝑧1 −𝑧2 | 2

Interpretare geometrică.
Aurel Doboşan, Lugoj

Clasa a XI-a

I. a) Să se arate că pentru orice 𝑛 ∈ ℕ∗ , ecuaţia :


√𝑛2 𝑥 + 1 + √𝑛2 𝑥 + 2 + ⋯ + √𝑛2 𝑥 + 𝑛 = 𝑛2 are o soluţie unică 𝑥𝑛 ≥ 0.
b) Să se demonstreze că lim𝑛→∞ 𝑥𝑛 = 1.
Dumitru Popa, Constanţa
2 (ℝ), ∗
II. Fie 𝑎, 𝑏 ∈ ℝ cu 𝑎 ≤ 4𝑏 şi matricele 𝐴, 𝐵 ∈ 𝑀𝑛 𝑛 ∈ ℕ , astfel încât 𝐴𝐵 = 𝐵𝐴. Să se
arate că 𝑑𝑒𝑡(𝐴2 + 𝑎𝐴𝐵 + 𝑏𝐵 2 ) ≥ 0.
D.M. Bătineţu-Giurgiu, Bucureşti

43
III. În triunghiul 𝐴𝐵𝐶, cu 𝐴̂ = 900 , considerăm bisectoarea 𝐴𝐸, 𝐸 ∈ 𝐵𝐶 şi mediana 𝐵𝐷, 𝐷 ∈
𝐴𝐵∙𝐴𝐶
𝐴𝐶. Fie {𝐾} = 𝐴𝐸 ∩ 𝐵𝐷 şi {𝑇} = 𝐴𝐵 ∩ 𝐶𝐾. Să se demonstreze relaţia: 𝐸𝑇 = 𝐴𝐵+𝐴𝐶 .
Gheorghe F. Molea, Curtea de Argeş

Clasa a XII-a

𝑎 𝑏
I. Fie 𝐺 = {( ) |𝑎, 𝑏 ∈ ℝ, 𝑎 ≠ 0 𝑠𝑎𝑢 𝑏 ≠ 0}. Să se arate că pentru orice 𝐴 ∈ 𝐺 şi orice
−𝑏 𝑎
𝑛 ∈ ℕ∗ , ecuaţia 𝑋 𝑛 = 𝐴 are exact 𝑛 soluţii distincte în 𝐺.
Marcel Ţena, Bucureşti
2𝜋 1
II. Să se calculeze ∫0 𝑑𝑥.
2+cos 𝑥
***

III. Fie (𝐴, +,∙) un inel cu 0 ≠ 1, care are un număr impar de elemente inversabile. Să se arate că
1 + 1 = 0.
***

Premianţii concursului

Clasa a V – a
Premiul I : Drăjneanu Diana – Şc. 3 Slobozia;
Premiul II : Antohe Andreea – Lic. de Artă Slobozia;
Premiul III : Naca Andrei – Şc. 7 Feteşti;
Menţiune : Ion Dragoş – Şc. 2 „Spriu Haret” Tăndărei,
Menţiune : Stoicescu Andreea – Şc. 3, Slobozia.

Clasa a VI-a

Premiul I : Drăgoi Octav – Şc. 3 Slobozia;


Premiul II : Popescu Teodor – Şc. 3 Slobozia;
Premiul III : Radu Oana – Şc. 3 Slobozia;
Menţiune : Iordache Mihaela – Şc. 3 Slobozia,
Menţiune : Marcu Andreea – Lic. de Artă Slobozia.

Clasa a VII – a

Premiul I : Voicu Bianca – C.N. – „ I.L. Caragiale” Ploieşti;


Premiul II : Drobrota Adina – Lic. Pedagogic Galaţi;
Premiul III : Dumitrascu Claudia – Şc. 7 Feteşti;
Menţiune : Ioniţă Carmen-Florentina – Şc. 2 Urziceni,
Menţiune : Damian Oana – Şc. „Fanuş Neagu” Brăila.

Clasa a VIII – a

Premiul I : Mincu Diana – Lic. de Artă Slobozia;


44
Premiul II : Buriceanu Dorin – Şc. 1 Urziceni;
Premiul III : Udrea Cosmin – Şc. 2 Urziceni;
Mentiune : Mihalcea Serban – Lic. de Artă Slobozia,
Menţiune : Gîndea Aura – Lic. Teoretic „M. Eminescu” Călăraşi.

Clasa a IX – a

Premiul I : Răşcanu Oana – C.N. – „Vasile Alecsandri” Galaţi;


Premiul II : Florea Mihai – C.N. „ Mihai Viteazul” Ploieşti;
Premiul III : Bunea Rodica – C.N. „Gh. Murgoci” Brăila;
Menţiune : Scarlat Georgiana C.N. „Gh. Murgoci” Brăila,
Menţiune : Teodor Alexandru – Lic. de Artă Slobozia.

Clasa a X – a

Premiul I : Ceobanu Victor „Lic. „Al. I. Cuza” Ploieşti,


Premiul II : Diaconu Irina – C.N. – „Gh. Murgoci” Brăila;
Premiul III : Lupu Maria-Gabriela – Lic. Teoretic Feteşti;
Mentiune : Mirica Emma – C.N. „Mihai Viteazul” Slobozia,
Menţiune : Moraru Ionuţ – C.N. „ Gh. Murgoci” Brăila.

Clasa a XI – a
Premiul I : Caragea Andrei – Lic. Pedagocic Galaţi;
Premiul II : Costea Sergiu – C.N. „ Mihai Viteazul” Slobozia;
Premiul III : Cristea Liviu – C.N. „ Mihai Viteazul” Slobozia;
Menţiune : Tănase Narcis-Ciprian – Lic. Teoretic Feteşti,
Menţiune : Munteanu Alexandru – C.N. „Mihai Viteazul” Ploieşti.

Clasa a XII – a

Premiul I : Sava Alexandru – C.N. „ Vasile Alecsandri” Galaţi;


Premiul II : Gudu Diana – C.N. „ Mihai Viteazul” Slobozia;
Premiul III : Petrăreanu Madi – Lic. Teoretic „Al.I. Cuza” Ploieşti;
Menţiune : Aldea Olga – C.N. „ I.L. Caragiale” – Ploieşti,
Menţiune : Ioniţă Mădălina – C.N. „Gh. Murgoci” Brăila.

EDIŢIA a XIII-a, 31 martie 2007


SUBIECTE

Clasa a VII-a

I. Arătaţi că, pentru orice 𝑛 ∈ ℕ, numărul 6𝑛 + 21𝑛 + 2004 nu este pătrat perfect.
Liliana Toderiuc, Bucureşti

45
II. Fie 𝐴𝐵𝐶𝐷 un pătrat şi 𝑀 un punct situat în interiorul său. Fie 𝐸, 𝐹, 𝐺, 𝐻 simetricele punctului
𝑀 în raport cu 𝐴𝐵, 𝐵𝐶, 𝐶𝐷, 𝐷𝐴.
i) Să se arate că tripletele (𝐸, 𝐵, 𝐹), (𝐹, 𝐶, 𝐺), (𝐺, 𝐷, 𝐻), (𝐻, 𝐴, 𝐸) sunt coliniare.
ii) Să se arate că aria patrulaterului 𝐸𝐹𝐺𝐻 este constantă, oricare ar fi poziţia punctului
𝑀 şi să se determine această constantă.
Rodica Marinescu şi Victor Marinescu, Craiova

III. Fie 𝐴𝐵𝐶 un triunghi dreptunghic cu unghiul drept în 𝐴, astfel încât 3𝐴𝐵 = 7𝐴𝐶.
Să se arate că măsura unghiului 𝐵 este strict mai mare decât 22° 30′.
Titu Zvonaru, Comăneşti şi Bogdan Ioniţă, Bucureşti

Clasa a VIII-a

I. Să se exprime toate numerele naturale 𝑥, 𝑦 cu proprietatea că:


1
(𝑥 + 𝑦) = √𝑥 − 1 + √𝑦 − 1 .
2
***
II. Se consideră o piramidă regulată având ca bază un poligon regulat cu 𝑛 laturi, 𝑛 ≥ 3. Să se
calculeze volumul piramidei, ştiind că latura bazei este 𝑎 şi muchia laterală este 𝑏.
***
𝑏
III. Fie 𝑎, 𝑏, 𝑐, 𝑑 numere reale strict pozitive. Să se arate că, dacă 𝑎 + 𝑏 > 𝑎𝑐 + 𝑑, atunci
𝑎
𝑎 + 𝑏 ≥ 𝑐 + 𝑏𝑑?
Liviu Oprişan, Bucureşti

Clasa a IX-a

1 1
I. Să se rezolve ecuaţia: [𝑥 2 − 𝑥 + 2] = [−𝑥 2 + 𝑥 + 2] , unde [𝑎] reprezintă partea întreagă a
numărului real 𝑎.
Ilie Diaconu, Cluj-Napoca

II. Fie 𝑛 ∈ ℕ∗ . Să se rezolve ecuaţia:


cos n 𝑥 − sinn x = 1 .
Inocenţiu Drăghicescu, Bucureşti

⃗⃗⃗⃗⃗ + 𝐴𝐶
III. Fie 𝐴𝐵𝐶 un triunghi şi 𝑀 ∈ 𝐵𝐶. Să se arate că 𝐴𝐵 ⃗⃗⃗⃗⃗ = 2𝐴𝑀
⃗⃗⃗⃗⃗⃗ , dacă şi numai dacă 𝑀 este
mijlocul lui [𝐵𝐶].
***
Clasa a X-a

I. Să se rezolve sistemul:
√2𝑥 + 1 + √3𝑦 + 1 = 4
{ .
22𝑥+√3𝑦+1 + 23𝑦+√2𝑥+1 = 64
Marcel Chiriţă, Bucureşti

46
II. Fie 𝑧 un număr complex cu proprietatea |𝑧| < 1 . Să se arate că există două numere complexe
𝑧1 , 𝑧2 cu proprietatea |𝑧1 | = |𝑧2 | = 1, astfel încât 𝑧 = 𝑧1 + 𝑧2 .
Marcel Ţena, Bucureşti

−1+𝑖√3 1−𝜆𝜀
III. Fie 𝜀 = 2 şi mulţimile 𝐴 = {𝑧 ∈ ℂ|𝑧 = 1−𝜆𝜀2 , 𝜆𝜖ℝ}, 𝐵 = {𝑧 ∈ ℂ|𝑧 = cos 𝛼 +
𝑖 sin 𝛼 , 𝛼𝜖[0,2𝜋)}.
Să se arate că:
a) 𝐴 ⊂ 𝐵.
b) Există 𝑧0 ∈ ℂ astfel încât 𝐵 ∖ 𝐴 = {𝑧0 }.
Lidia Păunescu, Târgovişte

Clasa a XI-a

I. Fie 𝐴 ∈ ℳ𝑝 (ℂ), cu 𝑑𝑒𝑡 𝐴 = 𝑑 ≠ 0, unde 𝑝 ≥ 2. Definim şirul (𝐴𝑛 )𝑛≥0 prin 𝐴0 = 𝐴, 𝐴𝑛 =


𝐴∗𝑛−1, pentru orice 𝑛 ≥ 1.
Să se calculeze 𝑑𝑒𝑡(𝐴𝑛 ), 𝑛 ∈ ℕ, în funcţie de 𝑛, 𝑝 ş𝑖 𝑑.
Inocenţiu Drăghicescu, Bucureşti

II. Să se determine toate numerele reale 𝛼, 𝛽 cu proprietatea că şirurile 𝑥𝑛 = sin 𝑛𝛼 şi 𝑦𝑛 =


cos 𝑛𝛽, 𝑛 ∈ ℕ∗ , sunt convergente.
Nelu Chichirim, Constanţa

III. Fie 𝐴, 𝐵 ∈ 𝑀2 (ℂ). Să se arate că:


𝑑𝑒𝑡(𝐴 + 𝐵)2 + 𝑑𝑒𝑡(𝐴 − 𝐵)2 = 2[𝑑𝑒𝑡 (𝐴2 + 𝐵 2 ) + 𝑑𝑒𝑡(𝐴𝐵 + 𝐵𝐴)] .
Marius Damian, Brăila

Clasa a XII-a

2̂𝑥 + 4̂𝑦 = 0̂
I. Să se rezolve în ℤ6 × ℤ6 sistemul: { .
𝑥 + 𝑦 = 2̂
***
1
II. Să se determine o primitivă a funcţiei 𝑓: ℝ → ℝ, 𝑓(𝑥) = 2+sin 𝑥 .
***

III. Fie (𝐺, +) grup abelian şi 𝐸𝑛𝑑(𝐺) mulţimea endomorfismelor grupului G. Definim 𝑓 + 𝑔 şi
𝑓 ∘ 𝑔 prin (𝑓 + 𝑔)(𝑥) = 𝑓(𝑥) + 𝑔(𝑥), (𝑓 ∘ 𝑔)(𝑥) = 𝑓(𝑔(𝑥)), ∀𝑥 ∈ 𝐺 , pentru 𝑓, 𝑔 ∈ 𝐸𝑛𝑑(𝐺) .
Să se arate că (𝐸𝑛𝑑(𝐺), +,∘) este inel.
***

47
Premianţii concursului

Clasa a VII – a

Premiul I : Drăgoi Octav, Şc. nr.3 Slobozia,


Premiul I :Radu Oana, Şc. nr.3 Slobozia;
Premiul II : Pisică Dana, C.N. Tonitza” Constanţa,
Premiul II : Popa Emanuela, Şc. „ M. Eminescu” Brăila;
Premiul III : Chitaru Vlad, Şc. „M. Eminescu” Brăila.

Clasa a VIII – a

Premiul I : Iacov Carmen, Şc. „ M. Eminescu” Călăraşi;


Premiul II : Bercaru Andrei, Lic. cu Progr. Sportiv Brăila;
Premiul III : Damian Oana, Şc. „Fănuş Neagu” Brăila;
Menţiune : Bengescu Laura, Lic. de Artă Slobozia;
Menţiune :Dumitraşcu Claudiu, Şc. nr.7 Feteşti.

Clasa a IX – a

Premiul I : Mihalcea Şerban, C.N. „Mihai Viteazul” Slobozia;


Premiul II : Dunaev Mihail „ Lic. Tehnic A. Saligny” Cernavodă);
Premiul III : Florea Răzvan, C.N. „Mihai Viteazul” Slobozia;
Menţiune : Bengescu Adela, C.N. „Mihai Viteazul” Slobozia.

Clasa a X - a

Premiul I : Căpâlnaşiu Adela, C.N. „Mircea cel Bătrân” Constanţa;


Premiul II : Toader Bogdan, C.N. „Gh. Murgoci” Brăila;
Premiul III : Scarlat Georgiana, C.N. „ N. Bălcescu” Brăila;
Mentiune : Bâncă Alexandra, Lic. Tehnic Urziceni,
Menţiune :Maria Bogdan, C.N. „Mihai Viteazul” Slobozia.

Clasa a XI - a

Premiul I : Maria Violeta, C.N. „Mihai Viteazul” Slobozia;


Premiul II : Herghelegiu Cristina, C.N. „Mihai Eminescu” Buzău;
Premiul III : Diaconu Irina, C.N. „Gh. Murgoci” Brăila;
MentiunE : Coteţ Silvia, C.N. „N. Bălcescu” Brăila,
Menţiune :Cazan Mădălina, Lic. Teoretic Urziceni.

Clasa a XII - a

Premiul I : Cristea Liviu, C.N. „ Mihai Viteazul” Slobozia;


Premiul II : Pană Emilia, C.N. „ Mihai Viteazul” Slobozia;

48
Premiul III : Tănase Narcis, Lic. Teoretic Feteşti;
Menţiune : Costea Sergiu, C.N. „Mihai Viteazul” Slobozia,
Menţiune :Zvincu Oana, Lic. M. Eminescu Călăraşi.

EDIŢIA a XIV-a, 4-6 aprilie 2008

SUBIECTE

Clasa a VII-a

I. Dacă 𝑥, 𝑦, 𝑧 > 0 atunci:


𝑥 3 +𝑦 3 𝑦 3 +𝑧 3 𝑧 3 +𝑥3 2
+ 𝑦 2 +𝑦𝑧+𝑧 2 + 𝑧 2 +𝑥𝑧+𝑥 2 ≥ 3 (𝑥 + 𝑦 + 𝑧).
𝑥 2 +𝑥𝑦+𝑦 2
D. M. Bătineţu-Giurgiu, Bucureşti

II. Fie triunghiul ascuţitunghic 𝐴𝐵𝐶 şi punctele 𝐷 ∈ (𝐵𝐶), 𝐸 ∈ (𝐴𝐶), 𝐹 ∈ (𝐴𝐵) astfel încât
dreptele 𝐴𝐷, 𝐵𝐸, 𝐶𝐹 sunt concurente în 𝐻.
Demonstraţi că 𝐻 este ortocentrul triunghiului 𝐴𝐵𝐶 dacă şi numai dacă
𝐻𝐴 ∙ 𝐻𝐷 = 𝐻𝐵 ∙ 𝐻𝐸 = 𝐻𝐶 ∙ 𝐻𝐹 .
***

III. Să se determine 𝑎 ∈ ℕ∗ pentru care numărul 𝑎2 + 621 este pătrat perfect.


***

Clasa a VIII-a

I. Să se determine funcţiile 𝑓: ℝ → ℝ care au proprietatea:


𝑓(1 − 𝑥) + 2𝑓(1 + 𝑥) = 3𝑥 2 + 2𝑥 + 6 , pentru orice 𝑥 ∈ ℝ.
***

II. Fie cubul 𝐴𝐵𝐶𝐷𝐴’𝐵’𝐶’𝐷’, 𝑀 simetricul punctului 𝐴 faţă de punctul 𝐵, 𝑁 piciorul


perpendicularei dusă din 𝐶 pe 𝐵𝐷’ şi 𝑃 centrul pătratului 𝐴𝐷𝐷’𝐴’.
Arătaţi că punctele 𝑀, 𝑁, 𝑃 sunt coliniare.
Luca Tuţă, Buzău

̅̅̅ pentru care are loc egalitatea: √100 − 𝑎𝑏


III. Aflaţi numerele de forma 𝑎𝑏 ̅̅̅ = 𝑎√𝑏 .
***

Clasa a IX-a

I. Se consideră funcţia 𝑓: (0, ∞) → ℝ şi 𝑎 > 0 astfel încât 𝑓(𝑎) = 1. Să se demonstreze că dacă:


𝑎 𝑎
(𝑥) ∙ 𝑓(𝑦) + 𝑓 ( ) ∙ 𝑓 ( ) = 2𝑓(𝑥𝑦) , pentru orice 𝑥, 𝑦 ∈ (0, ∞), atunci f este constantă.
𝑥 𝑦
Adriana Tonţ, Slobozia

49
II. Se consideră un poligon regulat 𝐴1 𝐴2 ⋯ 𝐴𝑛 înscris în cercul de centru O şi mulţimile de
⃗⃗⃗⃗⃗⃗⃗⃗1 , ⃗⃗⃗⃗⃗⃗⃗⃗
vectori: = {𝑂𝐴 𝑂𝐴2 , ⋯ , ⃗⃗⃗⃗⃗⃗⃗⃗ ⃗⃗⃗⃗⃗⃗⃗⃗1 , ⃗⃗⃗⃗⃗⃗⃗⃗
𝑂𝐴𝑛 } , 𝑌 = {𝑂𝐴 𝑂𝐴1 + ⃗⃗⃗⃗⃗⃗⃗⃗
𝑂𝐴2 , ⋯ , ⃗⃗⃗⃗⃗⃗⃗⃗
𝑂𝐴1 + ⃗⃗⃗⃗⃗⃗⃗⃗
𝑂𝐴2 + ⋯ + ⃗⃗⃗⃗⃗⃗⃗⃗
𝑂𝐴𝑛 } .
Să se determine mulţimea 𝑋 ∩ 𝑌.
Marcel Ţena, Bucureşti

𝑎 +𝑝 𝑎 +𝑝 𝑎 +𝑝
III. Se consideră suma 𝑆 = 𝑎1 +𝑞 + 𝑎2 +𝑞 + ⋯ + 𝑎𝑛 +𝑞, unde 𝑎𝑘 ∈ ℝ, 𝑎𝑘 > 0, 𝑘 ∈ {1,2, ⋯ , 𝑛},
1 2 𝑛
astfel încât 𝑎12 + 𝑎22 + ⋯ + 𝑎𝑛2 = 𝑐, 𝑐 > 0 şi 𝑝, 𝑞 ∈ ℝ, 𝑞 > 0, 𝑝 ≠ 𝑞 . Să se arate că:
𝑝√𝑛+√𝑐
a) 𝑆 ≤ 𝑛 ∙ 𝑞 , dacă 𝑝 < 𝑞;
√𝑛+√𝑐
𝑝√𝑛+√𝑐
b) 𝑆 ≥ 𝑛 ∙ 𝑞 , dacă 𝑝 > 𝑞.
√𝑛+√𝑐
Inocenţiu C. Drăghicescu, Bucureşti

Clasa a X-a

I. Fie 𝑛 un număr natural impar, 𝑛 ≥ 3. Să se arate că numărul:


𝑎𝑛 = (𝑛 − 2)(𝑛 − 1)(𝑛 + 1)𝑛+1 + 𝑛(𝑛 + 1)𝑛 − 2 este divizibil prin 𝑛3 .
Dumitru Acu, Sibiu

II. Rezolvaţi în mulţimea numerelor reale ecuaţia:


4𝑥 + (4𝑥 − 11)2𝑥 + 3𝑥 2 − 23𝑥 + 30 = 0
Nicolae Papacu, Slobozia

III. Fie O punctul de intersecţie al diagonalelor patrulaterului convex 𝐴𝐵𝐶𝐷, iar 𝑃, 𝑄, 𝑅, 𝑆


proiecţiile punctului 𝑂 respectiv pe 𝐴𝐵, 𝐵𝐶, 𝐶𝐷, 𝐷𝐴. Să se arate că următoarele afirmaţii sunt
echivalente:
1 1 1 1
a) 𝑂𝑃2 + 𝑂𝑅2 = 𝑂𝑄2 + 𝑂𝑆2 .
b) Patrulaterul 𝐴𝐵𝐶𝐷 are diagonalele perpendiculare.
Nicolae Bişboacă, Alba Iulia

Clasa a XI-a

I. Fie 𝑘 ∈ ℕ∗ şi numerele reale 𝑎1 , 𝑎2 , ⋯ , 𝑎𝑘 , 𝑎𝑘+1 astfel încât: [𝑛𝑎1 ] + [𝑛𝑎2 ] + ⋯ + [𝑛𝑎𝑘 ] =


[𝑛𝑎𝑘+1 ], pentru orice 𝑛 ∈ ℕ∗ .
Să se arate că 𝑎1 + 𝑎2 + ⋯ + 𝑎𝑘 = 𝑎𝑘+1 .
Vasile Popa, Galaţi

50
II. Să se arate că pentru orice număr natural 𝑛 ≥ 1, ecuaţia:
1 1 1 𝑥 1
√𝑛2 +1+𝑥
+ √𝑛2 + ⋯ + √𝑛2 = 1 are o soluţie unică reală notată 𝑥𝑛 şi lim𝑛→∞ 𝑛𝑛 = − 2.
+2+𝑥 +𝑛+𝑥
Dumitru Popa, Constanţa

III. Pentru 𝐴 ∈ ℳ𝑛 (ℤ), 𝑛 ≥ 2, definim 𝑓𝐴 : ℳ𝑛 (ℤ) → ℳ𝑛 (ℤ) prin : 𝑓𝐴 (𝑋) = 𝐴𝑋.


Să se arate că 𝑓𝐴 este injectivă dacă şi numai dacă 𝑑𝑒𝑡 𝐴 ≠ 0.
***

Clasa a XII-a

I. În inelul ℤ𝑛 , 𝑛 ≥ 2, considerăm 𝑎̂, 𝑏̂ ∈ ℤ𝑛 şi mulţimile 𝐴 = {𝑥̂ ∈ ℤ𝑛 |𝑎̂𝑥̂ = 0̂} , 𝐵 =


{𝑥̂ ∈ ℤ𝑛 |𝑎̂𝑥̂ = 𝑏̂}.
Fie 𝑑 cel mai mare divizor comun al numerelor 𝑎 ş𝑖 𝑛.
a) Determinaţi mulţimea 𝐴 în cazul 𝑛 = 100 , 𝑎 = 5.
b) Dacă 𝑑 divide 𝑏, arătaţi că 𝐵 ≠ ∅.
c) Dacă 𝑑 divide 𝑏, arătaţi că 𝐴 ş𝑖 𝐵 au acelaşi număr de elemente.
Nicolae Papacu, Slobozia

1 𝐼
II. Notăm 𝐼𝑛 = ∫0 𝑥 𝑛 𝑒 𝑥 𝑑𝑥, unde 𝑛 ∈ ℕ. Calculaţi lim𝑛→∞ ∑𝑛𝑘=1 𝑘+1 .
𝑘
Traian Tămâian, Carei, Satu Mare

III. Fie 𝐿, 𝐾 două corpuri şi 𝑓, 𝑔: 𝐿 → 𝐾 două funcţii astfel încât:


i) 𝑓(𝑥 + 𝑦) = 𝑔(𝑥) + 𝑔(𝑦), ∀𝑥, 𝑦 ∈ 𝐿;
ii) 𝑓(𝑥𝑦) = 𝑔(𝑥)𝑔(𝑦), ∀𝑥, 𝑦 ∈ 𝐿;
iii) ∀𝑦 ∈ 𝐾, ∃𝑥 ∈ 𝐿 astfel încât 𝑓(𝑥) = 𝑦 sau 𝑔(𝑥) = 𝑦.

Să se arate că 𝐿 ş𝑖 𝐾 sunt corpuri izomorfe.


Marcel Ţena, Bucureşti

Premianţii concursului

Clasa a VII – a

Premiul I : Drăjneanu Diana ( Şc. Nr.3 Slobozia);


Premiul II : Stegăroiu Petre ( C.N. „Mihai Viteazul” Ploieşti);
Premiul III : Albu Constantin ( C.N. „Mihai Viteazul” Ploieşti);
Menţiuni : Mihai Roxana ( Lic. de Artă „ Ionel Perlea” Slobozia), Pojer Corneliu ( Şc. Nr.2 „
Ion Heliade Rădulescu” Urziceni), Tănase Adrian ( Şc. Nr.2 „ Ion Heliade Rădulescu” Urziceni),
Găsdaru Bianca ( Lic. de Artă „Ionel Perlea” Slobozia), Naca Andrei ( Şc. nr.7 „Aurel Vlaicu”

51
Feteşti), Nichiforov Anca ( Şc. nr.7 „Aurel Vlaicu” Feteşti), Antohe Andreea ( Lic. de Artă
„Ionel Perlea” Slobozia).

Clasa a VIII – a

Premiul I : Drăgoi Octav ( Şc. Nr.3 Slobozia), Duma Andrei (Şc. „Sf. Vineri” Ploieşti),
Pisică Dana ( Şc. „N. Tonitza Constanţa”), Radu Oana ( Şc. Nr.3 Slobozia), Sevastian Ana
Denisei ( C.N. „Mihai Viteazul” Ploieşti);
Premiul II : Dragomir Mihaela ( Şc. Nr.2 „Ion Heliade Rădulescu” Urziceni);
Premiul III : Bulgaru Elena ( Şc. nr. 11 „ George Tutoveanu” Bârlad);
Menţiuni : Bulai Lavinia ( C.N. „Calistrat Hogaş” Tecuci), Hulea Alexandru ( C.N. „Calistrat
Hogaş” Tecuci), Marcu Andreea ( Lic. de Artă „ Ionel Perlea” Slobozia).

Clasa a IX – a

Premiul I : Mihai Bogdan ( Gr. Şc. Industrial „Ştefan Procopiu” Bârlad);


Premiul II : Valcu Georgiana ( C.N. „Barbu Ştirbei” Călăraşi);
Premiul III : Dochiţa Mihai ( Lic. Teoretic „Spiru Haret” Tecuci);
Menţiuni : Cârciumaru Andreea (C.N. „I. L. Caragiale” Ploieşti), Bosânceanu Andra ( Lic.
Teoretic „ Carol I” Feteşti), Drăgan Raluca ( Lic. de Artă „ Ionel Perlea” Slobozia),Drăjneanu
Alina ( C.N. „Mihai Vitezul” Slobozia), Ghidel Radu ( C.N. „ Mihai Vitezul” Ploieşti), Ionescu
Ovidiu ( C.N. „Barbu Stirbei” Călăraşi), Vasile Sorin ( C.N. „ I. L. Caragiale” Ploieşti).

Clasa a X - a

Premiul I : Mincu Diana ( Lic. de Artă „Ionel Perlea” Slobozia);


Premiul II : Mihalcea Şerban ( C.N. „ Mihai Viteazul” Slobozia);
Premiul III : Dunaev Mihail ( Lic. Teoretic „Anghel Saligny” Cernavodă);
Menţiuni : Florea Răzvan ( C.N. „Mihai Viteazul” Slobozia), Barbu Vlad ( C.N. „I.L.
Caragiale” Ploieşti), Bejgu Andreea ( C.N. „Mihai Viteazul” Slobozia), Dumitrescu Diana ( C.N.
„Mihai Viteazul” Slobozia), Petre Georgiana-Alina ( Gr. Şc. „Al.I. Cuza” Slobozia), Pâslaru
Alexandru ( L.I.C. Teoretic „Spiru Haret” Tecuci), Blănaru Adina ( Gr. Şc. Ind. „Ştefan
Procopiu” Bârlad).

Clasa a XI - a

Premiul I : Marin Bogdan-Adrian ( C.N. „Mihai Viteazul” Slobozia);


Premiul II : Cotac Oana ( Lic. Teoretic „ Anghel Saligny” Cernavodă);
Premiul III : Teodor Alexandru ( Lic. de Artă „Ionel Perlea” Slobozia);
Mentiuni : Nuţu Bogdan ( Colegiul „ M. Cantacuzino” Sinaia), Mutu Rares ( Lic. Teoretic
„Anghel Saligny” Cernavodă), Ene Vladimir ( Lic. Teoretic „ Anghel Saligny” Cernavodă),
Simion Sorin-Ionuţ ( Lic. Teoretic „ Al. I. Cuza” Ploieşti), Sachelarie Dan ( Lic. Teoretic „ Spiru
Haret” Tecuci), Stănescu George ( C.N. „ Mihai Eminescu” Buzău).

Clasa a XII - a

Premiul I : Damian Alexandru ( C.N. „I.L. Caragiale” Ploieşti);


52
Premiul II : Ion Ana – Maria ( Lic. Teoretic „Grigore Moisil” Urziceni);
Premiul III : Ceobanu Victor ( C.N. „ I.L. Caragiale” Ploieşti);
Mentiuni : Mirica Emma ( C.N. „ Mihai Viteazul” Slobozia), Antonache Marian ( Gr. Şc. Ind. „
Ştefan Procopiu” Bârlad), Cazan Mădălina ( Lic. Teoretic „Grigore Moisil” Urziceni), Alionte
Ana-Maria ( C.N. „Calistrat Hogaş” Tecuci), Bedea Bogdan ( Lic. Teoretic „Carol I” Feteşti),
Cuciureanu Cornelia ( Gr. Şc. „Al. I. Cuza” Slobozia), Urechea Laura ( Lic. Teoretic „ Carol I”
Feteşti).

Ediţia a XV-a, 3-5 aprilie 2009


SUBIECTE

Clasa a VII-a
Subiectul I.
Alegem la întâmplare 10 numere reale mai mari decât 1 şi mai mici decât 55. Arătaţi că printre
acestea există cel puţin trei care reprezintă lungimile laturilor unui triunghi.
Bianca Teodora Iordache
Subiectul II.
Să se arate că numărul a  n5  2009n este divizibil cu 30 pentru orice n număr natural.
Ion Neaţă
Subiectul III.
Se consideră punctul D situat pe latura BC a triunghiului ABC . Ducem DE AB , E  AC şi
DF AC , F  AB . Să se arate că S FBD  S EDC  S AEDF.
Gheorghe Stoica

Clasa a VIII-a
Subiectul I.
Să se determine funcţia de gradul întâi f :  , care verifică egalităţile: f 2  8 şi
f 3x  2 y  2  f 3x  1  f 2 y 1  2 , pentru orice x, y  .
Alfred Eckstein, Viorel Tudoran
Subiectul II.
Fie paralelipipedul dreptunghic cu muchiile de lungimi a, b, c şi diagonala de lungime d . Dacă
abc
d să se arate că paralelipipedul este cub.
3
Neculai Stanciu
Subiectul III.
Se consideră prisma triunghiulară regulată dreaptă ABCABC care are lungimea bazei de 12 3
cm, O este centrul bazei ABC , iar M este mijlocul segmentului BC . Ştiind că măsura
unghiului determinat de OM cu AC este de 600 , să se afle aria laterală şi volumul prismei
date.
E. Blăjuţ

53
Clasa a IX-a
Subiectul I.
Să se demonstreze că în orice triunghi ABC avem relaţia lui Sylvester: OH  OA  OB  OC ,
unde H este ortocentrul triunghiului ABC , iar O este centrul cercului circumscris triunghiului
ABC .

Subiectul II.
Într-un triunghi ascuţitunghic ABC se consideră, respectiv, medianele, înălţimile şi bisectoarele
sale interioare şi fie mulţimile lungimilor acestora M m  ma , mb , mc , M h  ha , hb , hc ,
Mi  ia , ib , ic . Să se demonstreze că luându-se câte un element din fiecare mulţime, suma
 9R 
acestora aparţine intervalului 9r, .
 2 
Inocenţiu Drăghicerscu

Subiectul III.
x  y  z  k

Să se rezolve în mulţimea numerelor reale sistemul de ecuaţii:  ,
xy  x  y z  a  a  k 2
1

 3
unde a, h .
I.V. Maftei, Mihai Haivas

Clasa a X-a
Subiectul I.
a x , x 

Fie a  0 , a  1, b  0 , b  1 şi funcţia f :   0,  , f  x    x . Să se arate că
b , x  

următoarele afirmaţii sunt echivalente:
p: f este injectivă;
q: f este surjectivă;
r: loga b .
Marcel Ţena
Subiectul II.
Fie a, b, c, d numere strict mai mari decât 1, cu proprietatea că produsul a oricare trei dintre ele
este diferit de al patrulea. Să se arate că log bcd a  log acd b  log abd c  log abc d  2 .
a b c d
D.M. Bătineţu - Giurgiu
Subiectul III.
Fie numerele complexe nenule şi distincte z k , k  1, n , n  3 , care verifică relaţiile
z  zk  zk , k  1, n , unde z  z1  z2  zn .
1. Să se demonstreze că z  0 .
2. Dacă n  3 şi z1  z2  z3 , atunci să se arate că z1 , z 2 , z3 sunt afixele vârfurilor unui
triunghi echilateral.
Nicolae Papacu

54
Clasa a XI-a

Subiectul I.
În reperul rectangular xOy se consideră punctele A2,3 şi B8,7. Să se determine punctul
M  AB astfel încât AM 2  AB  MB , AM  MB.
Gh. Tutulan
Subiectul II.
Pentru fiecare număr natural n  4 , fie  o rădăcină primitivă de ordinul n a unităţii.
Demonstraţi că pentru orice matrice A  M 2   cu TrA   şi det A   2 are loc egalitatea:

 
n


k 1
k
det A   k I 2  0 .
Traian Tămâian
Subiectul III.
Se consideră şirul cu termenul general dat de partea fracţionară a numărului n2  an  b ,
n   , unde a şi b . Să se arate că oricare ar fi numerele a şi b există limita acestui şir
şi să se afle această limită.
Inocenţiu Drăghicescu

Clasa a XII-a
Subiectul I.
Fie K un corp comutativ. Un polinom f  a0  a1 X   an X n  KX  cu an  0 se numeşte
reciproc dacă ak  an k , k  0, n . Să se arate că produsul a două polinoame reciproce este un
polinom reciproc.
Marcel Ţena
Subiectul II.
2
1
Să se calculeze  2  sin x dx .
0

Subiectul III.
1
Fie f : 1,1  o funcţie continuă şi I n  n  xn f xdx . Să se calculeze lim I 2n şi lim I 2n 1 .
n  n 
1
Dumitru Popa

Lista premianţilor

Clasa a VII-a

Premiul I: Custură Mihai, Şc. Nr. 3 Slobozia


Premiul al II-lea: Iacob Andrei, C.N. „Gh. M. Murgoci”, Brăila
Premiul al III-lea: Sava Loredana Şc. Nr. 2 „I.H. Rădulescu” Urziceni
Menţiuni: Dinu Valentina, Lic. Pedagogic „M. Basarab” Slobozia
Dănăilă Miruna, C.N. „N. Bălcescu”, Brăila

55
Huiu Andreea Cristina, Şc. „I. Creangă”, Brăila
Cazacu Andrei Gabriel, Şc. Nr. 2 „I.H. Rădulescu” Urziceni
Iancu Cristina Ioana, Lic. De Artă „I. Perlea” Slobozia
Miriţescu Cătălina, Gr. Şc. Agricol Fundulea
Peltea Marius, Şc. Nr. 7 Feteşti

Clasa a VIII-a
Premiul I: Naca Andrei, Şc. Nr. 7 Feteşti
Premiul al II-lea: Ciocîrlan Dan, Şc. Nr. 11 Buzău
Ţapu Liviu, Şc. „M. Eminescu” Brăila
Premiul al III-lea: Agrigoroaie Gabriela, Şc. „G. Palade” Buzău
Menţiuni: Lozneanu Dragoş, Şc. „M. Eminescu” Brăila
Iacob Robert, Şc. „M. Eminescu” Brăila
Bărzulea Amalia, Şc. „N. Titulescu” Călăraşi
Tatu Daniela, Şc. Nr. 2 „Sf. Andrei” Slobozia
Nan Mihai, Şc. Nr. 7 Feteşti
Antohe Andreea, Lic. De Artă „I. Perlea” Slobozia
Nichifor Ana, Şc. Nr. 7 Feteşti
Popina Sebastian Marius, , Şc. „G. Palade” Buzău
Pojar Mihai, Şc. Nr. 2 „I.H. Rădulescu” Urziceni
Buzoianu Diana, Şc. Nr. 2 „Sf. Andrei” Slobozia

Clasa a IX-a
Premiul I: Drăgoi Octav, Lic. Internaţional de Informatică, Bucureşti
Premiul al II-lea: David Octavian, Lic.Teoretic „A. Saligny”, Cernavodă
Premiul al III-lea: Radu Oana, C.N. de Informatică „Tudor Vianu” Bucureşti
Menţiuni: Bercu Andrei, C.N. „Gr. Moisil” Urziceni
Fieraru Mihai, C.N. de Informatică „Tudor Vianu” Bucureşti
Druncea Mihai, C.N. „M. Viteazul” Slobozia
Constantin Simona, C.N. „B. Ştirbei” Călăraşi
Popa Constantina, Lic. Teoretic „Al. I. Cuza” Ploieşti
Florea Alexandru, C.N. „M. Viteazul” Slobozia

Clasa a X-a
Premiul I: Matei Adrian, C.N. „I. L. Caragiale” Ploieşti
Premiul al II-lea: Bengescu Laura, C.N. „Mircea cel Bătrân” Constanţa
Premiul al III-lea: Cârciumaru Andreea, C.N. „I. L. Caragiale” Ploieşti
Conache Maria C.N. „N. Bălcescu” Brăila
Vîlcu Cristina Georgiana, C.N. „B. Ştirbei” Călăraşi
Menţiuni: Popia Andreea Marilena, C.N. „B. Ştirbei” Călăraşi

Clasa a XI-a
Premiul I: Blănaru Adina, Gr. Şc. „Şt. Procopiu” Vaslui
Premiul al II-lea: Străteanu Elena, C.N. „M. Eminescu ” Buzău
Premiul al III-lea: Mincu Diana, Lic. De Artă „I. Perlea” Slobozia
Menţiuni: Florea Răzvan, C.N. „Mihai Viteazul” Slobozia
56
Huiu Andreea Mariana, C.N. „Barbu Ştirbei” Călăraşi
Mihai Valentin, Lic. Teoretic „Gr. Moisil” Urziceni
Chivu Daniel Dumitru, Lic. Teoretic „Carol I” Feteşti
Sezciuc Radu, C.N. „Mihai Viteazul” Slobozia

Clasa a XII-a
Premiul I: Marin Bogdan, C.N. „Mihai Viteazul” Slobozia
Premiul al II-lea: Bunache Cristian, Lic. teoretic „Gr. Moisil” Urziceni
Premiul al III-lea: Coţac Oana Cristina, Lic.Teoretic „A. Saligny”, Cernavodă
Menţiuni: Ganea Raluca, , Lic. Teoretic „Al. I. Cuza” Ploieşti
Barbu Georgiana, , Lic. Teoretic „Al. I. Cuza” Ploieşti
Barbu Tiberiu, C.N. „M. Eminescu ” Buzău

Ediţia a XVI-a, 26-28 martie 2010


SUBIECTE

Clasa a VII-a
Subiectul I
Se dă triunghiul ABC . Fie AD bisectoarea unghiului BAC şi BE mediană, D BC ,
E  AC. Dacă mAFE  600 , unde F  AD  BE şi AD  BE , arătaţi că triunghiul
ABC este echilateral.

Subiectul II
Să se arate că dacă a reprezintă lungimea ipotenuzei unui triunghi dreptunghic, iar b, c
lungimile catetelor aceluiaşi triunghi dreptunghic, atunci are loc inegalitatea:
bc bc
2  2.
a
Când are loc egalitatea din stânga?

Subiectul III
1 n n2
Pentru ce valori ale numărului natural n , numărul s    este natural?
3 2 6
Iulian Vlăduţiu, G.M.

Clasa a VIII-a
Subiectul I
Fie triunghiul ABC cu lungimile laturilor egale cu a, b, c . Să se arate că dacă
a 4
 b4  c 4 
2
 
 2 a8  b8  c8 , atunci triunghiul ABC este dreptunghic.
Nicolae Papacu, G.M.
Subiectul II
1  a 2 1  b2 1  c 2
Dacă a, b, c  0 şi a  b  c  1 , atunci este adevărată egalitatea    4.
bc ac ab
Ion Neaţă
57
Subiectul III
În piramida triunghiulară regulată VABC cu muchia bazei AB  a , notăm cu M mijlocul
muchiei CV . Dacă mMBC  300 , aflaţi distanţa de la punctul A la dreapta BM .
Ion Tudor, G.M.

Clasa a IX-a
Subiectul I
AM 
Fie triunghiul ABC şi punctele M, N pe laturile AB , respectiv BC astfel încât  şi
MB 
BN 
 , unde  ,  ,   0 . Notăm cu P intersecţia dreptelor CM şi AN . Să se arate că:
NC 
1 1 1 
AP  BP  CP  0 .
  
G.M.
Subiectul II
Se consideră un triunghi ascuţitunghic ABC cu lungimile laturilor BC  a , CA  b , AB  c şi
aria egală cu 4. Notăm cu x, y, z distanţele de la ortocentrul triunghiului la vârfurile A, B
respectiv C . Să se arate că dacă a x  b y  c z  4 a  b  c , atunci triunghiul ABC este
echilateral.
Lucian Dragomir
Subiectul III
 1 
Să se determine toate funcţiile f :   ,   care satisfac egalitatea
 2010 
1  2 f x  1  3 f x  1  4 f x    1  2010 f x  2009, pentru orice x .
Dumitru Popa

Clasa a X-a
Subiectul I
Fie p, q numere reale strict pozitive. Să se arate că următoarele afirmaţii sunt echivalente:
i) p  q
ii) Pentru orice numere reale strict pozitive a1 , a2 , a3 are loc inegalitatea:

a   a 
1 1
q
1  a2q  a3q q
1
p
 a2p  a3p p

Subiectul II
Fie ABC un triunghi cu AB  AC şi M A  600 . Pe laturile AB şi AC se construiesc în
exteriorul triunghiului pătrate având centre O1 şi O2 , iar pe latura BC se construieşte în exterior
triunghiul echilateral de centru O3 . Să se arate că triunghiul O1O2O3 este echilateral.
Petru Todor
Subiectul III
Să se rezolve ecuaţia log 2 x  log3 x , x  0 .
Ion Nedelcu

58
Clasa a XI-a
Subiectul I
Fie matricea A  M n   cu  
A4  On . Să se arate că det A2  I n  1.
Viorica Frecuş
Subiectul II
1 3 1 1
1  1  n 1 n
Să se calculeze lim ln 2 ln 3 ln n .
n lnln n
Dumitru Popa
Subiectul III
a) Să se determine funcţiile f :  cu proprietatea lui Darboux, astfel încât:
cos f x  x sin f x, oricare ar fi x .
b) Să se arate că nu există funcţii g :  cu proprietatea lui darboux, astfel încât
cos ggx  x sin ggx , oricare ar fi x .
Marian Andronache

Clasa a XII-a
Subiectul I
Fie A,, un inel şi a, b  A două elemente inversabile. Să se demonstreze că următoarele două
afirmaţii sunt echivalente:
i) ba1  a 1  b1
ii) b  a  1
Marcel Ţena
Subiectul II
 x
1
x2 x3 xn 
0  1  x 1  x 2 1  x3
       ln 1  xdx
1  x n 
Să se calculeze lim .
n ln n
Dumitru Popa
Subiectul III
Fie f :  o funcţie continuă cu proprietatea că f x f  x  1, oricare ar fi x . Să se

4
dx
calculeze  1  2 sin x1  f x .
2

4
Radu Ghenghiu

Lista premianţilor

Clasa a VII-a

Premiul I: Radu Raluca, C.N. „Mihai Viteazul” Ploieşti.


Premiul al II-lea: Crestez Paul, C.N. „N. Bălcescu” Brăila
Premiul al III-lea: Muşat Alin, C.N. „N. Bălcescu” Brăila

59
Menţiuni: Gae Silviu, Şc. Nr. 2 „Sf. Andrei” Slobozia
Gheorghe Matei, Liceul de Artă „I. Perlea” Slobozia
Iordache Vladimir, Şc. Nr. 3 Slobozia
Iorgulescu Tiberiu, C.N. „Mircea cel Bătrân” Constanţa
Popa Andreea, C.N. „N. Bălcescu” Brăila
Iacov Mihai, Şc. „N. Titulescu” Călăraşi

Clasa a VIII-a

Premiul I: Turcu Denis, C.N. „N. Bălcescu”, Brăila


Premiul al II-lea: Custură Mihai, Şc. Nr. 3 Slobozia
Premiul al III-lea: Tănase Mircea, Şc. Movila, Ialomiţa
Menţiuni: Ardelean Daniela, Şc. Nr. 2 „I.H. Rădulescu” Urziceni
Toma Ioana, Şc. Nr. 2 „Sf. Andrei” Slobozia
Tapluk Aurelia, Şc. Nr. 2 „I.H. Rădulescu” Urziceni
Iacob Andrei, C.N. „Gh. M. Murgoci”, Brăila
Cazacu Andrei Gabriel, Şc. Nr. 2 „I.H. Rădulescu” Urziceni
Marin Alexandru, Şc. Nr. 2 „I.H. Rădulescu” Urziceni
.

Clasa a IX-a

Premiul I: Drăjneanu Diana, C.N. „M. Viteazul” Slobozia


Pojar Mihai, C.N. „Gr. Moisil” Urziceni

Premiul al III-lea: Nicolae Alexandra, C.N. „N. Bălcescu”, Brăila


Menţiuni: Petre Bogdan, C.N. „M. Viteazul” Ploieşti
Veigang Rădulescu Vlad, C.N. „I.L. Caragiale” Ploieşti
Mihai Roxana, C.N. „M. Viteazul” Slobozia
Pîrău Dan, Gr. Şc. „N. Iorga” Negreşti, Vaslui
Stegăroiu Petre C.N. „M. Viteazul” Ploieşti

Clasa a X-a

Premiul I: Drăgoi Octav, Lic. Internaţional de Informatică, Bucureşti


Premiul al II-lea: Gîţ Alexandru, Lic. Internaţional de Informatică, Bucureşti
Premiul al III-lea: Duma Andrei, C.N. „I.L. Caragiale” Ploieşti
Menţiuni: Geambaşu George, C.N. „M. Viteazul” Slobozia
Muşat Cristina, C.N. „Gh. M. Murgoci”, Brăila
Pîrvu Alexandra„C.N. „Gr. Moisil” Urziceni
David Octavian, Lic.Teoretic „A. Saligny”, Cernavodă

Clasa a XI-a

Premiul I: Mihai Bogdan, Gr. Şc. „Şt. Procopiu” Vaslui


Premiul al II-lea: Bîrsan Răzvan, Gr. Şc. „Şt. Procopiu” Vaslui
60
Premiul al III-lea: Bengescu Laura, C.N. „Tudor Vianu” Bucureşti
Menţiuni: Matei Adrian, C.N. „I. L. Caragiale” Ploieşti
Vasile Sorin, C.N. „I. L. Caragiale” Ploieşti
Purdea Lelia, Lic. Teoretic „A. Saligny” Cernavodă

Clasa a XII-a

Premiul I: Blănaru Adina, Gr. Şc. „Şt. Procopiu” Vaslui;


Premiul al II-lea: Sezciuc Radu, C.N. „Mihai Viteazul” Slobozia
Premiul al III-a: Sevastian Emma, C.N. „Mihai Viteazul” Ploieşti
Menţiuni: Florea Răzvan, C.N. „Mihai Viteazul” Slobozia,
Mincu Diana, Lic. de Artă „I. Perlea” Slobozia;
Scarlat Daniel, Lic. Teoretic „Carol I” Feteşti

Premiul special „Florin Cioacă”:


Drăgoi Octav, clasa a X-a, Lic. Internaţional de Informatică, Bucureşti

Ediţia a XVII-a, 25-27 martie 2011

SUBIECTE

Clasa a VII-a
Subiectul I
Fie ABCD un romb de latură 1 cu mABC  600 . Considerăm punctul E cu CE  AB ,
CE  AB şi astfel încât punctele C şi E sunt separate de dreapta AB. Fie punctul F pe dreapta AB
astfel încât DF  DE . Să se calculeze lungimea segmentului AF.
Neculai Stanciu, Gazeta Matematică

Subiectul II
Să se rezolve în numere naturale ecuaţia x 2  1  y 2 .
***
Subiectul III
Se consideră triunghiul ABC , E mijlocul laturii AC , F mijlocul laturii AB Bisectoarea
unghiului BAC intersectează medianele CF şi BE în punctele M , respectiv N . Să se
arate că:
FM EN BC 2
a) Dacă   , atunci triunghiul ABC este dreptunghic.
MC NB 2 AB  AC
FM EN 2
b)  
FC EB 3
Nicolae Papacu, Slobozia

61
Clasa a VIII-a
Subiectul I
 
Considerăm mulţimea: A  a3  b3  c3  3abc a, b, c   , a  b  c  a . Care este cel mai mic
element al mulţimii A?
Elena Iurea, Gazeta Matematică
Subiectul II
Fie ABCDABCD un paralelipiped dreptunghic. Drumul cel mai scurt (parcurs pe suprafaţa
paralelipipedului) între C şi A trece prin P AD şi are lungimea de 20cm. Ştiind că AP  3
CP 3
cm şi  , aflaţi volumul paralelipipedului.
AC 4
Verona Marin, Gazeta Matematică
Subiectul III
Se consideră o piramidă patrulateră regulată SABCD cu AB  2a , SO  a , unde
AC  BD  O. Să se afle lungimea minimă a segmentului AP, unde P este un punct variabil
pe cercul circumscris triunghiului SBC.
Sorana Dinu, Răzvan Dinu, Slobozia

Clasa a IX-a

Subiectul I
Fie şirul an n1 definit prin a1  1 şi an1  an 
1
2
 
1  8an  1 , n  1. Să se arate că termenii
şirului sunt numere naturale.
Cătălin Cristea, Gazeta Matematică
Subiectul II
În triunghiul ascuţitunghic ABC fie H A , M A , I A înălţimea, mediana, respectiv bisectoarea din A.
Considerând şi analoagele, să se arate că dacă H A , M B , I C  şi H B , M C , I A  sunt triplete de
drepte concurente, atunci triunghiul ABC este echilateral.
I.C. Drăghicescu, Bucureşti
Subiectul III
Fie ABC un triunghi oarecare şi x, y, z  cu x  y  z  0 . Considerăm pe laturile AB şi AC
MB NC
punctele variabile M, respectiv N astfel încât x  y  z . Să se arate că dreapta MN trece
MA NA
printr-un punct fix.
Marcel Popescu, Slobozia

Clasa a X-a
Subiectul I
Fie z1 , z2 numere complexe cu proprietatea că z15  z 25  2 , z13  z 23  2 , z1 z 2  1 . Să se arate
că z1  z 2  2 .
Aurel Doboşan, Gazeta Matematică

62
Subiectul II
log 7 x  4  log 2  y  5

Să se rezolve sistemul: log 7  y  4  log 2  z  5
log z  4  log  x  5
 7 2
Octav Drăgoi, Bucureşti
Subiectul III
y2  z 2 z 2  x2 x2  y2 1  1 1 1 
Să se arate că pentru orice x, y, z  0 , avem         6 .
x y z 2 x y z
Cătălin Cristea, Gazeta Matematică

Clasa a XI-a
Subiectul I
 
Fie an n1 un şir definit prin: a1  2 şi nan1  2n  2 an  n2n , n  1. Să se calculeze
lim n2 a1a2 an .
n
Traian Tămâian, Gazeta Matematică
Subiectul II
Fie funcţiile f , g :  , f x  e x sin x , gx  e x cos x . Să se arate că pentru orice n şi
orice x    
avem: f n x  g n x  2n e2 x .
2 2

Marcel Ţena, Bucureşti


Subiectul III
Se consideră matricele A, B  M n   astfel încât A  B4  A4  B4 . Să se arate că
AB2  On .
Liviu Smarandache, Gazeta Matematică

Clasa a XII-a

Subiectul I
2
 b2  1 
 
b
1
Fie 0  a  b . Să se arate că 4 b  a  3 3
dx  3 ln 2  .
a x2 1
2
  a  1
Constantin Rusu, Gazeta Matematică
Subiectul II
Fie K  un corp în care operaţiile sunt cele induse din şi astfel încât pentru orice
a  K  , există b  K  pentru care a  b  şi ab . Să se demonstreze că există

 d   x  y .
def
d  , liber de pătrate, astfel încât K  d / x, y 
Marcel Ţena, Bucureşti
Subiectul III
Să se arate rădăcinile polinomului f  X 5  4 X 4  aX 3  bX 2  2 X  c   X  nu pot avea
acelaşi modul.
I.C. Drăghicescu, Bucureşti

63
Lista premianţilor

Clasa a VII-a
Premiul I: Popescu Andreea, Şc. nr. 3, Slobozia
Premiul al II-lea: Oancea Emilia, C.N . „N. Balcescu” Brăila
Premiul al III-lea: Copaciu Tiberiu, C.N. „Al. I Cuza” Ploiesti
Menţiuni: Orlea Martina, Şc. Fănuş Neagu, Brăila,
Chiriac Andreea, Lic. „M Eminescu” Călăraşi
Niculescu Mihai Alexandru, Şc. „N. Titulescu” Călăraşi
Sarcoseli Claudiu, Şc. Fănuş Neagu, Brăila

Clasa a VIII-a

Premiul I: Popa Andreea, C.N . „N. Balcescu” Brăila


Premiul al II-lea: Gae Silviu, Şc. nr. 2 „Sf. Andrei”, Slobozia
Premiul al III-lea: Bîrsan Andrei, C.N. „Mihai Viteazul”, Ploieşti
Menţiuni: Bob Cristian, Lic. „M Eminescu” Călăraşi
Barbu Luiza, Şc. nr. 2 „I. H. Rădulescu”, Urziceni
Calotescu Andreea, Şc. nr.7 „Aurel Vlaicu”, Feteşti
Gheorghe Matei, Lic. de Artă „Ionel Perlea”, Slobozia

Clasa a IX-a

Premiul I: Vasilache Cosmin, Lic. Teoretic „A. Saligny”, Cernavodă


Premiul al II-lea: Radu Cristiana Laura, C.N. „B. Ştirbei” Călăraşi
Premiul al III-lea: Ţuţuianu Ana, C.N. „B. Ştirbei” Călăraşi
Menţiuni: Kusztos Andrei, C.N. „I. L. Caragiale”, Ploieşti
Custură Mihai, C.N. „Mihai Viteazul”, Slobozia
Ionete Sînziana, C.N. „I. L. Caragiale”, Ploieşti
Tănase Mircea, C.N. „Mihai Viteazul”, Slobozia

Clasa a X-a

Premiul I: Veigang Rădulescu Vlad, C.N. „I. L. Caragiale”, Ploieşti


Premiul al II-lea: Stegăroiu Petre, C.N. „Mihai Viteazul”, Ploieşti
Premiul al III-lea: Mihăilescu Maria, C.N. „I. L. Caragiale”, Ploieşti
Menţiuni: Mihai Roxana, C.N. „Mihai Viteazul”, Slobozia
Drăjneanu Diana, C.N. „Mihai Viteazul”, Slobozia
Ichim Georgiana, Lic. Teoretic „A. Saligny”, Cernavodă
Antohe Andreea, Liceul de artă „Ionel Perlea”, Slobozia

Clasa a XI-a

Premiul I: Ţăranu Cristian, C.N . „N. Balcescu” Brăila


64
Premiul al II-lea: Duma Andrei, C.N. „I. L. Caragiale”, Ploieşti
Premiul al III-lea: Sevastian Denisse, C.N. „Mihai Viteazul”, Ploieşti
Menţiuni: David Octavian, Lic. Teoretic „A. Saligny”, Cernavodă
Geambaşu George, C.N. „Mihai Viteazul”, Slobozia
Buzilă Eugen, Liceul Teoretic „Carol I”, Feteşti
Bercu Andrei, C.N. „Grigore Moisil”, Urziceni

Clasa a XII-a

Premiul I: Mihai Bogdan, Gr. Şc. „Şt. Procopiu”, Vaslui


Premiul al II-lea: Cojocaru Alexandru, C.N. „I. L. Caragiale”, Ploieşti
Premiul al III-lea: Conache Maria, C.N . „N. Balcescu” Brăila
Menţiuni: Bîrsan Vasile Razvan, Gr. Şc. „Şt. Procopiu”, Vaslui
Purdea Leila, Lic. Teoretic „A. Saligny”, Cernavodă
Vîlcu Cristina, C.N. „B. Ştirbei” Călăraşi
Bosînceanu Andrada, Lic. Teoretic „Carol I”, Feteşti

Premiul special „Florin Cioacă”:


Gae Silviu, clasa a VIII-a, Şcoala nr. 2 „Sf. Andrei”, Slobozia
Drăjneanu Diana, clasa a X-a, C.N. „Mihai Viteazul”, Slobozia

Ediţia a XVIII-a, 23-25 martie 2012


.SUBIECTE

Clasa a VII-a

Subiectul I
a) Fie numerele naturale nenule a, b cu a  b . Arătaţi că b  a  a, b, unde a, b reprezintă
c.m.m.d.c. pentru numerele a şi b.
b) Demonstraţi că oricare ar fi numerele naturale nenule a1 , a2 ,, a2011 cu proprietatea
1 1 1 1 1
a1  a2    a2011 are loc inegalitatea      , unde
a1 , a2  a2 , a3  a2010, a2011 a1 a2011
c, d  reprezintă c.m.m.m.c. al numerelor c şi d.
Alexandru Lăutaru, Petroşani, G.M. 12/2011

Subiectul II
Să se determine lungimile laturilor unui triunghi isoscel, ştiind că lungimile laturilor sale sunt
numere naturale şi că lungimea unei înălţimi este egală cu 2012 .
Nicolae Papacu, Slobozia

65
Subiectul III
Triunghiul ABC este dreptunghic în B, iar M este mijlocul înălţimii BD, D AC.
Perpendiculara din D pe AM intersectează pe BC în E. Demonstraţi că triunghiul AEC este
isoscel.
Romanţa Ghiţă, Ioan Ghiţă, Blaj

Clasa a VIII-a
Subiectul I
n  2010 n  2011
Determinaţi numerele naturale nenule n pentru care
 n 1
şi
n  
sunt numere naturale.

( a reprezintă partea întreagă a numărului real a)


Lucian Dragomir, Oţelu-Roşu, G.M. 12/2011
Subiectul II
Un paralelipiped dreptunghic are dimensiunile x  3, 7  x, x  2 . Determinaţi x astfel încât
diagonala paralelipipedului să aibă lungime minimă.
Ioan Voicu, Răduleşti, Ialomiţa

Subiectul III
Să se determine toate intervalele I  a, b , a  b care au proprietatea: pentru orice
 a  b a  b 
x  a,  , y , b rezultă x  y  I şi x  y  I .
 2   2 
Nicolae Papacu, Slobozia

Clasa a IX-a
Subiectul I
Rezolvaţi în mulţimea numerelor naturale ecuaţia:
 n   n 3 2   1  n .
Romanţa Ghiţă, Ioan Ghiţă, Blaj, G.M. 11/2011
Subiectul II
Într-un triunghi având lungimile laturilor în progresie aritmetică se consideră centrul I al cercului
înscris şi centrul O al cercului circumscris. Să se arate că:
a) Una din paralelele duse la laturile triunghiului prin centrul său de greutate conţine
punctul I.
b) Una din bisectoarele triunghiului este perpendiculară pe OI.
I.C. Drăghicescu, Bucureşti

66
Subiectul III
Fie ABC un triunghi şi P   AB, Q   AC, astfel încât
PB AC QC AB
 ,  . Ştiind că
PA BC QA BC
centrul cercului înscris în triunghiul ABC aparţine lui PQ, să se arate că ABC este triunghi
dreptunghic.
Vasile Berghea, Avrig, G.M. 11/2011

Clasa a X-a

Subiectul III
Considerăm numerele complexe distincte z1 , z 2 , z3 de modul 1 astfel încât
z1 z 2 z 2 z3 z3 z1
   1 . Să se demonstreze că z1 , z 2 , z3 sunt afixele unui
z1  z2  z2  z3  z3  z1 2
2 2

triunghi echilateral.
Florin Stănescu, Găieşti, G.M. 12/2011

Subiectul II
Fie z . Să se arate că următoarele afirmaţii sunt echivalente:
a) z  i
b) z satisface, simultan, inegalităţile
z 1 i  1
z 2  1  i  1.

Octav Drăgoi, Bucureşti


Subiectul III
Un patrulater este circumscris unui cerc. Dacă se cunosc lungimile a trei dintre laturile sale, se
cere să se afle maximul ariei patrulaterului şi să se precizeze când se realizează acesta.
I.C. Drăghicescu, Bucureşti

Clasa a XI-a

Subiectul I
Fie A, B  M 2   astfel încât AB  O2 . Să se demonstreze că detA  Bn  detAn  Bn  ,
oricare ar fi numărul n  1 natural.
Nicolae Bourbăcuţ, Sarmizegetusa Regia, G.M. 1/2012
Subiectul II
Fie numerele a  0, b  0, c  0 . Să se demonstreze inegalitatea

67
a  3b a  3c 1
  .
a  b3a  b a  c3a  c a
Benedict Niculescu, Bucureşti

Subiectul III
Fie a  1 . Se consideră o funcţie continuă F : 0,   0,  , astfel încât F1  1 şi Fa  a .

Să se arate că există c  1, a astfel încât Fc 1  ln Fc  a


Cătălin Cristea, Craiova, G.M. 4/2011

Clasa a XII-a

Subiectul I
Fie n un număr natural, n  2 , şi G,  un grup cu n 2  n  1 elemente. Ştiind că funcţia
f : G  G , f x  x n , este endomorfism al grupului, să se arate că G,  este abelian.
Marin Stroe, Hunedoara, G.M. 12/2011
Subiectul II
1
x 2n1
Se consideră şirul I n n1 , dat prin relaţia I n  1 (x 2  1) n dx , n  1.
2
1. Studiaţi monotonia şi mărginirea şirului I n n1 şi calculaţi lim nI n  .
n
n
1 1 1 
2. Calculaţi lim    k
k 1 k  2 5 
n k

Nicolae Papacu, Slobozia


Subiectul III

2
cos2 x  sin x2012
Să se calculeze 0 1  sin x2012  cos x2012 dx .
Florin Nicolaescu, Balş

Lista premianţilor

Clasa a VII-a

Premiul I: Zecheru Daniela, C.N. „Al. I. Cuza” Ploieşti


Premiul al II-lea: Dinu Sabina, Şcoala nr. 3 Slobozia
Premiul al III-lea: Hristescu Andrei, Şcoala nr. 3 Slobozia
Menţiuni: Băcanu Andreea, Şc. nr. 2 „Sf. Andrei” Slobozia
Lăcătuş Andreea, Şcoala nr. 3 Slobozia

68
Păunescu Adrian, Şcoala nr. 2 „I. H. Rădulescu” Urziceni
Oprea Adrian, Şcoala nr. 2 „I. H. Rădulescu” Urziceni
Piscan Ioana Sabina, Şcoala nr. 7 „A. Vlaicu” Feteşti

Clasa a VIII-a

Premiul I: Ceacîreanu Filip, C.N. „Mihai Viteazul” Ploieşti


Premiul al II-lea: Copaciu Tiberiu, C.N. „Al. I. Cuza” Ploieşti
Premiul al III-lea: Tiţa Iulian, Şcoala nr. 2 „I. H. Rădulescu” Urziceni
Menţiuni: Popescu Andreea, Şcoala nr. 3 Slobozia
Danciu Andra, C.N. „Mihai Viteazul” Slobozia

Clasa a IX-a

Premiul I: Gae Silviu, cls. a IX-a, C.N. „Mihai Viteazul” Slobozia


Premiul al II-lea: Bîrsan Andrei, C.N. „Mihai Viteazul” Ploieşti
Roşu Octavian, C.N. „I. L. Caragiale” Ploieşti
Premiul al III-lea: Matache Cristian, C.N. „I. L. Caragiale” Ploieşti
Menţiuni: Cană Baicu Cosmin, C.N. „Barbu Ştirbei” Călăraşi
Iordache Vlad, C.N. „Mihai Viteazul” Slobozia
Ilie Vlad, Liceul „A. Saligny”, Cernavodă

Clasa a X-a

Premiul I: Custură Adrian Mihai, C.N. „Mihai Viteazul” Slobozia


Premiul al II-lea: Ţuţuianu Ana Maria, C.N. „Barbu Ştirbei” Călăraşi
Vasilache Cosmin, Liceul „A. Saligny”, Cernavodă
Premiul al III-lea: Şchiaua Dragoş, C.N. „Mihai Viteazul” Slobozia
Menţiuni: Puia Cristian, C.N. „Mihai Viteazul” Slobozia
Kusztos Răzvan, C.N. „I. L. Caragiale” Ploieşti

Clasa a XI-a

Premiul I: Drăjneanu Diana, C.N. „Mihai Viteazul” Slobozia


Mihăilescu Elena, C.N. „I. L. Caragiale” Ploieşti
Premiul al II-lea: Antohe Andreea, Liceul de Arta „Ionel Perlea” Slobozia
Veigang Rădulescu Vlad Petre, C.N. „I. L. Caragiale” Ploieşti
Premiul al III-lea: Ichim Georgiana, Liceul „A. Saligny”, Cernavodă
Menţiuni: Tatu Daniela, C.N. „Mihai Viteazul” Slobozia
Pojar Mihai, C.N. „Gr. Moisil” Urziceni

Clasa a XII-a

Premiul I: David Octavian, Liceul „A. Saligny”, Cernavodă


69
Premiul al II-lea: Buzilă Eugen, Liceul „Carol I” Feteşti
Premiul al III-lea: Geambaşu George, C.N. „Mihai Viteazul” Slobozia
Panţîru Dragoş, Liceul „A. Saligny”, Cernavodă

Premiul „F. Cioacă” pentru originalitate:


Popescu Andreea, cls. a VIII-a, Şcoala nr. 3 Slobozia
Drăjneanu Diana, cls. a XI-a, C.N. „Mihai Viteazul” Slobozia

Ediţia a XIX-a, 22-24 martie 2013


SUBIECTE

Clasa a VII – a

Subiectul I
Să se arate că oricum am alege 23 de numere reale din intervalul 1,2012 , există trei printre
acestea care pot fi lungimile laturilor unui triunghi.
Mihai Bunget, Tg. Jiu, GM 10/2012

Subiectul II
Arătaţi că un triunghi este dreptunghic în A dacă şi numai dacă are loc egalitatea
a
ma  b2  c2  (se folosesc notaţiile uzuale).
2
Gheorghe Stoica, Petroşani
Subiectul III
1 1 1
Să se arate că: 86  1    ...   89 .
2 3 2005
Ion Burcă, Slatina

Clasa a VIII – a

Subiectul I
Arătaţi că, pentru orice n număr natural nenul, avem:
5 13 25 2n2  2n 1 n  n  2
   ...   .
2 2 2 2 2
Grigore Tarţa, Gherla, GM 1/2013
Subiectul II
Să se rezolve ecuaţiile:
 x  x  x  x  x  x
a)          2013 b)          2014
2 3 6 2 3 6
unde  y este partea întreagă a lui y  .
Nicolae Papacu, Slobozia

70
Subiectul III
Fie ABCD un dreptunghi şi E un punct în spaţiu. Dacă triunghiurile ECB şi CBA sunt
congruente şi AC  BE , arătaţi că E   ABC  . Calculaţi lungimea segmentului ED.
Romanţa şi Ioan Ghiţă, Blaj

Clasa a IX – a

Subiectul I
Să se arate că nu există pătrate perfecte de forma: n 9n  2012 , n  
.
G.M. 9/2012
Subiectul II
În triunghiul isoscel ABC ( AB = AC ), ortocentrul H este mijlocul unei înălţimi. Să se arate că:
AB BC AC
a) Punctul H este pe înălţimea din A şi avem  
3 2 3
b) Dacă I este centrul cercului înscris în triunghiul ABC atunci: HI  2  3 AH  
Marcel Ţena, Bucureşti
Subiectul III
Pe latura  BC  a triunghiului ABC se consideră un punct D. Arătaţi că :
a) Dacă mA  900 , atunci AD  min  BD, DC 
b) Dacă mA  900 , atunci AD  max  BD, DC 
Gheorghe Stoica, Petroşani

Clasa a X – a

Subiectul I
Fie a, b, c numere întregi cu a2  4b  c2 . Să se arate că numărul a2  2b se scrie ca sumă de
două pătrate perfecte.
Julieta Raicu, Blaj, GM 9/2012

Subiectul II
Se consideră trei numere reale nenule a, b, c cu următoarea proprietate: există trei numere
complexe z1, z2 , z3 de module r1, r2 , r3 astfel încât az1  bz2  cz3  0. Să se arate că
 a  b a  c r 2  b  cb  a  r 2  c  a c  b r 2  z  z2  z3
2
1 2 3 1
bc ca ab
I.C. Drăghicescu, Bucureşti
Subiectul III
Fie z  Definim mulţimile A  a  bz a, b   , i  a  bi a, b   . Să se arate că
avem echivalenţa: A  i  i  A şi z 2  A.
Marcel Ţena, Bucureşti

71
Clasa a XI – a

Subiectul I
Fie matricele A, B  M 2   cu proprietatea AB  BA  A. Să se arate că ABn A  O2 , pentru
orice n  
.
Marian Cucoaneş, Mărăşeşti, GM 11/2012

Subiectul II
 n  n   n 
Să se calculeze limita şirului  an n1 , unde an  1  1 2 ...1  2
n 1  n  2   n  n 
  .

2

Marcel Popescu, Slobozia

Subiectul III

Să se calculeze lim
 
ln 1  x2012   ln 1  x 
2012

.
x0 x2013
Daniel Sitaru, Drobeta Turnu-Severin

Clasa a XII – a

Subiectul I
Fie G un grup finit comutativ. Spunem că un element a din G are proprietatea (A) dacă există
un subgrup H al lui G astfel încât produsul tuturor elementelor din H este egal cu a. Să se arate
că mulţimea elementelor lui G cu proprietatea (A) este un subgrup al lui G.
Antonia Ciocan, Bucureşti, G.M. 9/2012
Subiectul II
1
xn
Fie n  
şi I n  0 x2  x 1dx . Să se arate că In este număr iraţional.
I.C. Drăghicescu, Bucureşti
Subiectul III
Spunem că un grup abelian G,  cu elementul neutru e are proprietatea (P), dacă se îndeplinesc
simultan condiţiile:
 
a) Hn  x  G / xn  e are cel mult n elemente pentru orice n   .
b) H  x  G k  , x  k

 e este finită.
1) Să se dea un exemplu de grup infinit G,  cu proprietatea (P).
2) Să se arate că există m  N  astfel încât H  Hm .
3) Să se arate că toate subgrupurile finite ale lui G sunt Hd , unde d este divizor al lui m.
Marcel Ţena, Bucureşti

72
Lista premianţilor

Clasa a VII-a

Premiul I: Antonie Iordache Ionuţ, Şc. Gimnazială „G. Enescu”, Sinaia


Premiul al II-lea: Gheorghe Alina, Şc. Gimnazială „I.H. Rădulescu, Urziceni
Premiul al III-lea: Turcea Adrian, Şc. Gimnazială nr. 3 Slobozia
Menţiuni: Bâra Andrei, Şc. Gimnazială „I.H. Rădulescu” Urziceni
Barbu Andrei, Şc. Gimnazială „I.H. Rădulescu” Urziceni
Marin Bogdan, Şc. Gimnazială „N. Titulescu” Călăraşi
Voinea Eduard, Şc. Gimnazială „Sf. Vasile” Ploieşti

Clasa a VIII-a

Premiul I: Zecheru Daniela, C.N. „Al. I. Cuza” Ploieşti


Premiul al II-lea: Chirea Miruna Maria, Şc. Gimnazială „M. Viteazul” Călăraşi
Premiul al III-lea: Hristescu Andrei, Şc. Gimnazială nr. 3 Slobozia
Menţiuni: Dinu Sabina, Şc. Gimnazială nr. 3 Slobozia
Nicoară Cătălina, Şc. Gimnazială „I.H. Rădulescu” Urziceni
Piscan Ioana, Şc. Gimnazială „A. Vlaicu” Feteşti
Sava Ştefan, Şc. Gimnazială „I.H. Rădulescu” Urziceni
Lăcătuş Andreea, Şc. Gimnazială nr. 3 Slobozia
Dincă Iustin, Şc. Gimnazială nr. 7 Giurgiu

Clasa a IX-a

Premiul I: Ceacîreanu Filip, C.N. „Mihai Viteazul” Ploieşti


Premiul al II-lea: Neagu Robert, C.N. „B. P. Haşdeu” Buzău
Premiul al III-lea: Chiriac Andreea, Lic. Teoretic „M. Eminescu” Călăraşi
Menţiuni: Comănici Codruţa, C.N. „Gr. Moisil” Urziceni
Danciu Andra, C.N. „M. Viteazul” Slobozia
Mocanu Diana, C.N. „Sf. Sava” Bucureşti
Popescu Andreea, C.N. „M. Viteazul” Slobozia

Clasa a X-a

Premiul I: Ciubotaru Alexandra, C.N. „Sf. Sava” Bucureşti


Premiul al II-lea: Bîrsan Andrei, C.N. „Mihai Viteazul” Ploieşti
Premiul al III-lea: Icăţoiu Cosmin, C.N. „Al. Vlahuţă” Râmnicu Sărat
Menţiuni: Gae Silviu, C.N. „M. Viteazul” Slobozia
Ichim Marina, C.N. „Sf. Sava” Bucureşti
Ilie Vlad, Lic. „A. Saligny”, Cernavodă
Roşu Octavian, C.N. „I.L. Caragiale” Ploieşti

73
Clasa a XI-a

Premiul I: Drăgan Relu, C.N. „B. P. Haşdeu” Buzău


Premiul al II-lea: Vasilache Teodor, Lic. „A. Saligny”, Cernavodă
Premiul al III-lea: Ţuţuianu Ana Maria, C.N. „Barbu Ştirbei” Călăraşi
Menţiuni: Ion Daniela, C.N. „B. P. Haşdeu ” Buzău
Radu Cristiana, C.N. „Barbu Ştirbei” Călăraşi
Custură Mihai, C.N. „M. Viteazul” Slobozia

Clasa a XII-a

Premiul I: Drăjneanu Diana, C.N. „Mihai Viteazul” Slobozia


Premiul al II-lea: Popina Sebastian Marius, C.N. „B. P. Haşdeu ” Buzău
Premiul al III-lea: Jercăianu Alexandru, C.N. „B. P. Haşdeu ” Buzău
Menţiuni: Pojar Mihai, C.N. „Gr. Moisil” Urziceni
David Rodica, C.N. „B. P. Haşdeu ” Buzău
Mihăilescu Elena, C.N. „I.L. Caragiale” Ploieşti
Tatu Daniela, C.N. „M. Viteazul” Slobozia

Premiul „F. Cioacă” pentru originalitate în rezolvarea unei probleme: Drăjneanu Diana,
C.N. „Mihai Viteazul” Slobozia.

Ediţia a XX-a, 28-30 martie 2014


SUBIECTE

Clasa a VII-a
Subiectul I. Se consideră triunghiul ABC cu m  A  1350 . Perpendiculara în A pe AB
intersectează latura  BC în D , iar bisectoarea unghiului B intersectează latura  AC în E .
Să se determine măsura unghiului BED .
Traian Preda, Bucureşti, G.M. nr. 1/2014
Subiectul II. Se consideră patrulaterul convex ABCD . Arătaţi că dacă triunghiurile ABC , CDB
, DAC și ABD au acelaşi perimetru, atunci ABCD este dreptunghi.
***
Subiectul III. Deteminaţi numerele naturale n pentru care numărul 3  22 este pătrat perfect.
n n

Nicolae Papacu, Slobozia

Clasa a VIII-a
Subiectul I. Se consideră cubul ABCDABCD . Dacă M este mijlocul muchiei CC ,
determinaţi măsura diedrului format de planele  ABM  şi  AMC .
Cezar Ozunu, Dăneţi, G.M. nr. 2/2014

74
Subiectul II. Să se determine x  0,1 cu proprietatea că:
x  x .
 x
(Notăm cu x partea fracţionară a lui x şi cu  x partea întreagă a lui x ).
***
Subiectul III. Fie k  . Rezolvaţi în mulţimea numerelor naturale ecuaţia:
 
x2  k 2 1 x  2k  1  0 .
***
Clasa a IX-a
Subiectul I. Fie ABCD un patrulater înscris într-un cerc de rază R. Să se arate că
AB  BC  CD  DA  4R4 .
Constantin Rusu, G.M. nr. 1/2014
Subiectul II. Arătaţi că numărul 1 2  3  2  3  4  3  4  5   n  n 1 n  2 nu poate fi pătrat
perfect, oricare ar fi n  
.
Inocenţiu Drăghicescu, Bucureşti
Subiectul III. Să se arate că, pentru orice triunghi ABC care nu este obtuzunghic şi orice numere
m, n, p3,4,5,  , este adevărată inegalitatea: cos A cos B cos C 3
m
 n  p  .
sin A sin B sin C 2sin Asin B sin C
Inocenţiu Drăghicescu, Bucureşti

Clasa a X-a

Subiectul I. Fie b un număr real strict pozitiv şi n un număr natural nenul. Să se arate că
ecuaţia z n1  bz n  z  b are o rădăcină complexă de modul 1 dacă şi numai dacă b  1 şi
n  4k 1 unde k  .
Ioan Băietu, Botoşani, G.M. nr. 10/2013
Subiectul II. Să se rezolve în numere reale ecuaţia:

2tgx  2ctgx  4 .
Marcel Ţena, Bucureşti
Subiectul III. Să se rezolve în numere complexe ecuaţia:
1 z  1 2z  3z2   
 nz n1   n 1 1 z  z 2  
 z n1 , unde n .
Marcel Chiriţă, Bucureşti

Clasa a XI-a
Subiectul I. Pentru un şir  an n0 dat, definim şirurile  xn n0 şi  yn n0 prin: xn  min  an , an1 
şi yn  max  an , an1  , pentru orice n  0 .
a) Să se arate că dacă şirul  an n0 este convergent, atunci şirurile  xn n0 şi  yn n0
sunt şi ele convergente.
b) Este adevărată reciproca?
*** G.M. nr. 11/2013
Subiectul II. a) Să se demonstreze că pentru orice t 0,1 , avem inegalitatea:
2t  1  t 2 .
b) Să se rezolve ecuaţia:

75
2sin x  2cos x  3 .
Marcel Ţena, Bucureşti

a 0 b 0 
0 a 0 b 
Subiectul III. Fie a, b și matricea M   . Să se calculeze M n , pentru
b 0 a 0 
 
0 b 0 a
n 
.
Inocenţiu Drăghicescu, Bucureşti

Clasa a XII-a
Subiectul I. Să se determine funcţiile derivabile f :  0,    0,  cu f 1 
1
, care au
2
2013 f  x 
proprietatea: f   x   , pentru orice x  0,  .
x2014  x
Supliment G.M. nr. 10/2013
Subiectul II. Pentru orice k  

, notăm Uk  x  k

x  1 . Fiind date m, n   , să se
determine:
a) Cel mai mare subgrup G al grupului  , cu proprietatea că G este subgrup al grupurilor
Um şi U n .
b) Cel mai mic subgrup al H grupului  , cu proprietatea că Um şi U n sunt subgrupuri ale lui
H.
Marcel Ţena, Bucureşti
1
Subiectul III. Se consideră şirul I n n1 , unde I n  xn arctg  nx  dx . Să se calculeze lim  nIn  .
 n
0
Nicolae Papacu, Slobozia

Lista premianţilor

Clasa a VII-a

Premiul I: Căpraru Adrian-Mircea-Gabriel, C.N."M. Viteazul" Ploieşti, Drăgoi Sabina, ICHB,


Lipan Dragoș, Șc. Gimnazială "F. Neagu" Brăila, Stănescu Andrei, Şc. Gimnazială „Sf. Andrei”
Slobozia
Premiul al II-lea: Călin Cristian, Şc. Gimnazială nr. 3 Slobozia
Mențiuni: Ilie Alexandru, C.N."M. Viteazul" Ploieşti, Belceanu Cătălin, Şc. Gimnazială nr. 3
Slobozia, Ovreiu Auraş, Sc. Gimnazială ,, A. Vlaicu “ Feteşti

Clasa a VIII-a

Premiul I: Barbu Andrei Cristian, Şc. Gimnazială „I. H. Rădulescu” Urziceni


Premiul al II-lea: Drăghici Delia, C.N."M. Viteazul" Ploieşti

76
Premiul al III-lea: Vlad Miruna, C.N. "N. Bălcescu" Brăila
Mențiuni: Turcea Adrian, Şc. Gimnazială nr. 3 Slobozia, Bâra Andrei Robert, Şc. Gimnazială
„I. H. Rădulescu” Urziceni, Gheorghe Alina Andreea, Şc. Gimnazială „I. H. Rădulescu”
Urziceni, Stănculescu Anca, Şc. Gimnazială nr. 3 Slobozia

Clasa a IX-a

Premiul I: Zecheru Daniela Cristina, C.N."Al.I.Cuza" Ploieşti


Premiul al II-lea: Lăcătuş Andreea, C.N. „M. Viteazul” Slobozia
Premiul al III-lea: Dinu Sabina, C.N. „M. Viteazul” Slobozia
Mențiuni: Hristescu Andrei, C.N. „M. Viteazul” Slobozia, Lazăr Andreea, C.N. „M. Viteazul”
Slobozia, Meiroșu Rareș, C.N. "N. Bălcescu" Brăila

Clasa a X-a

Premiul I: Neagu Robert, C.N.“B.P.Haşdeu” Buzău


Premiul al II-lea: Chiriac Andreea, C.N. “B. Ştirbei” Călăraşi, Tiţă Iulian, C.N.”Gr. Moisil”,
Urziceni
Premiul al III-lea: Kelesidis Evgnosia-Alexandra, C.N."M. Viteazul" Ploieşti
Mențiuni: Dragomir Horia Alexandru, C.N. "I. C. Brătianu" Pitești, Ceacîreanu Mihai Filip,
C.N."M. Viteazul" Ploieşti, Șerbănescu Corina Ioana, C.N. "I. L. Caragiale" , Ploieşti, Barbu
Andrei, C.N. "N. Bălcescu" Brăila

Clasa a XI-a

Premiul I: Ilie Cătălin-Andrei, C.N. "M. Viteazul" Ploieşti


Premiul al II-lea: Bîrsan Andrei, C.N. "M. Viteazul" Ploieşti
Premiul al III-lea: Gae Silviu, C.N. „M. Viteazul” Slobozia
Mențiuni: Matache Cristian-Răzvan, C.N."I. L. Caragiale", Ploieşti, Ilie Vlad, Lic. Teoretic "A.
Saligny" Cernavodă, Icătoiu Cosmin, C.N. “Al. Vlahuță”
Rm. Sărat, Minciu Oana, Liceul de Arte ”I. Perlea” Slobozia

Clasa a XII-a

Premiul I: Cosma Ioan-Adrian, C.N. „Gh. Șincai” București


Premiul al II-lea: Drăgan Relu, cls. a XII-a, C.N.“B.P.Hasdeu” Buzău
Premiul al III-lea: Kusztos Răzvan-Emanuel, C.N."I. L. Caragiale", Ploieşti
Mențiuni: Vasilache Teodor Cosmin, Lic. Teoretic "A. Saligny" Cernavoda, Toma Theodor,
C.N. "N. Bălcescu" Brăila, Țuțuianu Ana Maria, C.N. “B. Ştirbei” Călăraşi, Bantaş Mihai, C.N.
„Gh. Șincai” București

Premiul „Fl. Cioacă” pentru originalitate în rezolvarea unei probleme: Stănescu Andrei,
cls. a VII-a, Şc. Gimnazială „Sf. Andrei” Slobozia.

77
Ediția a XXI-a, 27-29 martie 2015
Subiecte

Clasa a VII-a
Subiectul I
Dacă a,b sunt cifre nenule astfel încât a  b  10 , iar c astfel încât ab  c  25 ,
arătați că numărul 2015a  2015b  c este pătrat perfect.
D.M. Bătinețu-Giurgiu, București, Neculai Stanciu, Buzău
Subiectul II
a) Determinaţi numerele naturale n pentru care 16n 12n este pătrat perfect.
b) Determinaţi numerele naturale n care verifică relaţia 16n 12n  11n  5n  4n .
Nicolae Papacu, Slobozia

Subiectul III
Fie paralelogramul ABCD. Se notează cu E simetricul punctului A față de diagonala BD.
Demonstrați că punctele B, C, E, D sunt pe același cerc.
Ion Voicu, Rădulești, Ialomița

Clasa a VIII-a
Subiectul I
Determinați numărul natural n și numărul natural prim p , astfel încât numărul pn  8
să fie un cub perfect.
Nicolae Papacu, Slobozia
Subiectul II
Determinați numerele prime p și q astfel încât
 p 1!1  q6  39
p
( Dacă n  1 este un număr natural, notăm n!  1 2  3   n ) .
Cristian Moanță, Lucian Tuțescu, Craiova, Supliment G.M., 2/2015
Subiectul III
Se consideră tetraedrul ABCD și punctele M   AB , N   AC  și P   AD  astfel încât
1 1
MA  AB , N este mijlocul lui AC și AP  AD . Fie Q și R punctele de intersecție ale
3 3
planului (MNP) cu dreptele BC, respectiv CD.
Să se arate că dreapta de intersecție a planelor (ABR) și (ADQ) este paralelă cu planul
(BCD).
Marian Mărgărit, Slobozia

Clasa a IX-a

Subiectul I
Să se rezolve ecuația x2  5 x  2x  0
2 2

Inocențiu Drăghicescu, București

Subiectul II
78
Dacă A, B, C sunt măsurile unghiurilor unui triunghi, să se demonstreze că numerele
A B C
cos ,cos ,cos sunt lungimile laturilor unui triunghi.
2 2 2
D.M. Bătinețu-Giurgiu, București, Neculai Stanciu, Buzău, G.M. nr.2/2015
Subiectul III
Fie H ortocentrul triunghiului ascuțitunghic ABC. Calculați măsura unghiului A, știind că
BH  CH  BA  CA  4 BC
Mihaela Berindeanu, București

Clasa a X-a

Subiectul I
Să se rezolve în numere reale ecuația: 2log5 x 1  5  xlog5 4  2 .
Camelia Macsut, Supliment G.M. 3/2015
Subiectul II
Să se arate că în orice triunghi ABC, are loc egalitatea
1 1 1 1 1
 2 2  2
2
AI BI CI AI  BI  CI r
unde I, r semnifică centrul, respectiv raza cercului înscris.
Florin Rotaru, Focșani
Subiectul III
4

 
n n
. Să se arate că:  zk   4n  8 Re zk2 .
 1
Fie z1, z2 , , zn  , n 

k 1 zk k 1

Daniel Sitaru, Drobeta Turnu Severin


Clasa a XI-a
Subiectul I
1 a ha hb 
 
Se consideră matricea A  1 b hb hc  , unde cu a, b, c se notează lungimile laturilor
1 c h h 
 c a

unui triunghi, iar cu ha , hb , hc lungimile înălțimilor triunghiului. Să se arate că det A  0 . În ce


condiții avem det A  0 ?
Lucian Dragomir, Oțelu Roșu, Supliment G.M., 12/2014
Subiectul II
Fie numărul real x0 . Considerăm șirul  xn n0 , cu termenul general satisfăcând relația
3xn2  xn  2
de recurență xn1 
2xn2  xn  3
Să se arate că șirul este convergent și să i se afle limita.
Inocențiu Drăghicescu, București
Subiectul III
Să se determine funcțiile derivabile f :  cu proprietățile:
(i) f  0  0 .

79
(ii) f   f  x   1, pentru orice x .
Marcel Țena, București

Clasa a XII-a
Subiectul I
 
Să se determine o primitivă a funcției f :  0,  , definită prin
 2
f  x   sin x ln tgx  ctgx  .
Vasile Mircea Popa, Sibiu, Supliment G.M., 2/2015
Subiectul II
Fie  A, ,  un inel cu proprietatea că x2  0 sau x2  1 , pentru orice x  A .
Să se demonstreze că:

a) Grupul elementelor inversabile din inelul A este U  A  x  A x2  1 . 
b) Inelul A este comutativ.
Lurențiu Panaitopol, Toma Albu, București
Subiectul III

 
1
Să se calculeze lim  ln xn  ex dx .
n
0
Traian Tămâian, Carei

Lista premianților

Clasa a VII-a

Premiul I: Săvulescu Ştefan, C.N. „I.L. Caragiale” Ploieşti


Premiul al II-lea: Sima Cosmin Alexandru, Şc. Gimnazială „A. Vlaicu” Feteşti
Premiul al III-lea: Vârlan Mihnea, Şc. Gimnazială Centrală Câmpina
Mențiuni: Bâra Sînziana, Șc. Gimnaziala „I. H. Radulescu” Urziceni; Mihai Raluca, Șc.
Gimnazială Nr. 1 Rîmnicu Sărat

Clasa a VIII-a

Premiul I: Dumitru Cătălin, C.N. ,,B.P. Haşdeu” Buzău; Ovreiu Auraş, Şc. Gimnazială „A.
Vlaicu” Feteşti; Stănescu Andrei, Şc. Gimnazială „Sf. Andrei” Slobozia
Mențiuni: Buf Sorina, Şc. Gimnazială Nr. 3 Slobozia; Căpraru Adrian, C.N. ,,Mihai Viteazul”
Ploieşti; Săceleanu Andrei, Şc. Gimnazială „A. Vlaicu” Feteşti

Clasa a IX-a

Premiul I: Bâra Andrei, Liceul Internațional de Informatică Constanţa; Corcescu Tiberiu, Liceul
Internațional de Informatică Constanţa; Gheorghe Alina, C.N. Militar ,,D.Cantemir” Breaza
Mențiuni: Voinea Eduard, C.N. ,,Mihai Viteazul” Ploieşti; Milcu Ana Maria, Liceul
Internațional de Informatică Constanţa; Zevri Radu, Liceul Internațional de Informatică
Constanţa; Turcea Adrian, C.N. ,,Mihai Viteazul” Slobozia;

80
Clasa a X-a

Premiul I: Zecheru Daniela, C.N. „Al. I. Cuza” Ploieşti


Premiul al II-lea: Sava Ştefan, C.N. Militar ,,D.Cantemir” Breaza
Premiul al III-lea: Constantin Maria, C.N. ,,Mihai Viteazul” Ploieşti
Mențiuni: Dinu Sabina, C.N. ,,Mihai Viteazul” Slobozia; Lăcătuș Andreea, C.N. ,,Mihai
Viteazul” Slobozia; Ioniță Cristian, C.N. ,,Mihai Viteazul” Slobozia; Manole Daniela, Liceul
Teoretic „A. Saligny” Cernavodă; Hristescu Andrei, C.N. ,,Mihai Viteazul” Slobozia

Clasa a XI-a

Premiul I: Dragomir Horia Alexandru, C.N. „I. C. Brătianu” Piteşti


Premiul al II-lea: Chiriac Andreea, C.N. ”Barbu Știrbei” Călăraşi
Premiul al III-lea: Tiţă Iulian, C.N. „Grigore Moisil” Urziceni
Mențiuni: Troscot Silviu, C.N. ,,Al. Vlahuţă“ Rîmnicu Sărat; Savu Mihnea, C.N. „I.L.
Caragiale” Ploieşti; Leca Silviu, C.N. ,,Mihai Viteazul” Slobozia

Clasa a XII-a

Premiul I: Ilie Vlad, Liceul Teoretic „A. Saligny” Cernavodă; Matache Cristian, C.N. „I.L.
Caragiale” Ploieşti.
Premiul al III-lea: Gae Silviu, C.N. ,,Mihai Viteazul” Slobozia
Mențiune: Icatoiu, Vlad, , C.N. ,,Al. Vlahuţă“ Rîmnicu Sărat

Ediția a XXII-a, 1-3 aprilie 2016

Subiecte

Clasa a V-a

Subiectul I.
4n  3
a) Aflați valorile numărului natural nenul n pentru care fracția este reductibilă.
5n 1
b) Arătați că suma numerelor care împărțite la 2016 dau restul de două ori mai mare decât câtul,
se poate scrie ca produsul a trei numere naturale consecutive.
Marcel Popescu, Slobozia
Subiectul II. Un număr natural se numește cub bipătratic dacă este cub perfect și se scrie ca
suma a două pătrate perfecte nenule diferite. Un număr natural se numește pătrat bicubic dacă
este pătrat perfect și se scrie ca suma a două cuburi perfecte nenule diferite.
a) Dați exemplu de un cub bipătratic și un exemplu de pătrat bicubic.
b) Arătați că există o infinitate de cuburi bipătratice și o infinitate de pătrate bicubice.
Cătălin Cristea, Craiova, G.M. nr. 11/2015
Subiectul III. Mulțimea numerelor naturale pare și nenule se scrie într-un tabel în felul următor:
Linia 1 2
Linia 2 4, 6

81
Linia 3 8, 10, 12
Linia 4 14, 16, 18, 20
Linia 5 22, 24, 26, 28, 30
...........................................................................................................
a) Aflați cu ce număr începe linia 2016.
b) Aflați pe ce linie se găsește numărul 2016
c) Aflați valoarea numărului natural nenul n astfel încât suma elementelor liniei n să fie
egală cu 8020.
Marcel Popescu, Slobozia
Clasa a VI-a

Subiectul I. Două dintre numerele raționale a, b, c sunt numere naturale. Numerele a, b, c sunt
direct proporționale cu numerele k , k 1, k  2 , unde k este număr natural impar mai mare sau
egal cu 3. Determinați numerele a, b, c, k știind că a  c  112  2b .
Dan Nedeianu, Drobeta Turnu Severin, G.M. nr. 12/2015
Subiectul II. Se consideră unghiurile DAB , BAC , CAE ascuțite și congruente două câte
două, astfel încât  AD   AB și  AC    AE  . Fie P  BE  DC , M   AB  DC și
N  BE  AC .
a) Să se arate că unghiurile AMP și ANP sunt suplementare.
b) Dacă AMP  ANP , atunci să se arate că dreptele AP și BC sunt perpendiculare.
Marian Mărgărit, Slobozia
2 3 4 2016
Subiectul III. Se consideră numărul S  1      .
2 22 23 22015
Să se arate că 3  S  4 .
Marian Mărgărit, Slobozia

Clasa a VII-a
Subiectul I. Arătați că numărul  2n  1   2n 1 se scrie ca suma a trei pătrate perfecte,
3 3

oricare ar fi n  
.
Aurel Doboșan, Lugoj, G. M. Nr. 1/2016
Subiectul II. Se consideră ABCD paralelogram cu AC=2AB, AC  AB . Dacă E este mijlocul
segmentului  AD , perpendiculara din D pe CE taie perpendiculara în B pe AB în punctul F, să
se arate că dreapta EF trece prin mijlocul segmentului  AC
Sorana Ionescu, Slobozia
Subiectul III. Să se arate că pentru orice număr natural n :
a) 5n  3n nu se divide prin 7;
b) 5n  3n nu este pătrat perfect.
Nicolae Papacu, Slobozia

Clasa a VIII-a
Subiectul I. În trapezul ABCD puncctele M și N sunt mijloacele laturilor neparalele AD,
AB
respectiv BC, CM  DN  Q , raportul bazelor este  k  1 , iar A DQC  t .
CD
a) Exprimați aria trapezului în funcție de k și t .

82
b) Există k, t naturale astfel încât aria trapezului să fie 2016?
Mihail Frăsilă, Constantin Petrea, Pașcani
Subiectul II.
x y
a). Fie x, y  0 numere reale. Demonstrați că x2  xy  y2  .
2
b). Tetraedrul ABCD îndeplinește condițiile: m  BAC   m  BAD  m  CAD  600 ,
AB  AC  AD  BC  BD  CD .
Demonstrați că ABCD este un tetraedru regulat.
Cătălin Cristea, Craiova, G.M. nr. 2/2016
Subiectul III. Se dă cubul ABCDABCD și fie P piciorul perpendicularei din C pe AC .
a) Să se arate că triunghiurile APD și ABC sunt congruente.
b) Aflați cosinusul unghiului format de planele  APD și  ABC  .
Ion Neață, Slatina

Clasa a IX-a
Subiectul I. Să se rezolve ecuația x  22x  10x  9 .
2

Constantin Nicolau, Curtea de Argeș, G.M. nr. 12/2015


Subiectul II. Se consideră triunghiul ascuțitunghic ABC cu a  b  c , unde a  BC , b  AC ,
c  AB .
1). Să se arate că există un unic pătrat cu două vârfuri pe BC și celelalte două vârfuri pe AB și
BC .
2). Notăm cu Sa aria acestui pătrat. Fie Sb și Sc ariile pătratelor construite analog pentru
laturile CA și AB . Să se arate că Sa  Sb  Sc .
Inocențiu Drăghicescu, București
1 1 1 4
Subiectul III. Să se arate că     pentru orice n  
.
n 1 n  2 3n 3
Valentin Nicula, București

Clasa a X-a
Subiectul I. Să se determine numărul n  pentru care n 29 2  41  n 29 2  41  2 .

Aurel Doboșan, Lugoj, G.M. nr. 12/2015


Subiectul II. Să se determine toate funcțiile f :  cu proprietatea:
n

 f  k  f  n  k   2 f  n , n 
k 0
n
.

Marcel Țena, București

a  1x  a y  a y 1

Să se rezolve în numere reale sistemul a  1  a z  a z 1 , unde
y
Subiectul III.

a  1  a  a
z x x 1

a  0,  .
Nicolae Papacu, Slobozia

83
Clasa a XI-a
n
n!
Subiectul I. Să se calculeze lim .
n 1 1  1   1
e 2 3 n

Cătălin Pană, Râmnicu Vâlcea


Subiectul II. Fie matricele de ordinul 2 cu elemente numere reale A, B, C astfel încât
A2015  B2015  C2015  AB  BC  CA  O2 . Să se arate că A2  B2  C2  O2 . Există matrice
nenule cu proprietățile din enunț?
Lucian Tuțescu, Craiova, G.M. nr. 12/2015
exi n
n n
Subiectul III. Fie x1, x2 , , xn   0,  . Să se arate că    x .
i 1 1  e 2 i 1 i
xi

Daniel Sitaru, Drobeta Turnu Severin

Clasa a XII-a
Subiectul I. Se consideră ecuația x  3x  5x2  mx  2  0 , unde m , m  0 . Notând cu
4 3

x1, x2 , x3 , x4 rădăcinile ecuației, să se demonstreze că exact 3 dintre punctele de afixe x1, x2 , x3 , x4


sunt situate în planul complex de aceeași parte a axei imaginare.
Inocențiu Drăghicescu, București
Subiectul II. Fie funcția f : 0,1  de două ori derivabilă, cu f  0  0 și f   0  0 . Să se
 x
1

 f  n  dx .
2
calculeze lim n
n
0
Florin Rotaru, Focșani, G.M. nr. 12/2015
Subiectul III. Numim serie formală peste un „polinom infinit”, adică o expresie scrisă unic

sub forma f  X   a0  a1 X  a2 X 2    an X n , unde an  , n  , iar X este o
n0
nedeterminată. Două serii formale se înmulțesc „precum polinoamele”, adică
 n 

n

  n   n   cn X , unde cn   aibj .
 n
a X b X
 n0  n0  n0 i  j n

Determinați seriile formale cu proprietatea: f 2  X   f  2 X 


Marcel Țena, București

Premianții acestui concurs sunt:

Clasa a V-a

Premiul I: Toader Sebastian (Șc. Gimn. nr. 3 Slobozia)


Premiul al II-lea: Dumitru Maria (Șc. Gimn. nr. 3 Slobozia)
Premiul al III-lea: Șerban Ioana (Șc. Gimn. nr. 3 Slobozia)

84
Mențiuni: Drăgan Mihaela (Șc. Gimn. „Al. Odobescu” Urziceni), Țigău Mara (Șc. Gimn. nr. 3
Slobozia), Dinu Raluca (Șc. Gimn. nr. 3 Slobozia), Iagăr Vlad (Șc. Gimn. nr. 3 Slobozia), Istrate
Mihai (Șc. Gimn. „Sf. Andrei” Slobozia), Duzi Mihai (Șc. Gimn. „Al. Odobescu” Urziceni),
Mihai Dragoș (Șc. Gimn. nr. 3 Slobozia).

Clasa a VI-a

Premiul I: Petre Alexandru (Șc. Gimn. nr. 3 Slobozia)


Premiul al II-lea: David Andrei (Șc. Gimn. nr. 3 Slobozia)
Premiul al III-lea: Costea Andreea (Șc. Gimn. nr. 3 Slobozia)
Mențiuni: Niță Eduard (C.N. „Gr. Moisil” Urziceni), Parfinescu Claudiu (Șc. Gimn. „A.
Vlaicu” Fetești), Berbece Rian (Șc. Gimn. „Sf. Andrei” Slobozia)

Clasa a VII-a

Premiul I: Ionescu Dan (Şc. Gimn. "Sf. Andrei" Slobozia)


Premiul al II-lea: Manea Lidia (Șc. Gimn. nr. 3 Slobozia)
Premiul al III-lea: Neacșu Liviu (C.N. „B.P. Hașdeu” Buzău)
Mențiuni: Băceanu Robert (C.N. „B.P. Hașdeu” Buzău), Frunză Vladimir (C.N. „I.L. Caragiale”
Ploiești), Gurău Denis (Șc. Gimn. „A. Vlaicu” Fetești), Negoiță Anca (C.N. „M. Viteazul”
Slobozia), Bordea Cosmin (C.N. „M. Viteazul” Slobozia)

Clasa a VIII-a

Premiul I: Dinu Andreea (Șc. Gimn. nr. 3 Slobozia), Donescu Irina (C.N. ”N. Bălcescu” Brăila),
Mincu Mihai (Șc. Spectrum Ploiești), Nicoară Laura (Șc. Gimn. nr. 3 Slobozia)
Mențiuni: Buzoianu Dan (Șc. Gimn. nr. 3 Slobozia), Vârlan Mihnea (Șc. Centrală Câmpina)

Clasa a IX-a

Premiul I: Dumitru Cătălin (C. N. „B.P. Hașdeu” Buzău), Olteanu Cătălina (C.N. „M.
Viteazul” Ploiești)
Premiul al III-lea: Gheorghe Antonia (C.N. „I.L. Caragiale” Ploiești)
Mențiuni: Călin Cristian (C.N. „M. Viteazul” Slobozia), Nastasia Alexandru (C.N. „M.
Viteazul” Slobozia), Surugiu Horia (C.N. "Gh. Șincai" București), Bercaru Teodora (C.N.
„Spiru Haret” București), Irimia Alexandra (C.N. „M. Viteazul” Ploiești), Moraru Mircea (C.N.
"Gh. Șincai" București)

Clasa a X-a

Premiul I: Bonifate Tudor (C.N. „I.L. Caragiale” Ploiești)


Premiul al II-lea: Gheorghe Alina (C. N. Militar „D. Cantemir” Breaza)
Premiul al III-lea: Voinea Eduard (C.N. „M. Viteazul” Ploiești)
Mențiuni: Popa Iulia (C.N. „M. Viteazul” Slobozia), Bazavan Cristian (C.N. „Spiru Haret”
București)

Clasa a XI-a

Premiul I: Sava Ștefan (C. N. Militar „D. Cantemir” Breaza)


85
Premiul al II-lea: Mușat Mihai (C.N. „I.L. Caragiale” Ploiești)
Premiul al III-lea: Lazăr Andreea (C.N. „M. Viteazul” Slobozia)
Mențiuni: Constantin Maria (C.N. „M. Viteazul” Ploiești), Dinu Sabina (C.N. „M. Viteazul”
Slobozia), Hristescu Andrei (C.N. „M. Viteazul” Slobozia)

Clasa a XII-a

Premiul I: Dragomir Horia (C.N. „I. C. Brătianu” Pitești)


Premiul al II-lea: Tiță Iulian (C.N. „Gr. Moisil” Urziceni)
Premiul al III-lea: Stănișor Ștefan (ICHB)
Mențiuni: Grosulea Andreea (C.N. "Gh. Șincai" București), Focșăneanu Bogdan (C.N. "Gh.
Șincai" București)

Ediția a XXIII-a, 31 martie-2 aprilie 2017

Subiecte

Clasa a V-a
Subiectul I.
Pe o tablă sunt scrise numerele 1,2,3,...,2017. Un pas constă în ștergerea a două numere de pe
tablă și punerea în loc a restului împărțirii sumei lor la 251. După mai mulți pași, pe tablă rămân
două numere. Dacă unul dintre aceste numere este 1007, aflați care este celălalt număr.
Cristina Militaru, București, G.M. nr. 1/2017
Subiectul II.
Aflați câtul și restul împărțirii numărului 2  32017  51017 la numărul 22  32015  51015
Marcel Popescu, Slobozia

Subiectul III.
Aflați pentru fiecare x4,5,6,7,8,9 numărul abcde cu proprietatea că dacă punem x la
sfârșitul lui, obținem un număr de 4 ori mai mic decât numărul obținut prin așezarea lui x la
începutul numărului abcde .
Marcel Popescu, Slobozia

Clasa a VI-a

Subiectul I.
2n2 1
Fie n un număr natural, n  2 . Arătați că se scrie ca o fracție zecimală periodică mixtă.
n5  n
Constantin Petrea, Pașcani, G. M. nr. 2/2017
Subiectul II.
Se consideră numerele naturale nenule a, b, c astfel încât a  b  c și 6c  17b 11a .
Să se arate că 1122 divide numărul n   c  a  c  bb  a  a  b  c  .
Marian Mărgărit, Slobozia
86
Subiectul III.
În triunghiul neisoscel ABC , bisectoarea unghiului BAC intersectează madiatoarea laturii BC
în punctul M .
Să se arate că unghiurile BAC și BMC sunt suplementare.
Marian Mărgărit, Slobozia

Clasa a VII-a

Subiectul I.
SE consideră triunghiul oarecare ABC și ceviana AA , A   BC  , astfel încât triunghiurile
ABA și ACA au perimetre egale. Similar, se constriesc cevienele BB și CC . Să se arate că
AA , BB și CC sunt concurente.
Inocențiu Drăghicescu, București
Subiectul II.
1. Să se determine numerele naturale nenule n pentru care 3n  7n  8n  4n  5n  9n .
2. Să se arate că 4n  5n  9n nu este pătrat perfect oricare ar fi numărul natural nenul n .

Nicolae Papacu, Slobozia


Subiectul III.
Se consideră triunghiul ABC în care m  ABC   800 , m  ACB   700 . Fie D  Int  ABC 
astfel încât triunghiul BCD este echilateral. Notăm DC  AB  E . Să se arate că ED  EA .
Sorana Ionescu, Slobozia

Clasa a VIII-a

Subiectul I.
Să se arate că există o infinitate de valori ale numărului natural n pentru care numărul
2017n 17 este divizibil cu 20.
Nicolae Papacu, Slobozia
Subiectul II.
BT 1
Fie ABCDA1B1C1D1 un cub și punctul T   BB1  astfel încât  . Fie M   D1T   ABC  .
TB1 2
Știind că aria triunghiului ACM este egală cu 36cm2 , calculați distanța de la punctul C la
planul  AC1M  .
Daniela Tilincă, Adriana Mihăila, Brăila, Supliment G.M. nr. 2/2017
Subiectul III.
Fie numerele a, b, c strict pozitive. Să se demonstreze inegalitatea:
ab bc ca ab  bc  ca
 2  2 1  4  2 2 2 .
a  ab  b
2 2
b  bc  c 2
c  ca  a 2
a b c
Costel Anghel, Slatina

87
Clasa a IX-a
Subiectul I.

Fie x, y, z  0 . Să se demonstreze că x2  y2  y 2
 
 z 2 z 2  x2  xyz  x  y  y  z  z  x  .
Florin Rotaru, Focșani
Subiectul II.
Se consideră triunghiul ABC de laturi BC  a , CA  b , AB  c și punctele A  (BC) ,
B  CA , C  ( AB) , astfel încât BA  CB  AC  x . Să se precizeze, în ce condiții există
x  0 astfel încât aria triunghiului ABC să fie minimă și în aceste condiții, să se calculeze x
în funcție de a, b, c .
Benedict Niculescu, București, G.M. nr. 3/2017
Subiectul III.
x 1 y 1 z 1
Demonstrați inegalitatea    6 unde x, y, z sunt numere reale strict
x  yz y  zx z  xy
pozitive care satisfac relația x  y  z  1 .
Vasile Mircea Popa, Sibiu

Clasa a X-a

Subiectul I.
Fie z1, z2 , z3 , z4 njumere complexe astfel încât z1  z2  z1  z2 și z3  z4  z3  z4 . Să se
arate că z13  z23 și z36  z46 .
Marcel Țena, București
Subiectul II.
x 2  5x
Rezolvați în mulțimea numerelor reale ecuația 3x 5x 
2
x
 1.
3
Vasile Șteopoaie, Sângeorz - Băi
Subiectul III.
   
a) Demonstrați că an2  bn2   a  b  an1  bn1  ab an  bn , pentru orice a, b și

n .

 
2
b) Demonstrați că sin14 x  cos14 x  1 7sin2 x cos2 x 1  sin2 x cos2 x .
169 6  
c) Rezolvați ecuația sin14 x  cos14 x  cos  2x  în intervalul  0, 2 
64  
Marius Iosifescu (adaptare)
Clasa a XI-a

Subiectul I.
5 x   2x 
Se consideră șirul  xn n1 , definit prin x1  0 , xn1  n n
, pentru orice n  1. Să se
3 x   4x n n
arate că șirul este convergent și să se calculeze lim xn .
n
Inocențiu Drăghicescu, București

88
Subiectul II.
1  an
Considerăm șirurile de numere reale  an n1 , bn n1 , unde a1  b1 
1
și an1  ,
2 2
1  an
bn1  , pentru n  1. Consideră de asemenea șirul de matrice  An n1 , unde
2
a bn 
An   n  . Se cere:
 bn an 
i. Să se calculeze lim an și lim bn .
n n
 
ii. Să se arate că pentru orice n  există k  astfel încât Ank  I2 .

iii. Notând cu kn cel mai mic număr k  pentru care Ank  I2 să se calculeze lim knbn .
n
Marcel Țena, București
Subiectul III.
Să se determine funcțiile derivabile f :   0,  cu proprietatea că tangenta la graficul
 
funcției în fiecare punct M x0 , f  x0  intersectează Ox în punctul M  x0 1,0 .
Andrei Vernescu, București

Clasa a XII-a
Subiectul I.
1  x ln  2  x  dx .
1n
Să se calculeze lim 
n 0
Ion Nedelcu, Ploiești, G.M. nr. 11/2016

Subiectul II. Fie L un corp comutativ și K  L un alt corp, operațiile din K fiind induse de
cele din L . Presupunem că orice polinom f  K  X  , este ireductibil în inelul K  X  , rămâne
ireductibil în inelul L  X  . Să se demonstreze egalitatea:

K    L h  K  X  , h  0, cu h    0 . 
Marcel Țena, București

 2
Subiectul III. Arătați că  cos(sin x)  sin(cos x) dx  4
0
2 .
Daniel Sitaru, Drobeta Turnu-Severin

Premianții concursului sunt:

Clasa a V-a

Premiul I: Costea Anamaria (Șc. Gimn. nr. 3 Slobozia)


Premiul al II-lea: Popescu Mihnea (Șc. Gimn. nr. 3 Slobozia)
Premiul al III-lea: Litu Bogdan (Șc. Gimn. ”A. Vlaicu” Fetești)

89
Mențiuni: Hughineață Rareș (Șc. Gimn. ”I.H. Rădulescu Urziceni), Agheorghesă Sebastian (Șc.
Gimn. „Sf. Andrei” Slobozia), Oprea Mircea (Șc. Gimn. ”A. Vlaicu” Fetești)

Clasa a VI-a

Premiul I: Dumitru Maria (Șc. Gimn. nr. 3 Slobozia), Toader Sebastian (Șc. Gimn. nr. 3
Slobozia)
Premiul al III-lea: Șerban Ioana (Șc. Gimn. nr. 3 Slobozia)
Mențiuni: Lazăr Razvan (Șc. Gimn. „Sf. Andrei” Slobozia).

Clasa a VII-a

Premiul I: Barbu Bianca (C.N. „M. Viteazul” Ploiești)


Premiul al II-lea: Berbece Rian (Șc. Gimn. „Sf. Andrei” Slobozia)
Premiul al III-lea: Stanca Victor (Șc. Gimn. nr. 56 București)
Mențiuni: Petre Alexandru (Șc. Gimn. nr. 3 Slobozia), Țărnică Dalia (C.N. „M. Viteazul”
Ploiești), Mihai Alexandru (C.N. „M. Viteazul” Slobozia)

Clasa a VIII-a

Premiul I: Badea Andrada (C.N. ”N. Bălcescu” Brăila), Drăgan Andrei (C.N. „M. Viteazul”
Ploiești), Tănăsescu Alexandru (C.N. „M. Viteazul” Ploiești)
Mențiuni: Ionescu Dan (Şc. Gimn. "Sf. Andrei" Slobozia), Manea Lidia (Șc. Gimn. nr. 3
Slobozia), Neacșu Liviu (C.N. „B.P. Hașdeu” Buzău), Trufaș Dafina (C.N. ”N. Bălcescu”
Brăila), Negoiță Anca (C.N. „M. Viteazul” Slobozia)

Clasa a IX-a

Premiul I: Donescu Irina (C.N. ”N. Bălcescu” Brăila)


Premiul al II-lea: Bâra Sînziana (C.N. „M. Viteazul” Slobozia)
Premiul al III-lea: Mincu Mihai (C.N. „M. Viteazul” Ploiești)
Mențiuni: Dinu Andreea (Lic. Internațional. de Informatică, Constanța), Vârlan Mihnea (C.N.
”N. Grigorescu” Câmpina), Nicoară Laura (Lic. Internațional. de Informatică, Constanța)

Clasa a X-a

Premiul I: Dumitru Cătălin (C. N. „B.P. Hașdeu” Buzău)


Premiul al II-lea: Ovreiu Auraș (Lic. Teoretic ”Carol I” Fetești)
Premiul al III-lea: Ducaru Andrei (C.N. "Sf. Sava" București)
Mențiuni: Gheorghe Antonia (C.N. „I.L. Caragiale” Ploiești), Staicu Astrid Diana (C.N. "Sf.
Sava" București), Stănescu Andrei (C.N. „M. Viteazul” Slobozia), Săceleanu Andrei (Lic.
Teoretic ”Carol I” Fetești), Buf Sorina (C.N. „M. Viteazul” Slobozia)

Clasa a XI-a

Premiul I: Bonifate Tudor (C.N. „I.L. Caragiale” Ploiești)


Premiul al II-lea: Voinea Eduard (C.N. „M. Viteazul” Ploiești)
Premiul al III-lea: Vlad Mihai (C.N. „B. Știrbei” Călărași)
90
Mențiuni: Popa Iulia (C.N. „M. Viteazul” Slobozia), Neculai Andreea (Lic. Teoretic ”Carol I”
Fetești), Bițineanu Virginia (C.N. „Gr. Moisil” Slobozia)

Clasa a XII-a

Premiul I: Constantin Maria (C.N. „M. Viteazul” Ploiești), Lazăr Andreea (C.N. „M. Viteazul”
Slobozia)
Premiul al III-lea: Dinu Sabina (C.N. „M. Viteazul” Slobozia), Ioniță Cristian (C.N. „M.
Viteazul” Slobozia) Sava Ștefan (C. N. Militar „D. Cantemir” Breaza)

Ediția a XXIV-a, 4-6 mai 2018

Subiecte

Clasa a V-a

Subiectul I. Determinați numărul ̅̅̅̅̅̅̅


𝑎𝑏𝑐𝑑 știind că ̅̅̅̅̅̅̅
𝑑𝑐𝑏𝑎 = 2 ∙ ̅̅̅̅̅̅̅
𝑎𝑏𝑐𝑑 +3068.
Ion Neață, Slatina, G.M., nr. 9/2017
Subiectul II. Considerăm 900 de numere naturale nenule a căror sumă este 45449. Arătați că cel
puțin 10 dintre aceste numere sunt egale.
Marcel Popescu, Slobozia
Subiectul III. Determinați forma generală a numerelor naturale care împărțite la 8 dau restul 5,
împărțite la 9 dau restul 6 și împărțite la 10 dau restul 7. Determinați cel mai mare număr de
cinci cifre cu proprietățile de mai sus.
Marcel Popescu, Slobozia

Clasa a VI-a

Subiectul I. Arătați că oricum am alege trei puteri cu baza 2, există două dintre ele care au suma
sau diferența divizibilă cu 5.
Traian Preda, București, G.M., nr. 2/2018
𝑚 𝑛 𝑝
Subiectul II. Să se determine numerele prime distincte 𝑚, 𝑛, 𝑝, 𝑞 astfel încât 𝑛+1 = 𝑝+1 = 𝑞+3 =
𝑞
.
𝑚+12
Marian Mărgărit, Slobozia
Subiectul III. Se consideră triunghiul ascuțitunghic 𝐴𝐵𝐶 și unghiurile drepte 𝐵𝐴𝐷 ̂ și 𝐶𝐴𝐸 ̂ astfel
încât 𝐵 și 𝐸 se află de-o parte și de alta a dreptei 𝐴𝐶, iar 𝐶 și 𝐷 se află de-o parte și de alta a
dreptei 𝐴𝐵.
a) Dacă 𝐴𝐸 ∩ 𝐷𝐶 = {𝐹}, 𝐵𝐸 ∩ 𝐴𝐶 = {𝐺} și 𝐵𝐸 ⊥ 𝐶𝐷, atunci arătați că 𝐹𝐺 ⊥ 𝐶𝐸.
b) Dacă 𝐵𝐷 ∩ 𝐶𝐸 = {𝐻}, 𝐵𝐸 ⊥ 𝐶𝐷 și 𝐵𝐸 ≡ 𝐶𝐷, atunci arătați că unghiurile 𝐵𝐴𝐶 ̂ și 𝐵𝐻𝐶 ̂ sunt
complementare.
Marian Mărgărit, Slobozia
Clasa a VII-a
Subiectul I. Rezolvați în mulțimea numerelor naturale ecuația 29𝑥 + 20𝑥 = 𝑦 2 .
Nicolae Papacu, Slobozia

91
Subiectul II. Fie triunghiul dreptunghic ABC cu 𝑚(∢𝐴) = 900 . Se consideră bisectoarea (AD a
unghiului A, 𝐷 ∈ (𝐵𝐶). Notăm cu r1 raza cercului înscris în triunghiul ABD și cu r2 raza cercului
1 1 1 1
înscris în triunghiul ACD. Arătați că 𝑟 − 𝑟 = √2 (𝑐 − 𝑏), unde b = AC și c = AB.
1 2
Constantin Petrea, Pașcani
Subiectul III. Rezolvați în mulțimea numerelor întregi ecuația 𝑥 𝑦 + 𝑥 2 + 𝑦 2 + 1 = 4𝑥𝑦.
2 2

Victor Săceanu, Drobeta Turnu Severin, Supliment G.M., nr. 3/2018

Clasa a VIII-a
𝑥𝑦 4(𝑥+𝑦)2
Subiectul I. Dacă 𝑥 și 𝑦 sunt numere reale astfel încât 𝑥𝑦 > 0, să se arate că (𝑥+𝑦)2
+ ≥
𝑥𝑦
65

4
Artur Bălăucă, Botoșani
Subiectul II. Să se arate că dacă 𝑥, 𝑦, 𝑧 sunt numere întregi distincte două câte două, astfel încât
𝑥𝑦 + 𝑦𝑧 + 𝑧𝑥 = 26, atunci 𝑥 2 + 𝑦 2 + 𝑧 2 ≥ 29.
Costin Negrii, București
Subiectul III. Fie VABC o piramidă regulată cu baza triunghiul echilateral ABC și fie M mijlocul
laturii (BC). Dacă triunghiul BMV este isoscel, determinați măsura unghiului dintre dreapta AV și
planul (VBC).
Dorina și Vladimir Baban, Supliment G.M. nr. 3/2018

Clasa a IX-a

Subiectul I. Să se determine mulțimea tripletelor de numere reale (𝑥, 𝑦, 𝑧) care verifică sistemul
𝑧 = 𝑥 + {𝑦}
{𝑥 = 𝑦 + 3{𝑧}
𝑦 = 𝑧 − 2{𝑥}
Inocențiu Drăghicescu, București
Subiectul II. Pe laturile triunghiului ABC se consideră punctele A’∈(BC), B’∈(CA), C’∈(AB)
astfel încât ariile triunghiurilor AB’C’, BC’A’, CA’B’ sunt egale. Să se demonstreze inegalitatea
1
𝑎𝑟𝑖𝑎(𝐴′𝐵′𝐶′) ≥ 4 𝑎𝑟𝑖𝑎(𝐴𝐵𝐶).
În ce caz avem egalitate?
Inocențiu Drăghicescu, București
Subiectul III. Fie 𝑎, 𝑏, 𝑐 numere naturale nenule astfel încât 𝑎 + 2𝑏 + 4𝑐 = 2017. Arătați că
ecuația 𝑎𝑥 2 + 𝑏𝑥 + 𝑐 = 0 nu are rădăcini raționale.
Gheorghe Stoica, Petroșani, Supliment G.M. nr. 12/2017

Clasa a X-a

log 𝑎+1(𝑥 + 1) = log 𝑎 𝑦


Subiectul I. Să se rezolve în mulțimea numerelor reale sistemul {log 𝑎+1(𝑦 + 1) = log 𝑎 𝑧 ,
log 𝑎+1 (𝑧 + 1) = log 𝑎 𝑥
unde 𝑎 ∈ (0,1) ∪ (1, ∞).
Nicolae Papacu, Slobozia

92
Subiectul II. Pentru 𝑛 ≥ 3, 𝑛 număr natural, se consideră numerele complexe distincte două câte
două 𝑧1 , 𝑧2 , 𝑧3 , … , 𝑧𝑛 . Să se arate că există un număr complex 𝑧, cu proprietatea |𝑧| = 1, astfel
încât ∑𝑛𝑖=1|𝑧 − 𝑧𝑖 | ≥ 𝑛.
Dan Nedeianu, Drobeta Turnu Severin
𝑛 𝑛
Subiectul III. Să se arate că √√2018 + √2017 + √√2018 − √2017 este număr irațional,
oricare ar fi 𝑛 ≥ 2, 𝑛 număr natural.
Liviu Smarandache, Craiova, G.M., nr. 11/2017

Clasa a XI-a
Subiectul I. Dacă 𝐴 ∈ 𝑀2 (ℤ) este o matrice cu proprietatea că mulțimea 𝑀 = {𝐴𝑛 | 𝑛 ∈ ℕ∗ } este
finită, demonstrați că această mulțime are cel mult 6 elemente.
Marcel Țena, București
𝑥𝑛+1
Subiectul II. Se consideră șirul (𝑥𝑛 )𝑛≥1 definit prin 𝑥1 = 1 și 𝑛2 = 1 + 3√𝑥𝑛 pentru orice
3
√𝑥𝑛+1 1+ 3√𝑥𝑛
𝑛 ∈ ℕ∗ . Să se calculeze lim𝑛→∞ 𝑛 și lim𝑛→∞ .
√𝑛! 𝑛
Florin Rotaru, Focșani
𝑘
Subiectul III. Fie șirul (𝑎𝑛 )𝑛≥1 definit prin 𝑎𝑛 = ∑𝑛𝑘=1 𝑘 4 +𝑘 2 +1 , 𝑛 ∈ ℕ∗ . Să se calculeze
lim𝑛→∞ ∑𝑛𝑘=1 𝑎𝑟𝑐𝑡𝑔(1 − 2𝑎𝑘 ).
Traian Tămâian, Carei, Supliment G.M., nr. 10/2017

Clasa a XII-a
𝜋
𝑎 𝑠𝑖𝑛𝑛 𝑥+𝑏𝑐𝑜𝑠𝑛 𝑥
Subiectul I. Să se calculeze integrala ∫0 2
(𝑠𝑖𝑛 𝑥+𝑐𝑜𝑠 𝑥)𝑛+2
𝑑𝑥, unde 𝑎, 𝑏 sunt numere reale și 𝑛 este
număr natural nenul.
Inocențiu Drăghicescu, București
Subiectul II. Fie 𝑝 un număr prim și ℤ𝑝 [𝑋] inelul polinoamelor cu coeficienți în corpul ℤ𝑝 .
Pentru fiecare 𝑢 ∈ ℤ𝑝 [𝑋] considerăm funcțiile 𝐷𝑢 , 𝑆𝑢 : ℤ𝑝 [𝑋] → ℤ𝑝 [𝑋], unde 𝐷𝑢 (𝑓) =
𝑓(𝑢), 𝑆𝑢 (𝑓) = 𝑢(𝑓), pentru orice 𝑓 ∈ ℤ𝑝 [𝑋]. Să se arate că:
a) Orice endomorfism al inelului ℤ𝑝 [𝑋] este o funcție 𝐷𝑢 , cu 𝑢 ∈ ℤ𝑝 [𝑋].
b) Orice automorfism al inelului ℤ𝑝 [𝑋] este de forma 𝐷𝑢 , cu 𝑔𝑟𝑎𝑑 𝑢 = 1.
c) Pentru 𝑢 = 𝑋 𝑝 , funcția 𝑆𝑢 este un endomorfism al inelului ℤ𝑝 [𝑋].
Marcel Țena, București
1 22018 −1
Subiectul III. Fie 𝑓: [0,1] → ℝ o funcție continuă cu ∫0 𝑓(𝑥)𝑑𝑥 = 2018 . Să se arate că există
𝑎 ∈ (0,1) cu 𝑓(𝑎) = (𝑎 + 1)2017 .
Florin Rotaru, Focșani, G.M., nr. 1/2018

Premianții concursului sunt:


Clasa a V-a
Premiul I: Boiangiu Ana (Șc. Gimn. ”Al. Ciucurencu” Tulcea)
Premiul al II-lea: Bîzgă Nicolescu Carmen (C.N. ”Z. Golescu” Pitești)
Premiul al III-lea: Papacioc Rareș (Șc. Gimn. nr. 3 Slobozia)
Mențiuni: Mușat Delia Andrada (Șc. Gimn. „Sf. Andrei” Slobozia), Negoescu Cîrstea Andrei
Daniel (C.N. ”Z. Golescu” Pitești), Șerban Rareș Mario (Șc. Gimn. „Sf. Andrei” Slobozia)

93
Clasa a VI-a
Premiul I: Popescu Mihnea (Șc. Gimn. nr. 3 Slobozia)
Premiul al II-lea: Litu Bogdan (Șc. Gimn. ”A. Vlaicu” Fetești)
Premiul al III-lea: Agheorghesă Sebastian (Șc. Gimn. „Sf. Andrei” Slobozia)
Mențiuni: Ion Diana (Șc. Gimn. ”I.H. Rădulescu Urziceni)

Clasa a VII-a
Premiul I: Toader Sebastian (Șc. Gimn. nr. 3 Slobozia)
Premiul al II-lea: Oprea Horațiu (C.N. ”M. Viteazul” Ploiești)
Premiul al III-lea: Donțu Alexandru (C.N. ”M. Viteazul” Ploiești)
Mențiuni: Dumitru Maria (Șc. Gimn. nr. 3 Slobozia), Ruse Petru (C.N. ”M. Viteazul” Ploiești),
Paraschiv Miruna Maria (Lic. Teoretic ”Gr. Moisil” Tulcea), Grădinaru Adela Victoria (Lic.
Teoretic ”Gr. Moisil” Tulcea),

Clasa a VIII-a
Premiul I: Biteș Rareș (Șc. Gimn. ”I.H. Rădulescu Urziceni)
Premiul al II-lea: Voicu Oana Georgiana (Șc. Gimn. ”Traian” Pitești)
Premiul al III-lea: Berbece Rian (Șc. Gimn. „Sf. Andrei” Slobozia), Ghiță Alexandru (C.N. „M.
Viteazul” Ploiești)
Mențiuni: Petre Alexandru (Șc. Gimn. nr. 3 Slobozia)

Clasa a IX-a
Premiul I: Ionescu Dan (C.N. „M. Viteazul” Slobozia)
Premiul al II-lea: Neacșu Miclea Liviu Ștefan (C.N. „B.P. Hașdeu” Buzău)
Premiul al III-lea: Manea Lidia (C.N. „M. Viteazul” Slobozia)
Mențiuni: Negoiță Anca (C.N. „M. Viteazul” Slobozia), Gaiță Ștefan Andrei (C.N. ”Al. I. Cuza”
Ploiești), Șcheaua Andreea (C.N. „M. Viteazul” Slobozia)

Clasa a X-a
Premiul I: Vârlan Mihnea (C.N. ”N. Grigorescu” Câmpina)
Premiul al II-lea: Donescu Irina (C.N. ”N. Bălcescu” Brăila)
Premiul al III-lea: Mincu Mihai (C.N. „M. Viteazul” Ploiești)
Mențiuni: Bâra Sînziana (C.N. „M. Viteazul” Slobozia)

Clasa a XI-a
Premiul I: Ovreiu Auraș (Lic. Teoretic ”Carol I” Fetești)
Premiul al II-lea: Dumitru Cătălin (C. N. „B.P. Hașdeu” Buzău)
Premiul al III-lea: Buf Sorina (C.N. „M. Viteazul” Slobozia)
Mențiuni: Nastasia Alexandru (C.N. „M. Viteazul” Slobozia), Moraru Mircea (C.N. ”Gh.
Șincai” București), Săceleanu Andrei (Lic. Teoretic ”Carol I” Fetești), Ducaru Andrei (C.N. "Sf.
Sava" București)

Clasa a XII-a
Premiul I: Bâra Andrei Robert (Lic. Teoretic Internațional de Informatică Constanța)
Premiul al II-lea: Miclăuș Adrian (C.N. ”Barbu Știrbei” Călărași)
Premiul al III-lea: Bazavan Cristian (C. N. „Spiru Haret” București), Popa Iulia (C.N. „M.
Viteazul” Slobozia)
Mențiuni: Vlad Radu Mihai (C.N. ”Barbu Știrbei” Călărași)
94
PARTEA A II-A

REZOLĂVRI
Ediţia a II-a, 8-10 noiembrie 1996
Clasa a VII-a

I.  x  a 2  b
b
Dacă x  a  0  2   , contradicţie. Avem deci x-a=0 şi b=0.
x a
Discuţie:
Pentru b≠0  ecuaţia nu are soluţie în .
Pentru b=0  ecuaţia are soluţia x=a în .

II. Fie AE ∩ DF={O}.

A F B

C
D E
Din AB II CD, AE secantă  EAB  AED (1).
Din [AE bisectoare a unghiului DAB  EAB  EAD (2).
Din (1) şi (2)  AED  EAD, deci ADE este isoscel şi cum [DO este bisectoare a
unghiului ADE  DO mediană, adică AO=OE (3).
Analog DO=OF (4).
Din (3) şi (4)  AFED paralelogram şi cum AE=DF  AFED dreptunghi  AD  AB.

III. Notând x-y=z, ecuaţia se scrie z2 =2(y+z-y2) sau y2+(y-1)2+(z-1)2=2 şi cum y,z   y=0,
z-1=  1 sau y=1, z-1=  1. Așadar y=0, z∈ 0,2 sau y=1,z∈ 0,2 . În final, mulțimea soluţiilor
ecuaţiei este 0,0,2,0,1,1,3,1 .
Clasa a VIII-a

1 1 1 1 1 1 1 1 1 1 1 1 1 1
I. (  )           evidentă.
2a 2a 2a 2  a a a  2a 2a 2a 2  a a a 

95
( )
1 1 1 1 1 1 1 1 1 1 1 1 1 1 1 1 1 1 1
                  
b c c a a  b 2a b c  b c c a a b 4 b c  4c a  4a b 
1 1 1 1
Arătăm că     1 , cu egalitate pentru b=c.
bc 4b c 
Într-adevăr, (1) se scrie succesiv
1 bc
  4bc   b  c    b  c   0 , cu egalitate pentru b=c.
2 2

b  c 4bc
Analog avem şi
1 1 1 1
     2 , cu egalitate pentru c=a.
c a 4c a
1 1 1 1
    3 , cu egalitate pentru a=b.
ab 4a b
Adunând (1), (2), (3) avem
1 1 1 1 1 1 1 1 1 1 1 1
              , care, conform ipotezei devine
bc c a ab 4b c  4c a 4a b
egalitate  a=b=c.

II. Din x, y, z  avem că relaţia de arătat se mai scrie


 x  zy   z  yx   y  xz2  1 x2  1 y 2  1 z2  1
2 2 2 2 2

  
Dar 1 x2 1 y 2  1 x2  y 2  x2 y 2  z2  2xyz  x2 y 2   z  xy  şi analog
2

1 y 1 z    x  yz și 1 z 1 x    y  zx  , care înmulţite dau relaţia (1).
2 2 2 2 2 2

III. Fie M,N,P mijloacele laturilor BC, CA, AB. Notăm AB=c, BC=a, CA=b, OA=OB=OC=m.

N
A C
P M

B
Folosind teorema medianei şi linia mijlocie în triunghi, avem că

OM  OP  OM  OP  MP  2 2 2 
2 OB2  OC2  BC2 
 
2 OA2  OB2  AB2

AC2
4 4 4
 4m2  a2  4m2  c2  b2  a2  b2  c 2  8m2 .
Analog OM  ON  a2  b2  c 2  8m2 , ON  OP  a2  b2  c 2  8m2
şi deci OM  OP  OM  ON  ON  OP şi cerinţa problemei este imediată.

96
Clasa a IX-a

I. Din a,b  *

 , 8a  b    0,1,2,...,a  b este echivalentă cu
a  b  8a  b  a  b  1. Avem a  b  8a  b  a  b  1 care după calcule devine
2 2

a a  2b  8  b  b  1  0 1

 8a  b  a  b  1  2
2

Din a,b  *  a  0; b(b-1)  0 şi cu (1)  a+2b-8  0, cu soluţiile (1,1), (1,2), (1,3), (2,1), (2,2),
(3,1), (3,2), (4,1), (5,1). Dintre acestea, varianta a=2, b=1 nu verifică (2), deci rămân soluţii
perechile (1,1), (1,2), (1,3), (2,2), (3,1), (3,2), (4,1), (5,1).

II. Fie Q mijlocul lui [AC] şi O centrul cercului circumscris triunghiului ABC.

Q
P

O
B A
M

OP  BC,NB  BC  OP BN OP  BC,NB  BC  OP BN
OQ  AC,NM  AC  OQ NM. OQ  AC,NM  AC  OQ NM
AB
Din OP BN , OQ NM şi PQ  BM   POQ  BNM U.LU . . 
2
PO=BN şi cum PO  PB,BN  PB  PONB dreptunghi, de unde NP=BO=R= raza cercului
circumscris triunghiului ABC.

III. a) Fie N exterior pătratului ABCD astfel ca MB=BN= 2 , MB  BN .

A B

D C

97
Cum şi AB  BC  ABM  CBN  ABM  CBN (L.U.L.)  CN=AM=1 şi cum
MN2  MB2  BN2  2  2  4  3  1  MC2  CN2  MC  CN , de unde avem că MCNB
   
este patrulater inscriptibil. Atunci m MCB  m MNB  450  M  AC şi deci MB=MD.

b) Fie a=AB  AC  a 2,1 3  a 2


1 3 2 6 1 3 2 6
a  a 
2 2 2 2

Clasa a X-a

x x
 5   12 
I. Ecuaţia se mai scrie:       1.
 13   13 
x x
 5   12 
Fie f :  ,f  x        care este strict descrescătoare, fiind sumă de funcţii strict
 13   13 
2 2
 5   12 
descrescătoare  f injectivă. Cum f  2        1 şi ecuaţia se scrie f(x)=f(2)  x=2
 13   13 
este soluţie unică.

II. Notăm:
   
m OBC  m OCB  x

m OAC   m OCA  y şi avem x  y  C, y  z  A, x  z  B .

m OAB   m OBA  z

B C

Cum A  B  C  1800 , avem imediat că x  90o  A, y  90o  B, z  90o  C .



o
 o

tgBAO  tgCAO tg 90  C  tg 90  B ctgC  ctgB

 
sin2 A sin2 A sin2 A
sin B  C  sin A 1
= 2  2  .
sin A sin B sinC sin A sin B sinC sin A sin B sinC
Analog avem şi

98
tgCBO  tgABO tgACO  tgBCO 1
2
 2
 .
sin B sin C sin A sinB sinC

III.. Soluţia 1
Fie 𝑎 = 𝑐𝑜𝑠𝜑1 + 𝑠𝑖𝑛𝜑1 , 𝑏 = 𝑐𝑜𝑠𝜑2 + 𝑠𝑖𝑛𝜑2 , 𝑐 = cos 3 + sin 3 , cu
1,2 ,3  [0,2 ),1,2 ,3 distincte și z=cos   i sin,  [0,2 ) .
(cos  cos1 )2  (sin  sin1 )2  (cos  cos2 )2  (sin  sin2 )2 
(cos  cos3 )2  (sin  sin3 )2  6 
 2(cos cos1  sin sin1  cos cos2  sin sin2  cos cos3  sin sin3 )  0
adică (cos1  cos2  cos3 )cos  (sin1  sin2  sin3 )sin  0,   [0.2 )
Cum putem alege  astfel încât
cos  0,cos  0;sin  0,sin  0  cos1  cos2  cos3  0,
sin1  sin2  sin3  0,
adică 𝑎 + 𝑏 + 𝑐 = 0 şi cum |a| = |b| = |c|=1  a,b,c sunt afixele vârfurilor unui triunghi
echilateral înscris în cercul C(0,1), deci a  ,| a | 1; b  a ,c  a 2 , unde  2    1  0 .

Soluţia 2
Din |a| = |b| = |c|=1  a, b, c sunt afixele punctelor A, B, C de pe cercul C(0,1). Trebuie sa
avem: MA2  MB2  MC2  6, M C(0,1) (1)
Fie H ortocentrul triunghiului ABC.
MA2  MA2  (MO  OA)2  MO2  OA2  2MO  OA  2  2MO  OA .
Analog MB2  2  2MO  OB,MC2  2  2MO  OC şi atunci (1) devine
2MO(OA  OB  OC)  0,()M C(o,1)
2MO  OH  0,()M C(0,1) (2)
Dacă H  O, atunci putem alege M  C(0,1) astfel ca să avem m(MOH )  90o şi atunci (2) este
falsă.
În concluzie, avem că H=O, adică ortocentrul lui ABC coincide cu centrul cercului circumscris
lui ABC  ABC este triunghi echilateral.

Clasa a XI-a

kxk 1 1 1 n1 1 1 n1


1
I. 1) xk 1      (  ) 
k  xk xk 1 xk k k 1 xk 1 xk k 1 k

1 1 1 1 1 x1
     ...  , de unde xn  .
xn x1 1 2 n 1  1 1 
1 2  ...  n  1 x1  1
 
1 1 1 1
2) Fie yn     ...  . Din
1 2 3 n

99
1  1 1  1 1 1 
y 2n  1      ...   n1  n1  ...  n1 n1  
2 3 4  2 1 2  2 2 2 
1  1 1  1 1 1 1 2 2n1 n
 1      ...   n  n  ...  n   1   ...  n  1
2 4 4 2 2 2  2 4 2 2
avem că lim y 2n   şi cum  yn n1 este strict crescător avem că lim yn   .
n n

x1
xn   lim x  0 .
yn1x1  1 n n

 a11 a12  b b  x y
II. Fie A    ,B   11 12 , X   .
 a21 a22   b21 b22  z t 
a  x a12  y a11  x a12  y
A  X   11   2 a22 x  a21y  a12 z  a11t 
a21  z a22  t a21  z a22  t
1) A  X   2  a22 x   a21y   a12 z   a11t     2 a22 x  a21y  a12 z  a11t   A  X , 2)
 X  M2     A  A
AB  x   2((a22  b22 )x  (a21  b21 )y  (a12  b12 )z  (a11  b11 )t ) 
 2(a22 x  a21y  a12 z  a11t )  2(b22 x  b21y  b12z  b11t ) 
 A  x   B  x , x  M2    AB  A  B
3) A  x1A  y1I2 , alegem x1  1, y1  0
A2  a11  a22  A  a11a22  a12a21  I2 , de unde cu 1),2)
A  a
2
11 a22 A  a11a22 a12a21I2  a11  a22 A  a11a22  a12a21 I2 , alegem
x2  a11  a22 , y2   a11a22  a12a21 
Din An  a11  a22  An1  a11a22  a12a21  An2 avem
A  a11  a22 A  a11a22  a12a21 A
n n 1 n 2 de unde
xn  a11  a22  xn1  a11a22  a12a21  xn2
yn  a11  a22  y n1  a11a22  a12a21  y n2

III.
0 1 0 0
0 0 1 0 
A  I4  B, unde B  
0 0 0 1
 
0 0 0 0
0 0 1 0 0 0 0 1
0 0 0 
1 0 0 0 0 
B 
2
, B 
3
, B4  B5  ...  O4
0 0 0 0 0 0 0 0
   
0 0 0 0 0 0 0 0

100
n  n  1 2 n  n  1 n  2 3
An  I4  B   Cn0I4  Cn1B  Cn2B2  Cn3B3  I4  nB  B  B 
n

2 6
 n  n  1 n  n  1 n  2 
1 n 
 2 6 
 n  n  1 
 0 1 n .
 2 
0 0 1 n 
 
0 0 0 1 

Clasa a XII-a

,
 1
 x 
I. I   2
1
dx    2
dx.
x  x a  1
2
1
x  2 a 4
 
1 1
x  a
 1 1 2 4
a) a   ,   I  ln  C.
 4 1 1 1
2 a x   a
4 2 4
1 1
b) a   I    C.
4 1
x
2
1
x
1  1 2  C.
c) a   ,    I  arctg
4  1 1
a a
4 4

II. Pentru x   ,0 sau x  0,  avem


1  x n1
,
1 1 1  1 1 1
 f  x  dx   sin x sin x2 ...sin xn dx    cos xn   n sin x sin x2 ...sin xn1 dx 
1 x n1 1 1 1 1 n 1 n 1 1 1
 cos n  sin sin 2 ...sin n1   cos n  [ x sin sin 2 ...sin n1 
x n x x x x n x x x
n 1 n 1
x 1 1 1 k 1 1 1
 
n k 1 x
sin sin 2 ...sin n1  k 1 cos k sin k 1 ... sin n1 ]dx.
x x x x x x
Fie g :  ,

101
 1 n  1 n1 1 1 1 x n n1 1 1 1 k
 x cos
x n
(
n
x sin
x
sin
x 2
...sin
x n 1
  sin sin 2 ...sin n1  k 1 
n k 1 x x x x

 1 1 1
g x    cos k sin k 1 ...sin n1 ), pentru x  0
 x x x
 0, pentru x  0


Se observă că g este continuă şi deci g are primitive; fie G :  o primitivă a sa.
Considerăm F :  ,
 1 x n1 1 1 1
 cos  sin sin 2 ...sin n1  G  x   c1, pentru x  0
F x   x n
n x x x
 c2 , pentru x  0

Impunem F continuă în 0  G 0  c1  c2  k.
 1 x n1 1 1 1
 cos  sin sin 2 ...sin n1  G  x   G 0  k, pentru x  0
F x   x n
n x x x
 k, pentru x  0

F  x   F  0  1 xn 1 1 1 G  x   G  0 
F ' 0  lim  lim  cos n   sin sin 2 ...sin n1   
x 0 x 0 x 0
 x n x x x x 0 
 G '(0)  g(0)  0 (1)
F '( x )  f ( x ),  x   0 (2)
Din (1), (2)  f are primitive dacă şi numai dacă   0 . În acest caz F este o primitivă a
lui f.

III. Alegem T=X. Definim  : X  F( X ) astfel:


def
a  X   a : X  X,  a  x   a  x,  x  X.
Arătăm că  este injectivă. Fie a,b  X cu  a    b   ae    be , adică
a  e  b  e  a  b ( e este elementul neutru al monoidului).
 a  b    a   b ,  a, b  X
Arătăm că
 a  b  x    a  b x    ab  x   a  b  x   a  b  x 
  a  b x ,  x  X   a   b   a  b.

102
Ediţia a III-a, 31 octombrie – 2 noiembrie 1997
Clasa a VII-a

I.
Y

D
b
C

O B A X

Din OAD  ODA  OA  OD  a  b


ab
CB mediană în OAC  OB  BA  .
2

II.

      1 1
 
m AIC  180o  m IAC  m ICA  180o  m BAC  m ABC
2 2
 
m  A m B  m C  m  A  m B   m C 

o
180 60o
      m B  60o 
n n 1 n2 3  n  1 3  n  1 n 1

  1
 m AIC  90o   60o  120o.
2
A

B C

III.
 24xx3yy  21  8x  2y  2x  3y  6x  5y  yx  56 .
4x  y 20k  6k 14k 1
  yx  56  5x  y6  k  x  5k, y  6k     .
2x  3y 10k  18k 28k 2

103
Clasa a VIII-a

I. Fie B ' D  A'C, D  A'C şi C ' E  A' B, E  A' B .


AB '2  A 'C '2  AB '2  A ' B2  BC '2  2A ' B  BE  AB '2  A ' B2  BC '2  2A ' B  BC 'sin B 1
AC '2  A ' B '2  AC '2  A 'C2  B 'C2  2A 'C  CD  AC '2  A 'C2  B 'C2  2A 'C  B 'C sinC 2
AB 2 AC 2
Dar AB’=B’C, A’B=A’C, BC’=AC’ şi BC 'sin B  sin B  sinC  B 'C sinC .
2 2
Acum din (1) şi (2) 
 AB '2  A 'C '2  AC '2  A ' B '2 de unde avem că AA '  B 'C ' .
A B’

C’

D
E

C
B

A

II. Fie O centrul pătratului şi L mijlocul lui [AB].


D M A

P O L

C N B

AB
OM  ML  OL  (raze în cele două cercuri)  OLM este triunghi echilateral 
2
   
m OLM  60o  m MPL  90o  60o  30o . Analog m NPL  30o   
 
 m MPN  60o şi cum MP=PN  PML  PNL   MPN echilateral.
I .U.

 

104
m
III. a) Presupunem că ; m, n  * , m, n   1.
5
n
5n  m  5 | m  m  5m1, m1  *
2 2

5n2  25m12  n2  5m12  5 | n  n  5n1, n1  *

şi deci (m,n)  5, în contradicţie cu (m,n)=1.


b) Se ia o dreaptă d, A,B d astfel încât AB=2 şi în B se construieşte perpendiculara d’ pe
d; se ia C  d ' cu BC=1.
AC  AB2  BC2  5 .
d’

C
1
d
2
A B

Clasa a IX-a

I. a) Fie AB=a latura tetraedrului regulat, AM=x.


D

Q P

A B

M N

Din faptul că (MNPQ) II AB  MN II AB, PQ II AB. Din (MNPQ) II CD  MQ II CD, PN


II CD. Deci MN II PQ, MQ II PN  MNPQ paralelogram. Dar MN II AB, MQ II CD şi AB 
CD  MN  MQ şi deci MNPQ este dreptunghi.
AM MQ x MQ
b) AMQ ACD   ,   MQ  x ( se putea observa direct că AMQ este
AC CD a a
triunghi echilateral).
MCN este triunghi echilateral  MN=MC=a-x.
 x  a  x  a2
2
a
A(MNPQ)  MN  MQ  x(a  x )     cu egalitate pentru x=a-x  x  şi deci
 2  4 2
M este la mijlocul lui [AC].

105
II.
M

A N B

Fie MN  AB  MA2  MB2  MN2  NA2  MN2  NB2  NA2  NB2 


a  AB2
 NA2   AB  NA  2NA  AB  AB2  a  NA   ct.
2

2AB
Cum A este fix  N este fix.
Locul geometric al lui M este planul perpendicular pe AB care trece prin N, unde
a  AB2
N  AB cu NA  .
2AB

III. x(a+1)=2
2 2a
1) a  1  0, a  1 x  , y .
a 1 a 1
2) a+1=0, a=-1  sistemul nu are soluţii.

Clasa a X-a

I. Fie MP  BC  S, PN  BC  T  .
A B
N T

P Q

S
M
D C

Avem că (PQ este bisectoarea lui MPN  SC  BT .


SC MC
MCS MDP  
PD MD
1
BT BN
BTN DPN  
PD ND
2
Fie triunghiul BDC şi punctele coliniare M, Q, N. Cu teorema lui Menelaus avem
MC ND QB MC NB
 
MD NB QC
 1 şi cum QB  QC  
MD ND
 3
Din (1), (2), (3)  SC=BT.

106
II. Pentru n par avem cosm x  1 sinn x  1, de unde sinx=0 şi
cosm x  1  x  2k , k  pentru m impar şi x  k , k  pentru m par .

Pentru n=1  cos2m x  1 sin x  . Notăm 1+sinx=t, t  0,2  2t  t 2  


2 m
 t2

1 2


t  0, t  1 sin x  1, sin x  0 , de unde x  2k sau   2k, k  pentru m
2

impar şi x  k sau   2k , k  pentru m par.
2
Pentru m=1, n  3 , n impar  cos x  sinn x  cos x  1  sin2m x . Notăm
2

 
1 cos x  t  0,2  t 2  2t  t 2 ; t  0, t  1  cos x  1, cos x  0 , de unde
n


x  k ,   2k , k  .
2
Pentru m=2, n  3 , n impar  cos2 x  sinn x  1 

 
 sinn x  sin2 x  0  sin2 x sinn2 x  1  0   x  k , 
2
 2k , k  .
Pentru m  3, n  3 , n impar  cosm x  sinn x  cos2 x  sin2 x 

 cos2 x 1 cosm2 x   sin2 x 1 sinn2 x   0 , de unde cos x  0 sau cosm2 x  1 şi 
sin x  0 sau 
sin x  1  sin x  1 sau cosm2 x  1
 
x    2k ,2k , k  pentru m impar şi x    2k , k , k  pentru m par.
2 2

 5
III. 3x 2  5  2x  3,5  x 2  1,  , de unde  x  2,1 .
3 
Pentru  x   2  3x 2  2  x  2  5 ,
1
3x2  2x  1  0, x1  1 x2 
care nu convin.
3
1 22 22  1
Pentru [x] = 1  3x 2  2  x  1  5, 3x 2  2x  7  0, x1,2  , convine x 
3 3
.

107
Clasa a XI-a

I. Fie A’1, A’2 simetricele lui A’ faţă de A1, A2.


A

A’1 A’2

A1 A2

B C
A

Avem că A1A2 este linie mijlocie în A '1 A ' A '2 şi A '1 AA '2 este triunghi isoscel cu
AA '1  AA '2  AA ' şi m  A '1 AA '2   2m A . 
A '1 A '2 B' B'
A1A2   AA 'sin A şi analog B1B2  1 2  BB 'sin B ,
2 2
C '1 C '2
C1C2   CC 'sinC .
2
 A1A2  B1B2  CC 2 2 2

1 2    AA 'sin A  BB 'sinB  CC 'sinC   AA '  BB '  CC ' .
2 2


3 a2  b2  c 2
 b2  b2 a 3 a 2
 b2  c 
2 2

 
2
 sin A  sin B  sin C 
2 2 2
 
4 4R2 16R 2

3  81R4 3  81R2 9R 3
 2
  A1A2  B1B2  C1C2  .
16R 16 4

 a b 1
II.. Notăm 5
x2  34x  64  a, 4
x2  34x  33  b  0 şi avem sistemul  5
a  b  31
4

 a  b 1  a  b 1
  5
 b  1  b  31 b  4b  10b  10b  5b  30
5 4 4 3 2

Funcţia f : 0,   , f  x   b5  4b4  10b3  10b2  5b este strict crescătoare şi cum


f 1  30  ecuaţia f  b  30 are unica soluţie b=1  a=2, de unde
x2  34x  33  1; x1,2  17  257 .

III. Lemă
Fie ABCD patrulater strâmb, adică ABCD tetraedru şi
MA NB PC QD
M   AB, N  BC , P  CD, Q  DA astfel încât     1. Atunci
MB NC PD QA
M,N,P,Q sunt coplanare.

108
Dem.
A
A

Q’
X T
M
Q
O
D
D B
B
Z
P Y
N
C C

Fie Q '  MNP    AD şi A1,B1,C1,D1 proiecţiile punctelor A,B,C,D pe planul


(MNP).
MA NB PC Q ' D AA1 BB1 CC1 DD1 MA NB PC QD
        1 şi cum avem şi    1
MB NC PD Q ' A BB1 CC1 DD1 AA1 MB NC PD QA
Q ' D QD
   Q '  Q şi deci M,N,P,Q coplanare.
Q ' A QA
Fie acum X,Y,Z,T punctele de contact dintre o sferă şi laturile AB, BC, CD, DA ale
pătratului strâmb ABCD. Dacă O este centrul sferei
 OX  AB, OY  BC, OZ  CD, OT  DA şi OX=OY=OZ=OT= razab sferei.
AOX  AOY I.C.  XA  AT. Analog XB=BY, YC=CZ, ZD=DT. Atunci
XA YB ZC TD
    1 şi , folosind lema  X,Y,Z,T sunt coplanare.
XB YC ZD TA

Clasa a XII-a

I. A  a  b In  bB , unde
1 1 1
1 1 1 
B este matrice de ordinul n, B2  nB, B3  n2B, …, Bk  nk 1B.
 
 
1 1 1

Ak  a  b  In  bB   Ck0 a  b  In  Ck1 a  b  bB  Ck2 a  b 


k k 1 k 2
b2nB 
k

1
Ck3  a  b   Ckk bk nk 1B  a  b  In  (Ck1 a  b  bn 
k 3 k 1
b3 n2B 
k

n
1
Ck2 a  b   Ckk bk nk )B  a  b  In  a  b  bn   a  b   B.
k 2
b2 n 2 
k k k

n 
II.

109
Y
n
P
N
m

O A X

a) Dacă m II n, cum m  AM, n  AN , atunci avem că AM II AN, ceea ce este imposibil.


b) Fie m  n  P, M 0,  , N 0,   k , A a,0 cu a, k fixe,  variabil.
yM  yM   0  a
mAM      mMP 
xM  xA 0  a a 
 a
   k 
  MP  : y     x 
xP 
 , de unde  a
NP  : y    k  a  y  2  k
x  P
  k
y k y  k yP  k
 P  axP  P   yP2  4axP  k 2 .
2 2 2
Deci locul geometric al lui P este parabola de ecuaţie yP2  4axP  k 2 .

cos 3x  3   cos3x


III. f  x       f  x    este perioadă pentru f.
sin  x     sin x
Pentru a reprezenta grafic f este suficient să reprezentăm funcţia pe un interval de lungime  , fie
  
acesta  ,  .
2 2 
cos  3x  cos3x
Cum f  x     f  x   f este impară şi deci Gf este
sin  x   sin x
 
simetric faţă de origine. Ne restrângem atunci studiul lui f la  0,  .
 2
 k   
f  x   0  cos3x  0  x   , x   , 
6 3 6 2
lim f  x     x  0 asimptotă verticală.
x 0,x 0

110
3sin3x  sin x  cos3x  cos x 2sin3x  sin x  cos2x
f 'x   
sin2 x sin2 x
2cos2x  cos4x 2cos2 2x  2cos2x  1
 
sin2 x sin2 x
1 3 1 1 3
f '  x   0  cos2x   x  arccos
2 2 2
x   1 1 3 
0 arccos
6 4 2 2 2
f’(x) | - - - - - - - - - - - - - 0 + + + +
f(x) | 0 -1  0

-1

Ediţia a IV-a, 30 octombrie – 1 noiembrie 1998


Clasa a VII-a

I. a) E(a, b,c )  (a  1)2  (b  1)2  (c  1)2


b) (a  1)2  (b  1)2  (c  1)2  9; a, b, c  , 9  02  02  32  02  02  (3)2
9  12  22  22  (1)2  22  22  12  (2)2  22  (1)2  (2)2  22 
 12  (2)2  (2)2  (1)2  (2)2  (2)2
(a, b,c)  {(1,1,4),(1,4,1),(4,1,1),(1,1, 2),(1, 2,1),(2,1,1),(2,3,3),(3,2,3),(3,3,2),(0,3,3),(3,0,3),(3,3,0),
(2, 1,3),(2,3, 1),(1,2,3),(1,3,2),(3,2, 1),(3, 1,2),(0, 1,3),(0,3, 1)(1,0,3),(1,3,0),(3,0, 1),

111
(3, 1,0),(2, 1, 1),(1,2, 1),(1, 1,2),(0, 1, 1),(1,0, 1),(1, 1,0)}

 1  1   1  1 2 n 1 1 1
II. A  n,1  1 1  1 n   2  3   n  n  2 , n  \ 1
*

 2 3   
 1  1  1  1 3 2  4 (n  1)  n  1 1 n  1 1
A  n,2  1 2 1 2  1 2   2  2      ,  n  *
\ 1
 2  3   n  2 3 n2 2 n 2
1 1 1 1 1 1
Pentru k  2  1 k  1 2  0, 1 k  1 2  0, ,1 k  1 2  0 de unde
2 2 3 3 n n
1
A n, k   A n,2  , n  * \ 1 .
2

III. Fie CM  AB, CN  AD.

A A

M N
B B D
D
M N A

C B
N
M
C D

C
Avem cazurile:
1) M  BA, N  DA
ACM  ACN I.U.  CM  CN şi AM  AN 1
BCM  DCN I.C.  BM  DN  2
Din (1) şi (2)  AB=AD
2) M  BA, N  DA . Este valabilă demonstraţia de la 1).
3) M  BA, N  DA .
ACM  ACN I.U.  CM  CN
BCM  DCN I.C.  BCM  DCN
   
de unde m MCN  m BCD  90o  AMCN are

 
trei unghiuri drepte şi deci şi m BAD  90o .
4) M  BA, N  DA . Analog ca la 3).

112
Clasa a VIII-a

 1  1
 ay  1  2 a  ay  1 2a
 x  1  x  1
I. x  1,   2
 a
 y  1 2a  a  ay  a  2a2
 x  1  x  1
1 a2 1 a1 a  1 a 2a  1
x 1
 1 a  2a2 ,
x 1
  
 1
 x  1  y  3 x    \ 1
1) 1 a  0  a  1    1
 1 y 3
 y  3   1
 x  1
1 a
2) 1 a  0  a  1  2a  1 * 
x 1
Dacă a=-1 sau a=  se observă că ecuaţia  *  nu are soluţie reală.
1
2
 1 a  a
 x 1  x
1  2a  1  2a  1
Dacă a  1 şi a     
2  a  2a  1 y  1 2a
y  1 2a 
 1 a  1 a
II. x=0, y=t, z= t cu t  este soluţie a ecuaţiei x  y  z2 .
2 3 4

III. BC  a 2 . Notăm AA’=x>0.

B’

A a

x
a
A B

a a

B 'C2  a2  2a2  3a2 , A 'C2  a2  x 2 , A ' B '2  a  x   a2  2a2  2ax  x 2


2

 
a) m A ' B 'C  90o  A ' B '2  B 'C2  A 'C2  2a2  2ax  x 2  3a2  a2  x 2 
 4a2  2ax  x  2a .

113
b) A'C2  B 'C2  a2  x2  3a2  x2  2a2  x  a 2 .

Clasa a IX-a

   a  b  0
 2
I.a) Fie   A  B     3  8  0,      2
a  b   8  0

 a
b) 1o A  0  a2  4b  0  A    şi cum A\B are un singur element
 2
3 3 3
a ab a a
   8  0,   8  0  a3  64  0  a  4
4 2 4 8
2 A  0  a2  4b  0  A are 2 elemente şi cum A\B are un singur element
o

a)
 A  B    A  B  2  4  2a  b  0  4b  8a  16
a2  4b  a2  8a  16  a  4  0  a  4 .
2

M(a,b) este pe parabola a2  4b  0 sau pe dreapta 4+2a+b=0, dar M a, b  C  4,4 .

4
C

a
-4 0 4

II. ab  bc  ca 
 a  b  c
2

 a2  b2  c 2 care după calcule devin


3
 a  b  c  ab  ac  bc  0
2 2 2
 a  b    b  c   c  a   0
2 2 2
 2
2a  2b  2c  2ab  2ac  2bc  0
2 2

III.

114
A1
D’
D1 C’

A B’

O
D C

B1
A
B

C1

a) A1A' CC1, A1A'  CC1  A1A'CC


1 este paralelogram şi deci mijlocul O al segmentului
[A’C] coincide cu mijlocul segmentului  AC
1 1  1

D1A ' D ' B ' DB CB1 


  D1A ' B1C este paralelogram şi deci mijlocul O al segmentului
D1A '  D ' B '  DB  CB1 
[A’C] coincide cu mijlocul segmentului D1B1   2
Din (1) şi (2)   AC
1 1  şi D1B1  au acelaşi mijloc şi anume O, care este centrul cubului

ABCDA’B’C’D’  A1D1, BC
1 1 coplanare.

b) Din a)   A1D1,B1C1   A 'C  O şi deci relaţia de arătat devine


d 2 O, ABC   d 2 O,BCC '   d 2 O, A ' B 'C '   d 2 O,  ADD '   a2 , adică
a2 a2 a2 a2
    a2 .
4 4 4 4
AC  CB1 
c) AC  BD, BD CB1    AC  CC1B1   AC  B1C1 .
AC  CC1 

Clasa a X-a

2 a  1 2  b  1
I. Notăm 3x  a  0, 2x  b  0 şi avem   1.
b a
b  2  5b  2 3b
2a  a  b  2  3b  3b  0,   5b  2 1,2
2 2 2 a   a1  , a2  b  1.
4 2

x
3b 3 3 3
A doua soluţie nu convine pentru că a>0, b>0, de a   3x   2x      x  1
2 2  2 2
este soluţia ecuaţiei.

II. Fie z=a+bi, a,b  .

115
 
4 a2  b2  2abi  4 a  bi   3
4a2  4b2  4a  3

 8ab  4b
3 1
a)b  0  4a2  4a  3  0  a1  , a2  
2 2
3 3
z1  | z1 | 
2 2
1 1
z2   | z2 | 
2 2
1 1
b)a    1 4b2  2  3  b  0 şi deci z   ca la a).
2 2

III. Fie AA1,BB1,CC1 cevienele concurente în M, A2, B2, C2 mijloacele laturilor [BC], [CA],
[AB] şi A3, B3, C3 mijloacele laturilor[AA1], [BB1], [CC1].

C2 A3 B2
B1
C1 B3
M C3

B C
A1 A2

A3 este pe linia mijlocie [B2C2] şi avem


A3C2 C3B2 B3 A2 BA1 AC1 CB1
      1 pentru că AA1, BB1, CC1 sunt concurente ( teorema
A3B2 C3 A2 B3C2 CA1 BC1 AB1
lui Ceva directă). Folosind reciproca teoremei lui Ceva, se obţine că cevienele A2A3, B2B3, C2C3
sunt concurente în triunghiul A2B2C2.

Clasa a XI-a
a b 
I. Fie A     M2    A2   A   I2  O2 , unde   Tr  A  a  d este urma lui A şi
c d 
  det A .
 
det A3   det A  8  9  1  det A  1  det A 
3


A2   A  I2  A3   A2  A    A  I2   A   2  1 A  I2 
      
2  Tr A3  Tr  2  1 A  I2   2  1 Tr  A  Tr I2    2  1   2  
  3  3   3  3  2  0    1   2  0
2

116
4 3
Pentru   1  A3  0  A  I2  I2   .
 3 2 
4 3  6 3  2 1
Pentru   2  A3  3A  2I2     3A    A .
 3 2   3 0   1 0 

II. xn xn1  xn1xn2  xn1xn2  xn2 xn3 ,  n  3 arată că şirul


yn  xn xn1  xn1xn2 , n  2 este constant, deci yn  y2 ,  n  2 , adică
xn xn1  xn1xn2 =x2x1,  n  2 , care arată că şirul zn  xn xn1, n  1 este progresie
aritmetică cu raţia x2x1 deci
zn  z1   n  1 r   n  1 x2 x1,  n  1, adică
xn xn1   n  1 x2 x1,  n  1.
Avem şi xn1xn2   n  2 x2 x1,  n  2 , de unde
xn n 1 n 1
 ,  n  3  xn  x ,  n  3 .
xn2 n  2 n  2 n2
1 x2n  22nn 21 x2n2  22nn 21  22nn  34 x2n4   22nn 21  22nn  34   32  x2,  n  2
2 x2n1  2n2n 1x2n1  2n2n 1 22nn  32 x2n3   2n2n 1 22nn  32   21  x1,  n  1
 , x1x2  0
Din lim  n  1 x1x2   şi xn xn1   n  1 x2 x1,  n  1  şirul  xn n0 nu este
, x1x2  0
n 

convergent.
Se poate arăta din (1) şi (2) că
 , x2  0  , x1  0
lim x2n   şi lim x2n1   .
n 
, x2  0 n 
, x1  0

III.

117
A

G
E

D
B I

C
3
a) Notăm V1=V(AEFG), V2=V(BEHI), V=V(ABCD) şi atunci V(EFGHCDI)  V
4
3 3
1 V V 1  AE   BE  1
 V1  V2  V  1  2   
4

V V 4  AB   AB  4
 1
Notând AE  x  0, BE  y  0 , (1) devine
x3  y 3 x2  xy  y 2 1

1
  2

  4 x2  xy  y 2   x  y   3  x  y   0 .
2

 x  y 3 4 x  y 2
4
AE BE V V
b)   1  3 1  3 2  1  3 V  3 V1  3 V2  3  5  8  V  83  512cm3 şi deci
AB AB V V
V (EFGHCDI )  V  V1  V2   512  27  125  512  152  360cm3 .

Clasa a XII-a

arctgx 1  1  arctgx 1 1
I.  ax  b 2
dx     
a  ax  b 
' arctgxdx    
a ax  b a ax  b x 2  1
dx .
 
  a2
   
 a    0  a2  b2
1  x    a
  2  a  b  0     2
 
ax  b x  1 ax  b x  1    b  1 
2
a  b2
  b
    a2  b2
 
arctgx 1    
Deci 
arctgx
dx   
  ln | ax  b |  ln x 2  1   arctgx   C cu ,  ,
a ax  b a  a
ax  b2
2 
determinaţi mai sus.

118
P x
II. Presupun că   f :  a, b bijectivă şi funcţie raţională  f  x  
Qx
cu P,Q  X .
Dacă gradP>gradQ  lim f  x    sau   şi deci Imf conţine un interval de forma
x 

c,  sau  ,c  în contradicţie cu Imf=(a,b).


a0
Deci gradP  gradQ  lim f  x   lim f  x   0 dacă gradP<gradQ şi dacă
x  x  b0
gradP=gradQ; a0,b0 coeficienţii dominanţi ai lui P, Q. (1)
Pe de altă parte, f derivabilă pe şi f injectivă  f strict monotonă. (2)
Din (1) şi (2) avem contradicţie.
Observaţie: Rezultatul se menţine şi dacă înlocuim f bijectivă cu f injectivă.
Într-adevăr, pentru gradP>gradQ demonstraţia este ca mai sus. Pentru gradP  gradQ
 lim f  x   lim f  x   număr finit. (1)
x  x 

Cum f derivabilă pe şi ecuaţia f '  x   0 are un număr finit de rădăcini, fiind


polinomială  f are un număr finit de puncte de extrem local. (2)
Din (1) şi (2)  f îşi atinge cel puţin un extrem global, în contradicţie cu Imf=(a,b).

III.
2
 a3  b3  c 3  3 a2  b2  c 2

3.a) 3 a3  b3  c 
3 2

 3 a2  b2  c 
2 3
   
 3  3

      1
2 2 2 2

a b c 3 3 3 3 a3 3  b3 3  c3 3
  
 3  3
2
Funcţia f : 0,   , f  x   x este concavă şi deci avem 3

 x  x  x  f  x1   f  x2   f  x3 
f 1 2 3   x1, x2, x3 0, .
 3  3
Luând x1  a3 , x2  b3 , x3  c3 se obţine exact (1).
b) | z1  z2 |2  | z2  z3 |2  | z1  z3 |2   z1  z2  z1  z2   z2  z3  z2  z3     
   
  z1  z3  z1  z3  2 z1 z1  z2 z2  z3 z3  z1 z2  z1z2  z2 z3  z2 z3  z3 z1  z3 z1 

      
 2 | z1 |2  | z2 |2  | z3 |2  z1 z2  z3  z2 z3  z1  z3 z1  z2 
 6  z  z  z  z  z   z z  z  z  z   z z  z  z  z  
1 1 2 3 1 2 1 2 3 2 3 1 2 3 3

 6   z  z  z   z  z  z   | z |  | z |  | z |   3 | z  z  z
1 2 3 1 2 3 1
2
2
2
3
2
1 2 3 |2
şi atunci | z1  z2 |2  | z2  z3 |2  | z3  z1 |2  3 | z1  z2  z3 |2  0  z1  z2  z3  0 şi cum
| z1 || z2 || z3 | 1  z1, z2 , z3 sunt afixele vârfurilor unui triunghi echilateral.

119
Ediţia a V-a, 20 – 21 noiembrie 1999
Clasa a VII-a

I. În avem a2000  b2000  a  b sau a  b , deci

 x  19992000   x  20012000  x  1999  x  2001, care este imposibilă, sau


x  1999  x  2001 2x  4000  x  2000 

x 1 2
II.    x  1 y  2  8 , de unde avem cazurile:
4 y 2
 x 1 1 x  2  x 1 2 x  3  x 1 4 x  5
  sau   sau   sau
y  2  8 y  6  y  2  4 y  2  y  2  2 y  0
x  1  8  x  9  x  1  1  x 0  x  1  2  x  1
  sau   sau   sau
y  2  1 y  1 y  2  8 y  10 y  2  4 y  6
 x  1  4  x  3  x  1  8  x  7
  sau  
y  2  2 y  4 y  2  1 y  3
Dar x, y  *
, deci nu convin toate soluţiile găsite. Soluţia este deci
2,6,3,2,9, 1, 1, 6, 3, 4, 7, 3 .
III.
A
B’
C’

T
C1 B1
G

S
B C
A1 A’

a) Fie A1, B1, C1 mijloacele segmentelor [BC], [CA], [AB].


A1B1 este linie mijlocie în triunghiurile ABC şi SA’B’, de unde avem
AB A1B1 A ' B ' 
  AB  A ' B ' şi AB A ' B '  ABA’B’ paralelogram.
AB  2A1B1  A ' B ' 
Analog ACA’C’ şi BCB’C’ paralelograme.
Avem deci AB  A ' B ', BC  B 'C ', CA  C ' A '  ABC  A ' B 'C ' L.L.L. .
b) Din ABA’B’ paralelogram  BB’ trece prin mijlocul lui [AA’] (1)

120
Din ACA’C’ paralelogram  CC’ trece prin mijlocul lui [AA’] (2)
Din (1) şi (2)  AA’, BB’, CC’ sunt concurente, punctul de concurenţă T fiind mijlocul lui
[AA’] ( de fapt şi mijlocul lui [BB’] şi mijlocul lui [CC’] ).
Observaţie:
Punctul T de concurenţă între AA’, BB’, CC’ se găseşte pe dreapta SG, unde G este
centrul de greutate al triunghiului ABC. Într-adevăr, AA1 şi ST sunt mediane în triunghiul ASA’
G ' A1 1
şi notând cu G '  AA1  ST   şi cum AA1 este şi mediană în triunghiul ABC,
G' A 2
rezultă că G’=G centrul de greutate al triunghiului ABC.

Clasa a VIII-a
I.

A
A

1 2-x

M N
B C
1-x
x-1
B C
M N
Cazul 1
2-x>1, x<1  M (AB), N (AC)
AM AN
T .Thales 1 (1 x )2
MN BC   ,   NC  (1 x)3
MB NC 1 x NC
Avem de ordonat numerele 1, 1-x, (1 x )2 , (1 x )3
Pt. 1-x<1  x (0,1), avem (1 x)3  (1 x)2  (1 x)  1
Pt. 1-x=1  x=0, avem 1=1-x= (1 x )2 = (1 x )3
Pt. 1-x>1  x  (,0) avem 1  1 x  (1 x)2  (1 x)3
Cazul 2
2-x<1, x>1  B (AM), C  (AN)
AM AN
T .Thales 1 ( x  1)2
MN BC   ,   NC  (1 x)3
MB NC x  1 NC
Avem de ordonat numerele 1, x-1, ( x  1)2 , ( x  1)3
Pt. x-1<1  x (1,2), avem ( x  1)3  ( x  1)2  x  1  1
Pt. x-1=1  x=2, avem 1  x  1  ( x  1)2  ( x  1)3 , dar AB=0, imposibil.
Pt. x-1>1  x>2 şi atunci AB = 2-x<0, imposibil.
Cazul 3
2-x=1  x=1, AN=0, imposibil.

121
3( x  2)( x  2) 3( x  2)
II. a) F( x)  
( x  2)( x  4) x4
3( x  4  2) 6
b) F( X )  3  * 
x4 x4
6
  şi cum x  
x4
 x  4  D6  {6, 3, 2, 1,1,2,3,6} .
6 6
Dar 3   *  3 şi deci x+4 {-6,-3,-2,-1, 3, 6}, de unde x {-10, -7, -6,-5,-1,2}.
x4 x4

III.
F E

G B
A

C
D

a) AC  a 2, AE  a 2, EC  a 2 din triunghiul dreptunghic isoscel EBC  AEC

 
triunghi echilateral şi deci m AC, AE  60o .

a 2 3 a 6
b) d E, AC   hAEC   .
2 2
c) FA   ABCD  T .3
  FD  DC
AD  DC 
d F,CD   FD  a 2
d) Fie AG  FD. Cum şi FD  DC, AD  DC, avem cu o reciprocă a teoremei celor trei
perpendiculare că AG  (FDCE).

d  A,ECD  AG 
FD a 2
 .
2 2

122
  
m  AEC ,  ABC   m EOB 
e) Fie AC  BD  O . Din BO  AC, EO  AC avem că tg EOB  EB 
  BO
a
a 2
 2

Clasa a IX-a
I.
1. a  n  2  2 2  n  2  2  n  2  2 2  n  2  2  2 2 

   
2 2
 n2  2  n2  2 2 2 

 n  2  2  | n  2  2 | 2 2
Cazul 1
n 2,3  a  n  2  2  2  n  2  2 2  2  
n 2  2  0.
Cazul 2
n  4 a  2 2 02 2  0.
Cazul 3
n 5,6,7, a  n 2  2  n 2  2 2 2  0.

II. Fie x  A \ C  x  A şi x C . Avem cazurile:


1o x  B  x   A \ B   x   A \ B   B \ C 
2o x  B  x  B \ C   x   A \ B   B \ C 
Am arătat deci că  x  A \ C  x   A \ B  B \ C  , adică avem A\C   A \ B  B \ C  .

A B

C
A\C

A \ C  (A \ B)  (B \ C)  ( A C) \ B   şi B \ ( A C)    ( A C)  B  ( A C)

123
A
C
B

 1
III.. f injectivă   x1, x2   ,   cu f  x1   f  x2   x1  x2
 2 
f  x1   f  x2   x1  x1  1  x2  x2  1  x1  x2  x1  x2  1  0 de unde x1  x2 sau
2 2

x1  x2  1  0 
 1
1 1   x1  x2  
x1   , x2    2
2 2
3   1
f surjectivă  y   ,  ,  x   ,   cu f  x   y
4   2
3 
x 2  x  1 y  0,   4y  3  0 pentru că y   ,  
4 
1 4y  3 1 4y  3 1
x1,2  convine x  
2 2 2
Din f injectivă şi surjectivă  f bijectivă  f inversabilă.
3   1 1 4y  3
f 1 :  ,     ,   , f 1  y   x  f  x   y  f 1  y   .
4   2 2

Clasa a X-a

I. f : 1,   , f  x   x  1  x  4  x  9  x  16  x  25 este funcţie strict


crescătoare. Ecuaţia se scrie f(x)=15=f(0).  x=0 este unica soluţie a ecuaţiei.

a b a sin 
II. Cu teorema sinusurilor avem   sin A 
sin A sin  b

1o 0   
2
Dacă
a sin 
b
   a sin  
 1  m A1  arcsin 
 b 
    a sin  
, m A2    arcsin 
 b 

A2

A1 b
b

B a C

Dacă
a sin 
b


 1 m A  .
2

124
2o  

2
 sin A 
a
b

 1, m A  arcsin
a
b

B a C

3o  

2
 sin A 
a sin 
b
   a sin  
 1  m A  arcsin 
 b 

A
b
C
a
B
b

A’
În concluzie, avem simultan a) şi b) 
 a sin   a  a sin 
0   ,  1 sau   ,  1 sau    ,  1.
2 2 2 b 2 2

III. Soluţia 1
a) Existenţa: Demonstrăm prin inducţie că   an , bn  a.î . zn  an z  bn .
Pentru n=1, z  1 z  0  a1  1, b1  0 .

 
Pentru n=2, z2  z  z z  zz  a2  z  z, b2  zz  .
Presupunem că   an , bn  a.î. zn  an z  bn şi arătăm că   an1, bn1  a.î.
zn1  an1z  bn1 .
zn1  zn z  an z  bn  z  an z2  bn z  an a2z  b2   bn z  a2an  bn  z  an b2 , alegem
an1  a2an  bn , bn1  an b2 şi avem că an1, bn1  .

125
b) Unicitatea: Presupunem că
zn  an z  bn  an ' z  bn ', cu an , bn ,an ', bn '   an  an '  z  bn ' bn .
bn ' bn
Dacă an  an '  0  z   , în contradicţie cu z \ . Atunci avem
an  an '
an  an '  0  bn ' bn  0 şi deci an  an ', bn '  bn .
Soluţia 2
z  r cos  i sin , r  0,   0,2  \   pentru că z \ .
zn  r n cos n  i sin n 
zn  an z  bn  r n cos n  i sin n   an r cos  i sin   bn
an r cos  bn  r n cos n

 an r sin  bn  r sin n
n

De unde
r n1 sin n
an  
sin
r n sin n
bn  r n cos n  cos  .
sin
Unicitatea se arată ca în soluţia 1.

Clasa a XI-a

I. Arătăm mai întâi că polinomul P(X) de grad n este reciproc dacă şi numai dacă
1 1
P( )  n P( X ) .
X X
Dacă P( X )  a0  a1 X  a2 X 2   an X n atunci
1 1 a a a 1
P( )  n P( X )  a0  1  22   nn  n (a0  a1 X  a2 X 2   an X n ) 
X X X X X X
 a0 X  a1 X  a2 X   an  a0  a1 X  a2 X 2   an X n  a0  an ,a1  an1, ,
n n 1 n 2

an  a0  ai  ani , pentru i  0,1, , n  P( X ) este polinom reciproc.


Fie acum P(X) de grad n şi Q(X) de grad m două polinoame reciproce. Avem deci
1 1 1
P( )  n P( X ) şi Q( X )  m Q( X ) de unde prin înmulţire avem
X X X
 
1 1
(PQ)    nm (PQ)  X  , care arată că PQ este polinom reciproc de grad n+m.
X X

II. Notăm

126
x y y y
y x y y 

Mx,y y y x y  matrice pătratică de ordinul n.
 
 
y x 
 y y
Bănuim că inversa lui M=Ma,1 este o matrice de tipul de mai sus.
Ma,1  Mx,y  Maxn1y,xan2y .
 ax   n  1 y  1
Impunem Ma,1  Mx,y  In  M1,0 şi avem  .
x  a  n  2 y  0
Scăzând relaţiile  a  1 x  y   1.
 1
x  y  |an2 an2
 a 1  x a  n  1  .
 x   a  n  2 y  0 a  1

an2 1
şi deci M 1  Ma,1   M an2
1
x , y .
a  1a  n  1 a  1a  n  1 ,
1
a1an1 a1an1

a a  1a  2 a  n  ,
III. Fie xn  n  1.
b  b  1 b  2 b  n
xn1 a  n  1
  1 pentru că 0  a  b  şirul  xn n1 este strict descrescător şi fiind minorat
xn b  n  1
de 0, rezultă că este convergent.
Notăm lim xn   0.
n

y1  y2   yn
 xn ,  y1  y 2   y n  nxn
Să determinăm şirul  yn n1 astfel ca n

şi y1  y2   yn1   n  1 xn1  yn1   n  1 xn1  nxn , n  1 şi y1  x1.

lim
 y1  y2   y n1    y1  y2   y n 
 lim  n  1 xn1  nxn  
n n  1 n n

 a  n 1  an  n2  n  a  n  1 bn  n2  n
 lim xn    n  1  n   lim xn  
n
 b  n 1  n b  n 1
n a  b  1  a  1
 lim xn   a  b  1.
n n  b 1
a  b  1 şi cum avem şi
\
Folosind lema Stolz- Cesaro  lim xn 
n

lim xn   a  b  1   a  b  0 şi cum a<b, rezultă =0.


n

127
Clasa a XII-a

I. Fie t  0 un timp oarecare.  M1 v1t,0, M2 0,v2t .

M2

O M1 x

 xM  kxM2 vt
 xM  1  1
MM1  1 k 1 k
Din k 0
MM2 y  yM1  kyM2  kv 2t
 M 1 k 1 k
kv2
kv2 xM  v1yM şi deci locul geometric al lui M este semidreapta de ecuaţie y  x , cu x  0
v1
.

II. a) Fie x, y Gn  x n  e, y n  e . Cum (G,  ) este comutativ


  x  y   x n  y n  e  e  e şi deci x  y  Gn .
n

   x  n 1
n
Fie x Gn  xn  e  x1  e1  e şi deci x1 Gn .
b) Fie H subgrup finit cu n elemente al grupului (G,  ).
H  x1, x2 , , xn  . Fie x  H  H  xx1, xx2 , , xxn  şi scriind produsul elementelor lui H
avem xx1  xx2   xxn  x1x2 xn  xn  e  x Gn .
Am arătat că H  Gn . Cum H are n elemente iar Gn are cel mult n elemente, rezultă că H=Gn.

III.   a2  4 .
a  
1)  0  a   , 2   2, , x1,2 
2
dx 1  1 1  1
I       dx  ln | x  x1 |  ln | x  x2 |  C 
 x  x1  x  x2  x1  x2  x  x1 x  x2  x1  x2
1 x  x1
 ln | | C
x1  x2 x  x2

128
a   a  
x1  x2    .
2 2
2)  0  a  2, x 2  ax  1   x  1
2

dx 1
I    C.
 x  1
2
x 1
3)  0  a   2,2
a
x
I
dx
 arctg
2 2  C  2 arctg 2x  a  C .
a  4  a2
2
 4  a2 4  a2 4  a2 4  a2
  
2 
x
 4 2

Ediţia a VI-a, noiembrie 2000


Clasa a VII-a
I.
Y

6 6
4

45o d
C1 A B’ C X
Fie d o dreaptă; fixăm A  d şi X  d. Ducem o semidreaptă [AY care face cu [AX un
unghi de 45o pe care luăm B astfel încât AB=4. Avem că distanţa de la B la d este
4
BB '   2 2  6 şi atunci putem alege C [AX astfel încât BC=6. Soluţia este unică
2
pentru că celălalt punct C1 de pe d cu BC1=6 nu se află pe [AX.
2. n  ab, a  b | ab  a  b |10a  b  a  b | 9a . Avem succesiv:

129
a  1  1 b | 9  b  0,2,8
a  2  2  b |18  b  0,1,4,7
a  3  3  b | 27  b  0,6
a  4  4  b | 36  b  0,2,5,8
a  5  5  b | 45  b  0,4
a  6  6  b | 54  b  0,3
a  7  7  b | 63  b  0,2
a  8  8  b | 72  b  0,1,4
a  9  9  b | 81  b  0
 ab 10,12,18,20,21,24,27,30,36,40,42,45,48,50,54,60,63,70,72,80,81,84,90
  
3. Fie ,  , măsurile unghiurilor exterioare. Avem    k    3k,   4k,  5k
3 4 5
şi cum       360  12k  360  k  30    90o ,   120o ,  150o şi deci
o o o

A,B,C au măsurile 90o ,60o ,30o.

Clasa a VIII-a

I. 0<a<b<c sunt lungimile laturilor unui triunghi 



 c  a  b a  b  c, b  a  c sunt evidente . 
0<x<x+5<x+10 sunt lungimile laturilor unui triunghi  x  0 şi x  10  x  x  5  x  5.
Trebuie arătat că x  10  k  x  k  x  5  k , unde k>0, adică
x  10  k  x  k  2  x  k  x  5  k   x  5  k  5  x  k  2  x  k  x  5  k  care
este evidentă, deoarece x  5 şi k  0.

II. 2000  10 20  20 5 şi cum 20  avem că 2000 este iraţional  5 este


iraţional.
a
Presupunem că 5 cu a, b  *
, (a,b)=1
b
a2
5  2
 a2  5b2  5 | a2  5 | a  a  5a1 cu a1  *  25a12  5b2 
b
5a12  b2  5 | b2  b  5b1 cu b1  * contradicţie cu (a,b)=1.

x 1 x  2
III.    x  1a  3   x  2a  1  ax  3x  a  3 
a 1 a  3
ax  x  2a  2  2x  1  a.

130
Clasa a IX-a

I. Notăm OA=OB=OC=OD=R.

B D
N
L M

OB2  OC2 BC2 OC2  OD2 CD2 BD2


   
OL2  OM 2  LM 2 2 4 2 4 4 
cos(LOM )  
2  OL  OM 2  OL  OM
BC2  CD2  BD2
2R   
2
4 8R2  BC2  CD2  BD2
  .
2  OL  OM 8  OL  OM

8R2  BC2  CD2  BD2 

Analog cos LON   8  OL  ON
şi


8R2  BC2  CD2  BD2 

cos NOM   8  ON  OM
.
Atunci
  
LOM  LON  NOM  cos LOM  cos LON  cos NOM  ON  OM  OL    
BD2 CD2 BC2
 R2   R2   R2   BD  CD  BC.
4 4 4
a  3 x  2y  4
II.   a  5 x  y   0.
2x  a  3 y  4
2x  2y  4
1o a  5    x  y  2 cu soluţia  x,2x  | x  
2x  2y  4

131
2o a  5  x  y şi a  1 x  4
4
pentru a  1  x  y 
a 1
pentru a  1  sistemul nu are soluţie.

III.  x2    x
2

k
 x   k  k  x  k  1.

1 k  0  k 2  x 2   k  1   x 2   k 2  p, cu p  0,1,2,...,2k şi ecuaţia devine
o 2

k 2  p  k 2  p  0 deci x2   k 2  k 2  x2  k 2  1  k  x  k 2  1,


x  k, k 2  1  k, k  1 .

2 k  0  k 2  x 2   k  1   x 2   k 2  p, p 2k  1,2k,...,0 şi ecuaţia devine
o 2

k 2  p  k 2  p  0 deci x2   k 2  k 2  x2  k 2  1  k  x   k 2  1
şi cum k  x  k  1 x  k.
 
S  1, 2, 3,...  0,1  1, 2  2, 5  3, 10  ... 
Clasa a X-a

I. Luând z1  z2  z  0 avem | 2z |p  2  2 | z |p  2p  4  p  2 care verifică problema


 
pentru că | z1  z2 |2  | z1  z2 |2  2 | z1 |2  | z2 |2 ,  z1, z2  se arată simplu.

II. Plecând de la x 2  y 2  z2  xy  yz  zx,  x, y, z avem  x  y  z   3  xy  yz  zx 


2

şi atunci sin A sin B  sin B sinC  sinC sin A 


 sin A  sin B  sinC 
2

. Rămâne de arătat că
3
 sin A  sinB  sinC  9

2
sin A  sin B  sinC
 
3
care este adevărată, pentru că
3 4 3 2
f : 0,   , f  x   sin x este concavă şi cu inegalitatea lui Jensen avem
 A  B  C  f  A  f B   f C 
f  care se scrie
 3  3
A  B  C sin A  sinB  sinC  sin A  sinB  sinC
sin   sin  .
3 3 3 3

x1  x2   xn1 n1
III. Se ştie că  x1x2 xn1 ,  x1, x2 , , xn1  0 , cu egalitate pentru
n 1
x1  x2   xn1. Rezultă că

132
1 1
a1  a2   an    n  1 n1 a1a2 an   n  1, cu egalitate pentru
a1a2 an a1a2 an
1
a1  a2   an  , de unde avem imediat că a1  a2   an  1.
a1a2 an

Clasa a XI-a

I. Fie OO '   ABC  cu O '   ABC  şi AO ' BC  D .

c
a
h
b
C
A O’
D
B

Din OA  OB, OA  OC  OA  BOC  , de unde OA  BC şi cum


OO '  BC  BC  OAO ' şi deci BC  AD, BC  OD .
OB  OC bc b2c 2
OD   , AD  OA  OD  a  2
2 2 2
BC b2  c 2 b  c2
bc
a
OA  OD b  c2
2
abc
h  OO '    .
AD a2 b2  a2c 2  b2c 2 a2 b2  a2c 2  b2c 2
b2  c 2
Rămâne de arătat că
a2  b2  c2  1 1 1
abc
 3 a 2

 b2  c2  2  2  2  care se arată fie
a b c 
a2b2  a2c2  b2c2 3
xy z 3
cu  xyz,  x, y, z  0 , fie efectuând înmulţirile în
3
a2  b2  c2   a12  b12  c12  şi ţinând cont că x  x1  2,  x  0.

 
II. P  X   n3 X n  n2  2 X  n2  n  1, n  *

Presupun că  a  
cu P a  0  n3an  n2  2 a  n2  n  1  0 şi cum
n2  n  n  n  1 este par,  n  *
, se observă că a trebuie să fie impar, a=2b+1, b  .

133
 
n3 2b  1  n2  2 2b  1  n2  n  1  0 de unde n3  n2  1 trebuie să fie par, ceea ce
n

este imposibil, pentru că n3  n2  1  n  n  n  1  1 este impar, indiferent de paritatea lui n.

III. Din  an n1 şi  bn n1 convergente  sunt mărginite şi deci   M  0 a.î.


| an | M, | bn | M,  n  1.
Fie acum k  *, k  2 şi n  * cu n  2k . Avem
| cn || an  b1 |  | an1  b2 |   | ank 2  bk 1 |  | ank 1  bk |   | ak  bnk 1 |  | ak 1  bnk 2 |  
 | a2  bn1 | a1  bn | 2M  2M   2M | ank 1  bk |   | ak  bnk 1 | 2M  2M   2M 
  2M 
2k 2
 | ank 1  bk |   | ak  bnk 1 | .
Din lim an  a şi lim bn  b avem că
n n

   0, N a.î . | an  a |  şi | bn  b | ,  n  N .


Luând n  2N cu cele de mai sus avem | cn | 2M 2N 2  | anN 1  bN |   | aN  bnN 1 | şi
avem | anN 1  bN || anN 1  a |  | a  b |  | b  bN |   | a  b |  şi analog pentru ceilalţi
factori. Obţinem : | cn |  2M    2  | a  b | , n  2N .
2N 2 n2N 2

1 | a  b |
Alegând    0 deoarece | a  b | 1, obţinem
4
n2N 2
 1 | a  b | 
| cn |  2M  , n  N .
2N 2
 
 2 
n2N 2
1 | a  b | 2N 2  1 | a  b | 
Dar 0   1 lim  2M       0 şi atunci cu criteriul majorării
2 n
 2 
 lim cn  0.
n

Clasa a XII-a
I. an1  2an  2an  1
2

2an1   2an  1  1
2

2an1  1   2an  1   2an1  1    2a0  1   2a  1


2 22 2n 1 2n 1
de unde

 2a  1
n
1
2

an  .
2
 1
 , 2a  1  1,1, a  0,1
2

lim an   1, 2a  1  1, a  0 sau 1
n 

, 2a  1  , 1  1,  , a   ,0  1,  

şi deci an n0 este convergent  a  0,1.

134
1
II.  ln( x  x 2  1)  dx 
x2  1
1
 (ln( x  x  1))  (ln( x  x2  1)' dx 
2 2

3
(ln( x  x 2  1))2 2
  C  ln( x  x 2  1) ln( x  x 2  1)  C
3 3
2

(  ) ( A  In )2  4In  det( A  In )2  det(4In )   det( A  In )   4


2 n
III. şi cum
det(A  In )2  0 pentru că A  Mn   avem că şi  4n  0 , de unde n este par.
  Fie n un număr par. Trebuie arătat că  A  Mn   astfel ca (A  In )2  4In , adică
 B  Mn   astfel ca să avem B2  4In .
Căutăm B de forma
a b 0 0 0 0
c d 0 0 0 0 

0 0 a b 0 0
 
B0 0 c d 0 0  cu a,b,c,d  şi impunând B2  4In , se obţine
 
 
0 0 0 0 a b
0 d 
 0 0 0 c

a2  bc  4

 b a  d   0

 c a  d   0
d 2  bc  4
Nu putem avea b=0 şi c=0 pentru că ar rezulta că a2  4 , imposibil pentru a  . Deci
a2  bc  4

 ad  0
O posibilitate este a=0, b=2, c= -2, d=0.

Ediţia a VII-a, noiembrie 2001


Clasa a VII-a

𝑎 b c
I. = 3 = 6 = k ⇒ a = 2k, b = 3k, c = 6k
2

135
(⇒) Ştim că 2𝑎 + 3𝑏 + 6𝑐 = 7 ⇒ 2 ∙ 2𝑘 + 3 ∙ 3𝑘 + 6 ∙ 6𝑘 = 7 ⇒ 4𝑘 + 9𝑘 + 36𝑘 = 7 ⇒
1 4 9 36
49𝑘 = 7 ⇒ 𝑘 = 7 şi atunci 𝑎2 + 𝑏 2 + 𝑐 2 = 49 + 49 + 49 = 1
1
(<=)Ştim că 𝑎2 + 𝑏 2 + 𝑐 2 = 1 ⇒ 4𝑘 2 + 9𝑘 2 + 36𝑘 2 = 1 ⇒ 𝑘 2 = 49 şi cum
1
a, b, c> 0 ⇒ 𝑘 = 7 şi deci 2𝑎 + 3𝑏 + 6𝑐 = 4𝑘 + 9𝑘 + 36𝑘 = 49𝑘 = 7

34
II. ∈ ℕ ⇔ 𝑥 2 + 𝑦 2 + 𝑧 2 ∈ {1,2,17,34}, 𝑥 2 + 𝑦 2 + 𝑧 2 = 1 ⇒ 𝑥 = 1, 𝑦 = 𝑧 = 0 ⇒
𝑥 2 +𝑦 2 +𝑧 2
𝑥𝑦𝑧 = 100, 𝑥 2 + 𝑦 2 + 𝑧 2 = 2 ⇒ 𝑥 = 1, 𝑦 = 1, 𝑧 = 0,
̅̅̅̅̅
𝑥 = 1, 𝑦 = 0, 𝑧 = 1 şi deci 𝑥𝑦𝑧
̅̅̅̅̅ = 110 şi 𝑥𝑦𝑧
̅̅̅̅̅ = 101
𝑥 2 + 𝑦 2 + 𝑧 2 = 17 ⇒ 𝑥𝑦𝑧
̅̅̅̅̅ poate fi 104, 140, 232, 223, 322, 401, 410
𝑥 2 + 𝑦 2 + 𝑧 2 = 34 ⇒ 𝑥𝑦𝑧
̅̅̅̅̅ poate fi 334, 343, 305, 350, 433, 503, 530.

III.

a) ∆𝐸𝐴𝐶 ≡ ∆𝐵𝐴𝐺(𝐿. 𝑈. 𝐿) ⇒EC=BG.


̂ ≡ 𝐴𝐵𝐺
b) Fie 𝐸𝐶 ∩ 𝐵𝐺 = {𝑁} şi 𝐸𝐶 ∩ 𝐴𝐵 = {𝑀}. Din ∆𝐸𝐴𝐶 ≡ ∆𝐵𝐴𝐺 ⇒ 𝐴𝐸𝐶 ̂

̂ ) = 𝑚(𝐵𝑀𝑁
Dar 𝑚(𝐸𝑀𝐴 ̂ ) – opuse la vârf şi atunci folosind ∆𝐸𝐴𝑀 𝑠𝑖 ∆𝐵𝑁𝑀 ⇒ 𝑚(𝐵𝑀𝑁
̂)=
̂ ) = 90° şi deci 𝐸𝐶 ⊥ 𝐵𝐺.
𝑚(𝐸𝑀𝐴

Clasa a VIII-a

𝑥=3
𝐈. { este soluţie.
𝑦=2
𝑦 3 = 35 − 𝑥 3 > 35 − 27 = 8 ⇒ 𝑦 > 2
- pentru 𝑥 < 3 ⟹ { şi deci avem
𝑦 5 = 𝑥 5 − 211 > 243 − 211 = 32 ⇒ 𝑦 < 2
contradicţie.
𝑦 3 = 35 − 𝑥 3 < 35 − 27 = 8 ⇒ 𝑦 < 2
- pentru 𝑥 > 3 ⟹ { şi deci avem
𝑦 5 = 𝑥 5 − 211 > 243 − 211 = 32 ⇒ 𝑦 > 2
contradicţie.

II. 𝐸 = 3𝑎2 − 6𝑎𝑏 + 3𝑏 2 + 2𝑏 2 − 12𝑏𝑐 + 18𝑐 2 + 𝑐 2 − 2𝑐 + 1 + 7,


𝐸 = 3(𝑎2 − 2𝑎𝑏 + 𝑏 2 ) + 2(𝑏 2 − 6𝑏𝑐 + 9𝑐 2 ) + (𝑐 2 − 2𝑐 + 1) + 7 ,
𝐸 = 3(𝑎 − 𝑏)2 + 2(𝑏 − 3𝑐)2 + (𝑐 − 1)2 + 7
136
𝐸 ≥ 2 egalitatea are loc pentru
𝑎−𝑏 =0 𝑎=3
{𝑏 − 3𝑐 = 0 ⇒ {𝑏 = 3
𝑐−1=0 𝑐=1

III. Fie E mijlocul lui [𝐷𝐴], F mijlocul lui [𝐵𝐶]

𝑃𝐺 ⊥ (𝐴𝐵𝐶𝐷), 𝐹𝐻 ⊥ (𝐴𝐵𝐶𝐷)

Cum 𝑁𝐷 ⊥ (𝐴𝐵𝐶𝐷) şi P mijlocul lui [𝑀𝑁] ⟹ G mijlocul lui [𝐷′𝐵] ⟹ H mijlocul lui [𝐷𝐵].
𝐴𝑀 𝐷𝑁
𝐸𝐺 = 2 şi 𝐺𝑃 = 2 ⇒ 𝐸𝐺 = 𝐺𝑃 (1)

𝐴𝑀 = 𝐷𝑁.
𝐴𝐵 𝐷𝐷 ′
𝐸𝐻 = , 𝐹𝐻 = , 𝐴𝐵 = 𝐷𝐷′ ⇒ 𝐸𝐻 = 𝐹𝐻 (2)
2 2

Din (1) ,(2) şi E,G,H coliniare ⇒E,P,F coliniare şi deci P este pe [𝐸𝐹] .
Dacă 𝑀 = 𝐴 ⇒ 𝑁 = 𝐷 şi 𝑃 = 𝐸. Dacă 𝑀 = 𝐵 ⇒ 𝑁 = 𝐷′ şi 𝑃 = 𝐹 deci când M descrie [𝐴𝐵] ⇒
P descrie [𝐸𝐹].

Clasa a IX-a

I. a) Numărătorul este :
𝑁1 = 𝑥 2000 + 𝑥1000 + 𝑥 998 + 𝑥 997 + 𝑥 996 + ⋯ + 𝑥 + 1 =
= 𝑥 2000 − 𝑥 999 + (𝑥1000 + 𝑥 999 + 𝑥 998 + ⋯ + 𝑥 + 1) =
= 𝑥 999 (𝑥1001 − 1) + (𝑥1000 + 𝑥 999 + 𝑥 998 + ⋯ + 𝑥 + 1) =
= 𝑥 999 (𝑥 − 1)(𝑥1000 + 𝑥 999 + 𝑥 998 + ⋯ + 𝑥 + 1) + (𝑥1000 + 𝑥 999 + 𝑥 998 + ⋯ + 𝑥 + 1) =
= (𝑥1000 + 𝑥 999 + 𝑥 998 + ⋯ + 𝑥 + 1)(𝑥1000 − 𝑥 999 + 1) (1)
Numitorul este:
𝑁2 = 𝑥 2001 + 𝑥 999 + 𝑥 998 + 𝑥 997 + 𝑥 996 + ⋯ + 𝑥 + 1 =
= 𝑥1000 (𝑥1001 − 1) + (𝑥 1000 + 𝑥 999 + 𝑥 998 + ⋯ + 𝑥 + 1) = 𝑥1000 (𝑥 − 1)(𝑥1000 + 𝑥 999 +
𝑥 998 + ⋯ + 𝑥 + 1) + (𝑥1000 + 𝑥 999 + 𝑥 998 + ⋯ + 𝑥 + 1) =
137
= (𝑥1000 + 𝑥 999 + 𝑥 998 + ⋯ + 𝑥 + 1)(𝑥1001 − 𝑥1000 + 1) (2)
𝑥 1000 −𝑥 999 +1
Din (1),(2) ⇒ 𝐹 = 𝑥 1001 −𝑥 1000+1
1 1 1
b) + 𝑏 + 𝑐 = 𝑎𝑏𝑐 ⇔ (𝑎𝑏𝑐)2 = 𝑎𝑏 + 𝑏𝑐 + 𝑎𝑐 (1)
𝑎
𝑐 2 (𝑎2 𝑏 2 +1) 𝑎2 𝑏 2 𝑐 2 +𝑐 2 𝑎𝑏+𝑏𝑐+𝑎𝑐+𝑐 2
𝑎2 𝑏 2 + 1 = = = =
𝑐2 𝑐2 𝑐2
𝑏(𝑎+𝑐)+𝑐(𝑎+𝑐) (𝑎+𝑐)(𝑏+𝑐)
= (2)
𝑐2 𝑐2

(𝑏+𝑎)(𝑐+𝑎)
Analog 𝑏 2 𝑐 2 + 1 = (3)
𝑎2
(𝑐+𝑏)(𝑎+𝑐)
𝑐 2 𝑎2 + 1 = (4)
𝑏2
(𝑎+𝑏)(𝑏+𝑐)(𝑐+𝑎) 2 (𝑎+𝑏)(𝑏+𝑐)(𝑐+𝑎)
Din (2),(3),(4)⇒𝐴 = [ ] ⇒𝐵= ∈ℚ
𝑎𝑏𝑐 𝑎𝑏𝑐

III.

Soluţia 1
Fie 𝐴′𝐶 ∩ 𝐶′𝑂 = {𝑀}. Fie 𝐴𝐷 = 𝑎, 𝐵𝐶 = 𝑏, 𝐴𝐴′ = 𝑐. Din ipoteză 𝑐 ≤ 𝑎 şi 𝑐 ≤ 𝑏.
𝑀𝑂 𝑂𝐶 1 𝑂𝐶′ 2𝑂𝐶′
△ 𝑀𝑂𝐶 ∼△ 𝑀𝐶′𝐴′ ⇒ 𝑀𝐶′ = 𝐴′𝐶′ = 2 ⇒ 𝑀𝑂 = 3 , 𝑀𝐶′ = 3
𝑙√2
(⇒) Presupun că 𝐴𝐵𝐶𝐷𝐴′𝐵′𝐶′𝐷′ este cub ⇒ 𝑎 = 𝑏 = 𝑐 = 𝑙, 𝐴𝐶 = 𝑙√2, 𝑂𝐶 =
2
2𝑙2 𝑙√6
𝑂𝐶′ = √𝑂𝐶 2 + 𝐶𝐶′2 = √ + 𝑙2 =
4 2
𝑙√6 𝑙√6
Avem 𝐶𝐶′2 = 𝐶′𝑀 ∙ 𝐶′𝑂 ⇒ 𝑙 2 = 3 ∙ 2 şi atunci cu reciproca teoremei catetei
⇒ 𝐶𝑀 ⊥ 𝑂𝐶′, adică 𝐶𝐴′ ⊥ 𝐶′𝑂.
(⇐) Presupunem că 𝐶′𝑂 ⊥ 𝐴𝐶′ ⇒ 𝐶𝑀 ⊥ 𝐶𝐶′ şi atunci cu teorema catetei avem

138
2 2 2
2𝑂𝐶 ′ 2 𝐶𝐶 ′ 2𝑂𝐶 ′
𝐶𝐶′2 = 𝐶′𝑀 ∙ 𝐶′𝑂 ⇔ 𝐶𝐶′2 = ⇒ 𝐶𝐶′2 = 3 (𝐶𝐶′2 + 𝑂𝐶 2 ) de unde = ⇒ 𝐶𝐶′2 =
3 3 3
𝐴𝐶 2 𝐴𝐶 2
2 ∙ ( 2 ) = 2 ⇒ 2𝐶𝐶′2 = 𝐴𝐵 2 + 𝐵𝐶 2 deci 2𝑐 2 = 𝑎2 + 𝑏 2 (1)
Dar 𝑐 ≤ 𝑎 şi 𝑐 ≤ 𝑏 ⇒ 𝑐 2 ≤ 𝑎2 , 𝑐 2 ≤ 𝑏 2 ⇒ 2𝑐 2 ≤ 𝑎2 + 𝑏 2 (2)
Din (1) şi (2) ⇒𝑐 = 𝑎, 𝑐 = 𝑏, adică 𝐴𝐵𝐶𝐷𝐴′𝐵′𝐶′𝐷′ este cub.
Soluţia 2
Fie 𝐴𝐵 = 𝑎, 𝐵𝐶 = 𝑏, 𝐶𝐶′ = 𝑐 şi vectorii ⃗⃗⃗⃗⃗ ⃗⃗⃗⃗⃗ , ⃗⃗⃗⃗⃗⃗⃗
𝐴𝐵 , 𝐵𝐶 𝐶𝐶 ′ .
1 𝐴𝐵+𝐵𝐶 ⃗⃗⃗⃗⃗
𝐴𝐵+𝐵𝐶 +2𝐶𝐶 ⃗⃗⃗⃗⃗ ⃗⃗⃗⃗⃗ ⃗⃗⃗⃗⃗ ⃗⃗⃗⃗⃗⃗⃗′
⃗⃗⃗⃗⃗⃗⃗
𝑂𝐶 ′ = 𝑂𝐶 ⃗⃗⃗⃗⃗ + ⃗⃗⃗⃗⃗⃗⃗
𝐶𝐶 ′ = 2 𝐴𝐶⃗⃗⃗⃗⃗ + ⃗⃗⃗⃗⃗⃗⃗
𝐶𝐶 ′ = 2 + ⃗⃗⃗⃗⃗⃗⃗
𝐶𝐶 ′ = 2
⃗⃗⃗⃗⃗⃗
𝐴 ⃗⃗⃗⃗⃗ − ⃗⃗⃗⃗⃗⃗⃗
′ 𝐶 = 𝐴𝐶 𝐴𝐴′ = 𝐴𝐵⃗⃗⃗⃗⃗ + 𝐵𝐶⃗⃗⃗⃗⃗ − ⃗⃗⃗⃗⃗⃗⃗
𝐶𝐶 ′
⃗⃗⃗⃗⃗
𝐴𝐵+𝐵𝐶 ⃗⃗⃗⃗⃗⃗⃗′
⃗⃗⃗⃗⃗ +2𝐶𝐶
𝑂𝐶′ ⊥ 𝐴′𝐶 ⟺ ⃗⃗⃗⃗⃗ + 𝐵𝐶
∙ (𝐴𝐵 ⃗⃗⃗⃗⃗ − ⃗⃗⃗⃗⃗⃗⃗
𝐶𝐶 ′ ) = 0 ⇔
2
⇔ 𝐴𝐵 2 + 𝐵𝐶 2 − 2𝐶𝐶′2 = 0 ⇒ (𝐴𝐵 ⃗⃗⃗⃗⃗ ∙ 𝐵𝐶
⃗⃗⃗⃗⃗ = ⃗⃗⃗⃗⃗ ⃗⃗⃗⃗⃗⃗ = 𝐵𝐶
𝐴𝐵 ∙ 𝐶𝐶′ ⃗⃗⃗⃗⃗⃗ = 0) ⇔
⃗⃗⃗⃗⃗ ∙ 𝐶𝐶′
⇔ 𝑎2 + 𝑏 2 = 2𝑐 2
Dar 𝑐 ≤ 𝑎 , 𝑐 ≤ 𝑏 şi atunci
𝑎2 + 𝑏 2 = 2𝑐 2 ⇔ 𝑐 2 = 𝑎2 , 𝑐 2 = 𝑏 2 ⇔ 𝑎 = 𝑏 = 𝑐 ⇔ 𝐴𝐵𝐶𝐷𝐴′𝐵′𝐶′𝐷′ este cub.

Clasa a X-a
I.

𝐴𝐶 ′ 𝐴𝐶 𝑏 𝐴𝐶 ′ 𝑏 𝑏𝑐 𝑎𝑐
̂ avem că: ′ =
a) Din [𝐶𝐶′ bisectoarea lui 𝐴𝐶𝐵 =𝑎⇒ = 𝑏+𝑎 ⇒ 𝐴𝐶′ = 𝑎+𝑏 , 𝐵𝐶′ = 𝑎+𝑏 .
𝐶 𝐵 𝐵𝐶 𝑐
𝐶 ′ 𝐴 𝐴′ 𝐵 𝐵′ 𝐶 𝑏 𝐴′ 𝐵
Din 𝐴𝐴′, 𝐵𝐵′, 𝐶𝐶′ concurente, cu teorema lui Ceva avem că 𝐶 ′ 𝐵 ∙ 𝐴′ 𝐶 ∙ 𝐵′ 𝐴 = 1 , deci 𝑎 ∙ 𝐴′ 𝐶 ∙ 1 =
𝐴′ 𝐵 𝑎 𝐶 ′𝐵 𝑎 𝐴′ 𝐵 𝐶′𝐵
1 ⇒ 𝐴′ 𝐶 = 𝑏 şi cum avem şi 𝐶 ′ 𝐴 = 𝑏 ⇒ 𝐴′ 𝐶 = 𝐶 ′ 𝐴 şi atunci cu teorema lui Thales ⇒ 𝐴′𝐶′ ∥ 𝐴𝐶 şi
cum 𝐵𝐵′ este mediană⇒ 𝐵𝐵′ ∩ 𝐴′𝐶′ = {𝑘} este mijlocul lui [𝐴′𝐶′].
𝐴′ 𝐵 𝑎 𝐴′ 𝐵 𝑎 𝑎2 𝑎𝑏
Mai avem =𝑏⟹ = 𝑎+𝑏 ⇒ 𝐴′ 𝐵 = 𝑎+𝑏 , 𝐴′ 𝐶 = 𝑎+𝑏 .
𝐴′ 𝐶 𝑎
𝑠 𝜎(𝐴′ 𝐵′ 𝐶 ′ ) 𝜎(𝐴𝐵𝐶)−𝜎(𝐴𝐶 ′ 𝐵′ )−𝜎(𝐵𝐶 ′ 𝐴′ )−𝜎(𝐶𝐴′ 𝐵′ ) 𝜎(𝐴𝐶 ′ 𝐵′ ) 𝜎(𝐵𝐶 ′ 𝐴′ ) 𝜎(𝐶𝐴′ 𝐵′ )
b) = = = 1−( + + )=
𝑆 𝜎(𝐴𝐵𝐶) 𝜎(𝐴𝐵𝐶) 𝜎(𝐴𝐵𝐶) 𝜎(𝐴𝐵𝐶) 𝜎(𝐴𝐵𝐶)
𝑏𝑐 𝑏 𝑎𝑐 𝑎2 𝑎𝑏 𝑏
𝐴𝐶 ′ ∙𝐴𝐵′ 𝐵𝐶 ′ ∙𝐵𝐴′ 𝐶𝐴′ ∙𝐶𝐵′ ∙ ∙ ∙ 𝑏 𝑎2
1 − ( 𝐴𝐵∙𝐴𝐶 + + )=1−( 𝑎+𝑏 2
+ 𝑎+𝑏 𝑎+𝑏
+ 𝑎+𝑏 2
) = 1 − (2(𝑎+𝑏) + (𝑎+𝑏)2 +
𝐵𝐴∙𝐵𝐶 𝐶𝐵∙𝐶𝐴 𝑏𝑐 𝑎𝑐 𝑎𝑏

139
𝑏 𝑏 𝑎2 𝑎𝑏+𝑏 2 +𝑎2 𝑎𝑏 1
) = 1 − (𝑎+𝑏 + (𝑎+𝑏)2 ) = 1 − (𝑎+𝑏)2
= (𝑎+𝑏)2 ≤ 4 ⇔ 4𝑎𝑏 ≤ (𝑎 + 𝑏)2 ⇔ 0 ≤
2(𝑎+𝑏)
(𝑎 − 𝑏)2

𝐈𝐈. 𝑎, 𝑏, 𝑐 > 0 cu 𝑎2 + 𝑏 2 + 𝑐 2 + 𝑎𝑏𝑐 = 4, 𝑐 2 + 𝑎𝑏𝑐 + (𝑎2 + 𝑏 2 − 4) = 0 ecuaţie în “c”


Δ = 𝑎2 𝑏 2 − 4(𝑎2 + 𝑏 2 − 4) = 𝑎2 𝑏 2 − 4𝑎2 − 4𝑏 2 + 16 = (𝑎2 − 4)(𝑏 2 − 4) > 0 deoarece din
𝑎2 + 𝑏 2 + 𝑐 2 + 𝑎𝑏𝑐 = 4 şi 𝑎, 𝑏, 𝑐 > 0 ⇒ 𝑎2 < 4, 𝑏 2 < 4.
𝜋
De asemenea putem nota 𝑎 = 2 cos 𝐴 , 𝑏 = 2 cos 𝐵 cu 𝐴, 𝐵 ∈ (0, 2 ) şi atunci
Δ = (4𝑐𝑜𝑠 2 𝐴 − 4)(4𝑐𝑜𝑠 2 𝐵 − 4) = 16(𝑐𝑜𝑠 2 𝐴 − 1)(𝑐𝑜𝑠 2 𝐵 − 1) = 16 ∙ 𝑠𝑖𝑛2 𝐴 ∙ 𝑠𝑖𝑛2 𝐵
−4 cos A cos B±4 sin A sin B
c1,2 = = −2(cos A cos B ± sin A sin B) = −2 cos(A ± B)
2
𝜋 𝜋 𝜋
Din 𝐴, 𝐵 ∈ (0, 2 ) ⇒ 𝐴 − 𝐵 ∈ (− 2 , 2 ) şi deci cos(𝐴 − 𝐵) > 0. Cum 𝑐 > 0 ⇒ convine 𝑐 =
𝜋
−2 cos(𝐴 + 𝐵) = 2 cos(𝜋 − (𝐴 + 𝐵)) = 2 cos 𝐶 unde am notat 𝑐 = 𝜋 − (𝐴 + 𝐵) ∈ (0, 2 )
deoarece 𝑐 > 0 . Deci 𝑎 = 2 cos 𝐴 , 𝑏 = 2 cos 𝐵,
𝜋
𝑐 = 2 cos 𝐶 ; 𝐴, 𝐵, 𝐶 ∈ (0, 2 ) , 𝐴 + 𝐵 + 𝐶 = 𝜋
𝐴
2−𝑎 2−2 cos 𝐴 1−cos 𝐴 2𝑠𝑖𝑛2 𝐴
= 2+2 cos 𝐴 = 1+cos 𝐴 = 2
𝐴 = 𝑡𝑔2 2
2+𝑎 2𝑐𝑜𝑠2
2
Atunci relaţia de demonstrat devine:
𝐴 𝐵 𝐵 𝐶 𝐶 𝐴
𝑡𝑔 2 𝑡𝑔 2 + 𝑡𝑔 2 𝑡𝑔 2 + 𝑡𝑔 2 𝑡𝑔 2 = 1 (1)
𝐴 𝐵
𝐶 𝜋−(𝐴+𝐵) 𝜋 𝐴+𝐵 𝐴+𝐵 1 1−𝑡𝑔 𝑡𝑔
Dar 𝑡𝑔 2 = 𝑡𝑔 = 𝑡𝑔 ( 2 − ) = 𝑐𝑡𝑔 = 𝐴+𝐵 = 𝐴
2
𝐵
2
de unde
2 2 2 𝑡𝑔 𝑡𝑔 +𝑡𝑔
2 2 2
𝐶 𝐴 𝐵 𝐴 𝐵
𝑡𝑔 2 (𝑡𝑔 2 + 𝑡𝑔 2 ) = 1 − 𝑡𝑔 2 𝑡𝑔 2 adică (1).

III. Arătăm că discul unitate 𝐷1 = {𝑧 ∈ ℂ| |𝑧| ≤ 1} este inclus în M. Din ipoteza 1° avem că
cercul unitate este inclus în M şi 0 = 1 + (−1) ∈ 𝑀 din 2° . Fie acum 𝑧 ∈ 𝐷1 , 𝑧 ≠ 0, |𝑧| < 1.

Dacă 𝐴(𝑧) este imaginea lui z în planul complex, notăm cu 𝐵(𝑧1 ) şi 𝐶(𝑧2 ) punctele în care
mediatoarea lui [𝑂𝐴] taie cercul-unitate.
Din rombul OBAC⇒𝑂𝐴 ⃗⃗⃗⃗⃗ = 𝑂𝐵
⃗⃗⃗⃗⃗ + 𝑂𝐶
⃗⃗⃗⃗⃗ şi deci 𝑧 = 𝑧1 + 𝑧2 . Dar 𝑧1 , 𝑧2 ∈ 𝑀(din 1° ) şi atunci 𝑧 ∈
°
𝑀(din 2 ).
Pentru fiecare 𝑛 ∈ ℕ notăm 𝐷2𝑛 discul de centru O şi rază 2𝑛 , adică 𝐷2𝑛 = {𝑧 ∈ ℂ| |𝑧| ≤ 2𝑛 }
Prin inducţie arătăm că 𝐷2𝑛 ⊆ 𝑀, (∀)𝑛 ∈ ℕ.

140
Pentru 𝑛 = 0 avem 𝐷1 ⊆ 𝑀 conform celor de mai sus. Dacă 𝐷2𝑘 ⊆ 𝑀, atunci 𝐷2𝑘+1 ⊆ 𝑀
𝑧 𝑧 𝑧
deoarece (∀)𝑧 ∈ 𝐷2𝑘+1 ⇒ |𝑧| ≤ 2𝑘+1 ⇒ |2| ≤ 2𝑘 ⇒ 2 ∈ 𝐷2𝑘 ⊆ 𝑀 şi deci 2 ∈ 𝑀 şi atunci
𝑧 𝑧
𝑧 = 2 + 2 ∈ 𝑀.
Deoarece ℂ = ⋃𝑛≥0 𝐷2𝑛 şi 𝐷2𝑛 ⊆ 𝑀, (∀)𝑛 ≥ 0 ⇒ ℂ ⊆ 𝑀 şi deci 𝑀 = ℂ.

Clasa a XI-a

I. a) Fie |𝑧1 | = |𝑧2 | = |𝑧3 | = 𝑟 > 0 ⇒ |𝑧1 |2 = |𝑧2 |2 = |𝑧3 |2 = 𝑟 2 adică 𝑧1 ∙ 𝑧̅1 = 𝑧2 ∙ 𝑧̅2 = 𝑧3 ∙
𝑟2 𝑟2 𝑟2
𝑧̅3 = 𝑟 2 ⇒ 𝑧̅1 = , 𝑧̅2 = , 𝑧̅3 = .
𝑧1 𝑧2 𝑧3
𝑟2 𝑟2 𝑟2
𝑧1 + 𝑧2 + 𝑧3 = 0 ⇔ 𝑧̅1 + 𝑧̅2 + 𝑧̅3 = 0 ⇔ +𝑧 +𝑧 =0⇔
𝑧1 2 3
𝑟 2 (𝑧2 𝑧3 +𝑧1 𝑧3 +𝑧1 𝑧2 )
⇔ = 0 ⇔ 𝑧2 𝑧3 + 𝑧1 𝑧3 + 𝑧1 𝑧2 = 0.
𝑧1 𝑧2 𝑧3
b) 𝑧1 + 𝑧2 + 𝑧3 = 0 ⇒ 𝑧2 + 𝑧3 = −𝑧1 ⇒ |𝑧2 + 𝑧3 | = |𝑧1 | (1) 𝑧2 𝑧3 +
𝑧1 𝑧3 + 𝑧1 𝑧2 = 0 ⇒ 𝑧1 (𝑧2 + 𝑧3 ) = −𝑧2 𝑧3 ⇒ |𝑧1 ||𝑧2 + 𝑧3 | = |𝑧2 ||𝑧3 | (2)
Din (1), (2) ⇒|𝑧1 |2 = |𝑧2 ||𝑧3 | . Analog se deduce că |𝑧2 |2 = |𝑧1 ||𝑧3 |, |𝑧3 |2 = |𝑧1 ||𝑧2 |.
Obţinem |𝑧1 |2 ∙ |𝑧1 ||𝑧3 | = |𝑧2 ||𝑧3 | ∙ |𝑧2 |2 şi cum 𝑧1 , 𝑧2 , 𝑧3 ∈ ℂ∗ ⇒ |𝑧1 |3 = |𝑧2 |3 ⇒|𝑧1 | = |𝑧2 |.
Analog |𝑧2 | = |𝑧3 |.

II.
𝑛
a). Cu binomul lui Newton avem că (2 + √3) = 𝑝𝑛 + 𝑞𝑛 √3 cu 𝑝𝑛 , 𝑞𝑛 ∈ ℕ
𝑛 𝑛 𝑛
Trecând la conjugată avem: (2 − √3) = 𝑝𝑛 − 𝑞𝑛 √3, de unde (2 + √3) + (2 − √3) = 2𝑝𝑛
𝑛 𝑛
şi atunci (2 + √3) = (2𝑝𝑛 − 1) + [1 − (2 − √3) ], ∀𝑛 ∈ ℕ∗ .
𝑛 𝑛 𝑛
Cum 2𝑝𝑛 − 1 ∈ ℤ şi 1 − (2 − √3) ∈ (0,1) obţinem că 𝑎𝑛 = {(2 + √3) } = 1 − (2 − √3) şi
atunci lim𝑛→∞ 𝑎𝑛 = 1 − 0 = 1.
La fel ca mai sus
𝑛 𝑛
(1 + √2) = 𝑝𝑛 + 𝑞𝑛 √2 , (1 − √2) = 𝑝𝑛 − 𝑞𝑛 √2 cu 𝑝𝑛 , 𝑞𝑛 ∈ ℕ
𝑛 𝑛
(1 + √2) + (1 − √2) = 2𝑝𝑛 de unde
𝑛 𝑛
(1 + √2) = 2𝑝𝑛 − (1 − √2) (1)
𝑛 𝑛 𝑛
Pentru n impar (1) se scrie : (1 + √2) = 2𝑝𝑛 + (√2 − 1) şi atunci 𝑏𝑛 = {(1 + √2) } =
𝑛
(√2 − 1) .
𝑛 𝑛
Pentru n par (1) se scrie: (1 + √2) = (2𝑝𝑛 − 1) + [1 − (√2 − 1) ] şi atunci 𝑏𝑛 =
𝑛 𝑛 2𝑛−1
{(1 + √2) } = 1 − (√2 − 1) . Aşadar lim𝑛→∞ 𝑏2𝑛−1 = lim𝑛→∞ (√2 − 1) =0.
2𝑛
lim𝑛→∞ 𝑏2𝑛 = lim𝑛→∞ [1 − (√2 − 1) ] = 1 ceea ce arată că (𝑏𝑛 )𝑛≥1 nu are limită.

𝑥12 𝑥22 2
𝑥𝑛 𝑥1 +𝑥2 +⋯+𝑥𝑛
III. Pentru 𝑎 = 1 avem:𝑥 +𝑥 + ⋯+ 𝑥 ≥ care rezultă
1 +𝑥2 +⋯+𝑥𝑛 1 +𝑥2 +⋯+𝑥𝑛 1 +𝑥2 +⋯+𝑥𝑛 𝑛
)2
din :(𝑥1 + 𝑥2 + ⋯ + 𝑥𝑛 = (1 ∙ 𝑥1 + 1 ∙ 𝑥2 + ⋯ + 1 ∙ 𝑥𝑛 ≤ + 1 + ⋯ + 1 ∙ (𝑥12 + 𝑥22 + )2 (12 2 2)

⋯ + 𝑥𝑛2 )
Pentru 𝑎 ≠ 1, notăm 𝑦1 = 𝑎𝑥1 + 𝑥2 + ⋯ + 𝑥𝑛 , 𝑦2 = 𝑥1 + 𝑎𝑥2 + ⋯ + 𝑥𝑛 ,....
𝑦𝑛 = 𝑥1 + 𝑥2 + ⋯ + 𝑎𝑥𝑛 ⇒ 𝑦1 = 𝑆 − (1 − 𝑎)𝑥1 ≥ 0, 𝑦2 = 𝑆 − (1 − 𝑎)𝑥2 ≥ 0 ,..

141
… 𝑦𝑛 = 𝑆 − (1 − 𝑎)𝑥𝑛 ≥ 0 .
Avem 𝑦1 + 𝑦2 + ⋯ + 𝑦𝑛 = 𝑛𝑆 − (1 − 𝑎)𝑆 = 𝑆(𝑛 + 𝑎 − 1)
𝑆−𝑦1 2 𝑆−𝑦2 2 𝑆−𝑦𝑛 2
𝑥12 𝑥22 2
𝑥𝑛 ( ) ( ) ( )
1−𝑎 1−𝑎 1−𝑎
+ + ⋯+ = + +⋯+ =
𝑎𝑥1 +𝑥2 +⋯+𝑥𝑛 𝑥1 +𝑎𝑥2 +⋯+𝑥𝑛 𝑥1 +𝑥2 +⋯+𝑎𝑥𝑛 𝑦1 𝑦2 𝑦𝑛
1 𝑆 2 −2𝑆𝑦1 +𝑦12 𝑆 2 −2𝑆𝑦2 +𝑦22 𝑆 2 −2𝑆𝑦𝑛 +𝑦𝑛2 1 1 1 1
= (1−𝑎)2 ( + + ⋯+ ) = (1−𝑎)2 [𝑆 2 (𝑦 + 𝑦 + ⋯ + 𝑦 ) − 2𝑛𝑆 +
𝑦1 𝑦2 𝑦𝑛 1 2 𝑛
𝑆 1 1 1 𝑆 1 1
𝑦1 + 𝑦2 + ⋯ + 𝑦𝑛 ] = (1−𝑎)2 [𝑆 (𝑦 + 𝑦 + ⋯ + 𝑦 ) − 2𝑛 + 𝑛 + 𝑎 − 1] = (1−𝑎)2 [𝑆 (𝑦 + 𝑦 +
1 2 𝑛 1 2
1
⋯ + 𝑦 ) + 𝑎 − 𝑛 − 1]
𝑛
1 1 1
Dar (𝑦 + 𝑦 + ⋯ + 𝑦 ) (𝑦1 + 𝑦2 + ⋯ + 𝑦𝑛 ) ≥ 𝑛2 de unde
1 2 𝑛
1 1 1 𝑛2 𝑛2
+𝑦 +⋯+𝑦 ≥ 𝑦 ≥ 𝑆(𝑛+𝑎−1)
𝑦1 2 𝑛 1 +𝑦2 +⋯+𝑦𝑛

În final avem
𝑥12 𝑥22 2
𝑥𝑛 𝑆 𝑛2
+𝑥 + ⋯+ 𝑥 ≥ (1−𝑎)2 [𝑆 ∙ 𝑆(𝑛+𝑎−1) + 𝑎 − 𝑛 − −1] =
𝑎𝑥1 +𝑥2 +⋯+𝑥𝑛 1 +𝑎𝑥2 +⋯+𝑥𝑛 1 +𝑥2 +⋯+𝑎𝑥𝑛
𝑆 𝑛2 +(𝑎−1)2 −𝑛2 𝑆
(1−𝑎)2
= 𝑛+𝑎−1 .
𝑛+𝑎−1

Clasa a XII-a

1 0 1 0 2 0
I. Matricea dată este 𝑀 = 𝑥𝐴 + 𝑦𝐵, unde 𝐴 = (0 2 0), 𝐵 = (1 0 1)
1 0 1 0 2 0
Avem prin calcul 𝐴2 = 2𝐴, 𝐵 2 = 2𝐴, 𝐴𝐵 = 𝐵𝐴 = 2𝐵 de unde
𝐴𝑘 = 2𝑘−1 𝐴, 𝐵 3 = 22 𝐵, 𝐵 4 = 23 𝐴, ⋯ , 𝐵 2𝑘+1 = 22𝑘 𝐵, 𝐵 2𝑘 = 22𝑘−1 𝐴
𝑀𝑛 = (𝑥𝐴 + 𝑦𝐵)𝑛 = (𝐶𝑛0 𝑥 𝑛 𝐴𝑛 + 𝐶𝑛2 𝑥 𝑛−2 𝐴𝑛−2 𝑦 2 𝐵 2 + 𝐶𝑛4 𝑥 𝑛−4 𝐴𝑛−4 𝑦 4 𝐵 4 + ⋯ ) +
(𝐶𝑛1 𝑥 𝑛−1 𝐴𝑛−1 𝑦𝐵 + 𝐶𝑛3 𝑥 𝑛−3 𝐴𝑛−3 𝑦 3 𝐵 3 + 𝐶𝑛5 𝑥 𝑛−5 𝐴𝑛−5 𝑦 5 𝐵 5 + ⋯ ) = (𝐶𝑛0 𝑥 𝑛 2𝑛−1 𝐴 +
𝐶𝑛2 𝑥 𝑛−2 2𝑛−3 𝐴 ∙ 2𝑦 2 𝐴 + 𝐶𝑛4 𝑥 𝑛−4 2𝑛−5 𝐴 ∙ 23 𝑦 4 𝐴 + ⋯ ) + (𝐶𝑛1 𝑥 𝑛−1 2𝑛−2 𝐴 ∙ 𝑦𝐵 + 𝐶𝑛3 𝑥 𝑛−3 2𝑛−4 𝐴 ∙
22 𝑦 3 𝐵 + 𝐶𝑛5 𝑥 𝑛−5 2𝑛−6 𝐴 ∙ 24 𝑦 5 𝐵 + ⋯ ) = 2𝑛−1 (𝐶𝑛0 𝑥 𝑛 + 𝐶𝑛2 𝑥 𝑛−2 𝑦 2 + 𝐶𝑛4 𝑥 𝑛−4 𝑦 4 + ⋯ )𝐴 +
2𝑛−1 (𝐶𝑛1 𝑥 𝑛−1 𝑦 + +𝐶𝑛3 𝑥 𝑛−3 𝑦 3 + 𝐶𝑛5 𝑥 𝑛−5 𝑦 5 + ⋯ )𝐵 =
(𝑥+𝑦)𝑛 +(𝑥−𝑦)𝑛 (𝑥+𝑦)𝑛 −(𝑥−𝑦)𝑛
2𝑛−1 [ 𝐴+ 𝐵] =
2 2
(𝑥 + 𝑦) + (𝑥 − 𝑦)𝑛
𝑛
2[(𝑥 + 𝑦)𝑛 − (𝑥 − 𝑦)𝑛 ] (𝑥 + 𝑦)𝑛 + (𝑥 − 𝑦)𝑛
2𝑛−2 ((𝑥 + 𝑦)𝑛 − (𝑥 − 𝑦)𝑛 2[(𝑥 + 𝑦)𝑛 + (𝑥 − 𝑦)𝑛 ] (𝑥 + 𝑦)𝑛 − (𝑥 − 𝑦)𝑛 )
(𝑥 + 𝑦)𝑛 + (𝑥 − 𝑦)𝑛 2[(𝑥 + 𝑦)𝑛 − (𝑥 − 𝑦)𝑛 ] (𝑥 + 𝑦)𝑛 + (𝑥 − 𝑦)𝑛

II. Relaţia 𝑓(2𝑥) = 𝑓 2 (𝑥) + 𝑥 2 + 2𝑥(1 − 𝑓(𝑥)) se mai scrie 𝑓(2𝑥) − 2𝑥 = (𝑓(𝑥) − 𝑥)2
Notăm 𝑔(𝑥) = 𝑓(𝑥) − 𝑥, 𝑔: ℝ → ℝ.
Din f derivabilă în 0, 𝑓 ′ (0) = 1 + 𝑓(0) = 2 ⇒ 𝑔 derivabilă în 0, 𝑔′ (0) = 𝑓 ′ (0) − 1 = 1,
𝑔(0) = 𝑓(0) − 0 = 1
𝑥 𝑥 𝑥 𝑥
Avem 𝑔(2𝑥) = 𝑔2 (𝑥) de unde avem :𝑔(𝑥) = 𝑔2 (2) = 𝑔4 (22 ) = 𝑔8 (23 ) = ⋯ = 𝑔2𝑛 (2𝑛) .
𝑥 𝑥
Din lim𝑛→∞ 2𝑛 = 0 şi g continuă în 0 (fiind derivabilă în 0)⇒ lim𝑛→∞ 𝑔 (2𝑛) = 𝑔(0) = 1 şi
atunci:

142
𝑥
1 2𝑛 (𝑔( 𝑛)−1)
2
𝑛 𝑥 𝑥 𝑥
lim𝑛→∞ 𝑔2 (2𝑛) = [1∞ ] = lim𝑛→∞ [(1 + 𝑔 (2𝑛 ) − 1) 𝑔( 𝑛 )−1
2 ] = 𝑒𝐿 ,
𝑥 𝑥
𝑥 𝑔( 𝑛)−1 𝑔( 𝑛)−𝑔(0)
𝑛
Unde, 𝐿 = lim𝑛→∞ 2 (𝑔 (2𝑛) − 1) = lim𝑛→∞ 2
1 = lim𝑛→∞ 2
𝑥 ∙ 𝑥 = 𝑔′ (0) ∙ 𝑥 = 𝑥
−0
2𝑛 2𝑛
.
Atunci :
𝑛 𝑥
𝑔(𝑥) = lim𝑛→∞ 𝑔2 (2𝑛) = 𝑒 𝑥 de unde 𝑓(𝑥) = 𝑒 𝑥 + 𝑥.

III.. Fie 𝑘1 , 𝑘2 ∈ ℤ cu (𝑘1 , 𝑛) = 1, (𝑘2 , 𝑛) = 1.


(𝑓𝑘1 °𝑓𝑘2 )(𝑥) = 𝑓𝑘1 (𝑓𝑘2 (𝑥)) = 𝑓𝑘1 (𝑥 𝑘1 ) = (𝑥 𝑘1 )𝑘2 = 𝑥 𝑘1 𝑘2 şi 𝑘1 𝑘2 ∈ ℤ, (𝑘1 𝑘2 , 𝑛) = 1 ⇒
𝑓𝑘1 °𝑓𝑘2 = 𝑓𝑘1 𝑘2 ∈ Γ.
Avem (𝑓𝑘1 °𝑓𝑘2 )°𝑓𝑘3 = 𝑓𝑘1 °(𝑓𝑘2 °𝑓𝑘3 ), (∀) 𝑓𝑘1 , 𝑓𝑘2 , 𝑓𝑘3 ∈ Γ deoarece compunerea funcţiilor est
asociativă.
Avem 𝑓𝑘1 °𝑓𝑘2 = 𝑓𝑘2 °𝑓𝑘1 = 𝑓𝑘1 𝑘2 , (∀) 𝑓𝑘1 , 𝑓𝑘2 ∈ Γ
Se ştie că 𝑓𝑘 °1𝐺 = 1𝐺 °𝑓𝑘 = 𝑓𝑘 , (∀)𝑓𝑘 ∈ Γ şi cum 1𝐺 = 𝑓1 ⇒ 𝑓1 este elementul neutru .
Arătăm acum un lucru important şi anume (∗)𝑘1 , 𝑘2 ∈ ℤ , (𝑘1 , 𝑛) = 1, (𝑘2 , 𝑛) = 1, 𝑘1 ≡
𝑘2 (𝑚𝑜𝑑 𝑛) ⇒ 𝑓𝑘1 = 𝑓𝑘2 .
Într-adevăr, scriind 𝑘2 = 𝑛𝑞 + 𝑘1 , 𝑞 ∈ ℤ avem 𝑓𝑘2 (𝑥) = 𝑥 𝑘2 = 𝑥 𝑛𝑞+𝑘1 = (𝑥 𝑛 )𝑞 𝑥 𝑘1 = 𝑥 𝑘1 =
𝑓𝑘1 (𝑥), (∀)𝑥 ∈ Γ şi deci 𝑓𝑘1 = 𝑓𝑘2 .
Fie acum 𝑓𝑘 ∈ Γ. Din (𝑘, 𝑛) = 1 ⇒ (∃)𝑎, 𝑏 ∈ ℤ astfel încât 𝑎𝑘 + 𝑏𝑛 = 1, deci 𝑎𝑘 ≡ 1(𝑚𝑜𝑑 𝑛)
şi atunci, cu cele de mai sus avem 𝑓𝑎𝑘 = 𝑓1 , adică 𝑓𝑘 °𝑓𝑎 = 𝑓𝑎 °𝑓𝑘 = 𝑓1 , ceea ce arată că inversa
lui 𝑓𝑘 ∈ Γ este 𝑓𝑎 ∈ Γ.
Aşadar (Γ, °) este grup abelian. Acest grup este finit, deoarece
Γ = {𝑓𝑘 |1 ≤ 𝑘 ≤ 𝑛, (𝑘, 𝑛) = 1} din (∗) şi atunci are cel mult 𝜑(𝑛) elemente (atenţie funcţiile 𝑓𝑘
, 1 ≤ 𝑘 ≤ 𝑛 , (𝑘, 𝑛) = 1 nu sunt neapărat distincte două câte două).

EDITIA a VIII-a, 15-16 noiembrie 2002


Clasa a V-a

I. 1) 20023 − 20022 · 2001 − 2002 · 2001 − 2002 = 2002(20022 − 2002 · 2001 −


2001 − 1) = 2002[2002(2002 − 2001) − 2001 − 1] = 0
0 2 5 20 0
2) [52002 + 55 ∙ 52 : (53 )19 + (52 ∙ 25 )4 : (57 ∙ 219 )] − 22 ∙ 33 = [5 + 525 ∙ 532 : 557 +
(58 ∙ 220 ): (57 ∙ 219 )] − 22 ∙ 3 = 5 + 525+32−57 + 58−7 ∙ 22 ∙ 3 = 5 + 1 + 5 ∙ 2 − 22 ∙ 3 = 5 +
1 + 10 − 12 = 4
100∙101
3) 𝑎 = 3 ∙ 42524 = 3 ∙ 25048 , 𝑏 = 21+2+3+⋯+100 − (16631 )2 = 2 2 − 24∙631∙2 = 250∙101 −
28∙631 = 25050 − 25048 = 25048 (22 − 1) = 25048 ∙ 3 ⇒ 𝑎 = 𝑏 şi atunci
(𝑎 − 𝑏) ∙ 20022002 : 20012001 = 0
4) “0” deoarece unul din factorii produsului este 24 − 42 .

143
II. 1) 1000𝑦 + 2(1 + 2 + 3 + ⋯ + 1001) = 1001 ∙ 2002, 1000𝑦 + 1001 ∙ 1002 = 1001 ∙
2002 , 1000𝑦 = 1001(2002 − 1002) = 1001 ∙ 1000 ⇒ 𝑦 = 1001
2) 4 ∙ (22𝑎 ∙ 5𝑎 + 𝑥𝑦𝑦
̅̅̅̅̅) + 2𝑏 = 2001.
Considerând paritatea numerelor de mai sus ⇒𝑏 = 0 şi avem : 20𝑎 + 𝑥𝑦𝑦
̅̅̅̅̅ = 500 ⇒ 𝑎𝜖{0,1,2}.
𝑎 = 0 ⇒ 𝑥𝑦𝑦̅̅̅̅̅ = 499 ⇒ 𝑥 = 4, 𝑦 = 9
𝑎 = 1 ⇒ 𝑥𝑦𝑦̅̅̅̅̅ = 480 nu convine
𝑎 = 0 ⇒ 𝑥𝑦𝑦̅̅̅̅̅ = 100 ⇒ 𝑥 = 1, 𝑦 = 0.
6∙7
3) 𝑥 + (𝑥 + 1) + (𝑥 + 2) + ⋯ + (𝑥 + 6) = 7𝑥 + 2 = 7𝑥 + 21 = 7(𝑥 + 3)
̅̅̅̅̅̅̅̅̅
1𝑎000=10000 + 𝑎 ∙ 1000 = 1000(10 + 𝑎) deci 7(𝑥 + 3) = 1000(10 + 𝑎)
⇒7|1000(10 + 𝑎)
𝑎 − 𝑐𝑖𝑓𝑟𝑎⇒𝑎 = 4.
7(𝑥 + 3) = 1000 ∙ 14 ⇒ 𝑥 + 3 = 2000 ⇒ 𝑥 = 1997
Numerele sunt 1997, 1998, ..., 2003.

III.
̅̅̅̅̅ ̅̅̅̅̅ 100𝑎 + 10𝑏 + 𝑐 = 200𝑐 + 20𝑏 + 2𝑎 + 100
{𝑎𝑏𝑐 = 2𝑐𝑏𝑎 + 100 ⇔ {
𝑎−𝑐 =4 𝑎−𝑐 = 4

98𝑎 = 199𝑐 + 10𝑏 + 100


⇔{
𝑎 =𝑐+4
98(𝑐 + 4) = 199𝑐 + 10𝑏 + 100
101𝑐 + 10𝑏 = 292 ⇒ 𝑐 = 2 ⇒ 𝑏 = 9, 𝑎 = 6 𝑑𝑒𝑐𝑖 ̅̅̅̅̅
𝑎𝑏𝑐 = 692

Clasa a VI-a

2660∙10𝑛 266 266


I. 1) 𝑎 = 10∙10𝑛(2+3∙52+7∙23 ) = 2+75+56 = 133 = 2, 𝑏 = 3
𝑎39 = 239 = (23 )13 = 813 , 𝑏 26 = 326 = (32 )13 = 913 deci 𝑎39 < 𝑏 26
2) 𝑛 = 2 ∙ 11(𝑎 + 𝑏 + 𝑐)
a) 11|22 ⇒ 11|𝑛
b) 9|𝑛 ⇒ 9|𝑎 + 𝑏 + 𝑐 ⇒ 9|𝑎𝑏𝑐 ̅̅̅̅̅
c) 𝑛 = 2 ∙ 11(𝑎 + 𝑏 + 𝑐)
𝑛 𝑒𝑠𝑡𝑒 𝑝𝑎𝑡𝑟𝑎𝑡 𝑝𝑒𝑟𝑓𝑒𝑐𝑡 ⇒ 𝑎 + 𝑏 + 𝑐 = 22 de unde
̅̅̅̅̅
𝑎𝑏𝑐 𝜖{985,976,967, 958, 895,859,796,769,697,679,598,589}

̂ = 50𝑜 , 𝐵𝑂𝐶
II. 1) 𝐴𝑂𝐵 ̂ = 130𝑜
2) 𝑀40 ∙ 𝑀50 = 𝑀0 ∙ 𝑀50 − 𝑀0 ∙ 𝑀40 = (1 + 2 + ⋯ + 50) − (1 + 2 + ⋯ + 40)
50∙51 40∙41
= 2 − 2 = 455 cm.

Clasa a VII-a

I. Pentru 𝑛 = 0 ⇒ 𝑏 + 𝑐√2 = 0 şi cum 𝑏, 𝑐𝜖ℚ ⇒ 𝑏 = 𝑐 = 0 ⇒ 𝑎 = 0.

II. Construim 𝑀𝑁||𝐴𝐷, MN=AD astfel încât N şi A sunt de o parte şi de cealaltă a dreptei BM.
̂ ) = 30𝑜 .
Demonstrând că ∆𝐵𝑀𝑁 este echilateral, obţinem 𝑚(𝑀𝐵𝐶

144
III. Din relaţia dată obţinem (𝑥 + 𝑦 − 1)(9𝑥 + 𝑦) = 90. Singurele cifre x,y care satisfac această
egalitate sunt 𝑥 = 1, 𝑦 = 6.

Clasa a VIII-a
I. 𝑎𝜖[1, ∞)

II. Dacă PA nu intersectează cercul (C) şi într-un alt punct decât A, atunci 𝑃𝐵 ⊥ 𝐴𝐵.
Dacă PB nu intersectează cercul (C) şi într-un alt punct decît B, atunci 𝑃𝐵 ⊥ 𝐴𝐵.
În caz contrar, fie {𝑀} = (𝐶) ∩ 𝐴𝑃 , {𝑁} = (𝐶) ∩ 𝐵𝑃. Avem că BM şi AN sunt înălţimi ale
∆𝐴𝐵𝐶 ⇒ {𝐻} = 𝐵𝑀 ∩ 𝐴𝑁 este ortocentrul ∆𝑃𝐴𝐵 ⇒ 𝑃𝐻 ⊥ 𝐴𝐵.

III.. Vezi exerciţiul I clasa a VII-a.

Clasa a IX-a

I. ∆= 𝑏 2 − 4𝑎𝑐 = 𝑏 2 − 2𝑐 ∙ 2𝑎 = 𝑏 2 − 2𝑐(−3𝑏 − 6𝑐) = 𝑏 2 + 6𝑏𝑐 + 12𝑐 2 = (𝑏 + 3𝑐)2 + 3𝑐 2


Dacă ∆= 0 ⇒ 𝑏 + 3𝑐 = 𝑐 = 0 ⇒ 𝑏 = 𝑐 = 0 şi deci şi 𝑎 = 0 contradicţie, deci ∆> 0 ⇒ ecuaţia
are rădăcini reale distincte.
𝑏 𝑐 𝑏 𝑐
𝑥1 , 𝑥2 rădăcinile ecuaţiei 𝑥 2 + 𝑎 𝑥 + 𝑎 = 0. Notăm 𝑚 = 𝑎 , 𝑛 = 𝑎 şi din
2𝑎 + 3𝑏 + 6𝑐 = 0 ⇒ 3𝑚 + 6𝑛 = −2 (1)

𝑥1 < 0
Presupunem că {𝑥1 , 𝑥2 } ∩ [0,1] = ∅ ⇔ 𝑥1 , 𝑥2 < 0 sau 𝑥1 , 𝑥2 > 1 sau {
𝑥2 > 1
𝑥1 < 0 𝑥1 + 𝑥2 < 0 −𝑚 < 0 𝑚>0
Dacă { ⇔{ ⇔{ ⇔{ contradicţie cu (1)
𝑥2 < 1 𝑥1 , 𝑥2 > 0 𝑛>0 𝑛>0
𝑥 >1 𝑥 − 1 + 𝑥2 − 1 > 0 −𝑚 − 2 > 0 −𝑚 − 2 > 0 𝑚 < −2
Dacă { 1 ⇔{ 1 ⇔{ ⇔ {−2−3𝑚 + 𝑚 + 1 > 0 ⇔ {𝑚 > − 4
𝑥2 > 1 (𝑥1 − 1)(𝑥2 − 1) > 0 𝑛 + +1 > 0 6 3
contradicţie.
𝑥 <0 𝑥1 𝑥2 < 0 𝑛<0 𝑛<0
Dacă { 1 ⇔ { ⇒{ ⇒ {𝑛 + −2−6𝑛 + 1 < 0
𝑥2 > 1 (𝑥1 − 1)(𝑥2 − 1) < 0 𝑛+𝑚+1>0 3
𝑛<0
⇒{ 𝑛 > 1 contradicţie.
3

II. Fie 𝐴′ , 𝐵 ′ proiecţiile lui A,B pe planul 𝛼 şi mijlocul lui [𝐴′ 𝐵 ′ ] şi 𝑀 ∈ 𝛼.

Avem teorema lui Pitagora: 𝐴𝑀2 + 𝐵𝑀2 = 𝐴𝐴′2 + 𝐴′𝑀2 + 𝐵′𝑀2 + 𝐵𝐵′2 (1)

145
2
2 2(𝑀𝐴′ +𝑀𝐵′ )−𝐴′ 𝐵′
Folosind teorema medianei 𝑀𝑂 = (2)
4
2 2
4𝑀𝑂 2 +𝐴′ 𝐵′ 𝐴′ 𝐵′
Din (1) şi (2) ⇒ 𝐴𝑀2 + 𝐵𝑀2 = 𝐴𝐴′2 + 𝐵𝐵′2 + = 𝐴𝐴′2 + 𝐵𝐵′2 + + 2𝑀𝑂2 şi
2 2
deci 𝐴𝑀2 + 𝐵𝑀2 este minimă pentru M = O.

III. Vezi I de la clasa a VIII-a.

Clasa a X-a

I. 1) Presupunem că 𝑎 ∈ ℚ\ℤ . Avem 𝑝𝑎 = 𝑝[𝑎]+{𝑎} = 𝑝[𝑎] + 𝑝{𝑎} şi cum 𝑝[𝑎] ∈ ℚ, 𝑝𝑎 ∈ ℚ ⇒


𝑝{𝑎} ∈ ℚ ⇒ {𝑎} ∈ ℚ.
𝑚
𝑚 𝑘 𝑘
Notăm {𝑎} = 𝑛 , 𝑚 < 𝑛 𝑚, 𝑛 ∈ ℕ; 𝑝{𝑎} = 𝑙 , 𝑘, 𝑙 ∈ ℕ∗ , (𝑘, 𝑙) = 1 ⇒𝑝 𝑛 = 𝑙 ⇒ 𝑝𝑚 ∙ 𝑙 𝑛 =
𝑘 𝑛 ⇒ 𝑝|𝑘 𝑛 şi cum p este număr prim ⇒ 𝑝|𝑘 deci 𝑘 = 𝑝𝑖 cu 𝑖 ∈ ℕ∗
𝑝𝑚 ∙ 𝑙 𝑛 = 𝑝𝑛 𝑖 𝑛 ⇒ 𝑙 𝑛 = 𝑝𝑚−𝑛 ∙ 𝑖 𝑛 ⇒ 𝑝|𝑙 dar 𝑝|𝑘 şi avem contradicţie cu (𝑘, 𝑙) = 1.
𝑚
𝑚
2) Fie 𝑎 = 𝑙𝑜𝑔𝑝 (𝑝 + 1). Presupunem că 𝑎 = 𝑛 cu 𝑚, 𝑛 ∈ ℕ∗ ⇒ 𝑝 + 1 = 𝑝 𝑛 ⇒ (𝑝 + 1)𝑛 = 𝑝𝑚
absurd pentru că 𝑝 şi 𝑝 + 1 au parităţi diferite. Deci 𝑎 ∈ ℝ\ℚ şi 𝑝𝑎 = 𝑝 + 1 ∈ ℚ.

II. vezi I de la clasa a IX-a.

(𝑏+𝑐)√𝑏𝑐
III. Folosind, eventual, prima teoremă a lui Ptolemeu calculăm 𝐴𝐴′ = . Atunci obţinem
2√𝑝(𝑝−𝑎)
𝐴𝐴′ 𝑎 2 𝐴𝐴′ 𝐵𝐵′ 𝐶𝐶 ′ 𝑎 2 𝑏 2 𝑐 2 9 𝑎 2
= 1 − (𝑏+𝑐) . Avem 𝐴𝐴′′ + 𝐵𝐵′′ + 𝐶𝐶 ′′ = 3 − (𝑏+𝑐) − (𝑐+𝑎) − (𝑎+𝑏) ≤ 4 ⇔(𝑏+𝑐) +
𝐴𝐴′′
𝑏 2 𝑐 2 3
(𝑐+𝑎) + (𝑎+𝑏) ≥ 4
𝑎 𝑏 𝑐 3
Utilizând inegalitatea cunoscută + 𝑐+𝑎 + 𝑎+𝑏 ≥ 2 pentru (∀) 𝑎, 𝑏, 𝑐 > 0 şi inegalitatea C-
𝑏+𝑐
B-S avem că:
𝑎 2 𝑏 2 𝑐 2 𝑎 𝑏 𝑐 2 9
[(𝑏+𝑐) + (𝑐+𝑎) + (𝑎+𝑏) ] (12 + 12 + 12 ) ≥ (𝑏+𝑐 ∙ 1 + 𝑐+𝑎 ∙ 1 + 𝑎+𝑏 ∙ 1) ≥ 4
𝑎 2 𝑏 2 𝑐 2 3
de unde (𝑏+𝑐) + (𝑐+𝑎) + (𝑎+𝑏) ≥ 4 .

Clasa a XI-a

𝑏 𝑏
I. 𝑥𝑛 = √𝑎2 𝑛2 + 𝑏𝑛 − [√𝑎2 𝑛2 + 𝑏𝑛]. Notăm [2𝑎] = 𝑘 , 𝑘 ∈ ℕ. Avem atunci 𝑘 ≤ 2𝑎 < 𝑘 + 1
𝑘2
Pentru 𝑛 ∈ ℕ, 𝑛 ≥ 𝑏−2𝑎𝑘 avem egalitatea √(𝑎𝑛 + 𝑘)2 ≤ √𝑎2 𝑛2 + 𝑏𝑛 < √(𝑎𝑛 + 𝑘 + 1)2 ⇒
[√𝑎2 𝑛2 + 𝑏𝑛] = 𝑎𝑛 + 𝑘
𝑥𝑛 = √𝑎2 𝑛2 + 𝑏𝑛 − 𝑎𝑛 − 𝑘.
𝑏 𝑏 𝑏 𝑏
Avem imediat lim𝑛→∞ 𝑥𝑛 = − 𝑘 = − [ ] = { }.
2𝑎 2𝑎 2𝑎 2𝑎

II. 1) 1 + 𝑧 + 𝑧 𝑛 = 0 ⇒ 𝑧 𝑛 = −1 − 𝑧 ⇒ |𝑧|𝑛 = |1 + 𝑧| ⇒ |1 + 𝑧| = 1.

146
𝑥2 + 𝑦2 = 1 1 √3
Dacă 𝑦 = 𝑥 + 𝑖𝑦; 𝑥, 𝑦 ∈ ℝ obţinem { 2 2 de unde 𝑥 = − 2, 𝑦 = ± 2 ⇒ 𝑧 =
(𝑥 + 1) + 𝑦 = 1
1 √3
− 2 ± 2 𝑖 şi de aici avem 𝑧 3 = 1.
2) Pentru 𝑛 = 3𝑘 , 𝑘 ∈ ℕ∗ ⇒ 1 + 𝑧 + 𝑧 𝑛 = 1 + 𝑧 + 𝑧 3𝑘 = 2 + 𝑧 ≠ 0
Pentru 𝑛 = 3𝑘 + 1 ⇒ 1 + 𝑧 + 𝑧 𝑛 = 1 + 𝑧 + 𝑧 3𝑘+1 = 1 + 𝑧 + 𝑧 = 1 + 2𝑧 ≠ 0 şi deci rămâne
numai 𝑛 = 3𝑘 + 2.

III. Vezi I clasa a X-a.

Clasa a XII-a

I. 1) 𝑥 ∗ (𝑦 ∗ 𝑧) = 𝑥 şi (𝑥 ∗ 𝑦) ∗ 𝑧 = 𝑥 ∗ 𝑧 = 𝑥 ⇒ operaţia “∗” este asociativă


𝑥 ∘ (𝑦 ∘ 𝑧) = 𝑦 ∘ 𝑧 = 𝑧 şi (𝑥 ∘ 𝑦) ∘ 𝑧 = 𝑧 ⇒ operaţia “∘” este asociativă.
2) “∗” are element neutru ⇔(∃) 𝑒 ∈ 𝑀 astfel încât 𝑥 ∗ 𝑒 = 𝑒 ∗ 𝑥 = 𝑥,
(∀)𝑥 ∈ 𝑀 ⇔ (∃)𝑒 ∈ 𝑀 astfel încât 𝑥 = 𝑒 = 𝑥
(∀)𝑥 ∈ 𝑀 ⇔ 𝑀 = {𝑒}. Analog pentru legea “∘”.

𝑎𝑝 𝑏𝑞
II. (⇒) 𝑎𝑏 ≤ + (∀)𝑎 > 0, 𝑏 > 0 (1)
𝑝 𝑞
𝑏𝑞 𝑏𝑞
Dacă 𝑝 = 1 ⇒ 𝑎𝑏 < 𝑎 + de unde 𝑏 ≤ 1 + , (∀)𝑎 > 0, 𝑏 > 0 şi făcând 𝑎 → ∞ ⇒ 𝑏 ≤ 1,
𝑞 𝑎𝑞
(∀)𝑏 > 0 imposibil.
𝑎𝑝 𝑏𝑞 𝑎𝑝 𝑏𝑞 𝑏𝑞
Dacă 𝑝 < 1 în 𝑎𝑏 − ≤ fixăm 𝑏 > 0 şi atunci lim𝑛→∞ (𝑎𝑏 − )≤ adică ∞ ≤
𝑝 𝑞 𝑝 𝑞 𝑞
imposibil. În concluzie avem 𝑝 > 1.
1 1
În (1) luăm 𝑎 = 𝑏 = 1 ⇒ 1 ≤ 𝑝 + 𝑞
1 1 1 1 1 1
𝑥 𝑥 − −1 1 1
În (1) luăm 𝑎 = 𝑥 𝑝 , 𝑏 = 𝑥 𝑞 cu 𝑥 > 0 şi avem 𝑥 𝑝 𝑥 𝑞 ≤ 𝑝 + 𝑞 ⇒ 𝑥 𝑝 𝑞 ≤ 𝑝 + 𝑞, (∀)𝑥 > 0 . Dacă
1 1
1 1 − −1 1 1 1 1
avem 𝑝 − 𝑞 − 1 > 0 atunci din lim𝑥→∞ 𝑥 𝑝 𝑞 ≤ 𝑝 + 𝑞 ⇒ ∞ ≤ 𝑝 + 𝑞 imposibil.
1 1
În concluzie avem + = 1.
𝑝 𝑞
1 1
(<=) 𝑝 > 1 şi 𝑝 + 𝑞 = 1
𝑥𝑝 𝑏𝑞
Fixăm 𝑏 > 0 şi atunci trebuie arătat că − 𝑏𝑥 + ≥ 0, (∀)𝑥 > 0 .
𝑝 𝑞
1
𝑥𝑝 𝑏𝑞
Fie 𝑓: (0, ∞) → ℝ, 𝑓(𝑥) = − 𝑏𝑥 + . 𝑓 " (𝑥) = 𝑥 𝑝−1 − 𝑏 are rădăcina 𝑥 = 𝑏 𝑝−1 care este
𝑝 𝑞
1
punct de minim pentru că 𝑝 > 1 . Atunci 𝑓(𝑥) ≥ 𝑓 (𝑏 𝑝−1 ), (∀)𝑥 > 0 .
𝑝
1 1 𝑝
1 1 𝑝 𝑏 𝑝−1 𝑏𝑞 𝑏𝑞 𝑏𝑞
Din + 𝑞 = 1 ⇒ 𝑞 = 𝑝−1 şi avem 𝑓 (𝑏 𝑝−1 )= − 𝑏 ∙ 𝑏 𝑝−1 + = − 𝑏 𝑝−1 + =
𝑝 𝑝 𝑞 𝑝 𝑞
1 1
𝑏 𝑞 (𝑝 − 1 + 𝑞) = 0 şi atunci avem 𝑓(𝑥) ≥ 0, (∀)𝑥 > 0 .

III. Soluţia 1. O dreaptă care trece prin 𝑀(5𝑎, 5𝑏) are ecuaţia 𝑦 − 5𝑏 =
𝑥2 𝑦2
𝑦 − 5𝑏 = 𝑚(𝑥 − 5𝑎)
𝑚(𝑥 − 5𝑎). Dreapta este tangentă la elipsa 𝑎2 + 𝑏2 = 1 ⇔ sistemul { 𝑥2 𝑦2 are o
2
+ 2
= 1
𝑎 𝑏

147
𝑥2 [5𝑏+𝑚(𝑥−5𝑎)]2
unică soluţie 𝑎2 + = 1 . Se obţine ecuaţie de gradul 2 în x şi atunci pentru a avea o
𝑏2
soluţie unică avem ∆= 0 de unde se obţine 𝑚1 şi 𝑚2 .
Soluţia 2. Fie d una din cele două tangente care se pot duce prin 𝑀(5𝑎, 5𝑏) la elipsă şi
𝑥𝑥 𝑦𝑦 5𝑥
𝑀(𝑥0 , 𝑦0 ) punctul de tangenţă cu elipsa. Atunci: 𝑑: 𝑎20 + 𝑏20 = 1. Din 𝑀(5𝑎, 5𝑏) ∈ 𝑑 ⇒ 𝑎0 +
5𝑥0 5𝑦0
5𝑦0
+ =1
𝑎 𝑏
= 1 şi cum 𝑀(𝑥0 , 𝑦0 ) este pe elipsă avem sistemul {𝑥 2 𝑦0 2
de unde se află 𝑥0 , 𝑦0 .
𝑏 0
+ =1
𝑎2 𝑏2

EDIŢIA a X-a, 24 aprilie 2004


Clasa a VI-a

I. 2004 = 22 ∙ 3 ∙ 167. 𝑁 = ̅̅̅̅̅̅̅̅̅̅


334𝑥𝑥𝑦 = 334000 + 𝑥𝑥𝑦̅̅̅̅̅ = 2 ∙ 167 ∙ 1000 + 𝑥𝑥𝑦
̅̅̅̅̅: 167 ⇔
𝑥𝑥𝑦 ̅̅̅̅̅ ∈ {167,334,501,668,835}, deci 𝑥𝑥𝑦
̅̅̅̅̅: 167 ⇔ 𝑥𝑥𝑦 ̅̅̅̅̅ = 334 sau 668.
1° ) 𝑥𝑥𝑦
̅̅̅̅̅ = 334 ⇒ 𝑁 = 334334 care nu se divide cu 22 ∙ 3.
2° )𝑥𝑥𝑦
̅̅̅̅̅ = 668 ⇒ 𝑁 = 334668 care se divide cu 22 ∙ 3, deci soluţia este 𝑁 = 334668, 𝑥 =
6, 𝑦 = 8.

1 1 1 2 1 2003
II. a) 𝑎 + 𝑏 = 1 + (2 + 2) + (3 + 3) + ⋯ + (2004 + 2004) = ⏟
1 + 1 + ⋯ + 1 = 2004 şi deci
𝑑𝑒 2004 𝑜𝑟𝑖
3|(𝑎 + 𝑏).
𝑥 𝑦 𝑧 𝑡
b) 2 = 3 = 4 = 5 = 𝑘 ⇒ 𝑥 = 2𝑘, 𝑦 = 3𝑘, 𝑧 = 4𝑘, 𝑡 = 5𝑘 şi deci 2𝑘 ∙ 3𝑘 ∙ 4𝑘 ∙ 5𝑘 = 1756920 ⇒
1756920
𝑘4 = = 14641 = 114 ⇒ 𝑘 = 11 deci 𝑥 = 22, 𝑦 = 33, 𝑧 = 44, 𝑡 = 55 .
120

III.

a) Fie 𝑀 astfel încât 𝐴 ∈ (𝐵𝑀) şi 𝐴𝐵 = 𝐴𝑀 ⇒ 𝑀𝐵 = 2𝐴𝐵 = 𝐵𝐶. Cum 𝑚(𝐸𝐵𝐶 ̂ ) = 𝑚(𝐸𝐶𝐵


̂) =
̂ ) ⇒ [𝐵𝐸 bisectoarea lui 𝑀𝐵𝐶
𝑚(𝐴𝐵𝐸 ̂ în triunghiul 𝑀𝐵𝐶 ⇒𝐵𝐸 şi mediană. Dar şi CE mediană
⇒ E centrul de greutate al ∆𝑀𝐵𝐶 şi deci 𝑀, 𝐸, 𝐷 coliniare. Dar 𝐸𝐷 ⊥ 𝐵𝐶 ⇒ 𝑀𝐷 ⊥ 𝐵𝐶 şi cum
𝐵𝐷 = 𝐷𝐶 ⇒ 𝑀𝐵 = 𝑀𝐶. Din 𝑀𝐵 = 𝐵𝐶, 𝑀𝐵 = 𝑀𝐶 ⇒ ∆𝑀𝐵𝐶 echilateral şi de aici avem
̂ ) = 90∘ , 𝑚(𝐴𝐵𝐶
imediat că 𝑚(𝐵𝐴𝐶 ̂ ) = 60∘ , 𝑚(𝐴𝐶𝐵̂ ) = 30∘ .
1 1 1 𝑎
b) Avem 𝐹𝐴 = 𝐹𝐷, 𝐵𝐷 = 𝐷𝐶 şi atunci Aria(𝐴𝐵𝐹) = 2 𝐴𝑟𝑖𝑎(𝐴𝐵𝐷) = 2 ∙ 2 𝐴𝑟𝑖𝑎(𝐴𝐵𝐶) = 4

Clasa a VII-a

148
I. Fie 𝐶𝐸 ∩ 𝐵𝐷 = {𝑀} şi 𝐶𝐹 ∩ 𝐵𝐷 = {𝑁}

𝑀𝐵 𝐸𝐵 1
Din 𝐸𝐵 ∥ 𝐷𝐶 ⇒ ∆𝐸𝑀𝐵~∆𝐶𝑀𝐷 (𝑇. 𝐹. 𝐴) de unde 𝑀𝐷 = 𝐶𝐷 = 2 (1)
𝑁𝐷 𝐷𝐹 1
Analog 𝑁𝐵 = 𝐵𝐶 = 2 (2)
Din (1) şi (2) ⇒𝑀𝐵 = 𝑀𝑁 = 𝐷𝑁.

𝐈𝐈. 1 + 2 + 22 + 23 + 24 = 1 + 2 + 4 + 8 + 16 = 31, 𝑁 = (1 + 2 + 22 + 23 + 24 ) +
25 (1 + 2 + 22 + 23 + 24 ) + 210 (1 + 2 + 22 + 23 + 24 ) + ⋯ + 22000 (1 + 2 + 22 + 23 +
24 ) == 31(1 + 25 + 210 + ⋯ + 22000 ) ⋮ 31
3𝑥−2
𝐈𝐈𝐈. 𝑦(4𝑥 + 1) = 3𝑥 − 2, 𝑦 = 4𝑥+1
𝑥, 𝑦 ∈ ℤ ⇒ 4𝑥 + 1|3𝑥 − 2 dar 4𝑥 + 1|4𝑥 + 1 ⇒ 4𝑥 + 1|12𝑥 − 8 şi 4𝑥 + 1|12𝑥 + 3 ⇒ 4𝑥 +
1 5
1|11 , deci 4𝑥 + 1 ∈ {−11, −1,1,11} ⇒ 4𝑥 ∈ {−12, −2,0,10} ⇒ 𝑥 ∈ {−3, − 2 , 0, 2} şi cum
𝑥 ∈ ℤ ⇒ 𝑥 ∈ {−3,0}
−11
Pentru 𝑥 = −3 ⇒ 𝑦 = −11 = 1
Pentru 𝑥 = 0 ⇒ 𝑦 = −2.

Clasa a VIII-a

𝑎+𝑏 2
𝐈. 𝑎2 + ( 2 ) ≤ 4(𝑎2 + 𝑏 2 )⇔4𝑎2 + 𝑎2 + 2𝑎𝑏 + 𝑏 2 ≤ 16𝑎2 + 16𝑏 2 ⇔ 11𝑎2 − 2𝑎𝑏 +
15𝑏 2 ≥ 0 ⇔ 𝑎2 − 2𝑎𝑏 + 𝑏 2 + 10𝑎2 + 14𝑏 2 ≥ 0 ⇔⇔(𝑎 − 𝑏)2 + 10𝑎2 + 14𝑏 2 ≥ 0 egalitate
ce are loc pentru 𝑎 − 𝑏 = 𝑎 = 𝑏 = 0, deci 𝑎 = 𝑏 = 0.

II.

149
𝐵𝐷 𝑙√2
a) 𝑀𝑃 = = , 𝑀𝑁 = 𝐵𝐶’ = 𝑙√2 . Avem 𝐶’𝐷’ ⊥ (𝐴𝐷𝐷′𝐴′)
2 2
𝑙2 𝑙2 𝑙√6
𝑃𝑁 = √𝑃𝐷′2 + 𝐷′𝑁 2 = √𝑃𝐷 2 + 𝐷𝐷′2 + 𝐷′𝑁 2 = √ 4 + 𝑙 2 + =
4 2
2𝑙2 6𝑙2
𝑀𝑃2 + 𝑃𝑁 2 = 4 + 4 = 2𝑙 2 = 𝑀𝑁 2 ⇒ 𝑚(𝑀𝑃𝑁 ̂ ) = 90∘
b) Fie 𝑄 mijlocul lui (𝐶𝐷) ⇒ 𝑃𝑄 ∥ 𝐴𝐶 şi cum 𝑃𝑀 ∥ 𝐵𝐷,
𝐴𝐶 ⊥ 𝐵𝐷 ⇒ 𝑃𝑄 ⊥ 𝑃𝑀, dar şi 𝑃𝑁 ⊥ 𝑃𝑀. Atunci
𝑚((𝑀𝑁𝑃), ̂ (𝐴𝐵𝐶𝐷)) = 𝑚(𝑁𝑃𝑄 ̂ = 𝑁𝑄 = 𝑙 =
̂ ), sin 𝑁𝑃𝑄 2
=
2√6
=
√6
𝑁𝑃 𝑙√6 √6 6 3
2
III. Notăm cu 𝑀 intersecţia bisectoarelor lui 𝐶̂ şi 𝐷 ̂ , cu 𝑁 intersecţia bisectoarelor lui 𝐵̂ şi 𝐶̂ ,
cu 𝑃 intersecţia bisectoarelor lui 𝐴̂ şi 𝐵̂ și cu 𝑄 intersecţia bisectoarelor lui 𝐴̂ şi 𝐷
̂.

̂ ) + 𝑚(𝑄𝑃𝑁
𝑚(𝑄𝑀𝑁 ̂ ) = 180∘ − 𝑚(𝑀𝐷𝐶
̂ ) − 𝑚(𝑀𝐶𝐷̂ ) + 180∘ − 𝑚(𝑃𝐴𝐵
̂) −
1 1 1 1 1
̂ ) = 360∘ − 𝑚(𝐷
𝑚(𝑃𝐵𝐴 ̂ ) − 𝑚(𝐶̂ ) − 𝑚(𝐴̂) − 𝑚(𝐵̂ ) = 360∘ − ⋅ 360∘ = 360∘ − 180∘ =
2 2 2 2 2
180∘ şi deci 𝑀𝑁𝑃𝑄 este inscriptibil.

Clasa a IX-a

|𝑎+𝑏|+|𝑎−𝑏| (𝑎+𝑏)2 +(𝑎−𝑏)2 +2|𝑎2 −𝑏2 |


𝐈. √2 ≤ √𝑎2 2 ≤ 2 ⇔ 2 ≤ ≤ 4 ⇔2(𝑎2 + 𝑏 2 ) ≤ 2(𝑎2 + 𝑏 2 ) +
+𝑏 𝑎2 +𝑏 2
2|𝑎2 − 𝑏 2 | ≤ 4(𝑎2 + 𝑏 2 ) .
0 ≤ 2|𝑎2 − 𝑏 2 | ≤ 2(𝑎2 + 𝑏 2 )

𝐈𝐈. (𝑓(𝑥) − 𝑥 2 )(4 + 𝑦 2 − 𝑓(𝑦)) ≥ 4, (∀)𝑥, 𝑦 ∈ ℝ


Pentru 𝑦 = 𝑥 ⇒ (𝑓(𝑥) − 𝑥 2 )(4 + 𝑥 2 − 𝑓(𝑥)) ≥ 4, (𝑓(𝑥) − 𝑥 2 )2 − 4(𝑓(𝑥) − 𝑥 2 ) + 4 ≤ 0 ,
(𝑓(𝑥) − 𝑥 2 − 2)2 ≤ 0 ⇒ 𝑓(𝑥) = 𝑥 2 + 2 care verifică relaţia din enunţ cu egalitate.

III.

150
Din 𝑀𝐴 = 𝑀𝐵 = 𝑀𝐶 notând cu O proiecţia lui M pe (ABC) obţinem cu cazul (I.U) că:
∆𝑀𝑂𝐴 ≡ ∆𝑀𝑂𝐵 ≡ ∆𝑀𝑂𝐶 şi deci O este centrul triunghiului echilateral ABC (𝑂𝐴 = 𝑂𝐵 =
𝑂𝐶) .
2 𝑎√3 𝑎√3 3𝑎2 𝑎√6 𝑎√6
Fie 𝐴𝐵 = 𝑎 ⇒ 𝐶𝑂 = 3 ∙ = , 𝐷𝑂√𝐶𝐷2 − 𝐶𝑂2 = √𝑎2 − = , 𝑀𝑂 = −1.
2 3 9 3 3
2 2
𝑎√6 𝑎√3 2𝑎√6
𝑀𝑂2 + 𝑂𝐶 2 = 𝑀𝐶 2 ⇔ ( − 1) + ( ) = 3 ⇔ 𝑎2 − − 2 = 0,
3 3 3
2√6+4√6
3𝑎2 − 2𝑎√6 − 6 = 0, Δ = 24 + 72 = 96 = 16 ∙ 6 , 𝑎 = = √6 .
6
𝑎2 √3 𝑎√6
𝐴𝑟𝑖𝑎(𝐴𝐵𝐶)∙𝐷𝑂 ∙ 𝑎3 √18 𝑎3 √2 6√6∙√2
𝑉𝑜𝑙(𝐴𝐵𝐶𝐷) = = 4 3
= = = = √3
3 3 36 12 12

Clasa a X-a

I. Presupun că 𝑧1 , 𝑧2 ∈ ℂ, |𝑧1 | < 1, |𝑧2 | < 1, 𝑧1 ≠ 𝑧2 sunt rădăcini ale lui P:


𝑎0 + 𝑎1 𝑧1 + 𝑎2 𝑧12 + ⋯ + 𝑎𝑛 𝑧1𝑛 = 0 și 𝑎0 + 𝑎1 𝑧2 + 𝑎2 𝑧22 + ⋯ + 𝑎𝑛 𝑧2𝑛 = 0 , de unde 𝑎1 (𝑧1 −
𝑧2 ) + 𝑎2 (𝑧12 − 𝑧22 ) + ⋯ + 𝑎𝑛 (𝑧12 − 𝑧2𝑛 ) = 0 sau 𝑎1 (𝑧1 − 𝑧2 ) + 𝑎2 (𝑧1 − 𝑧2 )(𝑧1 + 𝑧2 ) + ⋯ +
𝑎𝑛 (𝑧1𝑛−1 + 𝑧1𝑛−2 𝑧2 + ⋯ + 𝑧2𝑛−1 ) ∙∙ (𝑧1 − 𝑧2 ) = 0 şi cum 𝑧1 ≠ 𝑧2 avem : 𝑎1 + 𝑎2 (𝑧1 + 𝑧2 ) +
𝑎3 (𝑧12 + 𝑧1 𝑧2 + 𝑧22 ) + ⋯ + 𝑎𝑛 (𝑧1𝑛−1 + 𝑧1𝑛−2 𝑧2 + ⋯ + +𝑧2𝑛−1 ) = 0 , de unde : |𝑎1 | = |−𝑎1 | =
|𝑎1 + 𝑎2 (𝑧1 + 𝑧2 ) + 𝑎3 (𝑧12 + 𝑧1 𝑧2 + 𝑧22 ) + ⋯ + +𝑎𝑛 (𝑧1𝑛−1 + 𝑧1𝑛−2 𝑧2 + ⋯ + 𝑧2𝑛−1 )| ≤
|𝑎2 |(|𝑧1 | + |𝑧2 |) + |𝑎2 |(|𝑧1 |2 + |𝑧1 ||𝑧2 | + +|𝑧2 |2 ) + ⋯ + |𝑎𝑛 |(|𝑧1 |𝑛−1 + ⋯ + |𝑧2 |2 ) < 2|𝑎2 | +
3|𝑎3 | + ⋯ + 𝑛|𝑎𝑛 |, în contradicţie cu ipoteza 2|𝑎2 | + 3|𝑎3 | + ⋯ + 𝑛|𝑎𝑛 | ≤ |𝑎1 |.

II.
ln 𝑏𝑐 ln 𝑏+ln 𝑐 ln 𝑐+ln 𝑎 ln 𝑎+ln 𝑏
𝑥 = ln 𝑎 = ln 𝑎 , 𝑦 = ln 𝑏 , 𝑧 = ln 𝑐 ⇒ 𝑥𝑦𝑧 − 𝑥 − 𝑦 − 𝑧 =
(ln 𝑏+ln 𝑐)(ln 𝑐+ln 𝑎)(ln 𝑎+ln 𝑏) ln 𝑏+ln 𝑐 ln 𝑐+ln 𝑎 ln 𝑎+ln 𝑏
− ln 𝑎 − ln 𝑏 − ln 𝑐
ln 𝑎 ln 𝑏 ln 𝑐
Aducând la acelaşi numitor şi reducând termenii asemenea obţinem
2 ln 𝑎 ln 𝑏 ln 𝑐
𝑥𝑦𝑧 − 𝑥 − 𝑦 − 𝑧 = ln 𝑎 ln 𝑏 ln 𝑐 = 2

1
III. Pentru 𝑎 = 𝑏 = 𝑐 = 𝛼 ≠ 0, obţinem 3𝛼 2 ≥ 9𝑚𝛼 2 unde 𝑚 ≤ 3
1
Arătăm că 𝑎2 + 𝑏 2 + 𝑐 2 ≥ 3 (𝑎 + 𝑏 + 𝑐)2 (∀)𝑎, 𝑏, 𝑐 ∈ ℝ.
3𝑎2 + 3𝑏 2 + 3𝑐 2 ≥ 𝑎2 + 𝑏 2 + 𝑐 2 + 2𝑎𝑏 + 2𝑎𝑐 + 2𝑏𝑐 care este echivalentă cu
1
(𝑎 − 𝑏)2 + (𝑏 − 𝑐)2 + (𝑐 − 𝑎)2 ≥ 0 deci 𝑚 ∈ (−∞, ].
3

Clasa a XI-a

I. 𝑋𝐴𝑋 −1 = 𝐵 ⇒ 𝑋𝐴 = 𝐵𝑋, 𝑋 ∈ 𝑀𝑛 (ℂ) ⇒ 𝑋 = 𝑋1 + 𝑖𝑋2 cu 𝑋1 , 𝑋2 ∈ 𝑀𝑛 (ℝ) şi avem:


𝑋 𝐴 = 𝐵𝑋1
(𝑋1 + 𝑖𝑋2 )𝐴 = 𝐵(𝑋1 + 𝑖𝑋2 ) ⇒ { 1 (1)
𝑋2 𝐴 = 𝐵𝑋2
Fie 𝑓(𝑧) = 𝑑𝑒𝑡(𝑋1 + 𝑧𝑋2 )⇒ f polinom de grad cel mult n, şi cum 𝑓(𝑖) = 𝑑𝑒𝑡(𝑋1 + 𝑖𝑋2 ) =
𝑑𝑒𝑡𝑋 ≠ 0 ⇒ 𝑓 nu este polinomul identic nul şi deci are cel mult n rădăcini. Atunci (∃)𝛼 ∈ ℝ
astfel încât 𝑓(𝛼) = 𝑑𝑒𝑡(𝑋1 + 𝛼𝑋2 ) şi luând 𝑌 = 𝑋1 + 𝛼𝑋2 ∈ 𝑀𝑛 (ℝ) avem Y inversabilă şi
𝑌𝐴 = 𝐵𝑌 ⇒ 𝑌𝐴𝑌 −1 = 𝐵.

151
𝑥
II. Concluzia se mai scrie (∃)𝑎 ∈ ℝ astfel încât lim𝑛→∞ 2𝑛 (𝑎 − 2𝑛𝑛) = 0.
𝑥 1
Notând 𝑦𝑛 = 2𝑛𝑛, deci 𝑥𝑛 = 2𝑛 𝑦𝑛 ⇒ relaţia din enunţ devine: 0 ≤ 2𝑛+1 𝑦𝑛+1 − 2 ∙ 2𝑛 𝑦𝑛 ≤ 𝑛
1
adică 0 ≤ 𝑦𝑛+1 − 𝑦𝑛 ≤ 𝑛∙2𝑛+1 de unde avem şi
1
0 ≤ 𝑦𝑛+2 − 𝑦𝑛+1 ≤ (𝑛+1)∙2𝑛+2
............................................
1
0 ≤ 𝑦𝑛+𝑚 − 𝑦𝑛+𝑚−1 ≤ (𝑛+𝑚−1)∙2𝑛+𝑚 şi prin adunare

1 1 1 1 1 1 1 1
0 ≤ 𝑦𝑛+𝑚 − 𝑦𝑛 ≤ 𝑛∙2𝑛+1 + (𝑛+1)∙2𝑛+2 + ⋯ + (𝑛+𝑚−1)∙2𝑛+𝑚 ≤ 𝑛 (2𝑛+1 + 2𝑛+2 + ⋯ + 2𝑛+𝑚 ) = 𝑛 ∙
1 1 𝑚
(1−( ) ) 1 1 1 1
2𝑛+1 2
1 < 𝑛 ∙ 2𝑛 = 𝑛∙2𝑛 deci 0 ≤ 𝑦𝑛+𝑚 − 𝑦𝑛 ≤ 𝑛∙2𝑛 . Considerând n fixat şi nu variabil ⇒
1−
2
(𝑦𝑛 )𝑛≥1 este mărginit şi cum este şi crescător ⇒ este convergent . Notăm lim𝑛→∞ 𝑦𝑛 = 𝑙 ∈ ℝ.
1 1
Din 0 ≤ 𝑦𝑛+𝑚 − 𝑦𝑛 ≤ 𝑛∙2𝑛 făcând 𝑚 → ∞ obţinem 0 ≤ 𝑙 − 𝑦𝑛 ≤ 𝑛∙2𝑛 , de unde 0 ≤
1 𝑥
2𝑛 (𝑙 − 𝑦𝑛 ) ≤ 𝑛 ceea ce arată că lim 2𝑛 (𝑙 − 𝑦𝑛 ) = 0 care se scrie lim 2𝑛 (𝑙 − 2𝑛𝑛) = 0, deci
𝑛→∞ 𝑛→∞
𝑎 = 𝑙 ∈ ℝ.

III. ∑𝑛𝑘=0(𝑛 − 2𝑘)2 𝐶𝑛𝑘 = ∑𝑛𝑘=0(𝑛2 − 4𝑘𝑛 + 4𝑘 2 )𝐶𝑛𝑘 = 𝑛2 ∑𝑛𝑘=0 𝐶𝑛𝑘 − −4𝑛 ∑𝑛𝑘=1 𝑘𝐶𝑛𝑘 +
4 ∑𝑛𝑘=1 𝑘 2 𝐶𝑛𝑘 = 𝑛2 ∑𝑛𝑘=0 𝐶𝑛𝑘 − 4𝑛 ∑𝑛𝑘=1 𝑘𝐶𝑛𝑘 + 4 ∑𝑛𝑘=1(𝑘 2 − 𝑘 + 𝑘)𝐶𝑛𝑘 = 𝑛2 ∑𝑛𝑘=0 𝐶𝑛𝑘 −
𝑛!
(4𝑛 − 4) ∑𝑛𝑘=1 𝑘𝐶𝑛𝑘 + 4 ∑𝑛𝑘=2(𝑘 2 − 𝑘)𝐶𝑛𝑘 = 𝑛2 ∑𝑛𝑘=0 𝐶𝑛𝑘 − (4𝑛 − 4) ∑𝑛𝑘=1 (𝑘−1)!(𝑛−𝑘)! +
𝑛! 𝑛! (𝑛−2)!
4 ∑𝑛𝑘=2 (𝑘−2)!(𝑛−𝑘)! = 𝑛2 ∑𝑛𝑘=0 𝐶𝑛𝑘 − (4𝑛 − 4) ∑𝑛𝑘=1 (𝑘−1)!(𝑛−𝑘)! + 4𝑛(𝑛 − 1) ∑𝑛𝑘=2 (𝑘−2)!(𝑛−𝑘)! =
𝑛2 ∑𝑛𝑘=0 𝐶𝑛𝑘 − (4𝑛 − 4) ∑𝑛𝑘=1 𝐶𝑛−1
𝑘−1
+ 4𝑛(𝑛 − 1) ∑𝑛𝑘=2 𝐶𝑛−2
𝑘−2
= 𝑛2 ∙ 2𝑛 − (4𝑛 − 4) ∙ 𝑛 ∙ 2𝑛−1 +
𝑛−2 𝑛−2 (4𝑛 𝑛−2
4𝑛(𝑛 − 1)2 = 𝑛2 − 8𝑛 + 8 + 4𝑛 − 4) = 𝑛2 ∙ 4 = 𝑛2𝑛 .

Clasa a XII-a

1 0 0
I. a) 𝐺 = {(𝑎 1 0) |𝑎, 𝑏, 𝑐 ∈ ℂ}
𝑏 𝑐 1

1 0 0 1 0 0 1 0 0
𝑎
( 1 1 0)∙( 2𝑎 1 0 )=( 𝑎1 + 𝑎 2 1 0) arată că 𝐺 este parte stabilă
𝑏1 𝑐1 1 𝑏2 𝑐2 1 𝑏1 + 𝑐1 𝑎2 + 𝑏2 𝑐1 + 𝑐2 1
faţă de înmulţirea matricilor.
Se ştie că înmulţirea matricelor este asociativă, elementul neutru este 𝐼3 ∈ 𝐺.
1 0 0 1 0 0
|𝑎 1 0| = 1 ≠ 0 ⇒ (𝑎 1 0) este inversabilă şi avem
𝑏 𝑐 1 𝑏 𝑐 1
1 0 0 −1 1 0 0
(𝑎 1 0 ) = ( −𝑎 1 0) ∈ 𝐺.
𝑏 𝑐 1 𝑎𝑐 − 𝑏 − 𝑐 −𝑐 1

152
1 0 0 1 0 0 1 0 0 1 0 0
b) (𝑎 1 0) ∙ (𝑥 1 0) = (𝑥 1 0) ∙ (𝑎 1 0)⇒𝑏 + 𝑐𝑥 + 𝑦 = 𝑦 + 𝑎𝑧 + 𝑏,
𝑏 𝑐 1 𝑦 𝑧 1 𝑦 𝑧 1 𝑏 𝑐 1
(∀)𝑥, 𝑦, 𝑧 ∈ ℂ de unde 𝑐 = 0 şi 𝑎 = 0 deci
1 0 0
𝐴 = (0 1 0) , 𝑏 ∈ ℂ.
𝑏 0 1

1 𝑥 𝑛 𝑒 −𝑥 1 ′
𝐈𝐈. 𝑛 ∫0 𝑑𝑥 = ∫0 𝑥𝑒 −𝑥 (𝑎𝑟𝑐𝑡𝑔(𝑥 𝑛 )) 𝑑𝑥 =
𝑥 2𝑛 +1
1 𝜋 1
𝑥𝑒 −𝑥 𝑎𝑟𝑐𝑡𝑔(𝑥 𝑛 )|10 − ∫0 𝑒 −𝑥 (1 − 𝑥)𝑎𝑟𝑐𝑡𝑔(𝑥 𝑛 )𝑑𝑥 = 𝑒 −1 ∙ 4 − ∫0 𝑒 −𝑥 (1 − −𝑥)𝑎𝑟𝑐𝑡𝑔(𝑥 𝑛 )𝑑𝑥
0 ≤ 𝑒 −𝑥 (1 − 𝑥)𝑎𝑟𝑐𝑡𝑔(𝑥 𝑛 ) ≤ 𝑎𝑟𝑐𝑡𝑔(𝑥 𝑛 ) ≤ 𝑥 𝑛 , (∀)𝑥 ∈ [0,1] de unde
1 1 𝑥 𝑛+1 1 1
0 ≤ ∫0 𝑒 −𝑥 (1 − 𝑥)𝑎𝑟𝑐𝑡𝑔(𝑥 𝑛 ) 𝑑𝑥 ≤ ∫0 𝑥 𝑛 𝑑𝑥 = | = 𝑛+1 şi deci
𝑛+1 0
1 1 𝑥 𝑛 𝑒 −𝑥 𝜋
lim𝑛→∞ ∫0 𝑒 −𝑥 (1 − 𝑥)𝑎𝑟𝑐𝑡𝑔(𝑥 𝑛 ) 𝑑𝑥 = 0 . Rezultă lim𝑛→∞ 𝑛 ∫0 𝑑𝑥 = 4𝑒
𝑥 2𝑛 +1

𝑡2
III. a) Trebuie arătat că 𝑒 𝑡−2 ≥ 4 , (∀)𝑡 ∈ [0, ∞)
Pentru 𝑡 = 0 ⇒ 𝑒 −2 ≥ 0 adevărată.
𝑡2
Pentru 𝑡 > 0 inegalitatea este echivalentă cu 𝑡 − 2 ≥ ln 4 , 𝑡 − 2 − 2 ln 𝑡 + 2 ln 2 ≥ 0 ,
(∀)𝑡 > 0.
2 𝑡−2
Fie 𝑓: (0, ∞) → ℝ, 𝑓(𝑡) = 𝑡 − 2 − 2 ln 𝑡 + 2 ln 2, 𝑓 ′ (𝑡) = 1 − 𝑡 = 2 , 𝑓 ′ (𝑡) = 0 ⇒ 𝑡 = 2,
𝑓(2) = 0

t 0 2

𝑓 ′ (𝑡) - - - - - - - - 0++++++++++++
𝑓(𝑡) 0

Din tabel avem că 𝑓(𝑡) ≥ 0, (∀)𝑡 > 0 .


(𝑥+𝑦)2
b) Din 𝑥, 𝑦 ∈ [0, ∞) ⇒ 𝑥 + 𝑦 ∈ [0, ∞) şi atunci din a) 𝑒 𝑥+𝑦−2 ≥ cu egalitate pentru
4
𝑥+𝑦 =2
(𝑥+𝑦)2 𝑥 2 +𝑦 2 𝑥 2 +𝑦 2
≥ 4 ⇔ 2𝑥𝑦 ≥ 0 cu egalitate pentru 𝑥𝑦 = 0, deci avem 𝑒 𝑥+𝑦−2 ≥ , (∀)𝑥, 𝑦 ∈
4 4
[0, ∞) iar egalitate avem pentru 𝑥 = 2, 𝑦 = 0 sau 𝑥 = 0, 𝑦 = 2.

153
EDIŢIA a XI-a, 20 noiembrie 2004
Clasa a V-a

I. a) 𝑎 = 1 + (4 + 42 + 43 ) + (44 + 45 + 46 ) + ⋯ + (42002 + 42003 + 42004 ) = 1 + 4 ∙ 21 +


44 ∙ 21 + ⋯ + 42002 ∙ 21 = 21(4 + 44 + 47 + ⋯ + 42002 ) + 1 deci restul împărţirii lui a la 21
este 1.
2004∙2005
b) 𝑥 = 1 + 2 + 3 + ⋯ + 2004 ⇒ 2𝑥 + 2005 = 2 ∙ + 2005 = 2004 ∙ 2005 + 2005 =
2
20052
1998∙1999
c) 𝐴 = 𝑎1+2+3+⋯+1998 = 𝑎 2 = 𝑎999∙1999 , 𝐵 = 𝑎999 ∙ (𝑎999 )1999 = 𝑎999+999∙1999 =
999∙2000
𝑎 şi cum 𝑎 ∈ ℕ ⇒ 𝐴 = 𝐵 = 0 pentru 𝑎 = 0 şi 𝐴 = 𝐵 = 1 pentru 𝑎 = 1 şi 𝐴 < 𝐵 pentru
𝑎 ≥ 2.

II. 3𝑎+𝑏 + 3𝑏+𝑐 + 3𝑐+𝑎 = 35 + 34 + 33 ⇒ {𝑎 + 𝑏, 𝑏 + 𝑐, 𝑐 + 𝑎} = {5,4,3} ⇒ 𝑎 + 𝑏 + 𝑐 = 6,


̅̅̅̅̅
𝑎𝑏𝑐 ∈ {231,321,213,312,123,132}

III. 𝑥 = 4𝑎 + 𝑏 , 𝑏 < 4 , 𝑥 = 10𝑏 + 𝑎 , 𝑎 < 10 ⇒𝑎 = 3𝑏, 𝑏 ∈ {1,2,3}.


𝑏 = 1 ⇒ 𝑎 = 3, 𝑥 = 13
𝑏 = 2 ⇒ 𝑎 = 6, 𝑥 = 26
𝑏 = 3 ⇒ 𝑎 = 9, 𝑥 = 39

Clasa a VI-a

I. a) 𝐴𝐴𝑛 = 𝐴𝐴1 + 𝐴1 𝐴2 + ⋯ + 𝐴𝑛−1 𝐴𝑛 = 1 + 2 + 22 + ⋯ + 2𝑛−1 = 2𝑛 − 1, 𝐴𝐴𝑛 < 𝐴𝐵 ⇔


2𝑛 − 1 < 1055 ⇔ 2𝑛 < 1056 ⇔ 𝑛 ≤ 10
b) Pentru 𝑛 = 10, 𝐴10 𝐵 = 𝐴𝐵 − 𝐴𝐴10 = 1055 − (210 − 1) = 32

II. a) Dacă 𝑛 = 0, ultima cifră a lui 5 ∙ 6𝑛 este 5. Dacă 𝑛 ∈ ℕ∗ , ultima cifră a lui 5 ∙ 6𝑛 este 0.
36𝑛 ∙5𝑛+1 ∙7−22𝑛+1 ∙32𝑛+1 ∙5𝑛 62𝑛 ∙5𝑛 ∙5∙7−62𝑛 ∙2∙3∙5𝑛 62𝑛 ∙5𝑛 (35−6) 62𝑛 ∙5𝑛
b) 𝐴 = = = 6𝑛 ∙5𝑛−1 (19+10) = 6𝑛∙5𝑛−1 =
6𝑛 ∙5𝑛−1 ∙19+2𝑛+1 ∙15𝑛 6𝑛 ∙5𝑛−1 ∙19+6𝑛 ∙2∙5𝑛
= 6𝑛 ∙ 5

𝑎+4 7
III. Pentru 𝑎 = 3 ⇒ 𝑎 + 8 = 11 număr prim şi 𝑎+10 = 13 ireductibilă. Pentru 𝑎 > 3, atunci
𝑎 = 3𝑘 + 1 sau 𝑎 = 3𝑘 + 2 (a este prim), 𝑘 ∈ ℕ∗ .
Dacă 𝑎 = 3𝑘 + 1 ⇒ 𝑎 + 8 = 3𝑘 + 9 ⋮ 3
𝑎+4 3𝑘+6 3(𝑘+2)
Dacă 𝑎 = 3𝑘 + 2 ⇒ = = reductibilă.
𝑎+10 3𝑘+12 3(𝑘+4)

Clasa a VII-a

154
𝐈. 9 ∙ 22𝑛 + 12 ∙ 2𝑛 + 4 = (3 ∙ 2𝑛 + 2)2
II. Unghiurile date sunt împărţite de semidreapta Ox în unghiuri ale căror măsuri le notăm
𝑎𝑘 , 𝑎𝑘′ cu 0 < 𝑎𝑘 ≤ 𝑎𝑘′ , 𝑘 ∈ {1,2, ⋯ ,61}, 𝑎𝑘 , 𝑎𝑘′ ∈ ℕ∗ < 𝑎𝑘 + 𝑎𝑘′ = 120 . Atunci 𝑎𝑘 ∈
{1,2, ⋯ ,60}, (∀)𝑘 ∈ {1,2, ⋯ ,61} obţinem atunci că (∃)𝑘1 , 𝑘2 ∈ {1,2, ⋯ ,61} cu 𝑘1 ≠ 𝑘2 astfel
ca să avem 𝑎𝑘1 = 𝑎𝑘2 şi atunci evident că (∃)𝑖 ≠ 𝑗, 𝑖, 𝑗 ∈ {1,2, ⋯ ,122} cu ∡𝐴𝑖 𝑂𝑋 ≡ ∡𝐴𝑗 𝑂𝑋.
1 1 1 1 1 1
III. Fie 𝑎 = 1 + 2 + 22 + ⋯ + 2𝑛. Atunci 2𝑎 = 2 + 1 + 2 + 22 ⋯ + 2𝑛−1 de unde 𝑎 = 2𝑎 − 𝑎 =
1
2 − 2𝑛 < 2

Clasa a VIII-a

I. Adunând ecuaţiile sistemului avem: 0 = (4𝑥 2 − 4𝑥 + 1) + (9𝑦 2 − 12𝑦 + 4) + (16𝑧 2 −


1 2 3
24𝑧 + 9) sau 0 = (2𝑥 − 1)2 + (3𝑦 − 2)2 + (4𝑧 − 3)2 de unde 𝑥 = 2 , 𝑦 = 3 , 𝑧 = 4 care însă nu
verifică ecuaţiile iniţiale ale sistemului.

𝐈𝐈. (⇒) Din ABCD circumscriptibil avem că 𝐴𝐵 + 𝐶𝐷 = 𝐴𝐷 + 𝐵𝐶 (1)


Fie 𝐴𝐶 ∩ 𝐵𝐷 = {𝑃}, 𝑃𝐴 = 𝑎, 𝑃𝐵 = 𝑏, 𝑃𝐶 = 𝑐 şi 𝑃𝐷 = 𝑑. Cum 𝐴𝐶 ⊥ 𝐵𝐷 ⇒ 𝐴𝐵 2 = 𝑎2 + 𝑏 2 ,
𝐵𝐶 2 = 𝑏 2 + 𝑐 2 , 𝐶𝐷2 = 𝑐 2 + 𝑑 2 , 𝐷𝐴2 = 𝑑 2 + 𝑎2 . Atunci (1) devine:
√𝑎2 + 𝑏 2 + √𝑐 2 + 𝑑2 = √𝑎2 + 𝑑2 + √𝑏 2 + 𝑐 2 ⇔2√𝑎2 + 𝑏 2 ⋅ √𝑐 2 + 𝑑 2 = 2√𝑎2 + 𝑑 2 ⋅
√𝑏 2 + 𝑐 2 . Relație care este echivalentă cu

(𝑎2 + 𝑏 2 )(𝑐 2 + 𝑑 2 ) = (𝑎2 + 𝑑 2 )(𝑏2 + 𝑐 2 ) ⇔ 𝑎2 𝑑 2 + 𝑏 2 𝑐 2 = 𝑎2 𝑏 2 + +𝑐 2 𝑑 2 ⇔ (𝑎2 −


𝑐 2 )(𝑏 2 − 𝑑 2 ) = 0 ⇔ 𝑎 = 𝑐 sau 𝑏 = 𝑑 ⇔ 𝐵𝐷 mediatoarea lui [𝐴𝐶] sau 𝐴𝐶 mediatoarea lui
[𝐵𝐷].
(⇐) Dacă una din diagonale este mediatoarea celeilalte, evident că diagonalele sunt
perpendiculare.

III. a) Fie O centrul bazei 𝐴𝐵𝐶, {𝑃} = 𝑉𝑂 ∩ 𝑀𝑁 şi Q mijlocul lui [𝑉𝐴].

155
1
𝑉𝐴 3 1 𝐴𝑄 𝑉𝐴 2 𝐴𝑂
Din 𝑉𝐴 = 4𝑉𝑀 ⇒ 𝑉𝑀 = deci 𝐴𝑀 = 4 𝑉𝐴 şi cum 𝐴𝑄 = 2 𝑉𝐴 avem 𝐴𝑀 = 2
3 = 3 = 𝐴𝑁.
4 𝑉𝐴
4
Atunci, cu reciproca teoremei lui Thales ⇒ 𝑄𝑂 ∥ 𝑀𝑁, dar se observă că M este mijlocul lui
[𝑉𝑄], şi atunci cum în ∆𝑉𝑄𝑂 avem că 𝑀𝑃 este linie mijlocie deoarece 𝑀𝑃 ∥ 𝑄𝑂 ⇒ 𝑃 este
mijlocul lui [𝑉𝑂].
𝑂𝑄 𝐴𝑂 2 3𝑄𝑂 3 𝑉𝐴 3
b) ∆𝐴𝑂𝑄~∆𝐴𝑁𝑀 ⇒ 𝑀𝑁 = 𝐴𝑁 = 3 ⇒ 𝑀𝑁 = 2 = 2 ∙ 2 = 4 𝑉𝐴 = 𝐴𝑀

Clasa a IX-a

I. Vezi problema 1 de la clasa a VIII-a.

𝑛+2
𝑛+2
∈ℕ 𝑛 = 5𝑝 − 2, 𝑝 ∈ ℕ∗
5
II. [√𝑛] = ⇔ {𝑛+2 𝑛+2 ⇔{ ⇔
5
≤ √𝑛 < +1 𝑝 ≤ √5𝑝 − 2 < 𝑝 + 1
5 5
𝑛 = 5𝑝 − 2 𝑝2 − 5𝑝 + 2 ≤ 0
⇔{ 2 . Avem { ⇔ 𝑝2 − 5𝑝 + 2 ≤ 0, 𝑝 ∈ ℕ∗ ⇒
𝑝 ≤ 5𝑝 − 2 < 𝑝2 + 2𝑝 + 1 𝑝2 − 3𝑝 + 3 > 0
𝑝 ∈ {1,2,3,4} de unde 𝑛 ∈ {3,8,13,18}.

III. Vezi problema 3 de la clasa a VIII-a.

Clasa a X-a

I. Se vede imediat că implicaţia este corectă pentru 𝑛 ∈ {1,2}. Pentru 𝑛 ≥ 3, fie 𝑧1 , 𝑧2 , ⋯ , 𝑧𝑛


soluţiile ecuaţiei 𝑧𝑛 − 1 = 0. Acestea au proprietatea că ∑𝑛𝑘=1 𝑧𝑘 = 0 şi ∑𝑛𝑘=1 𝑧𝑘2 = 0 dar nu
verifică 𝑧1 = 𝑧2 = ⋯ = 𝑧𝑛 = 0.

𝐈𝐈. 𝑓: ℝ → ℝ injectivă şi 𝑓 (𝑥 + 𝑓(𝑦 + 𝑓(𝑧))) = 𝑓(𝑥 + 𝑦 + 𝑧 + 1) + 1, (∀) 𝑥, 𝑦, 𝑧 ∈ ℝ


Pentru 𝑦 = 𝑧 = 0 se obţine:𝑓 (𝑥 + 𝑓(𝑓(0))) = 𝑓(𝑥 + 1) + 1, (∀) 𝑥 ∈ ℝ (1)
Notăm 𝑓(0) = 𝑎 şi avem: 𝑓(𝑥 + 𝑓(𝑎)) = 𝑓(𝑥 + 1) + 1, (∀) 𝑥 ∈ ℝ (1’)
Pentru 𝑦 = −𝑧 se obţine:𝑓 (𝑥 + 𝑓(−𝑧 + 𝑓(𝑧))) = 𝑓(𝑥 + 1) + 1, (∀) 𝑥 ∈ ℝ (2)

156
Din (1) şi (2) deducem :𝑓 (𝑥 + 𝑓(−𝑧 + 𝑓(𝑧))) = 𝑓(𝑥 + 𝑓(𝑎)), (∀) 𝑥, 𝑧 ∈ ℝ (3)
Dar f este injectivă şi atunci din (3) avem 𝑥 + 𝑓(−𝑧 + 𝑓(𝑧)) = 𝑥 + 𝑓(𝑎) sau încă
𝑓(−𝑧 + 𝑓(𝑧)) = 𝑓(𝑎) (4)
Aplicând din nou injectivitatea lui f avem:
−𝑧 + 𝑓(𝑧) = 𝑎 ⇒ 𝑓(𝑧) = 𝑧 + 𝑎, (∀) 𝑧 ∈ ℝ (5)
Acum folosind (5) relaţia iniţială devine:𝑥 + 𝑓(𝑦 + 𝑓(𝑧)) + 𝑎 = 𝑥 + 𝑦 + 𝑧 + 1 + 𝑎 + 1
𝑦 + 𝑓(𝑧) + 𝑎 = 𝑦 + 𝑧 + 2 , +1 + 𝑎 = 𝑧 + 2 ⇒ 𝑎 = 1 , deci 𝑓(𝑧) = 𝑧 + 1, (∀) 𝑧 ∈ ℝ.

𝐈𝐈𝐈. (⇒) Este evidentă.


(⇐) Fie 4𝑆 = 𝑎2 + 𝑏 2 + 𝑐 2 + 𝑑2 (1)
𝐴𝐵 = 𝑎, 𝐵𝐶 = 𝑏, 𝐶𝐷 = 𝑐, 𝐷𝐴 = 𝑑 .

𝑎𝑏 sin 𝐵 𝑐𝑑 sin 𝐷
Dar 𝑆 = 𝑆𝐴𝐵𝐶 + 𝑆𝐵𝐶𝐷 = 2 + 2 (2)
Din (1), (2) ⇒ 𝑎 + 𝑏 + 𝑐 + 𝑑 = 2𝑎𝑏 sin 𝐵 + 2𝑐𝑑 sin 𝐷 de unde (𝑎 − 𝑏)2 + (𝑐 − 𝑑)2 +
2 2 2 2

2𝑎𝑏(1 − sin 𝐵) + 2𝑐𝑑(1 − sin 𝐷) = 0 de unde ţinând cont că 𝑎, 𝑏, 𝑐, 𝑑 > 0, sin 𝐵 ≤ 1,


̂ = 𝜋 şi cum ∆𝐴𝐵𝐶 şi
sin 𝐷 ≤ 1 obţinem 𝑎 = 𝑏, 𝑐 = 𝑑, sin 𝐵 = 1, sin 𝐷 = 1 . Deci 𝐵̂ = 𝐷 2
∆𝐴𝐷𝐶 sunt dreptunghice isoscele obţinem imediat că 𝐴𝐵𝐶𝐷 este pătrat.
Clasa a XI-a

I. Soluţia 1. 𝑎0 , 𝑏0 > 0, 𝑎𝑛 = 3𝑎𝑛−1 + 2𝑏𝑛−1 ; 𝑏𝑛 = 2𝑎𝑛−1 + 3𝑏𝑛−1, (∀)𝑛 ≥ 1


𝑎 −3𝑎 𝑎 −3𝑎 3𝑎 −9𝑎
𝑏𝑛−1 = 𝑛 2 𝑛−1 ⇒ 𝑛+12 𝑛 = 2𝑎𝑛−1 + 𝑛 2 𝑛−1 de unde 𝑎𝑛+1 = 6𝑎𝑛 − 5𝑎𝑛−1 , (∀)𝑛 ∈ ℕ∗ .
Din ecuaţia caracteristică ⇒ (∃)𝐴, 𝐵 ∈ ℝ astfel încât 𝑎𝑛 = 𝐴 + 𝐵 ∙ 5𝑛 , (∀)𝑛 ≥ 0 ⇒𝑏𝑛 =
𝑎𝑛+1 −3𝑎𝑛 𝐴+𝐵∙5𝑛+1 −3(𝐴+𝐵∙5𝑛 ) 𝑎 𝐴+𝐵∙5𝑛
= = −𝐴 + 𝐵 ∙ 5𝑛 ⇒ 𝑏𝑛 = −𝐴+𝐵∙5𝑛 care are limita 1.
2 2 𝑛
3𝑥𝑛−1 +2
𝑎𝑛 3𝑎𝑛−1 +2𝑏𝑛−1 3𝑥𝑛−1 +2 𝑥𝑛 −1 −1 𝑥 −1
2𝑥𝑛−1 +3
Soluţia 2. 𝑥𝑛 = = 2𝑎 = 2𝑥 de unde 𝑥 = 3𝑥𝑛−1 +2 = 5(𝑥𝑛−1 +1) deci
𝑏𝑛 𝑛−1 +3𝑏𝑛−1 𝑛−1 +3 𝑛 +1 +1 𝑛−1
2𝑥𝑛−1 +3
𝑥𝑛 −1 1 𝑥 −1 1 2𝑥 −1 1 𝑛 𝑥0 −1 1+𝑦
= 5 ∙ 𝑥𝑛−1 +1 = (5) 𝑥𝑛−2 +1 = ⋯ = (5) = 𝑦𝑛 → 0 ⇒ 𝑥𝑛 = 1−𝑦𝑛 → 1
𝑥𝑛 +1 𝑛−1 𝑛−2 𝑥0 +1 𝑛

II. Cu inegalitatea mediilor avem:


4 4 4 4 4 4
≥ 2√2 𝑦2 ∙ 2 𝑥2 = 2√2 𝑥2
𝑥+ 2 𝑦+ 𝑥+ 𝑦+ 𝑥+ +𝑦+ 2 4 4
16 = 2 𝑦 + 2 𝑥2 𝑦 cu egalitate pentru 𝑥 + =𝑦+ .
𝑦2 𝑥2
Aplicând din nou inegalitatea mediilor avem:
4 𝑥 𝑥 4 𝑥 𝑥 4 𝑥 4
𝑥 + 𝑥 2 = 2 + 2 + 𝑥 2 ≥ 3√2 ∙ 2 ∙ 𝑥 2 = 3 cu egalitate pentru 2 = 𝑥 2 ⇒ 𝑥 = 2

4 𝑦 4
Analog 𝑦 + 𝑦 2 ≥ 3 cu egalitate pentru = 𝑦2 ⇒ 𝑦 = 2
2

157
Atunci avem: 16 ≥ 2√23+3 = 16. Deci se obţine egalitate, de unde 𝑥 = 𝑦 = 2.

III. Cazul 1 A şi B de aceeaşi parte a lui 𝜋.

Fie 𝐴𝐵 ∩ 𝜋 = {𝑃}. Dacă 𝑀 = 𝜋, 𝑀 ≠ 𝑃 punctele 𝑀, 𝐴, 𝐵 determină un triunghi şi deci |𝑀𝐴 −


𝑀𝐵| < 𝐴𝐵 = |𝑃𝐵 − 𝑃𝐴| = |𝑃𝐴 − 𝑃𝐵| şi deci maximul lui |𝑀𝐴 − 𝑀𝐵| când M este arbitrar în
𝜋 se atinge pentru 𝑀 = 𝑃.
Cazul 2. A şi B de o parte şi de cealaltă a lui 𝜋.

α
Fie B’ simetricul lui B faţă de 𝜋. Atunci pentru 𝑀 ∈ 𝜋 avem 𝑀𝐵 = 𝑀𝐵’ şi |𝑀𝐴 − 𝑀𝐵 | =
|𝑀𝐴 − 𝑀𝐵’| care este maximă pentru {𝑀} = 𝐴𝐵′ ∩ 𝜋 conform cazului 1.

Clasa a XII-a

I. Din 3𝐴𝐵𝐴−1 + 𝐴 = 2𝐴−1 𝐵𝐴 avem:2(𝐴𝐵𝐴−1 − 𝐴−1 𝐵𝐴) = −𝐴 − 𝐴𝐵𝐴−1 = 𝐴(−𝐴 − 𝐵)𝐴−1


de unde 𝑑𝑒𝑡(2(𝐴𝐵𝐴−1 − 𝐴−1 𝐵𝐴)) = 𝑑𝑒𝑡(𝐴(−𝐴 − 𝐵)𝐴−1 ) .
Avem 2𝑛 𝑑𝑒𝑡(𝐴𝐵𝐴−1 − 𝐴−1 𝐵𝐴) = 𝑑𝑒𝑡𝐴 ∙ 𝑑𝑒𝑡(−𝐴 − 𝐵) ∙ 𝑑𝑒𝑡𝐴−1 = 𝑑𝑒𝑡(−𝐴 − 𝐵) (1)
Tot din 3𝐴𝐵𝐴−1 + 𝐴 = 2𝐴−1 𝐵𝐴 obţinem: 3(𝐴𝐵𝐴−1 − 𝐴−1 𝐵𝐴) = −𝐴 − 𝐴−1 𝐵𝐴 =
𝐴−1 (−𝐴 − 𝐵)𝐴 de unde 3𝑛 𝑑𝑒𝑡(𝐴𝐵𝐴−1 − 𝐴−1 𝐵𝐴) = 𝑑𝑒𝑡(−𝐴 − 𝐵) (2)
−1 −1
Din (1), (2) ⇒ 𝑑𝑒𝑡(𝐴𝐵𝐴 − 𝐴 𝐵𝐴) = 0.

II. 1) Simplă, prin calcul şi grupare.


2) Prin inducţie, folosind forma lui 𝑥 ∗ 𝑦 din 1).

158
3) Fie 𝑀 ⊂ ℤ, 𝑀 ≠ ∅, 𝑀 finită astfel încât M este parte stabilă faţă de legea “*”.Pentru 𝑥 ∈ 𝑀
avem că şi 𝑥 ∗ 𝑥, 𝑥 ∗ 𝑥 ∗ 𝑥, ⋯ , 𝑥 ∗ 𝑥 ∗ ⋯ ∗ 𝑥(𝑑𝑒 𝑛 𝑜𝑟𝑖), ⋯ ∈ 𝑀 deci conform lui 2)
𝑥𝑛 = 𝑥 ∗ 𝑥 ∗ ⋯ ∗ 𝑥(𝑑𝑒 𝑛 𝑜𝑟𝑖) = 𝑎𝑛−1 (𝑥 − 𝑏)𝑛 + 𝑏 ∈ 𝑀, (∀)𝑛 ∈ ℕ∗
Cum 𝑎 ∈ ℤ, 𝑎 ≠ 0 şi 𝑥, 𝑏 ∈ ℤ din 𝑥𝑛 ∈ 𝑀, (∀)𝑛 ∈ ℕ∗ şi M finită ⇒ 𝑥 − 𝑏 ∈ {−1,0,1}, asfel şirul
(𝑥𝑛 )𝑛≥1 este nemărginit, ceea ce contrazice finititudinea lui M. Avem deci că 𝑀 ⊂ {𝑏 − 1, 𝑏, 𝑏 +
1}.
Pentru 𝑥 = 𝑏 ⇒ 𝑥𝑛 = 𝑏
Pentru 𝑥 = 𝑏 + 1 ⇒ 𝑥𝑛 = 𝑎𝑛−1 + 𝑏 ∈ 𝑀 ⇒ 𝑎 = ±1
Pentru 𝑥 = 𝑏 − 1 ⇒ 𝑥𝑛 = 𝑎𝑛−1 (−1)𝑛 + 𝑏 ∈ 𝑀 ⇒ 𝑎 = ±1 obţinem că dacă 𝑎 ∈ ℤ\{−1,0,1}
avem numai varianta 𝑀 = {𝑏} .
Pentru 𝑎 = 1 , 𝑥 ∗ 𝑦 = (𝑥 − 𝑏)(𝑦 − 𝑏) + 𝑏 şi 𝑀 = {𝑏}, 𝑀 = {𝑏 + 1},
𝑀 = {𝑏 − 1, 𝑏 + 1}, 𝑀 = {𝑏, 𝑏 − 1, 𝑏 + 1}

Pentru 𝑎 = −1 , 𝑥 ∗ 𝑦 = −(𝑥 − 𝑏)(𝑦 − 𝑏) + 𝑏 şi 𝑀 = {𝑏}, 𝑀 = {𝑏 − 1},


𝑀 = {𝑏 + 1, 𝑏 − 1} , 𝑀 = {𝑏, 𝑏 − 1}, 𝑀 = {𝑏, 𝑏 + 1, 𝑏 − 1}.

III. ii) Din 𝑓’ mărginită ⇒ (∃)𝑀 > 0 astfel încât |𝑓’(𝑥)| < 𝑀, (∀)𝑥 ∈ (𝑎, 𝑏).
Fie 𝑐 ∈ (𝑎, 𝑏) ⇒ cu teorema lui Lagrange avem că (∀)𝑥 ∈ (𝑎, 𝑏) există 𝑃𝑥 ∈ (𝑎, 𝑏) cu 𝑓(𝑥) −
𝑓(𝑐) = 𝑓’(𝑃𝑥 )(𝑥 − 𝑐), de unde |𝑓(𝑥) − 𝑓(𝑐)| = (𝑓’(𝑃𝑥 ))(𝑥 − 𝑐) < 𝑀(𝑏 − 𝑎)
𝑓(𝑐) − 𝑀(𝑏 − 𝑎) < 𝑓(𝑥) < 𝑓(𝑐) + 𝑀(𝑏 − 𝑎), (∀)𝑥 ∈ (𝑎, 𝑏) ⇒ f mărginită
iii) |𝑥𝑓’(𝑥) ln 𝑥 − 𝑓(𝑥)| ≤ 𝑀𝑥(ln 𝑥)2 , (∀)𝑥 ∈ (0,1) ∪ (1, ∞)
1
𝑓’(𝑥) ln 𝑥− 𝑓(𝑥) 𝑓(𝑥) ′
| 𝑥
(ln 𝑥)2
| ≤ 𝑀 ⇔ |( ln 𝑥 ) | ≤ 𝑀

𝑓(𝑥)
Fie 𝑔: (0,1) → ℝ, 𝑔(𝑥) = ln 𝑥 ⇒ |𝑔′(𝑥)| ≤ 𝑀, (∀)𝑥 ∈ (0,1) şi atunci din ii) ⇒ (∃)𝑚 > 0 astfel
încât |𝑔(𝑥)| < 𝑚, (∀)𝑥 ∈ (0,1) de unde |𝑓(𝑥)| < 𝑚|ln 𝑥|, (∀)𝑥 ∈ (0,1) ⇒ lim𝑥→1 𝑓(𝑥) = 0 .
𝑥<1
𝑓(𝑥)
Analog luând ℎ: (1,2) → ℝ , ℎ(𝑥) = se obţine că lim𝑥→1 𝑓(𝑥) = 0 şi deci lim𝑥→1 𝑓(𝑥) = 0.
ln 𝑥 𝑥>1

Ediţia a XII-a, 25 martie, 2006


Clasa a V-a

I. a) 𝑑|𝑛 , 𝑑|8𝑛 + 1 ⇒ 𝑑|8𝑛, 𝑑|8𝑛 + 1 ⇒ 𝑑|(8𝑛 + 1 − 8𝑛), adică 𝑑|1 ⇒ 𝑑 = 1


𝑛+1
b) 𝑑|𝑛 + 1 , 𝑑|8𝑛 + 1 ⇒ 𝑑|8𝑛 + 8, 𝑑|8𝑛 + 1 ⇒ 𝑑|(8𝑛 + 8 − 8𝑛 − 1), adică 𝑑|7 . Fracţia 8𝑛+1
este reductibilă ⇒ 𝑑 = 7.
7|𝑛 + 1 ⇒ 𝑛 + 1 = 7𝑘, 𝑘 ∈ ℕ∗ , 𝑛 = 7𝑘 − 1, 8𝑛 + 1 = 8(7𝑘 − 1) + 1 = 56𝑘 − 7 = (8𝑘 − 1)
, deci se simplifică prin 7. Răspuns 𝑛 = 7𝑘 − 1, 𝑘 ∈ ℕ∗ .

̅̅̅̅̅ + 11(𝑎 + 𝑏 + 𝑐) = 𝑐𝑏𝑎


II. a) 𝑎𝑏𝑐 ̅̅̅̅̅, 𝑎 < 𝑏 < 𝑐 se scrie 100𝑎 + 10𝑏 + 𝑐 + 11(𝑎 + 𝑏 + 𝑐) =
100𝑐 + 10𝑏 + 𝑎 , sau 110𝑎 + 11𝑏 = 88𝑐 , deci 10𝑎 + 𝑏 = 8𝑐, 𝑎𝑏 ̅̅̅ = 8𝑐. Avem succesiv:

159
𝑐 = 2 ⇒ ̅̅̅
𝑎𝑏 = 16 , 𝑐 = 3 ⇒ ̅̅̅
𝑎𝑏 = 24 , 𝑐 = 4 ⇒ ̅̅̅
𝑎𝑏 = 32 , 𝑐 = 5 ⇒ ̅̅̅
𝑎𝑏 = 40 , 𝑐 = 6 ⇒ ̅̅̅
𝑎𝑏 =
̅̅̅ ̅̅̅ ̅̅̅
48 , 𝑐 = 7 ⇒ 𝑎𝑏 = 56 , 𝑐 = 8 ⇒ 𝑎𝑏 = 64 , 𝑐 = 9 ⇒ 𝑎𝑏 = 72 .
Dar 𝑎 < 𝑏 < 𝑐 ⇒ 𝑎𝑏𝑐̅̅̅̅̅ = 567.
1 1 1 𝑦+1
b) 𝑥 + 𝑦 + 𝑥𝑦 = 1, 𝑥𝑦
̅̅̅ =?. Avem 𝑦 + 𝑥 + 1 = 𝑥𝑦 ⇒ 𝑥 = 𝑦−1 , 𝑦 − 1|𝑦 + 1 şi 𝑦 − 1|𝑦 − 1 ⇒
𝑦 − 1|(𝑦 + 1 − 𝑦 + 1 ) deci 𝑦 − 1|2 ⇒ 𝑦 − 1 = 1 sau 𝑦 − 1 = 2
̅̅̅ ∈ {23,32}.
Pentru 𝑦 = 2 ⇒ 𝑥 = 3, pentru 𝑦 = 3 ⇒ 𝑥 = 2 deci 𝑥𝑦

III. a) Ultima cifră a lui 20022002 este (20022002 ) = 𝑢(22 ) = 4 , 𝑢(20032003 ) = 𝑢(33 ) = 7 ,
𝑢(20042004 ) = 𝑢(44 ) = 6 , 𝑢(20052005 ) = 𝑢(5) = 5 , 𝑢(20062006 ) = 𝑢(6) = 6 de unde
𝑢(𝐴) = 𝑢(4 + 7 + 6 + 5 + 6) = 8 ⇒ A nu este pătrat perfect.
b) 123321 = 1232∙123+75 > (1232 )123 = 15129123 > 321123 .

Clasa a VI-a

I. a) ̅̅̅̅̅
𝑎𝑏𝑐 = 100𝑎 + 10𝑏 + 𝑐 = 98𝑎 + 7𝑏 + (2𝑎 + 3𝑏 + 𝑐) = 7(14𝑎 + 𝑏) + (2𝑎 + 3𝑏 + 𝑐) şi
̅̅̅̅̅ ⇔ 7|2𝑎 + 3𝑏 + 𝑐 .
de aici avem că 7|𝑎𝑏𝑐
b) Fie 𝑆 = 1 + 2 ∙ 3 + 3 ∙ 32 + 4 ∙ 33 + 5 ∙ 34 + ⋯ + 365 ∙ 3364 . Atunci 3𝑆 = 3 + 2 ∙ 32 + 3 ∙
33 + 4 ∙ 34 + 5 ∙ 35 + ⋯ + 365 ∙ 3365 .
Scăzând aceste relaţii avem :
2𝑆 = 365 ∙ 3365 − (1 + 3 + 32 + 33 + ⋯ + 3364 ) (1)
Fie 𝑇 = 1 + 3 + 3 + 33 + ⋯ + 3364 . Atunci 3𝑇 = 3 + 32 + 33 + 34 + ⋯ + 3365 .
2

Scăzând aceste relaţii avem :


3365 −1
2𝑇 = 3365 − 1 ⇒ 𝑇 = (2)
2
3365 −1 730∙3365 −3365 +1 729∙3365 +1 36 ∙3365 +1
Din (1) și (2) ⇒ 2𝑆 = 365 ∙ 3365 − = = ⇒𝑆 = =
2 2 2 4
3371 +1
.
4

5𝑦+4𝑧−𝑥 5𝑧+4𝑥−𝑦 5𝑥+4𝑦−𝑧 5𝑦+4𝑧−𝑥+5𝑧+4𝑥−𝑦+5𝑥+4𝑦−𝑧 8𝑥+8𝑦+8𝑧 8(𝑥+𝑦+𝑧) 8


II. = = = = = = .
3𝑥 3𝑦 3𝑧 3𝑥+3𝑦+3𝑧 3𝑥+3𝑦+3𝑧 3(𝑥+𝑦+𝑧) 3
5𝑦+4𝑧−𝑥 8 5𝑧+4𝑥−𝑦 8 5𝑥+4𝑦−𝑧 8
Avem acum: = 3 ⇒ 5𝑦 + 4𝑧 = 9𝑥, = ⇒ 5𝑧 + 4𝑥 = 9𝑦 , =3⇒
3𝑥 3𝑦 3 3𝑧
(5𝑦+4𝑧)(5𝑧+4𝑥)(5𝑥+4𝑦) 9𝑥∙9𝑦∙9𝑧
5𝑥 + 4𝑦 = 9𝑧, de unde = = 93 = 729
𝑥𝑦𝑧 𝑥𝑦𝑧

̂.
III. Fie [𝐴𝐴′ bisectoarea lui 𝐵𝐴𝐶

160
̂ ≡ 𝐴𝐵𝐷
Cum 𝐷𝐵 ∥ 𝐴𝐴′, 𝐷 ∈ 𝐴𝐶 şi 𝐸𝐶 ∥ 𝐴𝐴′, 𝐸 ∈ 𝐴𝐵 se obţine imediat că 𝐴𝐷𝐵 ̂ şi 𝐴𝐸𝐶̂ ≡ 𝐴𝐶𝐸 ̂,
deci 𝐴𝐵 = 𝐴𝐷, 𝐴𝐶 = 𝐴𝐸.
̂ ≡ 𝐴𝐶𝐵
Cu cazul (𝐿. 𝑈. 𝐿. ) ⇒ ∆𝐴𝐵𝐶 ≡ ∆𝐴𝐷𝐸 ⇒ 𝐴𝐸𝐷 ̂.
Din 𝐴𝐵 > 𝐴𝐶 ⇒ 𝐴𝐶𝐵 ̂ > 𝐴𝐵𝐶
̂ şi atunci 𝐴𝐸𝐷
̂ > 𝐴𝐵𝐶
̂ ceea ce arată că 𝐷𝐸 nu este paralelă cu
𝐵𝐶.
̂ + 𝐴𝐵𝐶
̂ = 𝐴𝐵𝐷
b) Fie 𝐷𝐸 ∩ 𝐵𝐶 = {𝐹}. 𝐹𝐵𝐷 ̂= 𝐴𝐷𝐵̂ + 𝐴𝐷𝐸 ̂= 𝐹𝐷𝐵 ̂ ⇒ ∆𝐹𝐵𝐷 isoscel cu
𝐹𝐵 = 𝐹𝐷. Cum 𝐸𝐶 ∥ 𝐷𝐵 ⇒ ∆𝐹𝐸𝐶 isoscel cu 𝐹𝐸 = 𝐹𝐶.

Clasa aVII-a
1 1 1 1 1 1 1 1
I. a) 𝑛1 = 1∙2 + 3∙4 + 5∙6 + ⋯ + 2003∙2004, 𝑛2 = 2∙3 + 4∙5 + 6∙7 + ⋯ + 2004∙2005
1 1 1 1 1 1 1 1 1 1 1 1
Avem, 𝑛1 + 𝑛2 = 1∙2 + 2∙3 + 3∙4 + 4∙5 + ⋯ + 2004∙2005 = 1 − 2 + 2 − 3 + 3 − 4 + ⋯ + 2004 −
1 1 1 2004 𝑛1 +𝑛2 1002
= 1 − 2005 = 2005 de unde 𝑚𝑎𝑟𝑖𝑡𝑚 = = 2005 .
2005 2
1 1 1 1 1 1 𝑛1 +𝑛2
b) Din 1∙2 > 2∙3 > 3∙4 > 4∙5 > ⋯ > 2003∙2004 > 2004∙2005 ⇒ 𝑛1 > 𝑛2 şi atunci 𝑛1 > > 𝑛2 ,
2
1002
adică 𝑛1 > 2005 > 𝑛2 .

II. a) ̅̅̅̅̅̅̅̅̅̅̅̅̅̅̅̅̅̅̅
𝑎1 𝑎2 ⋯ 𝑎𝑛−1 𝑎𝑛 = 10𝑎 ̅̅̅̅̅̅̅̅̅̅̅̅̅̅̅̅
1 𝑎2 ⋯ 𝑎𝑛−1 + 𝑎𝑛 = 10(𝑎 1 𝑎2 ⋯ 𝑎𝑛−1 − 5𝑎𝑛 ) + 3 ∙ 17𝑎𝑛 de unde
̅̅̅̅̅̅̅̅̅̅̅̅̅̅̅̅
𝑎1 𝑎2 ⋯ 𝑎𝑛−1 𝑎𝑛 ⋮ 17 ⇔ ̅̅̅̅̅̅̅̅̅̅̅̅̅̅̅̅
̅̅̅̅̅̅̅̅̅̅̅̅̅̅̅̅̅̅̅ 𝑎1 𝑎2 ⋯ 𝑎𝑛−1 − 5𝑎𝑛 ⋮ 17
b) 2006 ; 𝑎1 = 2; 𝑎2 = 0; 𝑎3 = 0; 𝑎4 = 6 . 2006 ⋮ 17 ⇔ 200 − 5 ∙ 6 ⋮ 17 ⇒ 170 ⋮ 17
adevărat.

III. Fie 𝐴𝐷 ⊥ 𝐵𝐶, 𝐷 ∈ 𝐵𝐶 şi 𝐶’ mijlocul lui (𝐴𝐵).

161
Presupun că ∆𝑃𝑄𝑅 este echilateral. Atunci 𝑚(𝑃̂ ) = 60° de unde 𝑚(𝑃𝐵𝐷
̂ ) = 𝑚(𝑃𝐵𝐴 ̂ ) = 30° şi
̂ ) = 30° . Din 𝑚(𝐵𝐴𝐷
𝑚(𝐵𝐴𝐷 ̂ ) = 30° şi
̂ ) = 90° , deci 𝐶𝐶’ ⊥ 𝐴𝐵 şi cum 𝐶𝐶’ mediană ⇒ 𝐴𝐶 = 𝐵𝐶 şi cum
𝑚(𝑅̂ ) = 60° ⇒ 𝑚(𝑅𝐶’𝐴
̂ ) = 60° ⇒ ∆𝐴𝐵𝐶 echilateral şi atunci 𝐴𝑅, 𝐶𝑄, 𝐵𝑃 concurente, contradicţie.
𝑚(𝐴𝐵𝐶

Clasa a VIII-a

I. 4𝑥−2 + 4𝑦+2 ≤ 2𝑥+𝑦+1 se mai scrie 22𝑥−4 + 22𝑦+4 − 2 ∙ 2𝑥+𝑦 ≤ 0 , (2𝑥−2 )2 + (2𝑦+2 )2 − 2 ∙
2𝑥−2 ∙ 2𝑦+2 ≤ 0 sau (2𝑥−2 − 2𝑦+2 )2 ≤ 0 de unde 2𝑥−2 = 2𝑦+2 şi deci 𝑥 − 2 = 𝑦 + 2 ⇒ 𝑥 =
𝑦+4.
2𝑥 + 2𝑦 = 2𝑦+4 + 2𝑦 = 2𝑦 (24 + 1) = 2𝑦 ∙ 17 ⋮ 34 pentru că 𝑦 ∈ ℕ∗

𝑥+1 𝑥+𝑛+1
II. [𝑛+1] + 𝑛 = , 𝑛 ∈ ℕ∗ . Trebuie determinat 𝑛 pentru care ecuaţia are 2006 soluţii.
𝑛
𝑥+1 𝑛𝑦+𝑦−1+𝑛+1 𝑛𝑦+𝑦+𝑛
Fie 𝑛+1 = 𝑦 ⇒ 𝑥 = 𝑛𝑦 + 𝑦 − 1 şi ecuaţia devine [𝑦] + 𝑛 = , [𝑦] + 𝑛 = ,
𝑛 𝑛
adică
𝑛𝑦+𝑦+𝑛
[𝑦] = −𝑛 (1) 𝑛
𝑛𝑦+𝑦+𝑛 𝑛2 +𝑛𝑘−𝑛
−𝑛 =𝑘 𝑦= 𝑛+1
Atunci notând [𝑦] = 𝑘 ∈ ℤ avem : { 𝑛 ⇔{ , 𝑛𝑘 + 𝑘 ≤
𝑛2 +𝑛𝑘−𝑛
𝑘 ≤𝑦 ≤𝑘+1 𝑘 ≤ 𝑛+1 < 𝑘 + 1
𝑛2 + 𝑛𝑘 − 𝑛 < 𝑛𝑘 + 𝑘 + 𝑛 + 1 sau 𝑘 ≤ 𝑛2 − 𝑛 < 𝑘 + 𝑛 + 1 de unde 𝑛2 − 2𝑛 − 1 < 𝑘 ≤
𝑛2 − 𝑛 , adică 𝑘 ∈ {𝑛2 − 2𝑛, 𝑛2 − 2𝑛 + 1, 𝑛2 − 2𝑛 + 2, ⋯ , 𝑛2 − 𝑛}
Atunci ecuaţia (1) şi cea iniţială are 𝑛2 − 𝑛 − (𝑛2 − 2𝑛 − 1) = 𝑛 + 1 soluţii. 𝑛 + 1 = 2006 ⇒
𝑛 = 2005 .

III.
a) Din 𝑉𝐴𝑀𝑁, 𝑉𝐴𝐵𝐶𝐷𝐸𝐹 piramide regulate ⇒ 𝐴𝑀𝑁 este triunghi echilateral , 𝐴𝐵𝐶𝐷𝐸𝐹
hexagon regulat, ambele având ca centru pe O care este proiecţia lui V pe planul bazelor. Notăm
cu P intersecţia lui 𝐴𝑂 cu 𝑀𝑁⇒ P mijlocul lui (𝑀𝑁). Fie 𝐶𝐷 = 𝑎 latura hexagonului regulat.
𝐴𝑂 𝑎 3𝑎 𝑙√3
Atunci 𝐴𝑂 = 𝑎, 𝑂𝑃 = = ⇒ 𝐴𝑃 = şi având 𝐴𝑃 = , unde 𝑙 este latura triunghiului
2 2 2 2
echilateral 𝐴𝑀𝑁, obţinem 𝑙√3 = 3𝑎 ⇒ 𝑙 = 𝑎√3.

Fie 𝑉𝑂 = ℎ şi T mijlocul lui [𝐷𝐸]. Calculăm apotemele celor două piramide:

162
𝑎2 √4ℎ2 +𝑎2 3𝑎2 √4ℎ2 +3𝑎2
𝑉𝑃 = √𝑉𝑂2 + 𝑂𝑃2 = √ℎ2 + = , 𝑉𝑇 = √𝑉𝑂2 + 𝑂𝑇 2 = √ℎ2 + = .
4 2 4 2
𝐴𝑙 (𝑉𝐴𝐵𝐶𝐷𝐸𝐹) 6𝑎∙√4ℎ2 +3𝑎2
Avem = = 𝑘>0.
𝐴𝑙 (𝑉𝐴𝑀𝑁) 3𝑎√3√4ℎ2 +𝑎2
2 2
4(4ℎ +3𝑎 )
2
Atunci 𝑘 = ⇔ 12ℎ 𝑘 + 3𝑘 2 𝑎2 = 16ℎ2 + 12𝑎2 ⇔ 3𝑎2 (𝑘 2 − 4) = 4ℎ2 (4 −
2 2
3(4ℎ2 +𝑎2 )
𝑘 2 −4 4ℎ2 𝑘 2 −4
3𝑘 2 ) ⇔ 4−3𝑘 2 = 3𝑎2 deci > 0.
4−3𝑘 2

2 𝑘2 > 4
1° ) { 𝑘 − 42 > 0 ⇒{ 2 4 imposibil
4 − 3𝑘 > 0 𝑘 <3

2 𝑘2 < 4
2° ) { 𝑘 − 42 < 0 ⇒{ 2 4 ⇒ 𝑘 ∈ ( 3 , 2)
2√2
4 − 3𝑘 < 0 𝑘 > 3

b) Fie 𝑂𝑆 ⊥ 𝑉𝑃. Cum 𝑉𝑂 ⊥ (𝐴𝑀𝑁), 𝑂𝑃 ⊥ 𝑀𝑁, 𝑉𝑃 ⊥ 𝑀𝑁 ⇒ 𝑂𝑆 ⊥ (𝑉𝑀𝑁) şi deci


𝑎
𝑉𝑂⋅𝑂𝑃 𝑉𝑂⋅𝑂𝑃 ℎ⋅ 𝑎ℎ
𝑑(𝑂, (𝑉𝑀𝑁)) = 𝑂𝑆 = = √𝑉𝑂2 = 2
= √4ℎ2 de unde
𝑉𝑃 +𝑂𝑃 2 2 +𝑎2
√ℎ2 +𝑎
4
𝑑(𝑂,(𝑉𝑀𝑁)) ℎ 1 1 1 3√2
= √4ℎ2 = < = < ; s-a folosit faptul că 𝑉𝑂 < 𝐴𝐵 ⇒ ℎ < 𝑎 ⇒
𝑎 +𝑎2 √4+(𝑎)
2 √4+1 √5 8

𝑎
> 1.

Clasa a IX-a

3 3 3 3
1 1 √𝑘+1− √𝑘 √𝑘+1− √𝑘
I. 3 3 3 = 3 3 3 3 3 = 3 3 = 3 3 =
√𝑘(𝑘+1)+ √𝑘 2 (𝑘+1)2 +𝑘 √𝑘+1 √𝑘 √𝑘+1( √(𝑘+1)2 + √𝑘(𝑘+1)+ √𝑘 2 ) √𝑘 √𝑘+1(𝑘+1−𝑘) √𝑘 √𝑘+1
1 1
3 −3 de unde
√𝑘 √𝑘+1
𝑛 1 1 1 1 1
∑𝑘=1 3 3 2 = ∑𝑛𝑘=1 ( 3 − 3 )= 3 −3 <1
√𝑘 (𝑘+1)+ √𝑘 (𝑘+1)2 +𝑘 3√𝑘+1 √𝑘 √𝑘+1 √1 √𝑛+1

II. i) ⇒ii) Ştim că ecuaţia 𝑎𝑥 2 + 𝑏|𝑥| + 𝑐 = 0 ; 𝑎 ∈ ℝ∗ , 𝑏, 𝑐 ∈ ℝ. Are soluţie unică. Fie această
soluţie 𝛼, se observă atunci că şi −𝛼 este soluţie şi cum soluţia este unică ⇒ 𝛼 = −𝛼, 𝛼 = 0,
de unde 𝑐 = 0 şi ecuaţia devine 𝑎𝑥 2 + 𝑏|𝑥| = 0.
Pentru 𝑏 = 0 ⇒ 𝑎𝑏 = 0 şi deci avem 𝑐 = 0 şi 𝑎𝑏 ≥ 0.
𝑏
Pentru 𝑏 ≠ 0, avem 0 soluţie şi pentru 𝑥 > 0 ⇒ 𝑎𝑥 2 + 𝑏𝑥 = 0 ⇒ 𝑥 = − şi pentru 𝑥 < 0 ⇒
𝑎
𝑏 𝑏 𝑏 𝑏 𝑏
𝑎𝑥 2 − 𝑏𝑥 = 0 ⇒ 𝑥 = 𝑎 şi pentru că − 𝑎 şi 𝑎 să nu fie soluţii trebuie ca − 𝑎 < 0, 𝑎 > 0 şi deci
𝑎𝑏 > 0 de unde 𝑐 = 0 şi 𝑎𝑏 ≥ 0.

ii) ⇒i) Avem 𝑐 = 0 şi 𝑎𝑏 ≥ 0. Ecuaţia devine 𝑎𝑥 2 + 𝑏|𝑥| = 0.

163
Dacă 𝑏 = 0 ⇒ 𝑎𝑥 2 = 0 cu unica soluţie 𝑥 = 0, iar dacă 𝑏 ≠ 0 avem 𝑥 = 0 soluţie şi pentru
𝑏
𝑥 > 0 avem 𝑎𝑥 2 + 𝑏𝑥 = 0 ⇒ 𝑥 = − 𝑎 < 0 nu convine şi pentru 𝑥 < 0 avem 𝑎𝑥 2 − 𝑏𝑥 = 0 ⇒
𝑏
𝑥 = 𝑎 > 0; nu convine.

⃗⃗⃗⃗⃗⃗
⃗⃗⃗⃗⃗⃗ = ⃗0⇔⃗⃗⃗⃗⃗⃗⃗⃗
𝑀𝑀+𝑀𝐵 ⃗⃗⃗⃗⃗⃗⃗ +𝑀𝐶
III. ⃗⃗⃗⃗⃗⃗⃗
𝐴𝐺𝐴 + ⃗⃗⃗⃗⃗⃗⃗⃗
𝐵𝐺𝐵 + ⃗⃗⃗⃗⃗⃗⃗⃗
𝐶𝐺𝐶 = ⃗0 ⇔ ⃗⃗⃗⃗⃗⃗⃗⃗⃗
𝑀𝐺𝐴 − 𝑀𝐴 ⃗⃗⃗⃗⃗⃗ + ⃗⃗⃗⃗⃗⃗⃗⃗⃗
𝑀𝐺𝐵 − 𝑀𝐵 ⃗⃗⃗⃗⃗⃗ + ⃗⃗⃗⃗⃗⃗⃗⃗⃗
𝑀𝐺𝐶 − 𝑀𝐶 +
3
⃗⃗⃗⃗⃗⃗ +𝑀𝐶
⃗⃗⃗⃗⃗⃗⃗⃗ +𝑀𝐴
𝑀𝑀 ⃗⃗⃗⃗⃗⃗ ⃗⃗⃗⃗⃗⃗ +𝑀𝐵
⃗⃗⃗⃗⃗⃗⃗⃗ +𝑀𝐴
𝑀𝑀 ⃗⃗⃗⃗⃗⃗⃗ ⃗⃗⃗⃗⃗⃗ +𝑀𝐵
−(𝑀𝐴 ⃗⃗⃗⃗⃗⃗ )
⃗⃗⃗⃗⃗⃗⃗ +𝑀𝐶
+ ⃗⃗⃗⃗⃗⃗ + 𝑀𝐵
− (𝑀𝐴 ⃗⃗⃗⃗⃗⃗ ) = ⃗0⇔
⃗⃗⃗⃗⃗⃗ + 𝑀𝐶 = ⃗0 ⇔ −𝑀𝐺
⃗⃗⃗⃗⃗⃗ = ⃗0 ⇔ 𝑀 =
3 3 3
𝐺.

Clasa a X-a

I. Pentru 𝑥 > 𝑎 ecuaţia se scrie:


10log𝑎(𝑥−𝑎) − 𝑎𝑙𝑔(𝑥+𝑏) = 𝑎 + 𝑏 (1)
𝑢
Notăm log 𝑎 (𝑥 − 𝑎) = 𝑢 şi 𝑙𝑔(𝑥 + 𝑏) = 𝑡 de unde 𝑥 − 𝑎 = 𝑎 şi
𝑥 + 𝑏 = 10𝑡 ⇒ 10𝑡 − 𝑎𝑢 = 𝑎 + 𝑏 (2)
Din (1) avem
10𝑢 − 𝑎𝑡 = 𝑎 + 𝑏 (3)
Din (2) și (3) avem
10𝑡 − 𝑎𝑢 = 10𝑢 − 𝑎𝑡 ⇔ 10𝑡 + 𝑎𝑡 = 10𝑢 + 𝑎𝑢 (4)
Cum funcţia 𝑓: ℝ → ℝ, 𝑓(𝑥) = 10𝑥 + 𝑎 𝑥 , 𝑎 > 1 este strict crescătoare, deci injectivă, din
(4)⇒𝑡 = 𝑛 şi atunci (2) devine
10𝑡 − 𝑎𝑡 = 𝑎 + 𝑏 ⇔ 𝑎𝑡 + 𝑎 + 𝑏 = 10𝑡 |: 10𝑡 , deci
𝑎 𝑡 1 𝑡
(10) + (𝑎 + 𝑏) (10) = 1 5)

𝑎
Din 𝑏 > 0, 2𝑎 + 𝑏 = 0 ⇒ 𝑎 < 5 şi deci 10 ∈ (0,1)
𝑎 𝑥 1 𝑥
Funcția 𝑔: ℝ → ℝ, 𝑔(𝑥) = (10) + (𝑎 + 𝑏) (10) este strict descrescătoare, deci injectivă ⇒
ecuaţia (5) are cel mult o soluţie.
𝑎 𝑎+𝑏 2𝑎+𝑏
Dar 𝑡 = 1 este soluţie pentru că 10 + 10 = 10 = 1 şi deci 𝑡 = 1 este soluţie unică ⇒ 𝑥 +
𝑏 = 10, 𝑥 = 10 − 𝑏 = 2𝑎 care verifică ecuaţia (1).

II. Fie 𝐺𝐴′ , 𝐺𝐵′ , 𝐺𝐶 ′ , 𝐺𝐷′ centrele de greutate ale triunghiurilor 𝐵𝐶𝐷, 𝐴𝐶𝐷, 𝐴𝐵𝐷, 𝐴𝐵𝐶.
⃗⃗⃗⃗⃗⃗⃗
𝑀𝐵+𝑀𝐶 ⃗⃗⃗⃗⃗⃗ +𝑀𝐷
⃗⃗⃗⃗⃗⃗⃗
⃗⃗⃗⃗⃗⃗⃗⃗⃗⃗
𝑀𝐺𝐴′ = 3
Se ştie că (∀)𝑥 ∈ ℝ considerând vectorul 𝑀𝑇 ⃗⃗⃗⃗⃗⃗ = 𝑥𝑀𝐴⃗⃗⃗⃗⃗⃗ + (1 − x)𝑀𝐺 ⃗⃗⃗⃗⃗⃗⃗⃗⃗⃗
𝐴′ , avem că 𝑇 ∈ 𝐴𝐺𝐴′ .
1 ⃗⃗⃗⃗⃗⃗⃗ ⃗⃗⃗⃗⃗⃗⃗ ⃗⃗⃗⃗⃗⃗ ⃗⃗⃗⃗⃗⃗⃗
Luând 𝑥 = ⇒ 𝑇 ∈ 𝐴𝐺𝐴′ , unde 𝑀𝑇 ⃗⃗⃗⃗⃗⃗ = 1 𝑀𝐴
⃗⃗⃗⃗⃗⃗ + 3 𝑀𝐺
⃗⃗⃗⃗⃗⃗⃗⃗⃗⃗
𝐴′ =
MA+𝑀𝐵+𝑀𝐶 +𝑀𝐷
4 4 4 4
⃗⃗⃗⃗⃗⃗ , 𝑀𝐵
Datorită simetriei în raport cu MA ⃗⃗⃗⃗⃗⃗ , 𝑀𝐷
⃗⃗⃗⃗⃗⃗ , 𝑀𝐶 ⃗⃗⃗⃗⃗⃗ avem că 𝑇 ∈ 𝐵𝐺𝐵′ , 𝑇 ∈ 𝐶𝐺𝐶 ′ , 𝑇 ∈ 𝐷𝐺𝐷′ şi deci
⃗⃗⃗⃗⃗⃗⃗ ⃗⃗⃗⃗⃗⃗⃗ ⃗⃗⃗⃗⃗⃗ ⃗⃗⃗⃗⃗⃗⃗
T este centrul de greutate al tetraedrului [𝐴𝐵𝐶𝐷] . Avem deci 𝑀𝐺 ⃗⃗⃗⃗⃗⃗ = MA+𝑀𝐵+𝑀𝐶+𝑀𝐷.
4
În final avem :𝐴𝐺 ⃗⃗⃗⃗⃗⃗⃗𝐴 + ⃗⃗⃗⃗⃗⃗⃗⃗
𝐵𝐺𝐵 + 𝐶𝐺 ⃗⃗⃗⃗⃗⃗⃗⃗𝐶 + 𝐷𝐺 ⃗⃗⃗⃗⃗⃗⃗⃗𝐷 = 0 ⃗ ⇔ ⃗⃗⃗⃗⃗⃗⃗⃗⃗
𝑀𝐺𝐴 − MA ⃗⃗⃗⃗⃗⃗ + ⃗⃗⃗⃗⃗⃗⃗⃗⃗
𝑀𝐺𝐵 − 𝑀𝐵 ⃗⃗⃗⃗⃗⃗ + ⃗⃗⃗⃗⃗⃗⃗⃗
𝑀𝐺𝑐 − MC ⃗⃗⃗⃗⃗⃗ +
⃗⃗⃗⃗⃗⃗⃗⃗ +𝑀𝐵
MM ⃗⃗⃗⃗⃗⃗ +𝑀𝐷
⃗⃗⃗⃗⃗⃗⃗ +𝑀𝐶 ⃗⃗⃗⃗⃗⃗⃗ ⃗⃗⃗⃗⃗⃗⃗⃗ +𝑀𝐴
MM ⃗⃗⃗⃗⃗⃗ +𝑀𝐶
⃗⃗⃗⃗⃗⃗ +𝑀𝐷
⃗⃗⃗⃗⃗⃗⃗ ⃗⃗⃗⃗⃗⃗⃗⃗ +𝑀𝐴
MM ⃗⃗⃗⃗⃗⃗ +𝑀𝐵
⃗⃗⃗⃗⃗⃗⃗ +𝑀𝐷
⃗⃗⃗⃗⃗⃗⃗ ⃗⃗⃗⃗⃗⃗⃗⃗⃗
MM+𝑀𝐴 ⃗⃗⃗⃗⃗⃗ +𝑀𝐵
⃗⃗⃗⃗⃗⃗⃗ +𝑀𝐶⃗⃗⃗⃗⃗⃗
⃗⃗⃗⃗⃗⃗⃗⃗⃗𝐷 − MD
𝑀𝐺 ⃗⃗⃗⃗⃗⃗ = ⃗0 ⇔ + + +
4 4 4 4
⃗⃗⃗⃗⃗⃗⃗ +𝑀𝐵
−(MA ⃗⃗⃗⃗⃗⃗ +𝑀𝐷
⃗⃗⃗⃗⃗⃗⃗ +𝑀𝐶 ⃗⃗⃗⃗⃗⃗⃗ )
⃗⃗⃗⃗⃗⃗ + 𝑀𝐵
−(MA ⃗⃗⃗⃗⃗⃗ + 𝑀𝐷
⃗⃗⃗⃗⃗⃗ + 𝑀𝐶 ⃗⃗⃗⃗⃗⃗ ) = ⃗0 ⇔ = ⃗0 ⇔ −MG
⃗⃗⃗⃗⃗⃗ = ⃗0 ⇔ M = G.
4

164
𝑧2 |𝑧1 −𝑧3 |+𝑧3 |𝑧1 −𝑧2 | (𝑧2 +𝑧3 )|𝑧1 −𝑧2 | 𝑧2 +𝑧3
III. (⇒) |𝑧1 − 𝑧2 | = |𝑧1 − 𝑧3 |. Atunci |𝑧1 −𝑧3 |+|𝑧1 −𝑧2 |
= =
2|𝑧1 −𝑧2 | 2

𝑧2 |𝑧1 −𝑧3 |+𝑧3 |𝑧1 −𝑧2 | 𝑧2 +𝑧3


(⇐) = sau 2𝑧2 |𝑧1 − 𝑧3 | + 2𝑧3 |𝑧1 − 𝑧2 | = (𝑧2 + 𝑧3 )|𝑧1 − 𝑧3 | +
|𝑧1 −𝑧3 |+|𝑧1 −𝑧2 | 2
(𝑧2 + 𝑧3 )|𝑧1 − 𝑧2 | , adică (𝑧2 − 𝑧3 )|𝑧1 − 𝑧3 | = (𝑧2 − 𝑧3 )|𝑧1 − 𝑧2 | şi cum 𝑧2 ≠ 𝑧3 avem
|𝑧1 − 𝑧3 | = |𝑧1 − 𝑧2 | .
Să interpretăm geometric echivalenţa propusă.
Fie 𝐴(𝑧1 ), 𝐵(𝑧2 ), 𝐶(𝑧3 ) . Atunci |𝑧1 − 𝑧2 | = |𝑧1 − 𝑧3 | ⇔ 𝐴𝐵 = 𝐴𝐶 ⇔ triunghiul ABC este
𝑧 +𝑧
isoscel cu baza BC. Se ştie că 2 2 3 este afixul mijlocului lui (𝐵𝐶).
̂.
Fie 𝐷 ∈ (𝐵𝐶) astfel încât [𝐴𝐷 bisectoarea lui 𝐵𝐴𝐶

𝐵𝐷 AB
⃗⃗⃗⃗⃗ , iar la numere
⃗⃗⃗⃗⃗⃗ = AB ∙ 𝐷𝐶
Cu teorema bisectoarei avem 𝐷𝐶 = AC care vectorial se scrie 𝐶 ∙ 𝐵𝐷
complexe notând z afixul lui D se scrie 𝐴𝐶(𝑧 − 𝑧2 ) = AB(z3 − z),
𝐴𝐶∙𝑧2 +𝐴𝐵∙𝑧3 |𝑧 −𝑧3 |𝑧2 +|𝑧1 −𝑧2 |𝑧3
𝑧 = 𝐴𝐶+𝐴𝐵 = 1|𝑧 −𝑧 |+|𝑧 −𝑧 |
.
1 3 1 2
Deci geometric relaţia de demonstrat se scrie 𝐴𝐵 = 𝐴𝐶⇔piciorul bisectoarei din A coincide cu
mijlocul lui (𝐵𝐶).

Clasa a XI-a

1
I. a) Fie 𝑓: [− 𝑛2 , ∞) → ℝ, 𝑓(𝑥) = √𝑛2 𝑥 + 1 + √𝑛2 𝑥 + 2 + ⋯ + √𝑛2 𝑥 + 𝑛 − 𝑛2 . Deoarece 𝑓
este strict crescătoare , 𝑓 continuă, 𝑓(1) = √𝑛2 + 1 + √𝑛2 + 2 + ⋯ + √𝑛2 + 𝑛 − 𝑛2 > √𝑛2 +
1
√𝑛2 + √𝑛2 + ⋯ + √𝑛2 − 𝑛2 = 0 și 𝑓 (1 − 𝑛) = √𝑛2 − 𝑛 + 1 + √𝑛2 − 𝑛 + 2 + ⋯ +
√𝑛2 − 𝑛 + 𝑛 − 𝑛2 = √𝑛2 − (𝑛 − 1) + √𝑛2 − (𝑛 − 2) + ⋯ + √𝑛2 − (𝑛 − 𝑛) − 𝑛2 ≤ √𝑛2 +
1
√𝑛2 + √𝑛2 + ⋯ + √𝑛2 − 𝑛2 = 0 , atunci ecuaţia 𝑓(𝑥) = 0 are o soluţie unică 𝑥𝑛 ∈ [1 − 𝑛 , 1) şi
deci 𝑥𝑛 ≥ 0 .
1 1
b) 1 − 𝑛 ≤ 𝑥𝑛 < 1 şi lim𝑛→∞ (1 − 𝑛) = 1 ⇒lim𝑛→∞ 𝑥𝑛 = 1

𝑎2 𝑎2
II. 𝐴2 + 𝑎𝐴𝐵 + 𝑏𝐵 2 = 𝐴2 + 𝑎𝐴𝐵 + 𝐵 2 + (𝑏 − ) 𝐵 2 şi cum 𝐴𝐵 = 𝐵𝐴, obţinem 𝐴2 +
4 4
𝑎 2 𝑎2 𝑎 2 𝑎2 𝑎
𝑎𝐴𝐵 + 𝑏𝐵 2 = (𝐴 + 2 𝐵) + (𝑏 − ) 𝐵 2 = (𝐴 + 2 𝐵) − 𝑖 2 (𝑏 − ) 𝐵 2 = (𝐴 + 2 𝐵 −
4 4

𝑎2 𝑎 𝑎2
𝑖 √𝑏 − 𝐵) (𝐴 + 2 𝐵 + +𝑖√𝑏 − 𝐵) şi cum 𝐴, 𝐵 ∈ 𝑀𝑛 (ℝ) obţinem 𝑑𝑒𝑡(𝐴2 + 𝑎𝐴𝐵 +
4 4

165
𝑎 𝑎2 𝑎 𝑎2 𝑎
𝑏𝐵 2 ) = 𝑑𝑒𝑡 (𝐴 + 2 𝐵 − 𝑖 √𝑏 − 𝐵) ∙ 𝑑𝑒𝑡 (𝐴 + 2 𝐵 + +𝑖√𝑏 − 𝐵) = 𝑑𝑒𝑡 (𝐴 + 2 𝐵 +
4 4
2
𝑎2 𝑎 𝑎2 𝑎 𝑎2
𝑖 √𝑏 − 𝐵) ∙ 𝑑𝑒𝑡 (𝐴 + 2 𝐵 + +𝑖 √𝑏 − 𝐵) = |𝑑𝑒𝑡 (𝐴 + 2 𝐵 + 𝑖 √𝑏 − 𝐵)| ≥ 0
4 4 4

III.

𝐸𝐶 𝐴𝐶
⃗⃗⃗⃗⃗ = 𝐴𝐶 ∙ 𝐸𝐵
Cu teorema bisectoarei ⇒ 𝐸𝐵 = 𝐴𝐵 care vectorial se scrie : 𝐴𝐵 ∙ 𝐶𝐸 ⃗⃗⃗⃗⃗ −
⃗⃗⃗⃗⃗ ⇒ 𝐴𝐵(𝐴𝐸
𝐴𝐵∙𝐴𝐶 +𝐴𝐶∙𝐴𝐵 ⃗⃗⃗⃗⃗ ⃗⃗⃗⃗⃗
⃗⃗⃗⃗⃗ ) = 𝐴𝐶(𝐴𝐵
𝐴𝐶 ⃗⃗⃗⃗⃗ − ⃗⃗⃗⃗⃗
𝐴𝐸 ) de unde ⃗⃗⃗⃗⃗
𝐴𝐸 = 𝐴𝐵+𝐴𝐶
Din 𝐴𝐸, 𝐵𝐷, 𝐶𝑇 concurente în K aplicând teorema lui Ceva avem :
𝑇𝐴 𝐸𝐵 𝐷𝐶 𝑇𝐴 𝐴𝐵 𝑇𝐴 𝐴𝐶 𝑇𝐴 𝐴𝐶 𝐴𝐶
=1, ∙ ∙1=1⇒ = ⇒ = ⇒ ⃗⃗⃗⃗⃗
𝐴𝑇 = ⃗⃗⃗⃗⃗
𝐴𝐵
𝑇𝐵 𝐸𝐶 𝐷𝐴 𝑇𝐵 𝐴𝐶 𝑇𝐵 𝐴𝐵 𝐴𝐵 𝐴𝐶+𝐴𝐵 𝐴𝐶+𝐴𝐵
⃗⃗⃗⃗⃗ +𝐴𝐶∙𝐴𝐵
𝐴𝐵∙𝐴𝐶 ⃗⃗⃗⃗⃗ 𝐴𝐶 ⃗⃗⃗⃗⃗
𝐴𝐵∙𝐴𝐶 𝐴𝐵∙𝐴𝐶
𝑇𝐸 = ⃗⃗⃗⃗⃗
⃗⃗⃗⃗⃗ 𝐴𝐸 − ⃗⃗⃗⃗⃗
𝐴𝑇 = − 𝐴𝐵+𝐴𝐶 ∙ ⃗⃗⃗⃗⃗
𝐴𝐵 = 𝐴𝐵+𝐴𝐶 de unde 𝑇𝐸 = 𝐴𝐵+𝐴𝐶 .
𝐴𝐵+𝐴𝐶

Clasa a XII-a

𝑎 𝑏
I. Soluţia 1. Se arată uşor că (𝐺,∙) este grup comutativ şi 𝑓: ℂ∗ → 𝐺, 𝑓(𝑎 + 𝑏𝑖) = ( )
−𝑏 𝑎
este izomorfism de grupuri de la (ℂ∗ ,∙) la (𝐺,∙).
Cum în (ℂ∗ ,∙) ecuaţia 𝑧 𝑛 = 𝛼 , cu 𝛼 ∈ ℂ∗ fixat şi 𝑛 ∈ ℕ∗ fixat are exact 𝑛 rădăcini , atunci şi în
(𝐺,∙) ecuaţia 𝑋 𝑛 = 𝐴 are 𝑛 soluţii în 𝐺.
𝑎 𝑏
𝑎 𝑏 2 2 √𝑎2 +𝑏 2 cos 𝜃 sin 𝜃
Soluţia 2. Fie 𝐴 = ( ) = √𝑎2 + 𝑏 2 (√𝑎−𝑏+𝑏 𝑎 ) = √𝑎2 + 𝑏 2 ( ) cu
−𝑏 𝑎 − sin 𝜃 cos 𝜃
√𝑎2 +𝑏2 √𝑎2 +𝑏 2
𝜃 ∈ [0,2𝜋]
𝑥 𝑦
𝑥 𝑦 2
2 + 𝑦 2 (√𝑥 +𝑦
2 √𝑥 2 +𝑦2 cos 𝜑 sin 𝜑
𝑋 = (−𝑦 𝑥 ) = √𝑥 −𝑦 𝑥 ) = √𝑥 2 + 𝑦 2 ( ) cu 𝜑 ∈ [0,2𝜋]
− sin 𝜑 cos 𝜑
√𝑥 2 +𝑦2 √𝑥 2 +𝑦2
cos 𝑛𝜑
𝑛 sin 𝑛𝜑 sin 𝜃 cos 𝜃
𝑋 𝑛 = (√𝑥 2 + 𝑦 2 ) ( ) = √𝑎2 + 𝑏 2 ( )
− sin 𝑛𝜑 cos 𝑛𝜑 cos 𝜃− sin 𝜃
𝑛 𝑛
⇒(√𝑥 2 + 𝑦 2 ) cos 𝑛𝜑 = cos 𝜃 √𝑎2 + 𝑏 2 , (√𝑥 2 + 𝑦 2 ) sin 𝑛𝜑 = sin 𝜃 √𝑎2 + 𝑏 2 , ridicând la

166
𝑛
pătrat şi adunând , avem 𝑥 2 + 𝑦 2 = √𝑎2 + 𝑏 2 şi atunci cos 𝑛𝜑 = cos 𝜃, sin 𝑛𝜑 = sin 𝜃 și deci
𝜃+2𝑘𝜋
𝑛𝜑 = 𝜃 + 2𝑘𝜋 , 𝜑 = 𝑛 cu 𝑘 ∈ {0,1,2, ⋯ , 𝑛 − 1} deoarece 𝜑 ∈ [0,2𝜋).
Avem soluţiile
𝜃+2𝑘𝜋 𝜃+2𝑘𝜋
2𝑛
cos sin
𝑛 𝑛
𝑋𝑘 = √𝑎2 + 𝑏 2 ( 𝜃+2𝑘𝜋 𝜃+2𝑘𝜋
), 𝑘 ∈ {0,1,2, ⋯ , 𝑛 − 1}.
−sin cos
𝑛 𝑛

2𝜋 1 1
II. 𝐼 = ∫0 𝑑𝑥. Fie 𝑓: [0,2𝜋] → ℝ, 𝑓(𝑥) = 2+cos 𝑥 care este continuă şi fie 𝐹: [0,2𝜋] → ℝ
2+cos 𝑥
𝑥
o primitivă a ei . Pentru 𝑥 ∈ [0,𝜋) sau (𝜋,2𝜋], facem notaţia 𝑡𝑔 2 = 𝑡 . 𝑥 = 2𝑎𝑟𝑐𝑡𝑔 𝑡 = 𝜑(𝑡) ,
2
𝜑′(𝑡) = 1+𝑡 2.
𝑥
1 2 2 2 𝑡 1 2 𝑡𝑔
∫ 1−𝑡2
∙ 1+𝑡 2 𝑑𝑥 = ∫ 𝑡 2 +3 𝑑𝑡 = 𝑎𝑟𝑐𝑡𝑔 + ∁ şi ∫ 2+cos 𝑥 𝑑𝑥 = 𝑎𝑟𝑐𝑡𝑔 2
+ +∁ . Atunci
2+ 2 √3 √3 √3 √3
1+𝑡
𝑥
2 𝑡𝑔
𝑎𝑟𝑐𝑡𝑔 2
+ 𝑐1 , 𝑥 ∈ [0,𝜋)
√3 √3
𝐹(𝑥) = 𝑐2 , 𝑥=𝜋
𝑥
2 𝑡𝑔
𝑎𝑟𝑐𝑡𝑔 2
+ 𝑐3 , 𝑥 ∈ (𝜋, 2𝜋]
{√3 √3

2 𝜋 2 𝜋
Din continuitatea lui 𝐹 în 𝜋 ⇒ 𝐹𝑆 (𝜋) = 𝐹𝑑 (𝜋) = 𝐹(𝜋), adică ∙ + 𝑐1 = ∙ (− ) + 𝑐3 =
√3 2 √3 2
𝑐2 = 𝑘
𝑥
2 𝑡𝑔 𝜋
𝑎𝑟𝑐𝑡𝑔 2
+𝑘− , 𝑥 ∈ [0,𝜋)
√3 √3 √3
𝐹(𝑥) = 𝑘, 𝑥=𝜋
𝑥
2 𝑡𝑔 𝜋
{√3 𝑎𝑟𝑐𝑡𝑔 √3 + 𝑘 + √3 , 𝑥 ∈ (𝜋, 2𝜋]
2

2𝜋 𝜋 𝜋 2𝜋
Atunci ∫0 𝑓(𝑥) 𝑑𝑥 = 𝐹(𝑥)|2𝜋
0 = 𝐹(2𝜋) − 𝐹(0) = (𝑘 + √3) − (𝑘 − √3) = √3 .

III. Fie 𝑈(𝐴) = {𝑥 ∈ 𝐴|𝑥 𝑒𝑠𝑡𝑒 𝑒𝑙𝑒𝑚𝑒𝑛𝑡 𝑖𝑛𝑣𝑒𝑟𝑠𝑎𝑏𝑖𝑙 𝑖𝑛 (𝐴, +,∙)}. Conform ipotezei 𝑈(𝐴) are un
număr impar de elemente , fie acesta 2𝑛 + 1.
Se ştie că (𝑈(𝐴),∙) este un grup şi cum acesta are 2𝑛 + 1 elemente ⇒𝑥 2𝑛+1 = 1, (∀)𝑥 ∈ 𝑈(𝐴) .
Din (−1) ∙ (−1) = 1 ⇒ −1 este inversabil, adică (−1) ∈ 𝑈(𝐴) ⇒ (−1)2𝑛+1 = 1, −1 = 1,
1 + 1 = 0.

EDIŢIA a XIII-a, 31 martie 2007


Clasa a VII-a

I. Fie 𝐴𝑛 = 6𝑛 + 21𝑛 + 2004 . Avem 𝐴0 = 2006 care nu este pătrat perfect și 𝐴1 = 2031 care
nu este pătrat perfect.
Pentru 𝑛 ≥ 2 ⇒ 6𝑛 = 𝑀9 şi 21𝑛 = 𝑀9, deci 𝐴𝑛 = 𝑀9 + 9 ∙ 222 + 6 = 𝑀9 + 6
Să vedem cum se comportă un pătrat perfect faţă de împărţirea cu 9. Fie 𝑥 ∈ ℕ, pătrat perfect :
𝑥 = (9𝑎 ± 1)2 = 𝑀9 + 1
167
𝑥 = (9𝑎 ± 2)2 = 𝑀9 + 4
𝑥 = (9𝑎 ± 3)2 = 𝑀9 + 9 = 𝑀9
𝑥 = (9𝑎 ± 4)2 = 𝑀9 + 16 = 𝑀9 + 7
𝑥 = (9𝑎)2 = 𝑀9

Deci 𝑥 = 𝑀9 sau 𝑀9 + 1 sau 𝑀9 + 4 sau 𝑀9 + 7 , ceea ce arată că 𝐴𝑛 = 𝑀9 + 6 nu este


pătrat perfect.

II. i) Fie 𝐸𝑀 ∩ 𝐴𝐵 = {𝑥} , 𝑀𝐹 ∩ 𝐵𝐶 = {𝑦}

∆𝐸𝑋𝐵 ≡ ∆𝐵𝑌𝐹 (𝐶𝐶) ⇒ 𝑌𝐹𝐵 ̂ ≡ 𝑋𝐵𝐸


̂ , 𝑌𝐵𝐹
̂ ≡ 𝑋𝐵𝐸 ̂ = 𝑌𝐵𝐹 ̂ + 𝑌𝐹𝐵 ̂ = 90° de unde 𝐸𝐵𝐹 ̂=
° ° °
90 + 90 = 180 ⇒𝐸, 𝐵, 𝐹 coliniare. Analog 𝐹, 𝐶, 𝐺 coliniare; 𝐺, 𝐷, 𝐻 coliniareşi 𝐻, 𝐴, 𝐸
coliniare.
ii) Fie 𝑀𝐺 ∩ 𝐶𝐷 = {𝑧}, 𝑀𝐻 ∩ 𝐴𝐷 = {𝑇}. Patrulaterul 𝐸𝐹𝐺𝐻 are diagonalele perpendiculare şi
𝐸𝐺∙𝐻𝐹 (2𝑀𝑋+2𝑀𝑍)(2𝑀𝑌+2𝑀𝑇)
deci : 𝑆𝐸𝐹𝐺𝐻 = 2 = , de unde
2
𝑆𝐸𝐹𝐺𝐻 = 2(𝑀𝑋 + 𝑀𝑍)(𝑀𝑌 + 𝑀𝑇) = 2 ∙ 𝑋𝑍 ∙ 𝑌𝑇 = 2𝑎2 = 2𝑆𝐴𝐵𝐶𝐷

III.

𝐴𝐶 3 3
𝑡𝑔 𝐵 = 𝐴𝐵 = 7 . Atunci 𝐵̂ > 22° 30′ ⇔ 𝑡𝑔 𝐵 > 𝑡𝑔 22° 30′ , adică 𝑡𝑔 22° 30′ < 7
̂,
Fie ∆𝑀𝑁𝑃 dreptunghic isoscel cu 𝑀𝑁 = 𝑁𝑃 = 1 ⇒𝑀𝑃 = √2 şi fie [𝑃𝑄 bisectoarea lui 𝑀𝑁𝑃
̂ ̂
𝑄 ∈ 𝑀𝑁 ⇒ 𝑁𝑃𝑄 = 𝑀𝑃𝑄 = 22 30′ . °

168
Cu teorema bisectoarei ⇒
𝑀𝑄 𝑀𝑃 √2 𝑀𝑁 √2+1
= = ⇒ = de unde
𝑄𝑁 𝑁𝑃 1 𝑄𝑁 1
1
𝑄𝑁 = = √2 − 1 şi deci 𝑡𝑔 22° 30′ =
√2+1
̂ = 𝑄𝑁 = √2 − 1 < 3 ⇔ √2 < 10 ⇔ 7√2 < 10 ⇔ 98 < 100 .
= 𝑡𝑔 𝑄𝑃𝑁 𝑁𝑃 7 7

Clasa a VIII-a

2 2
I. Relația 𝑥 + 𝑦 = 2√𝑥 − 1 + √𝑦 − 1 se scrie (√𝑥 − 1 − 1) + (√𝑦 − 1 − 1) = 0 , de unde
√𝑥 − 1 − 1 = 0, √𝑦 − 1 − 1 = 0 , deci 𝑥 = 2, 𝑦 = 2.

II. Fie 𝑉𝐴1 𝐴2 ⋯ 𝐴𝑛 piramida regulată cu 𝐴1 𝐴2 = 𝑎, 𝑉𝐴1 = 𝑏 şi O centrul bazei


𝐴1 𝐴2 ⋯ 𝐴𝑛 .

180 𝑎 𝑎 𝑎
Fie ⊥ 𝐴1 𝐴2 ⇒ 𝑚(𝐴̂ ̂
1 𝑂𝑀 ) = 𝑚(𝐴2 𝑂𝑀) = , 𝐴1 𝑀 = 𝑀𝐴2 = 2, 𝑂𝑀 = , 𝑂𝐴1 =
180 180
𝑛 2𝑡𝑔 2 sin
𝑛 𝑛
180
√4𝑏 2 sin2 − 𝑎2
𝑎2
. 𝑉𝑂 = √𝑉𝐴1 2 − 𝑂𝐴1 2 = √𝑏 2 −
𝑛
180 = 180 . Avem 𝑉𝑜𝑙[𝑉𝐴1 𝐴2 ⋯ 𝐴𝑛 ] =
4 sin2 2 sin
𝑛 𝑛
𝑎
𝑛∙𝑎∙ 180 √4𝑏 2 sin2
180
− 𝑎2
𝑆𝐴1 𝐴2 ⋯𝐴𝑛 ⋅𝑉𝑂 𝑛⋅𝑆𝐴1 𝑂𝐴2 ⋅𝑉𝑂 1 2𝑡𝑔 𝑛
= = 3∙ 𝑛
∙ 180 =
3 3 2 2 sin
𝑛
180 180
𝑛𝑎2 cos ∙√4𝑏 2 sin2 − 𝑎2
𝑛 𝑛
= 180 .
24 sin2
𝑛
𝑎 𝑏
III. Fie 𝑎, 𝑏, 𝑐, 𝑑 > 0 cu 𝑎 + 𝑏 ≥ 𝑐 + 𝑏𝑑. Presupun că 𝑎 + 𝑏 > 𝑎𝑐 + 𝑑 . Atunci adunând
𝑎 𝑏 1 1
inegalităţile avem: 2(𝑎 + 𝑏) > 𝑐 + 𝑎𝑐 + 𝑑 + 𝑏𝑑 = 𝑎 (𝑐 + 𝑐) + 𝑏 (𝑑 + 𝑑) ≥ 2𝑎 + 2𝑏 deoarece
1 1
𝑐 + 𝑐 ≥ 2 , 𝑑 + 𝑑 ≥ 2. Atunci 2(𝑎 + 𝑏) > 2𝑎 + 2𝑏 contradicţie.

169
𝑏
Arătăm printr-un exemplu că dacă 𝑎, 𝑏, 𝑐, 𝑑 > 0 şi 𝑎 + 𝑏 ≤ 𝑎𝑐 + 𝑑 atunci nu avem neapărat
𝑎 1 3
𝑎 + 𝑏 ≥ 𝑐 + 𝑏𝑑, 𝑎 = 3, 𝑏 = 1, 𝑐 = 2, 𝑑 = 3 ⇒ 3 + 1 ≥ 3 ∙ 2 + + 3, dar 3 + 1 ≥ 2 + 3 este
falsă.

Clasa a IX-a
1 1
I. [𝑥 2 − 𝑥 + 2] = [−𝑥 2 + 𝑥 + 2].
1 1 1 1 2 1 1 1
Deoarece 𝑥 2 − 𝑥 + 2 = 𝑥 2 − 𝑥 + 4 + 4 = (𝑥 − 2) + 4 ≥ 4 atunci [𝑥 2 − 𝑥 + 2] ≥ 0. De
1 1 3 1 2 3 3 1
asemenea, cum −𝑥 2 + 𝑥 + 2 = −𝑥 2 + 𝑥 − 4 + 4 = − (𝑥 − 2) + 4 ≤ 4 atunci [𝑥 2 − 𝑥 + 2] ≤
0 conform ecuaţiei date unica posibilitate este:

1 1
[𝑥 2 − 𝑥 + 2] = 0 0 ≤ 𝑥2 − 𝑥 + 2 < 1 1 1
{ 1
⇔{ 1 şi cum 𝑥 2 − 𝑥 + 2 ≥ 4,
2 2
[−𝑥 + 𝑥 + 2] = 0 0 ≤ −𝑥 + 𝑥 + 2 < 1

1 3
−𝑥 2 + 𝑥 + 2 ≤ 4 .
Obţinem:
1 1−√3 1+√3
𝑥2 − 𝑥 + 2 < 1 𝑥∈( 2 , 2 ) 1−√3 1+√3
{ 1 ⇔{ deci 𝑥 ∈ ( 2 , 2 ).
1−√3 1+√3
0 ≤ −𝑥 2 + 𝑥 + 2 𝑥∈[ 2 , 2 ]

sinn x = 0
II. Soluţia 1. Pentru 𝑛 par avem cos n 𝑥 = 1 + sinn x ≥ 1, de unde { n ⇔
cos 𝑥 = 0
sin x = 0 ⇔ x = kπ, 𝑘 ∈ ℤ
Pentru 𝑛 impar , 𝑛 ≥ 3 avem cosn 𝑥 − sinn x = cos 2 𝑥 + sin2 x.
cos2 𝑥 (1 − cosn−2 𝑥) = 0
cos2 𝑥 (1 − cosn−2 𝑥) + sin2 x (1 + sinn−2 x) = 0 de unde , { ⇔
sin2 x (1 + sinn−2 x) = 0
cos x = 0 n−2
{ n−2 sau {cos 𝑥=1
sin x = −1 sin 𝑥 = 0
cos x = 0 cos x = 1 π
Adică { sau { ⇔ x = − 2 + 2kπ, 𝑘 ∈ ℤ sau x = 2kπ, 𝑘 ∈ ℤ
sin 𝑥 = −1 sin 𝑥 = 0
√2 π π √2 π √2
Pentru 𝑛 = 1⇒ cos x − sin x = 1 | ⋅ 2 ⇒ cos 𝑥 cos 4 − sin 𝑥 sin 4 = 2 ⇒cos (𝑥 + 4 ) = 2 ⇒
π π
𝑥 + 4 = ± 4 + 2kπ, 𝑘 ∈ ℤ.
π π
Soluţia 2. Pentru 𝑛 = 1⇒ cos x − sin x = 1 cu 𝑥 + = ± + 2kπ, 𝑘 ∈ ℤ.
4 4
Pentru 𝑛 = 2⇒cos2 𝑥 − sin2 x = 1, cos2 𝑥 = 1, 2𝑥 = 2𝑘𝜋, 𝑘 ∈ ℤ
Pentru 𝑛 ≥ 3 nu putem avea simultan sin 𝑥 ≠ 0 şi cos x ≠ 0 deoarece ar rezulta că :
1 = |cosn 𝑥 − sinn x| ≤ |cos 𝑥|2 + |sin 𝑥|2 < cos 2 𝑥 + sin2 x = 1 contradicţie.
a) Dacă sin 𝑥 = 0 ⇒ cosn 𝑥 = 1 şi are soluţia 𝑥 = 2𝑘𝜋, 𝑘 ∈ ℤ pentru 𝑛 impar şi
𝑥 = 𝑘𝜋, 𝑘 ∈ ℤ pentru 𝑛 par.

170
π
b) Dacă avem cos x = 0 ⇒ sinn x = −1 care are soluţia x = − + 2kπ, 𝑘 ∈ ℤ
2
pentru 𝑛 impar şi nu are soluţie pentru 𝑛 par.

III. Fie 𝐷 mijlocul lui [𝐵𝐶].

⃗⃗⃗⃗⃗ = 𝐴𝐵
𝐴𝐷 ⃗⃗⃗⃗⃗ + 𝐵𝐷 ⃗⃗⃗⃗⃗ = 𝐴𝐶
⃗⃗⃗⃗⃗⃗ şi 𝐴𝐷 ⃗⃗⃗⃗⃗ + 𝐶𝐷⃗⃗⃗⃗⃗ ⇒2𝐴𝐷
⃗⃗⃗⃗⃗ = 𝐴𝐵
⃗⃗⃗⃗⃗ + 𝐴𝐶
⃗⃗⃗⃗⃗
Atunci ⃗⃗⃗⃗⃗
𝐴𝐵 + ⃗⃗⃗⃗⃗
𝐴𝐶 = 2𝐴𝑀 ⃗⃗⃗⃗⃗⃗ ⇔ 2𝐴𝐷 ⃗⃗⃗⃗⃗ = 2𝐴𝑀
⃗⃗⃗⃗⃗⃗ ⇔ 𝑀 = 𝐷.

Clasa a X-a

I. Ridicăm la pătrat prima ecuaţie şi folosim inegalitatea mediilor.


Obţinem: √2𝑥 + 1 + √3𝑦 + 1 = 4 ⇒ 2𝑥 + 1 + 3𝑦 + 1 = 16 − 2√(2𝑥 + 1)(3𝑦 + 1) , deci
2𝑥 + 3𝑦 + 2 ≥ 16 − (2𝑥 + 1 + 3𝑦 + +1) , adică 2(2𝑥 + 3𝑦 + 2) ≥ 16 ⇒ 2𝑥 + 3𝑦 ≥ 6.
În a doua ecuaţie folosim inegalitatea mediilor obţinută mai şi avem:64 = 22𝑥+√3𝑦+1 +
23𝑦+√2𝑥+1 ≥ 2√22𝑥+√3𝑦+1+3𝑦+√2𝑥+1 = 2√22𝑥+3𝑦+4 ≥ 2√210 = 26 = 64
Trebuie să avem egalitate şi deci 2𝑥 + √3𝑦 + 1 = 3𝑦 + √2𝑥 + 1, dar 2𝑥 + 1 = 3𝑦 + 1. Atunci
3
√2𝑥 + 1 + √3𝑦 + 1 = 4 ⇒ √2𝑥 + 1 = √3𝑦 + 1 = 2 de unde 𝑥 = 2, 𝑦 = 1.
II. Soluţia 1.

P(z1)
M(z)

OO
O
Q(z2)

Dacă 𝑧 = 0 putem lua 𝑧1 = 1, 𝑧2 = −1.


Dacă 𝑧 ≠ 0 fie 𝑀 imaginea geometrică a lui 𝑧 ⇒𝑀 ≠ 𝑂 şi cum |𝑧| < 1, avem că M este interior
cercului trigonometric. Mediatoarea [𝑂𝑀] taie cercul în P şi Q. Fie 𝑧1 , 𝑧2 afixele lui P şi Q.
Din POQM romb ⇒𝑧 = 𝑧1 + 𝑧2 şi |𝑧1 | = |𝑧2 | = 1.
Soluţia 2. Din |𝑧| < 1 ⇒ 𝑧 = 𝑟(cos 𝜑 + 𝑖 sin 𝜑) cu 𝑟 ≥ 0, 𝜑 ∈ [0,2𝜋).
Arătăm că (∃)𝜑1 , 𝜑2 astfel încât luând 𝑧1 = cos 𝜑1 + 𝑖 sin 𝜑1 şi 𝑧2 = cos 𝜑2 + 𝑖 sin 𝜑2 să avem
𝜑 +𝜑 𝜑 −𝜑 𝜑 +𝜑 𝜑 −𝜑
𝑧1 + 𝑧2 = 𝑧. 2 cos 1 2 2 cos 1 2 2 + 2𝑖 sin 1 2 2 cos 1 2 2 = 𝑟(cos 𝜑 + 𝑖 sin 𝜑).

171
𝜑1 −𝜑2 𝜑1 +𝜑2 𝜑1 +𝜑2
2cos (cos + 𝑖 sin ) = 𝑟(cos 𝜑 + 𝑖 sin 𝜑).
2 2 2
𝜑1 −𝜑2 𝜑1 −𝜑2 𝑟
2cos =𝑟 𝜑1 −𝜑2 𝑟 1
= 𝑎𝑟𝑐𝑐𝑜𝑠 2
2 2
O posibilitate este { 𝜑1 +𝜑2 , cos = 2 ∈ [0, 2) şi putem lua { 𝜑1 +𝜑2
=𝜑 2 =𝜑
2 2
𝑟
𝜑1 = 𝑎𝑟𝑐𝑐𝑜𝑠 2 + 𝜑
⇒{ 𝑟
𝜑2 = 𝑎𝑟𝑐𝑐𝑜𝑠 2 − 𝜑

−1+𝑖√3 1−𝜆𝜀
III. = , 𝐴 = {𝑧 ∈ ℂ|𝑧 = 1−𝜆𝜀2 , 𝜆𝜖ℝ}, 𝐵 = {𝑧 ∈ ℂ|𝑧 = cos 𝛼 + 𝑖 sin 𝛼 , 𝛼𝜖[0,2𝜋)}
2
−1+𝑖√3 2
1−𝜆𝜀 1−𝜆 2+𝜆−𝑖√3𝜆 √(2+𝜆)2 +(√3𝜆)
a) Fie ∈ 𝐴 ⇒ 𝑧 = 1−𝜆𝜀2 , cu 𝜆𝜖ℝ, 𝑧 = 2
−1−𝑖√3
= 2+𝜆+𝑖√3𝜆 de unde |𝑧| = = 1 şi
1−𝜆 2
2 √(2+𝜆)2 +(√3𝜆)

deci (∃) 𝛼𝜖[0,2𝜋) cu 𝑧 = cos 𝛼 + 𝑖 sin 𝛼. Ceea ce arată că 𝑧 ∈ 𝐵.


b) Deoarece 𝐵 = {𝑧 ∈ ℂ||𝑧| = 1} trebuie arătat că (∃)𝑧0 ∈ 𝐵, adică cu |𝑧0 | = 1 , 𝑧0 unic astfel
încât 𝑧0 ∈ 𝐴.
1−𝜆𝜀
Fie 𝑧 ∈ 𝐵, deci |𝑧| = 1. Studiem dacă (∃)𝜆𝜖ℝ cu = 1−𝜆𝜀2 ,z − z𝜆𝜀 2 = 1 − 𝜆𝜀 , 𝜆𝜀(𝑧𝜀 − 1) =
𝑧 − 1.
1
𝑧̅ −1𝑧−1 −1 1−𝑧 1−𝑧
Pentru 𝑧𝜀 − 1 ≠ 0, adică z ≠ 𝜀 ⇒ 𝜆 = 𝜀(𝑧𝜀−1) , 𝜆̅ = 𝜀̅(𝑧̅ 𝜀̅−1) =
2
2
𝑧
1 2 = 𝜀2 (𝜀2 −𝑧) = 𝜀(𝜀3 −𝜀𝑧) =
𝜀 ( 𝜀 −1)
𝑧
1−𝑧 1−𝑧
= 𝜀(𝜀𝑧−1) = 𝜆 deci 𝜆𝜖ℝ. Pentru 𝜀𝑧 − 1 = 0, adică 𝑧 = 𝜀 2 ⇒ 𝜆𝜀 ∙ 0 = 𝜀 2 − 1 fals şi deci
𝜀(1−𝜀𝑧)
𝑧0 = 𝜀 2 .

Clasa a XI-a
0
I. 𝐴0 = 𝐴 ⇒ 𝑑𝑒𝑡 𝐴0 = 𝑑𝑒𝑡 𝐴 = 𝑑 = 𝑑 (𝑝−1) . 𝐴1 = 𝐴∗0 = 𝐴∗ = 𝑑𝑒𝑡 𝐴 ∙ 𝐴−1 = 𝑑 ∙ 𝐴−1 ⇒
1
𝑑𝑒𝑡 𝐴1 = 𝑑𝑒𝑡 (𝑑 ∙ 𝐴−1 ) = 𝑑 𝑝 ∙ 𝑑𝑒𝑡 𝐴−1 = 𝑑 𝑝 ∙ 𝑑𝑒𝑡 𝐴 = 𝑑 𝑝−1 care se mai scrie 𝑑𝑒𝑡 𝐴∗ =
= (𝑑𝑒𝑡 𝐴)𝑝−1 .
𝑛 𝑛+1
Presupun că 𝑑𝑒𝑡 𝐴𝑛 = 𝑑 (𝑝−1) şi să arătăm că 𝑑𝑒𝑡 𝐴𝑛+1 = 𝑑 (𝑝−1) .
𝑛 𝑝−1 𝑛+1
𝐴𝑛+1 = 𝐴∗𝑛 ⇒ 𝑑𝑒𝑡 𝐴𝑛+1 = 𝑑𝑒𝑡 𝐴∗𝑛 = 𝑑𝑒𝑡 (𝐴𝑛 )𝑝−1 = (𝑑 (𝑝−1) ) = 𝑑(𝑝−1) . În concluzie,
(𝑝−1)𝑛
conform principiului inducţiei 𝑑𝑒𝑡 (𝐴𝑛 ) = 𝑑 .

II. a) Fie 𝛼𝜖ℝ astfel încât lim𝑛→∞ sin 𝑛𝛼 = 𝑙𝜖ℝ. Atunci 2𝑙 = lim𝑛→∞ (sin(𝑛 + 2)𝛼 +
sin 𝑛𝛼) = lim𝑛→∞ 2sin(𝑛 + 1)𝛼 cos 𝛼 = 2𝑙 cos 𝛼 ⇒ 2𝑙(cos 𝛼 − 1) = 0 .
Dacă 𝑙 ≠ 0 ⇒ cos 𝛼 − 1 = 0, 𝛼 = 2𝑘𝜋, 𝑘 ∈ ℤ. Atunci sin 𝑛𝛼 = sin(2𝑛𝑘𝜋) = 0 ⇒ 𝑙 = 0
contradicţie.
Dacă 𝑙 = 0 ⇒ sin 𝑛𝛼 → 0 şi deci cos2 𝑛𝛼 = 1 − sin2 𝑛𝛼 → 1 , sin(𝑛 + 2)𝛼 − sin 𝑛𝛼 → 0,
adică 2 sin 𝛼 cos(𝑛 + 1)𝛼 → 0. 4 sin2 𝛼 cos2 (𝑛 + 1)𝛼 → 0 şi cos2 (𝑛 + 1)𝛼 → 1 ⇒4 sin2 𝛼 = 0
⇒𝛼 = 𝑘𝜋, 𝑘 ∈ ℤ şi sin 𝑛𝛼 = sin 𝑛𝑘𝜋 = 0 .
b) Fie 𝛽𝜖ℝ astfel încât lim𝑛→∞ cos 𝑛𝛽 = 𝑙𝜖ℝ. Atunci 2𝑙 = lim𝑛→∞ (cos(𝑛 + 2)𝛽 +
cos 𝑛𝛽) = lim𝑛→∞ 2cos(𝑛 + 1)𝛽 cos 𝛽 = 2𝑙 cos 𝛽 ⇒ 2𝑙(cos 𝛽 − 1) = 0 .
Dacă 𝑙 ≠ 0 ⇒ cos 𝛽 = 1, 𝛽 = 2𝑘𝜋, 𝑘 ∈ ℤ şi cos 𝑛𝛽 = cos 2𝑛𝑘𝜋 = 1

172
Dacă = 0 ⇒ cos 𝑛𝛽 → 0 , deci şi cos 2𝑛𝛽 → 0 ⇒ 2 cos 2 𝑛𝛽 − 1 → 0 , contradicţie cu cos 𝑛𝛽 →
0.

III. Arătăm că 𝑑𝑒𝑡(𝑋 + 𝑌) + 𝑑𝑒𝑡(𝑋 − 𝑌) = 2(𝑑𝑒𝑡 𝑋 + 𝑑𝑒𝑡 𝑌) pentru orice 𝑋, 𝑌 ∈ 𝑀2 (ℂ)


𝑎 𝑏1 𝑎 𝑏2 𝑎 + 𝑎2 𝑏1 + 𝑏2 𝑎 − 𝑎2 𝑏1 − 𝑏2
Fie = ( 1 ),𝑌=( 2 ) . Atunci | 1 |+| 1 |=
𝑐1 𝑑1 𝑐2 𝑑2 𝑐1 + 𝑐2 𝑑1 + 𝑑2 𝑐1 − 𝑐2 𝑑1 − 𝑑2
𝑎 𝑏1 𝑎 𝑏2
2 (| 1 |+| 2 |)
𝑐1 𝑑1 𝑐2 𝑑2
(𝑎1 + 𝑎2 )(𝑑1 + 𝑑2 ) − (𝑏1 + 𝑏2 )(𝑐1 + 𝑐2 ) + (𝑎1 − 𝑎2 )(𝑑1 − 𝑑2 ) − −(𝑏1 − 𝑏2 )(𝑐1 − 𝑐2 ) =
2(𝑎1 𝑑1 − 𝑏1 𝑐1 + 𝑎2 𝑑2 − 𝑏2 𝑐2 ) cere este imediată.
Luăm 𝑋 = 𝐴2 + 𝐵 2, 𝑌 = 𝐴𝐵 + 𝐵𝐴 şi observăm că avem 𝑋 + 𝑌 = 𝐴2 + 𝐵 2 + 𝐴𝐵 + 𝐵𝐴
= (𝐴 + 𝐵)2 și 𝑋 − 𝑌 = 𝐴2 + 𝐵 2 − 𝐴𝐵 − 𝐵𝐴 = (𝐴 − 𝐵)2.Relaţia cerută este imediată.

Clasa a XII-a

I. 𝑦 = 2̂ − 𝑥 şi prima ecuaţie devine :2̂𝑥 + 4(2̂ − 𝑥) = 0̂ ⇔ −2̂𝑥 + 2̂ = 0̂ ⇔ 2̂(𝑥 − 1̂) = 0̂ de


unde 𝑥 − 1̂ = 0̂ sau 𝑥 − 1 = 3̂.
Pentru 𝑥 = 1̂⇒𝑦 = 1̂, iar pentru 𝑥 = 4̂⇒𝑦 = 2̂ − 4̂ = −2̂ = 4̂ ⇒ 𝑆 = {(1̂, 1̂), (4̂, 4̂)}

𝑥 2
II. Pentru 𝑥 ∈ (−𝜋, 𝜋) notăm 𝑡𝑔 = 𝑡, 𝑥 = 2𝑎𝑟𝑐𝑡𝑔 𝑡, 𝜑(𝑡) = 2𝑎𝑟𝑐𝑡𝑔 𝑡, 𝜑 ′ (𝑡) = . Avem
2 1+𝑡 2
1
1 2 1 1 2 𝑡+ 2 2𝑡+1
∫ 2𝑡 ∙ 1+𝑡 2 𝑑𝑡 = ∫ 𝑡 2 +𝑡+1 𝑑𝑡 = ∫ 2 𝑑𝑡 = 𝑎𝑟𝑐𝑡𝑔 2
+𝐶 = 𝑎𝑟𝑐𝑡𝑔 +𝐶 ,
2+ 1 2 √3 √3 √3 √3 √3
1+𝑡2 (𝑡+ ) +( ) 2
2 2
𝑥
1 2 2𝑡𝑔 +1
deci ∫ 2+sin 𝑥 𝑑𝑥 = 𝑎𝑟𝑐𝑡𝑔 2
+𝐶 .
√3 √3
1
Fie 𝐹: ℝ → ℝ o primitivă a lui : ℝ → ℝ , 𝑓(𝑥) = 2+sin 𝑥 .
𝑥
2 2𝑡𝑔 +1
𝐹(𝑥) = 𝑎𝑟𝑐𝑡𝑔 2
+ 𝑐𝑘
{ √3 √3 , (∀)𝑥 ∈ ((2𝑘 − 1)𝜋, (2𝑘 + 1)𝜋)
𝐹((2𝑘 + 1)𝜋) = 𝛼𝑘
𝑥
2 2𝑡𝑔 +1 𝜋
Luăm 𝑐0 = 0 ⇒ 𝛼0 = 𝐹(𝜋) = lim𝑥→𝜋 𝑎𝑟𝑐𝑡𝑔 2
=
𝑥<𝜋 √3 √3 √3
𝑥
2 2𝑡𝑔 +1 𝜋 𝜋 2𝜋
𝛼0 = 𝐹(𝜋) = 𝑥→𝜋
lim 𝑎𝑟𝑐𝑡𝑔 2
+ 𝑐1 = − + 𝑐1 ⇒ 𝑐1 = 𝛼0 + =
√3 √3 √3 √3 √3
𝑥>𝜋
𝑥
2 𝜋 2𝑡𝑔 +1
3𝜋
𝛼1 = 𝐹(3𝜋) = lim𝑥→3𝜋 𝑎𝑟𝑐𝑡𝑔 + 𝑐1 = + 𝑐1 = 2
𝑥<3𝜋 √3 √3 √3 √3
𝑥
2 2𝑡𝑔 +1 𝜋 𝜋 4𝜋
𝛼1 = 𝐹(3𝜋) = lim𝑥→3𝜋 ( 𝑎𝑟𝑐𝑡𝑔 2
+ 𝑐2 ) = − + 𝑐2 ⇒ 𝑐2 = 𝛼1 + = . Inductiv
𝑥>3𝜋 √3 √3 √3 √3 √3
2𝑘𝜋 (2𝑘+1)𝜋
găsim 𝑐𝑘 = şi 𝛼𝑘 = şi deci o primitivă 𝐹 a lui 𝑓 este:
√3 √3
𝑥
2 2𝑡𝑔 +1 2𝑘𝜋
𝐹(𝑥) = 𝑎𝑟𝑐𝑡𝑔 2
+
{ √3 √3 √3 , (∀)𝑥 ∈ ((2𝑘 − 1)𝜋, (2𝑘 + 1)𝜋), pentru orice 𝑘 ∈ ℤ .
(2𝑘+1)𝜋
𝐹((2𝑘 + 1)𝜋) =
√3

III. Mai întâi se arată că 𝑓 + 𝑔, 𝑓 ∘ 𝑔 ∈ 𝐸𝑛𝑑(𝐺), (∀)𝑓, 𝑔 ∈ 𝐸𝑛𝑑(𝐺). (De exemplu: (𝑓 + 𝑔)(𝑥 +
𝑦) = 𝑓(𝑥 + 𝑦) + 𝑔(𝑥 + 𝑦) = 𝑓(𝑥) + 𝑓(𝑦) + 𝑔(𝑥) + 𝑔(𝑦) = 𝑓(𝑥) + 𝑔(𝑥) + 𝑓(𝑦) + 𝑔(𝑦) =
(𝑓 + 𝑔)(𝑥) + (𝑓 + 𝑔)(𝑦), ∀𝑥, 𝑦 ∈ 𝐺)
173
Apoi se verifică axiomele inelului (schităm):
A1. (𝐸𝑛𝑑(𝐺), +) este un grup abelian, în care elementul neutru este morfismul nul 0, definit prin
0(𝑥) = 0𝐺 , ∀𝑥 ∈ 𝐺, iar opusul unui morfism 𝑓 este – 𝑓 , definit prin (−𝑓)(𝑥) = −𝑓(𝑥) , ∀𝑥 ∈
𝐺(Am notat cu 0𝐺 elementul neutru în grupul (𝐺, +)).
A2. (𝐸𝑛𝑑(𝐺),∘) este un monoid , care elementul neutru este morfismul identic 1𝐺 , definit prin
1𝐺 (𝑥) = 𝑥, ∀𝑥 ∈ 𝐺.
A3. " ∘ " este distributivă faţă de " + " pe mulţimea 𝐸𝑛𝑑(𝐺). Într-adevăr, pentru oricare 𝑓, 𝑔, ℎ ∈
𝐸𝑛𝑑(𝐺) şi 𝑥 ∈ 𝐺 avem: [(𝑓 + 𝑔) ∘ ℎ](𝑥) = (𝑓 + 𝑔)(ℎ(𝑥)) = 𝑓(ℎ(𝑥)) + 𝑔(ℎ(𝑥)) =
(𝑓 ∘ ℎ)(𝑥) + (𝑔 ∘ ℎ)(𝑥) = (𝑓 ∘ ℎ + 𝑔 ∘ ℎ)(𝑥), prin urmare
(𝑓 + 𝑔) ∘ ℎ = 𝑓 ∘ ℎ + 𝑔 ∘ ℎ .
De asemenea [ℎ ∘ (𝑓 + 𝑔)](𝑥) = ℎ((𝑓 + 𝑔)(𝑥)) = ℎ(𝑓(𝑥) + 𝑔(𝑥)) = ℎ(𝑓(𝑥)) + ℎ(𝑔(𝑥)) =
(ℎ ∘ 𝑓)(𝑥) + (ℎ ∘ 𝑔)(𝑥) = (ℎ ∘ 𝑓 + ℎ ∘ 𝑔)(𝑥) , prin urmare
ℎ ∘ (𝑓 + 𝑔) = ℎ ∘ 𝑓 + ℎ ∘ 𝑔

Ediţia a XIV, 4-6 aprilie 2008


Clasa a VII-a

x3  y 3 xy
I. Vom arăta că  şi având încă doua relaţii analoage, adunându-le va
x  xy  y
2 2
3
rezulta inegalitatea dorită.
x3  y 3 xy

x  xy  y
2 2
3
 x  y   x 2  xy  y 2  xy x  y 0
x 2  xy  y 2 1 x,y 0
    
x 2  xy  y 2 3 x 2  xy  y 2 3
x,y 0
 3x 2  3xy  3y 2  x 2  xy  y 2  2( x  y )2  0 care este adevărată,
 x, y  0, cu egalitate pentru x  y.

II.

174
A

x
z E

F H

y y
x z
B C
D
(  )Din H ortocentrul triunghiului ABC 
HAE ~ HBD ( AHE  BHD opuse la vârf şi m( AEH )  m(BDH )  90 ) 
HA HE
   HA  HD  HB  HE .
HB HD
Analog se obţine că HB  HE  HC  HF din HBF ~ HCE.

  Din HA  HD  HB  HE  HA  HE şi cum AHE  BHD, opuse la vârf


HB HD
 HAE    
HBD  m HAE  m HBD  x .

Analog din HB  HE  HC  HF  m HBF   m HCE   y şi din

HA  HD  HC  HF  m HAF   m HCD   z .
Dar în ABC avem
  
m A  m B  m C  180o  2  x  y  z   180o   x  y  z   90o şi de aici avem că

m  ADB  90 , m BEA  90
o o
 AD  BC, BE  AC  H ortocentrul ABC .

III. a2  621  b2 ; a, b  *
 b2  a2  621  b  ab  a   33  23, b  a  b  a
 b a 1  b  311
10   deci a  310
b  a  621 a  310
 b a  3 b  105
20   deci a  102
b  a  207 a  102
 b a  9 b  39
30   deci a  30
b  a  69 a  30
b  a  23 b  25
40   deci a  2
b  a  27  a  2

Clasa a VIII-a
I.

175
f 1 x   2f 1 x   3x 2  2x  6,  x  . Facem x  x.

f 1 x   2f 1 x   3x  2x  6
2

Înmulţind prima relaţie cu -2 şi adunând cu cea de a doua, avem


3f 1 x   6x 2  4x  12  3x 2  2x  6
3f 1 x   3x 2  6x  6  f 1 x   x 2  2x  2  f 1 x   1 x   1
2

de unde f  x   x 2  1,  x  care se arată simplu că verifică relaţia din enunţ.

II.
D C’

A
’ B’
P N
D C

A
S B M

M  AB, N  BD ', P  AD '  M,N,P   ABD ' 


Fie a latura cubului. Cu teorema catetei în BCD ' avem
a2 a a 3 BD '
BC2  BN  BD '  BN     .
a 3 3 3 3
Fie NS  AB; cum AD '  AB pentru că AB   ADD ' A '   NS AD ' şi deci
NS BN 1 NS 2
   
AD ' BD ' 3 AP 3
BS NS 1 MS 2
Pe de altă parte     (MB  AB) şi deci
AB AO ' 3 MA 3
NS MS
  M, N, P coliniare.
AP MA

Observaţie
MA NB PD ' 1
   2   1  1 şi atunci cu reciproca teoremei lui Menelaus avem că M, N, P
MB ND ' PA 2
coliniare.

III. 100  ab  a b  100  ab  a2b 


 100  10a  b  a2b  10(10  a)  b(a2  1)
de unde 5 | b(a2  1)  5 | b sau 5 | a2  1
10 5 | b cum b se observă că nu poate fi 0  b=5

176
10(10-a)=5( a2  1)  a2  1=2(10-a)  ( a  1) 2 =20, nu convine.

10(10  a)
20 5 | a2  1 a  2, 3, 7, 8; b 
a2  1
10  8
a=2  b   16 nu convine.
5
10  7
a=3  b  7
10
10  3 3
a=7  b   nu convine.
50 5
10  2 4
a=8  b   nu convine, deci a =3, b=7.
65 13

Clasa a IX-a
a a
I. În f  x  f  y   f   f    2f  xy ,  x, y  0,  luăm x=y=1 
   
x y
 f 2 1  f 2 a  2f 1  f 1  1  0  f 1  1 deoarece f(a)=1 din ipoteză.
2

a a
Luăm y=1 şi avem f  x   f 1  f    f a   2f  x   f  x   f  ,  x  0,  1
  x   x
a a a a
Luăm y  şi avem f  x   f    f    f  x   2f a   f  x   f    1,  x  0,   2
x x x x
Din (1) şi (2)  f (x)=1,  x  0,  . Dar luând x  x, y  x avem
2

0f2  xf 2  a 

 x
  2f  x ,  x  0,  şi atunci f(x)=1,  x  0,  , care verifică
relaţia din enunţ.

II.

B4
B5 B3

B6 B2
A3 A2

B7=A4 A1=B1
B8=O
A5 A8

A6 A7
10. n  4 .

177
Fie C cercul circumscris poligonului A1A2…An şi R raza sa (în figura de mai sus s-a luat
n=8). Construim punctele B1, B2, B3,…, Bn cu proprietăţile B1=A1,
B1B2  OA2 , B2B3  OA3 , ..., Bn1Bn  OAn . Deoarece OA1  OA2  ...  OAn  0 , rezultă
OB1  B1B2  ...  Bn1Bn  0, adică OBn  0 , prin urmare punctul Bn este chiar punctul O,
originea cercului C.
Poligonul B1B2B3…Bn este regulat şi are latura R. Pentru k 1,2,3,..., n avem
OA1  OA2  ...  OAk  OB1  B1B2  ...  Bk 1Bk  OBk
Dacă un vector OAi ,1  i  n , aparţine intersecţiei X Y , atunci
OAi  OA1  OA2  ...  OAk pentru un anumit k, 1 k  n , adică OAi  OBk sau încă
Ai  Bk 1
Însă toate vârfurile B2, B3, …, Bn-2 sunt exterioare cercului C, deoarece O fiind vârful Bn al
acestui poligon, înseamnă că OB2, OB3, …, OBn-2 sunt diagonale, deci au lungimea mai mare ca
latura poligonului, care este R. Aşadar, singurele vârfuri ale poligonului B1B2B3…Bn situate pe
cercul C sunt B1=A1 şi Bn-1. Prin urmare, egalitatea (1) este posibilă doar pentru k=1 (când i=1),
sau k=n-1.
Când k=n-1, avem Ai=Bn-1 şi deoarece
 n  2 ,

m B1OBn1   n

iar m AOA
1 
i   i  1 
2
n
 n  2  2  i  1 , adică n=2i (n par) şi
n
i  . N acest caz OA1  OA2  ...  OAn1  OAn  OAn .
2 2

20. n  3 . Se arată imediat că X Y  OA1 . 



 
 OA1 , dacă n este impar
Deci X Y     
 OA1,OAn , dacă n este par .
  2 
a  p a2  p a p
III. S  1   ...  n . Notăm xk=ak+q, k=1, 2, …, n şi atunci
a1  q a2  q an  q
x1  p  q x2  p  q x  pq 1 1 1
1 S    ...  n  n   p  q     ...  .
x1 x2 xn  x1 x2 xn 
Cu inegalitatea Cauchy-Buniakovski avem
a1  a2  ...  an 2  n a12  a22  ...  an2  şi cum
a12  a22  ...  an2  c  a1  a2  ...  an  nc care se scrie
x1  q  x2  q  ...  xn  q  nc , adică x1  x2  ...  xn  nc  nq.

178
1 1 1
Dar  x1  x2  ...  xn     ...    n2 , de unde
 x1 x2 xn 
1 1 1 n2 n2
  ...    .
x1 x2 xn x1  x2  ...  xn nc  nq
S n n2
Atunci, cu (1) avem  , de unde pentru p<q 
p q nc  nq
n2  p  q  n2 p  n nc np  nc p n c
S  n   n  n , iar pentru p>q se obţine
nc  nq nc  nq nc  nq q n c
p n c
că S  n  .
q n c

Clasa a X-a
I. an   n  2 n  1 n  1  n n  1  2 
n1 n

 
  n  2 n2  1  n  1  n  n  1  2 
n n

   
 n3  2n2  n  2 Mn3  Cn2 n2  Cn1n  1  n Mn3  Cn2n2  Cn1n  1  2. 
n 1
Cum n este impar   *.
2
n  n  1 2 n  1 3
Cn2 n2  n  n  Mn3
2 2
an   n  2n  n  2Mn3  n2  1  n Mn2  1  2 
3 2

 Mn3  2n2  n  2n2  2  n  2  Mn3 .

II. 4x   4x  11 2x  3x 2  23x  30  0


Notăm 2x  t şi avem t 2   4x  11 t  3x 2  23x  30  0 .

 
   4x  11  4 3x2  23x  30  4x2  4x  1  2x  1
2 2

4x  11  2x  1
t1,2  , de unde
2
10 t= -x+6  2x  x  6  0. Cum f :  , f  x   2x  x  6 este strict crescătoare, rezultă
că ecuaţia f(x)=0 are cel mult o soluţie şi cum f(2)=0, rezultă că x=2 este unica soluţie.
20 t= -3x+5  2x  3x  5  0. Ca mai sus, avem că x=1 este soluţie unică.
În concluzie ecuaţia are soluţiile x1=1, x2=2.

III.

179
B
A P
S Q

D
R C

 
Fie   m AOB , a  OA, b  OB, c  OC, d  OD.
Cu teorema cosinusului avem
AB2  a2  b2  2ab  cos, BC2  b2  c 2  2bc  cos,
CD2  c 2  d 2  2dc  cos, DA2  d 2  a2  2da  cos.
AB  OP ab  sin ab  sin
Aria  AOB    OP  şi analog
2 2 AB
bc  sin cd  sin ad  sin
OQ  , OR  , OS  . Acum avem
BC CD AD
1 1 1 1 a2  b2  2ab cos c 2  d 2  2cd cos
     
OP 2 OR2 OQ2 OS2 a2 b2 sin2  c 2d 2 sin2 
b2  c 2  2bc cos d 2  a2  2da cos 1 1 1 1
   2 2 2 2
b c sin 
2 2 2
d a sin 
2 2 2
a b c d
 1 1 1 1 1 1  1 1
2    cos  2  2  2  2  2    cos 
 ab cd  a b c d  bc da 
 1 1 1 1
     cos  0  cos  0    90o 
 ab bc cd da 
ABCD este ortodiagonal.

Clasa a XI-a

I. na1   na2   ...  nak   nak 1 ,  n  *

na1   na2   ...  nak   nak 1  ,


1 n n n n
 n  *

   a,  a 
na
Se arată uşor că lim
n n
  şi atunci, trecând la limită în (1), obţinem
a1  a2  ...  ak  ak 1.

II. Soluţia 1 :

 
Fie fn : n2  1,   , fn  x  
1

1
 ... 
1
 1.
n2  1 x n2  2  x n2  n  x

180
fn este funcţie strict descrescătoare şi continuă.
n n
1 1
fn 0   1  1 0 ,
k 1 n2  k k 1 n2
n n
1 1
fn  n    1 0 . 1 
n2  k  nk 1 k 1 n
2

Atunci ecuaţia fn  x   0 are o soluţie unică xn   n,0 , deci avem


n
1

k 1 n  k  xn
2
 1,  n  *
. Fie

n
1  1 n
1  n n2  k  x  n 2  k
an    1     n

n k k 1  n  k n  k  xn  k 1 n  k  n2  k  x
2 2 2 2
k 1
  n
n
xn

k 1 n2  k  n2  k  xn  n2  k  xn  n2  k 
n
an 1

xn k 1 n2  k  n2  k  x
n n2  k  xn  n2  k  
n2  n2  n   n2  n  n 2
n2  k  n2  k  xn  n2  k  xn  n2  k  
 n2  n  n2  n   n2  n  n2  n  şi atunci
n  n2 n2an n  n2
  ,  n  *

n n n  2
 n n  n
2 2
 xn 
2 n2  n n2  n 
n2an 1
de unde lim  1
n x 2
n
n 
n
1 1 1 
Pe de altă parte avem an  1     
k 1 n2  k k 1  n2 n2  k 
n
k

 
şi cum
k 1 n n2  k n  n2  k
k k k
  ,  k  1, n
n n2  n n  n 2  n   n n2  k n  n 2  k   
n n2 n  n2 
n  n  1 n  n  1 1
Obţinem  an  , de unde lim  nan   2
2n n  n n  n  n
2
 2
 2n n n  n
2
 2
 n 4

xn 1
Din (1) şi (2)  lim  .
n n 2
Soluţia 2 :

181
1 1 1
Ca şi la soluţia 1 se arată că ecuaţia   ...   1 are o
n2  1 x n2  2  x n2  n  x
n
unică soluţie reală xn   n,0 şi deci avem
1

k 1 n2  k  xn
 1.

Pentru a  1 avem că
1
a

2

 2
a  a  a  a 1 a  a 1
,
2 

  2 a  1  a ,2 a  a  1 deci   

2 a 1 a   1
a
 
 2 a  a  1 ,  a  1, de unde

   
n n n
1

k 1
2 n 2
 k  xn  1  n 2
 k  xn  
k 1 n  k  x
2
 
k 1
2 n2  k  xn  n2  k  xn  1
n

adică 2  
n2  n  xn  1  n2  1 xn  1  2  n2  n  xn  n2  xn 
2 n2  n  x  1  2 n2  1 x  1
n n

 2 n  xn  1  2 n  n  xn
2 2

4n2  4n  4x  4  4n2  4  4x  4 n2  1 x  1
n n n

 4n  4xn  4 n  xn  1  4n  4n  4xn
2 2 2

4n  1  4 n2  1 x 16n2  8n  1  16n2  16  16xn



n
 
 16n  16xn  16n  8n  1
2 2
 4 n2  xn  4n  1
x 1
 8n  15  16xn  8n  1, de unde  lim n   .
n n 2
 x1   0 
x   
0
III.   Presupunem că detA=0. Rezultă că sistemul liniar omogen A  2     are şi soluţii
   
   
 xn   0 
nenuli. Atribuim necunoscutelor secundare valori raţionale şi atunci obţinem soluţii raţionale
nenuli. Deoarece din (x1, x2, …, xn) soluţie a sistemului rezultă că şi (mx1, mx2, …, mxn) este
soluţie ,  m  * , obţinem că sistemul are şi soluţii întregi nenuli, adică
 x1   0 
x   
0
 x1,x2 , , xn  , nu toate nule, astfel încât A  2     . Luăm
   
 xn   0 

182
 x1 x1 x1 
x x2 x2 
X  2  Mn   şi avem X  On iar fA  X   AX  On . Dar
 
 
 xn x xn 
fA On   AOn  On , ceea ce contrazice faptul că f este injectivă.
  Fie X1, X2  Mn   cu fA  X1   fA  X2   AX1  AX2 .
Din det A  0    A1  Mn   şi atunci A1AX1  A1AX2  X1  X2  fA este injectivă.
Clasa a XII-a

I. a) 5x  0 în 100 
 100 | 5x  20 | x şi atunci A  0, 20, 40, 60, 80 . 
b) Din (n,a)=d    p,q  cu pa  qn  d. De asemenea din d | n  n  dn1 cu n1  ,
iar din d | b  b  db1 cu b1  obţinem că pa  d şi luând x0  pb1 avem

ax0  apb1  db1  b  x0  B.


c) Din b) avem că B este nevidă şi fie x0  B .
Considerăm f : A  B, f  x   x  x0 . Funcţia f este bine definită pentru că
a  x  x0   ax  ax0  0  b  b  x  x0  B.
Arătăm că f este bijectivă   y  B,  x  A unic cu f(x)=y, x  x0 =y, x=y-x0 unicitatea
este evidentă.
ax  a  y  x0   ay  ax0  b  b  0  x  A .
Din f injectivă şi A, B finite ( sunt incluse în n ), rezultă că au acelaşi număr de elemente.

II.
1 1 1 1

 
In   x n ex dx   x n ex ' dx  x n ex 0   nx n1ex dx  e  n  x n 1ex dx  In  e  nIn1, n 
1 *

0 0 0 0

Avem şi In1  e   n  1 In  2
Din (1) şi (2)  In1   n  1 In  In  nIn1  In1  n In1  In   3
Din 0  x n ex  x n e,  x  0,1 prin integrare avem 0  In  de unde lim In  0  4
e
n 1 n
n n
Ik 1
Din (3), (4) avem lim   lim Ik 1  Ik   lim I0  In   I0  e  1.
n n n
k 1 k k 1

f 0  f 3  0  3g 0 
III. Din i) avem   3g 0  2g 0  g    0.
f 0  f  2  0  2g 0 
f(x)=f(x+0)=g(x)+g(0)=g(x),  x  L , deci f=g.

183
 
Din ii) avem f 1  f 12  g 2 1 şi cum f 1  g 1  g 2 1  g 1 de unde g(1)=1
sau g(1)=0, adică în definitiv f(1)=1 sau f(1)=0.
Dacă am avea f(1)=0, ar rezulta că f  x   f  x  1  f  x   f 1  0, x  L , adică f ar fi
funcţie constantă, ceea ce contrazice iii).
Prin urmare f(1)=1 (1). Ipotezele i), ii) şi condiţia obţinută (1) arată că f este un morfism
de inele, deci un morfism de corpuri.
Fiind morfism de corpuri, este injectiv, iar din iii) deducem că acest morfism f este şi
surjectiv. În concluzie, f este un izomorfism de corpuri.

Ediţia a XV-a, 3-5 aprilie 2009


Clasa a VII-a
Subiectul I.
Fie a1, a2 , , a10 cele 10 numere. Putem presupune că a1  a2   a10 . Presupunem prin absurd
că nu există printre acestea trei numere cu proprietatea din enunţ. Deci
a3  a1  a2  2
Rezultă că a4  a2  a3  1 2  3 . În continuare a5  a3  a4  2  3  5 , a6  a4  a5  3  5  8 ,
a7  a5  a6  5  8  13 , a8  a6  a7  8 13  21, a9  a7  a8  13  21  34 ,
a10  a8  a9  21  34  55 , ceea ce contrazice ipoteza. Rezultă că printre cele 10 numere există
trei care pot reprezenta lungimile laturilor unui triunghi.

Subiectul II.
 
Avem a  n n 4  2009 care evident se divide prin 2.
Dacă n se divide prin 3 , atunci şi a se divide prin 3. Dacă n nu se divide prin 3 ( n  3k  1 sau
n  3k  2 ), atunci n2  M3  1 ( M 3 înseamnă multiplu de 3), n4  M3  1 şi atunci
n4  2009  M3  2010  M3 , aşadar a se divide prin 3.
Dacă n se divide prin 5, atunci şi a se divide prin 5. Dacă n nu se divide prin 5 ( n  3k  1 sau
n  3k  2 ), atunci n2  M5  1 sau n2  M5  4 şi atunci n4  M5  1 şi prin urmare
n4  2009  M5  2010  M5 , aşadar a se divide prin 5.
Cum a se divide prin 2, 3 şi 5 rezultă că a este divizibil cu 30 pentru orice n număr natural.

Subiectul III.
Fie S  S ABC , S1  SFBD , S 2  S EDC , atunci S AEDF  S  S1  S2 . Inegalitatea din enunţ devine
S S 
S1  S2  S  S1  S2 , deci trebuie să demonstrăm că 2S1  S2   S adică 2 1  2   1 .
S S 

184
 x  0,1 , deci
BD DC
Fie 1  x . Deoarece triunghiurile FBD şi ABC sunt asemenea,
BC BC
atunci 1  x 2 , analog triunghiurile EDC şi ABC sunt asemenea şi atunci 2  1  x . Avem
S S 2

S S
S S 
S S 
  S S 
2 1  2   2 x 2  1  x2  4x 2  4x  2 , adică 2 1  2   2x  12  1  1 , egalitate
S S 
pentru x  , adică D mijlocul laturii BC .
1
2

Clasa a VIII-a

Subiectul I.
Fie f :  , f x  ax  b . În relaţia f 3x  2 y  2  f 3x 1  f 2 y  1 1 facem
x  y  0 şi obţinem f 2  2 f 1  2 şi cum f 2  8 , avem f 1  5 . Deoarece f 1  5 şi
a  b  5
f 2  8 , avem sistemul  , care are soluţia a  3 , b  2 şi prin urmare f x  3x  2 .
2a  b  8
Arătăm în final că funcţia f x  3x  2 verifică relaţia din enunţ pentru orice x, y  R :
f 3x  2 y  2  33x  2 y  2  2  9x  6 y  8 ;
f 3x 1  f 2 y 1  2  33x 1  2  32 y 1  2  2  9x  6 y  8

Subiectul II.
Pentru diagonala unui paralelipiped dreptunghic, avem relaţia d 2  a 2  b 2  c 2 . Ţinând cont de

relaţia din enunţ, avem a  b  c


2 2 2

a  b  c2
sau 2a 2  2b 2  2c 2  2ab  2ac  2bc  0 ,
3
adică a  b  b  c  c  a  0 şi atunci a  b  b  c  c  a  0 , adică a  b  c , ceea ce
2 2 2

înseamnă că paralelipipedul este cub.

Subiectul III.
Fie N mijlocul lui AB . Avem MN AC şi deci mNMO  m AC, OM   600 .

185
Cum triunghiurile OBC şi OAB sunt congruente (LLL), atunci OM  ON şi pentru că
mNMO  600 , atunci triunghiul OMN este echilateral. Deoarece AB  BC  CA  12 3 ,
AC
atunci OM  ON  MN   6 3 . În triunghiul dreptunghic OMB , avem
2
OM  MB  6 3 şi aplicând teorema Pitagora, obţinem OB  6 6 . În mod analog
OA  OC  6 6 . Dar AB2  3 122  12  36  2  6  36  OA2  OB 2 , deci AO  OB .
Analog obţinem BO  OC şi CO  OA , deci tetraedrul OABC este tridreptunghic cu
AB  BC  CA  12 3 şi OA  OB  OC  6 6 , iar înălţimea tetraedrului dusă din O
S ABC  h S OBC  OA
(notată h ) este înălţimea prismei. Deoarece vOABC   , atunci
3 2
S OBC  OA OA  OB  OC
h  . Avem
S ABC 2S ABC
OA  OB  OC
vABCABC  S ABC  h   648 6 . Cum triunghiul ABC este echilateral,
2
BC 2  3 122  3  3 62  62  6
atunci S  ABC    62  3  3 , atunci h  2  6 2 şi avem
4 4 6 6 3
Sl  ABCABC  3  AB  h  3 12 3  6 2  432 6

Clasa a IX-a
Subiectul I.
În cazul în care triunghiul ABC este dreptunghic, relaţia este evidentă. Dacă triunghiul nu este
dreptunghic, fie D punctul diametral opus lui A în cercul circumscris şi P mijlocul lui BC.
Patrulaterul BHCD este paralelogram, rezultă că punctele H , P, D sunt coliniare. În triunghiul
AHD, OP este linie mijlocie, deci AH  2OP . Cum OP este mediană în triunghiul OBC ,
OB  OC
rezultă OP  . Obţinem OB  OC  AH  OH  OA, de unde OH  OA  OB  OC .
2

Subiectul II.

186
În orice triunghi avem ha  ia  ma . Trebuie să arătăm că ha  hb  hc  9r şi
9R  1 1 1  1 1 1
ma  mb  mc  . Avem ha  hb  hc  2S      2rp    , dar
2 a b c a b c
 1 1 1  1 1 1
2 p     a  b  c     9 , atunci ha  hb  hc  9r . De asemenea
a b c a b c
9a 2  b 2  c 2 
ma  mb  mc  3ma2  mb2  mc2   . Folosind inegalitatea cunoscută
4
9R
a 2  b 2  c 2  9R 2 , obţinem ma  mb  mc  .
2

Subiectul III.
S  z  k

Cu notaţiile x  y  S , xy  P sistemul devine  2 şi obţinem S  k  z
 P  S  z  a  
1
a  k 
3

P
a  k   k  z z  a. Atunci x, y sunt soluţiile ecuaţiei t 2  St  P  0 cu
2

3

 9z 2  6kz  12az  4a 2  k 2  4ak

3z  2a  k 2 . Din   0 , obţinem z  k  2a şi
3 3 3
ak
apoi x  y  .
3
Clasa a X-a
Subiectul I.
p  r Prin absurd presupunem că loga b  . Atunci f 1  a  f log b a , ceea ce este o
contradicţie cu f injectivă.
r  p Prin absurd presupunem că f nu este injectivă, deci există x1, x2  , x1  x2 astfel
încât f x1   f x2  . Dacă x1, x2  , atunci din f x1   f x2  , rezultă a x1  a x2 , deci x1  x2 ;
dacă x1, x2   , atunci din f x1   f x2  , rezultă b x1  b x2 , deci x1  x2 ; dacă x1  ,
x2   , atunci din f x1   f x2  , rezultă a  b , deci x1  x2 log a b , adică
x1 x2

loga b  , ceea ce este o contradicţie.


Deci p  r
q  r Cum funcţia f este surjectivă, pentru b  0,  , există x astfel încât f x  b .
Dacă x  , atunci din f x  b , avem b x  b , adică b  1  . Prin urmare x şi
atunci din f x  b , avem a x  b , adică x  loga b  .
r  q Avem loga b şi demonstrăm că funcţia f este surjectivă. Fie y  0,  oarecare.
Dacă loga y  , atunci pentru x  loga y  , avem f x  a loga y  y , iar dacă loga y  

  , x  logb y avem f x  b logb y  y .


loga y
, atunci pentru x 
loga b
Deci q  r .
În concluzie cele trei propoziţii sunt echivalente.

187
Subiectul II.
Notăm E  log bcd a  log acd b  log abd c  log abc d x  lg a , y  lg b , z  lg c , t  lg d şi
a b c d
lg a lg a x
S  x  y  z  t . Avem log bcd a    şi deci
a lg
bcd lg b  lg c  lg d  lg a S  2 x
a
2
S  
x x
. Folosind inegalitatea cunoscută (caz particular al inegalităţii
S  2x xS  2x 2
a 2  a
2

Cauchy – Buniakovski – Schwartz)   , avem


 
x2  x  S 2 . Vom demonstra că S 2  2 , inegalitate
2

 xS  2x 2 S x  2 x 2 S 2  2 x 2

S 2  2 x 2
  
care este echivalentă cu inegalitatea 4 x  x
2 2
 
. Folosind inegalitatea 2 a 2  b2  a  b ,
2

  2
 
avem 2 x 2  y 2  x  y şi 2 z 2  t 2  z  t  . Avem
2

2

4 x  2 x  y  z  t 
2 2
  x  y  z  t  , aşadar 2
2 S2
S  2 x 2
 2 şi deci
x2
 xS  2x 2  2
.

Subiectul III.
1. Din z  z k  z k , avem ( z  z k )(z  z k )  z k z k , de unde z  z  z  zk  z  zk , k  1, n .
n n
Însumând relaţiile de mai înainte, obţinem n  z  z  z   z k  z   z k , adică n  z  z  2  z  z ,
k 1 k 1

deci (n  2) z  0 . Prin urmare z  0 şi deci z  0 .


z2 z
2. Fie u  , v  3 . Cum z1  z2  z3 şi z1  z 2  z3  0 (din punctul precedent), atunci
z1 z1
u  v  1 şi 1  u  v  0 . Fie u  cos  i sin  , v  cos   i sin  , cu  ,  ,[0,2 ) . Cum
1  cos  cos   0
1  u  v  0 , atunci obţinem  . Avem
sin   sin   0
1
1  sin 2   cos2   (1) 2 sin 2   (1) 2 (1  cos ) 2 , adică 1  2  2 cos sau cos   .
2
 2 4  2 4
Analog cos    . Atunci  ,     ,  . Fie  
1
,  , atunci u   , v   2 ,
2  3 3  3 3
unde  este rădăcină nereală de ordin 3 a unităţii. Imediat triunghiul ABC, A(z1 ) , B( z2 ) , C( z3 )
, z2    z1 , z3   2  z1 , este echilateral.
Observaţie. Se poate arăta că z12009  z22009  z32009  0 .

Clasa a XI-a

188
Subiectul I.
x  xA y  yA x 2 y 3
Ecuaţia dreptei AB este  , de unde  , adică , eventual
xB  x A y B  y A 6 4
x  xB y  yB
se obţine y  7  x  8 . Fie M a, b AB
2
considerând ecuaţia sub forma 
x A  xB y A  y B 3
, atunci ţinând cont de cele două forme (echivalente!) ale ecuaţiei dreptei AB , avem
b  3  a  2 şi b  7  a  8 . Avem AM 2  xM  xA 2   yM  y A 2  a  22  b  32
2 2
3 3
şi cum b  3  a  2 , obţinem AM 2  a  2 . De asemenea MB  a  8  b  7 şi
2 13 2 2 2

3 9
cum b  3  a  2 , obţinem MB 
2 13
a  8 . Cum AB  36  16  2 13 , relaţia din enunţ
3 3
devine
13
a  22  2 13 a  8 , adică a  22  6 a  8 . Avem ecuaţia a 2  4a  4  6 a  8 ,
9 3
care pentru a  8 devine a 2  10a  52  0 iar pentru a  8 devine a 2  2a  44  0 . Prima
ecuaţie nu are soluţii, iar a doua are soluţiile a1,2  1  3 5   ,8.
13 13
Din AM  MB, rezultă a2  a  8 , adică a  2  a  8 , ceea ce înseamnă că
3 3
a  5 . Din cele două soluţii convine doar a  1 3 5 ; imediat b  1 2 5 şi atunci

M 1  3 5,1  2 5 . 
Subiectul II.
Avem polinomul caracteristic det A  X  I 2   X 2  Tr A  X  det A şi ţinând cont de condiţii
 
det A  X  I 2   X 2  X   2 şi atunci det A   k I 2   2k   k 1   2 , deci
 k detA   k I 2    3k   2k 1   k 2 . Avem  detA   I     
n n
k k
2
3k
  2k 1   k 2 . Ţinând
k 1 k 1
n
 3n  1 n
 2n  1 n
 n 1
con că  n  1 , avem  3k   3
k 1  3 1
 0,  2k 1   3
k 1  2 1
 0,  k 2   3
k 1  2 1
0

   
n n
şi atunci 
k 1
k
det A   k I 2    3k   2k 1   k 2  0
k 1

 
Subiectul III.
Avem şirul xn n1 , xn  n 2

 an  b  n 2  an  b  n 2  an  b şi vom calcula partea
întreagă a numărului n 2  an  b .
2
a2  a2 a
Dacă a este număr par, atunci n 2  an  b   n    b  . Dacă b   0 , pentru n
4  4 2
a a
suficient de mare, avem n   1  n 2  an  b  n  , adică n 2  an  b  n   1 şi
2 2
a
2
 
 a a 2
atunci lim xn  lim  n 2  an  b  n  1    1 . Dacă b   0 , pentru n suficient de mare,
n n
 2 4

189
avem n 
a
2
a
 n 2  an  b  n   1, adică
2
 n  an  b n  a2 şi atunci
2

 a
lim xn  lim  n 2  an  b  n    0 .
n n
 2
Dacă a este număr impar, pentru n suficient de mare, avem
n
a 1
2
 n 2  an  b  n 
a 1
2
 1, adică  n  an  b n  a 21 şi atunci
2

 a 1 1
lim xn  lim  n 2  an  b  n   .
n n
 2  2

Clasa a XII-a
Subiectul I.
1
Din definiţie, avem f este reciproc  f  X   X n f   . Fie f , g două polinoame reciproce,
X
1 1 1 1
deci f  X   X n f   , g  X   X m g  . Rezultă f  X g  X   X nm f   g  , deci
X X X X
1
h X   X nm h  , h  f  g este reciproc.
X

Subiectul II.
Determinăm o primitivă a funcţiei f x  pe 0,  . Cu substituţia t  tg , avem
1 x
2  sin x 2
x
2tg  1
2t  1 2
F x    2
dt 2 2  C . Aceeaşi formă are o primitivă pe
 arctg  arctg
t  t 1
1
3 3 3 3
 x
2 2tg 1
 arctg
2  C1 , x  0,  
 3 3

intervalul  ,2  . În concluzie F are forma F x  k x  . Din
 x
 2tg  1
 2 arctg 2  C , x   ,2 
 3 3
2

 
condiţia de continuitate, avem  C1  k    C2 şi rezultă că
3 3
 x
2 2tg  1

 arctg
2   k , x  0,  
 3 3 3

F x  k x  . Se verifică că F este derivabilă şi că
 x
 2tg  1

 k , x   ,2 
2
 arctg 2 
 3 3 3

190
F x  f x pentru orice x 0,2  , deci F este o primitivă a lui f . Atunci
2
2
0 2  sin x dx  F x 0  F 2   F 0  3 .
1 2

Subiectul III.
Demonstrăm întâi un rezultat binecunoscut:
1
Lemă. Dacă f : 0,1  este o funcţie continuă, atunci lim n x n hxdx  h1 .

n
0

Demonstraţie. Cum funcţia h este continuă, atunci există M  0 astfel încât f x  M ,
x  0,1. De asemenea din continuitatea funcţiei h în punctul x0  1 avem:   0 , a  0,1
astfel încât pentru orice x  a,1, avem hx  h1   . Avem succesiv
1 a 1
K n  n  1 x n hx  h1dx  n  1 x n hx  h1 dx  n  1 x n hx  h1 dx ,
0 0 a
a a
n  1 x n hx  h1 dx  2M n  1 x n dx  2Man1 ,
0 0

n  1 x n hx  h1 dx  n  1   x n dx   1  a n1  şi atunci


1 1

a 2a 2
 
Kn  2Man1 
2
1  a   2Ma
n1 n1
 . Dar cum a  0,1 , avem lim a n1  0 , adică n 
2 n

  
astfel încât 2Man1  , n  n . Atunci Kn     , n  n , ceea ce înseamnă că
2 2 2
1 1 1
lim K n  0 . Dar cum K n  n  1 x n hxdx  n  1 x n h1dx  n  1 x n hxdx  h1 ,
n
0 0 0
1 1
rezultă că lim n  1 x n hxdx  h1 şi deci şi lim n x n hxdx  h1 .
 
n n
0 0
Trecem acum la rezolvarea problemei. Avem
1
0 1

I 2n  2n  x 2n f xdx  2n  x 2n f xdx   x 2n f xdx  şi cum cu schimbarea de variabilă x  t
1  1 0 
0 1
 1 1

avem  x 2n f xdx   t 2n f  t dx , atunci I 2n  2n  x 2n f  xdx   x 2n f xdx  . Folosind lema
1 0 0 0 
de mai înainte, obţinem lim I 2n  f  1  f 1 .
n

 1 2n1 1

Procedând analog, vom avea I 2n1  2n  12n  x f xdx   x 2n1 f  xdx  şi atunci

0 0 
lim I 2n1  f 1  f  1
n

191
Ediţia a XVI-a, 26-28 martie 2010
Clasa a VII-a
Subiectul I
Fie EP AD şi EP  AD.

Patrulaterul AEPD este paralelogram şi mAFE  mBEP  600 , deci triunghiul BEP este
echilateral. Din ET ( T este mijlocul lui DC linie mijlocie în triunghiul ADC rezultă că T
este mijlocul lui EP, deci BT este mediană, bisectoare, înălţime şi mediatoare în triunghiul ,
ceea ce implică BT  EP . Dar cum EP AD , rezultă că AD  BC .
Folosind faptul că AD este bisectoare, obţinem că triunghiul ABC este isoscel cu AB  AC .
BT
Deoarece DT  TC şi BD  DC , rezultă că DT  , deci D este centrul de greutate al
3
triunghiului BEP şi avem că DE  BD.
AC BC
Din triunghiul dreptunghic ABD şi DE mediană, rezultă că DE  . Dar BD  , deci
2 2
BC  AC , adică triunghiul ABC este echilateral.

Subiectul II
bc bc
Demonstrăm întâi inegalitatea  2 . Dacă b  c , inegalitatea este echivalentă cu
a
b  a , iar dacă b  c , inegalitatea este echivalentă cu c  a , ceea ce în ambele cazuri este
adevărat pentru că orice catetă este strict mai mică decât ipotenuza.
bc bc
Demonstrăm în continuare inegalitatea din stânga: 2 . Deoarece ambii membri
a

sunt stric pozitivi, inegalitatea este echivalentă cu 2 


 b  c  b  c
2

sau ţinând cont că


a2
b  c  b  c2 , 2a 2  b  c2  b  c2  2 b  c b  c, iar aceasta, dacă ţinem cont de
2

teorema lui Pitagora, a 2  b 2  c 2 , este echivalentă cu 2 b  c b  c  0 , ceea ce este evident


adevărat. Egalitate dacă şi numai dacă b  c , adică în triunghiurile dreptunghice isoscele.

Subiectul III

192
1 n n 2 n 2  3n  2 n  1n  2
Avem s      . Numărătorul fiind un produs de două
3 2 6 6 6
numere naturale consecutive, este divizibil cu 2. Rezultă că s este număr natural dacă şi numai
dacă numărătorul de divide cu 3. Acest lucru se întâmplă dacă unul din factorii n  1 sau n  2
1 n n2
se divide cu 3. Aşadar s    este natural dacă şi numai dacă n nu se divide cu 3.
3 2 6

Clasa a VIII-a

Subiectul I


Relaţia din enunţ se mai scrie a8  b8  c8  2 a 4b 4  b 4 c 4  a 4 c 4 sau 
  
a8  b8  2a 4b 4  c8  2 b 4 c 4  a 4 c 4  4a 4b 4 , adică a 4  b 
4 2
 
 c8  2c 4 b4  a 4  4a 4b4 şi

prin urmare a 4  b4  c 4 
2
 2
 4a 4b4 . Imediat a 4  b4  c 4  4a 4b4  0 , adică
a 4  b4  c 4  2a 2b2 a 4  b4  c 4  2a 2b2   0 sau (a 2  b2 ) 2  c 4 (a 2  b2 )2  c 4   0 , adică
a 2  b2  c 2 a 2  b2  c 2 a 2  b2  c 2 a 2  b2  c 4   0 . Distingem cazurile:
1. a 2  b 2  c 2  0 şi conform teoremei reciproce a lui Pitagora, avem m(Cˆ )  900 .
2. a 2  b 2  c 2  0 şi conform teoremei reciproce a lui Pitagora, avem m(Bˆ )  900 .
3. a 2  b 2  c 2  0 şi conform teoremei reciproce a lui Pitagora, avem m( A ˆ )  900 .
Prin urmare triunghiul este dreptunghic.

Subiectul II
Avem succesiv:
1 a2 1 a2
  1  a (1)
b  c 1 a
1  b2 1  b2
  1  b (2)
a  c 1 b
1 c2 1 c2
  1  c (3)
a  b 1 c
Adunând relaţiile (1), (2), (3) se obţine concluzia

Subiectul III

Construim VN  BC , deci N este mijlocul muchiei BC .

193
Notând G  BM VN , rezultă că G este centrul de greutate al feţei VBC şi atunci
VG  2GN (1)
Din triunghiul dreptunghic BNG şi mB  300 , deducem de asemenea că
BG  2GN (2)
Din (1) şi (2) rezultă că VG  BG , deci triunghiul VBG este isoscel.
Notând   mVBG  mBVG , din triunghiul VNB dreptunghic, rezultă
 
    300  900 , de unde   300 .
Rezultă că triunghiul VBC este echilateral, deci VABC este un tetraedru echilateral. Distanţa de
la A la dreapta BM este AG şi reprezintă tocmai înălţimea tetraedrului regulat de muchie a
a 6
care este egală cu
3

Clasa a IX-a

Subiectul I

Să notăm BP  AC  S.

AM BN CS   CS CS 
Din teorema lui Ceva, rezultă    1 , adică    1 , deci  . Din
MB NC SA   SA SA 

AB  AC
   AB   AC
NB    NC , rezultă AN  , adică AN  .
   
1

194
AP AM AS       
Din relaţia lui Van Aubel avem      . Rezultă
PN MB SC   
AP                 AB   AC
 , iar de aici AP   AN   , adică
AN                 
AP 

    
 
 AB   AC . Atunci:

1  AB   AC
AP  (1)
     
În mod analog se obţin relaţiile:
1  BC   BA
BP  (2)
     
1  CA   CB
CP  (3)
     
1 1 1 
Adunând cele relaţiile (1), (2), (3), obţinem AP  BP  CP  0
  

Subiectul II
Avem 2SABC  a  AA  ax  HA , deci ax  2SBCH  8 . Analog obţinem
by  2SACH  8 şi cz  2SABH  8 .

Deoarece SBCH  SACH  SABH  SABC  4 , atunci adunând cele trei relaţii obţinute
mai sus, obţinem ax  by  cz  8  24, adică ax  by  cz  16 .
Pe de altă din inegalitatea Cauchy – Buniakovski – Schwartz, avem
a  b  cax  by  cz   a ax  b by  c cz  , adică
2

a  b  cax  by  cz  a x  b y  c z  şi folosind relaţia din enunţ, obţinem


2

ax  by  cz  16 . Deoarece ax  by  cz  16 , rezultă că în inegalitatea CBS, avem egalitate şi


ax by cz
atunci   , deci x  y  z şi atunci centrul cercului înscris în triunghiul ABC
a b c
coincide cu ortocentrul său, aşadar triunghiul ABC este echilateral

Subiectul III
Relaţia din enunţ se mai scrie  1  2 f x 1  1  3 f x 1  1  2010 f x 1  0 ,
kf x
 1  kf x  1  0 sau 
2010 2010 2010
 0 , deci f x  
k
adică  0 . Pentru
k 2 k 2 1  kf x  1 k 2 1  kf x  1

195
2010 2010
că f x     0 şi deci din f x  
1 k k
, x  , atunci 0
2010 k 2 1  kf x  1 k 2 1  kf x  1

rezultă f x  0 , x  .

Clasa a X-a

Subiectul I
i)  ii) . Vom arăta că pentru orice numere reale strict pozitive b1 , b2 , b3 şi orice x  1 are loc
inegalitatea:
b 
1

1
x
 b2x  b3x x  b1  b2  b3 (1)
Funcţia exponenţială cu baza subunitară este descrescătoare adică dacă 0  a  1 , atunci a y  a x
, x  y .
b1
Cum 0   1, iar x  1, din monotonia mai sus amintită, rezultă
b1  b2  b3
x 1
 b1   b1 
     sau
 b1  b2  b3   b1  b2  b3 
b1x b1

b1  b2  b3  b1  b2  b3
x

Analog obţinem
b2x b2

b1  b2  b3  b1  b2  b3
x

b3x b3

b1  b2  b3  b1  b2  b3
x

Adunând inegalităţile de mai înainte, obţinem


b1x b2x b3x
   1 sau b1x  b2x  b3x  b1  b2  b3  , de unde
x

b1  b2  b3  b1  b2  b3  b1  b2  b3 


x x x

b 
1

1
x
 b2x  b3x x  b1  b2  b3 .
q
Aplicăm acum (1) pentru b1  a1p , b2  a2p , b3  a3p şi x   1 şi vom obţine
p
a   a  şi imediat a   a 
p 1 1
q
1 a a
q
2
q q
3 1
p
a a
p
2
p
3
q
1 a a
q
2 3
q q
1
p
a a
p
2
p p
3 .
1 1
1 1
ii)  i) Pentru a1  a2  a3  1 , relaţia din ii) devine 3  3 , de unde q p
 , adică p  q .
q p

Subiectul II
Să notăm cu a lungimea comună a laturilor AB şi AC .

196
Din teorema cosinusului rezultă BC2  a 2  a 2  2a 2 cos1200  3a 2 , deci BC  a 3 . Avem

AO1  AO2 
a 2
2

a
 
, iar mO1 AO2   3600  450  1200  450  1500 . Teorema
2
cosinusului în triunghiul O1 AO2 dă O1O22 
a2 a2
  2
a a
 cos1500 
a2 2  3
şi
 
2 2 2 2 2
atunci

O1O2 
a
42 3 

a 1 3
(1)

2 2
2 BC 3
Avem O3C    a . Teorema cosinusului în triunghiul O2CO3 , dă
3 2
a2
O2O   a 2  2 
2
3
a
 a cos1050  a 2 
2  3 a2 1 3


şi deci
 2

2 2 2 4

O2O3 
a 1 3(2)

2
Deoarece triunghiurile O2CO3 şi O1 BO3 sunt congruente (L.U.L.), rezultă O1O3  O2O3 , deci
a 1 3
O1O3 
 (3)

2
Din (1), (2), (3) rezultă că O1O2  O2O3  O1O3 , deci triunghiul O1O2O3 este echilateral.

Subiectul III
 
Fie log 2 x  log 3 x  k , k  . Aşadar k  log2 x  k 1 , adică x  2k ,2k 1 . Analog

x  3 ,3 k k 1
 . În concluzie
  
x  2k ,2k 1  3k ,3k 1 (1) 
Tratăm două cazuri după cum k  0 , respectiv k  0 .
I) k  0 , deci k  . Avem în acest caz 2k  3k şi atunci intersecţia (1) este nevidă dacă şi
numai dacă 3k  2k 1 . Această inegalitate se verifică doar pentru k 0,1, deoarece pentru
k  2 se poate arăta prin inducţie că 3k  2k 1 .
Pentru k  0 , soluţia ecuaţiei este S1  1,2  1,3  1,2.
Pentru k  1 , soluţia ecuaţiei este S2  2,4  3,9  3,4.

197
II) k  0 , deci k   p , cu p   . Avem acum 3k  2k , deci intersecţia (1) este nevidă dacă şi
numai dacă 2k  3k 1 , inegalitate care se mai scrie 2 p  3 p1 sau notând q  p 1 , 2 p  3 p
avem 2q1  3q , inegalitate (la fel ca la cazul de mai înainte) are doar soluţiile q 0,1, adică
p 1,2 şi prin urmare k  2,1.
1 1  1 1   1 1 
Pentru k  2 , soluţia ecuaţiei este S3   ,    ,    ,  .
 4 2  9 3   4 3 
1  1   1 
Pentru k  1, soluţia ecuaţiei este S 4   ,1   ,1   ,1 .
 2  3   2 
În concluzie mulţimea soluţiilor ecuaţiei date este
1 1  1 
S  S1  S 2  S3  S 4   ,    ,2   3,4
4 3 2 

Clasa a XI-a

Subiectul I
      
Deoarece A2  I n A2  I n  A4  I n  I n , atunci det A2  I n det A2  I n  det I n    1
n

. Pentru că A  M n   , avem det  A  I  ,det  A  I  


2
n
2
n . De asemenea
  
det A2  I n  det A  iIn det A  iIn   det A  iIn   0 , rezultă că det A2  In 
2
 şi cum
detA 2
 In  este divizor al lui 1
n
 
se obţine că det A2  I n  1

Subiectul II
1 3 1 1
Notăm Sn  1   1  n 1  n, n  2
ln 2 ln 3 ln n

 
Din Stolz-Cesaro, cazul   , rezultă

n 1
1
1
Sn S n  S n1 ln n
lim  lim  lim . Doarece
n ln ln n n ln ln n  ln ln n  1 n ln ln n  ln ln n  1

 ln n  ln n  1 
ln ln n  ln ln n  1  ln
ln n
 ln1   , adică
ln n  1  ln n  1 
 n    1
 ln   ln1   
ln ln n  ln ln n  1  ln1  n  1   ln1  
n
, avem
 ln n  1  ln n  1 
   
   

198
 1 
ln1 
n 1
1
1  ln n 
Sn ln n e n 1
lim  lim  lim . Din acest moment, având în vedere
n ln ln n 
n
 1   n   1
 ln1     ln1   

ln1   n
ln1  
n
ln n  1  ln n  1 
   
   
e 1
x
ln1  x
limitele binecunoscute lim  1 , lim  1 , continuăm astfel
x0 x x 0 x
 1 
 1  ln 1   1 
ln 1   ln n 
ln1  
 ln n 
e n
 1  ln n  1 1
e n
1     an  bn  , iar
 1  1 n ln n n ln n
ln1  
 ln n  ln n
n
  1
 ln1   
ln1  
n  1
ln1  
  1 lnn  1 
  
n
 ln1    
ln1  
n
    lnn  1  n  c  d 
n
. De aici
lnn  1   1 1 lnn  1 n n
lnn  1
  ln1  
n lnn  1
 
  n
lnn  1
 1 
ln 1 
 ln n  1
an  bn 
e n
1
 n ln n  an  bn  ln n  1 . Deoarece
  1 n cn  d n ln n
 ln1    cn  d n 
ln n  1
ln1  
n
ln n  1 
 
 
 1  1
ln n n   ln n  ln n  
lnn  1  n  n
lim an  lim bn  lim cn  lim d n  1 şi lim  lim  lim  1,
n n n n n ln n n  ln n n  ln n
1 3 1 1
1  1  n 1 n
Sn ln 2 ln 3 ln n
obţinem lim  1 şi deci lim 1
n ln ln n n ln ln n

Subiectul III
a) Observăm că n  Im f , căci dacă ar exista n şi x0  cu f x0   n , ipoteza pentru
x  x0 s-ar scrie 1  0 , absurd.
n

Cum Im f este interval, înseamnă că există k  astfel încât Im f  k , k 1  .


Atunci sin f x  0 , x  şi ipoteza se scrie tgf x  x sau ctg f x  k   x , x  .

199
Deoarece f x  k  0,  , aplicând arccotangentă în ultima egalitate, obţinem
f x  k  arcctgx, adică f k x  arcctgx k , pentru un anumit k  . Se constată uşor, că
toate aceste funcţii verifică enunţul.
b) Dacă prin absurd, există o funcţie g ca în enunţ, atunci funcţia f  g  g verifică punctul a),
prin urmare există k  cu
ggx  arcctgx k (1)
Funcţia din membrul drept este o bijecţie de la la k , k  1  şi prin urmare funcţia g este
injectivă. Fiind injectivă şi având proprietatea lui Darboux, funcţia g va fi strict monotonă.
Rezultă că funcţia g  g este strict crescătoare, ceea ce contrazice (1) căci funcţia arcctgx k
este strict descrescătoare.

Clasa a XII-a

Subiectul I
Avem ba  a 1  b1  a 1b1  a 1  b1 . Prin înmulţire la stânga cu a şi la dreapta cu b ,
1

avem a 1b 1  a 1  b 1  1  b  a şi deci ba  a 1  b1  b  1  a .


1

Observaţie. Dacă în locul inelului A luăm corpul numerelor complexe, implicaţia ii)  i)
1 1 1
revine la „clasica” egalitate:  
aa  1 a a  1

Subiectul II
1
 x x2 x3 xn 
Notăm S n        ln 1  xdx , n  1. Pentru n  2 , avem
n 
0 1  x 1  x 2
1  x 3
1  x 
1
xn
S n  S n1   ln 1  xdx .
0 1 x
n

1
  S S S S n  S n1 n
Din Stolz-Cesaro, cazul   , rezultă lim n  lim n n1
 lim 
 n  ln n n  ln n  ln n  1 n 1  1
 ln1  
n  n
1
S  S n1 1
xn
ln 1  xdx .
Sn
şi cum lim n  1, avem lim  lim n  lim n
 1 0 1 x
n
n n ln n n 1 n
 ln1  
 n n
1 n
ln 1  xdx  ln 2 2 .
x
Dar lim n
0 1 x
n n

n 
  
1 1
xn
Într-adevăr, integrând prin părţi avem I n  n 0 1  x n ln 1  x dx  0 ln 1  x x ln1  xdx ,
    x 
 
1
I n  ln 1  x x ln 1  x 0    ln 1  x 
1
n
 ln 1  x n dx , adică
0 1 x 

200
 x 
 
1
I n  ln 2 2    ln 1  x   ln 1  x n dx , iar
0 1 x 
 x 
     
1 1 1

0  ln 1  x    ln 1  x n
dx   ln 1  x  
x
ln 1  x n
dx  M  ln 1  x n dx . Deoarece
1 x  0 1 x 0

   x 
 
1 1 1
 0 , rezultă că lim   ln 1  x 
1
 ln 1  x dx   x dx   ln 1  x n dx  0 şi atunci
n n

0 0 n 1 n
0 1  x 
1
xn
lim n ln 1  xdx  ln 2 2 .
0 1 x
n n

Subiectul III
Vom folosi în rezolvare următoarea proprietate:
b b
(p) Dacă funcţia f : a, b  este continuă, atunci  f xdx   f a  b  xdx .
a a
Relaţia se demonstrează uşor folosind schimbarea de variabilă t  a  b  x .

4

 1  2 sin  x1  f  x . Deoarece f x f  x  1,


dx
Folosind proprietatea (p), avem I  2

4
  
4
f xdx 4
dx 4
dx
avem I   1  2 sin 2 x1  f x  J şi atunci 2I  I  J    
1  2 sin x 0 1  2 sin 2 x ,
2
2
 
4 4

4
dx t2
deci I  
0 1  2 sin 2 x
. Cu
schimbarea de variabilă tgx  t , x  arctgt , cum sin 2
x 
1 t 2
,

dt 1
11 dt 1 1 
avem I   2  
1
, deci I   3arctgt 3  arctg 3 
0 3t  1
2
30  1  3 0 3 3 3
t 2   
 3

Ediţia a XVII-a, 25-27 martie 2011


Clasa a VII-a
Subiectul I
Din CE  AB şi AB CD rezultă că CE  CD şi imediat DE  2 .

201
Fie H  AB astfel încât DH  AB . Cum mDABH  600 , rezultă că AH 
1 3
şi DH  .
2 2
. Deoarece A HF , avem
5
Din triunghiul dreptunghic DHF , găsim FH 
2
5 1
AF  HF  HA 
2

Subiectul II
Relaţia din enunţ se mai scrie x 2  y 2  1 , sau x  yx  y  1 şi atunci x  y  x  y  1 şi
deci x  1, y  0

Subiectul III
Fie a  BC , b  AC şi c  AB şi D piciorul bisectoarei unghiului BAC .

Cum AM este bisectoare în triunghiul AFC , aplicând teorema bisectoarei, avem


FM AF c EN AE b
  . Analog  
MC AC 2b NB AB 2c

FM EN c b b 2  c 2 FM EN BC 2
1. Avem     . Cum   , rezultă BC 2  b 2  c 2
MC NB 2b 2c 2bc MC NB 2 AB  AC
şi conform teoremei reciproce a lui Pitagora triunghiul ABC este dreptunghic

FM c FM FM c EN b
2. Din  rezultă   . Analog  . Atunci
MC 2b FC FM  MC c  2b EB b  2c
FM EN c b 2b 2  c 2  bc
   
FC EB c  2b b  2c 2b 2  c 2   5bc
. Rămâne să demonstrăm că

2b 2  c 2  bc 2
  3b 2  c 2  bc  2b 2  c 2   5bc  b  c2  0 .
2b  c   5bc 3
2 2

Clasa a VIII-a
Subiectul I
Vom folosi identitatea a 3  b3  c 3  3abc 
1
2
 
a  b  ca  b2  b  c2  c  a2 .
Considerăm o ordine între numerele naturale nenule şi distincte a, b, c : a  b  c . Cum numerele
sunt naturale şi distincte, pentru a  1, b  2 , c  3 , rezultă

202
1
2
   
a  b  ca  b2  b  c2  c  a2  1 1  2  3 12  12  22 sau a3  b3  c3  3abc  18 .
2
Cum 1  2  3  3 1 2  3  18 , rezultă că cel mai mic element al mulţimii A este 18.
3 3 3

Subiectul II
Desfăşurăm faţa ADDA pe planul în care se află faţa ABCD.

CP 3
Drumul cel mai scurt este CA . Din  şi AC  20 cm, găsim CP  15 cm şi AP  5 cm.
AC 4
Din triunghiul dreptunghic AAP cu teorema Pitagora, găsim AA  4 cm, iar din asemănarea
triunghiurilor AAP şi ABC obţinem AB  12cm şi BC  12 cm. Rezultă atunci că volumul
paralelipipedului este egal cu 576cm.

Subiectul III
Notăm cu M mijlocul segmentului BC, Q centrul cercului circumscris triunghiului SBC , T
proiecţia punctului A pe planul SPC şi R proiecţia punctului A pe planul SPC. Găsim
a 2 a 6 a 2 3a 2
OT  , AR  a 2 , RT  TC  , QM  TQ  , SC  a 3 , SQ  .
2 2 4 4
. Lungimea lui AP este
a 34
Cu teorema cosinusului în triunghiul RTQ , obţinem QR 
4
minimă dacă lungimea segmentului PR este minimă care se realizează dacă punctul P se află
la intersecţia dintre QR şi cercul circumscris triunghiului SBC.

Avem PR  RQ  PQ 
a 2
4
 
17  3 . Aplicând teorema Pitagora în triunghiul dreptunghic
a
ARP, obţinem AP  21 3 17
2

Clasa a IX-a
Subiectul I
bn2  1
Fie bn  8an  1 , n  1. Avem bn  0 şi an  , de unde b1  3 şi
8
bn21  1 bn2  1 1  bn
  , adică bn21  bn  2 şi deci bn1  bn  2 , n  1. Prin urmare şirul
2

8 8 2
bn n1 este o progresie aritmetică cu raţia 2 şi rezultă bn  2n  1, apoi an  nn  1 şi
2
problema este rezolvată.

203
Subiectul II

Folosind teorema lui Ceva din H A , M B , I C  concurente, avem


BD CE AF
   1 . Dar cum
DC EA FB
BD c cos B CE AF b
 ( AD înălţime),  1 ( BE mediană),  ( CF bisectoare), atunci relaţia
DC b cosC EA FB a
c cos B
din teorema lui Ceva devine  1 , adică a cosC  c cos B . Dacă admitem şi
a cosC
H B , M C , I A  concurente, obţinem b cos A  a cosC , deci a cosC  b cos A  c cos B . Folosind
a 2  b2  c 2 b2  c 2  a 2 a 2  c 2  b2
teorema cosinusului, ultima relaţie devine   . Din
b c a
a 2  b2  c 2 b2  c 2  a 2 a 2  c 2 a 2  c 2 b2
 , avem    b , adică
b c b c c
a  ca  cb  c  bb  c sau
bc c
a  ca  cb  c  b2 b  c (1)
b2  c 2  a 2 a 2  c 2  b2
Din  , în mod analog se obţine
c a
b  ab  aa  c  c 2 c  a (2)
a 2  c 2  b2 a 2  b2  c 2
Din  , în mod analog se obţine
a b
c  bc  ba  b  a 2 a  b (3)
Înmulţim relaţiile (1), (2), (3) şi obţinem
a  cb  ac  ba  ba  cb  c2  abc2 b  cc  aa  b .
Dacă admitem că a  b  c  a , atunci relaţia de mai înainte devine
a  ba  cb  c2  abc2 , ceea ce nu este posibil. Rezultă că cel puţin două laturi sunt
egale, spre exemplu fie b  c . Atunci din (1) sau (3) rezultă că a  b  c şi deci triunghiul ABC
este echilateral.

Subiectul III
  , cu ,   0,1 .
AM AN
Fie  ,
AB AC

204
Atunci avem AM   AB şi AN   AC .
AM AM  MB 1   NC 1  
Din   se obţine  , adică  şi în mod analog obţinem 
AB MB 1   MA  NA 
1 1 
. Înlocuind aceste ultime două rapoarte în relaţia din ipoteză, avem x  y  z , care se
 
x y
mai scrie   x  y  z sau
 
x 1 y 1
    1 (1)
x yz  x yz 
Fie acum P  MN arbitrar. Atunci există   R , astfel încât AP   AM  1   AN , adică
AP      AB   1     AC (2)
Pentru a arăta că dreapta MN trece printr-un punct fix trebuie să arătăm că există   R astfel
încât expresia vectorului AP din (2) să nu depindă de  şi  . Privind cu atenţie (1) se observă
x 1 y 1
că luând    , atunci 1     şi deci
x yz  x yz 
AP      AB   1     AC 
x y
AB  AC , ceea ce arată că P  MN este
x yz x yz
punct fix.

Observaţii.
MB NC 1 1
1. Luând x  y  z , avem   1 şi AP  AB  AC , de unde P  G este centrul de
MA NA 3 3
greutate al triunghiului ABC .
MB NC
2. Luând x  b  AC , y  c  AB , z  a  BC , avem b  c  a şi
MA NA
b c
AP  AB  AC , de unde P  I este centrul cercului înscris în triunghiul
a bc a bc
ABC .
MB NC
3. Luând x  tgB , y  tgC , z  tgA, avem tgB   tgC   tgA şi
MA NA
tgB tgC
AP  AB  AC , de unde P  H este ortocentrul triunghiului
tgA  tgB  tgC tgA  tgB  tgC
ABC .

205
MB NC
4. Luând x  sin 2B , y  sin 2C , avem sin 2B   sin 2C   sin 2 A şi
MA NA
sin 2B sin 2C
AP  AB  AC , de unde P  O este centrul
sin 2 A  sin 2B  sin 2C sin 2 A  sin B  sin C
cercului circumscris triunghiului ABC

Clasa a X-a
Subiectul I
 
Avem z1  z2   z15  z25  5z1 z2 z13  z23  10z12 z22 z1  z2  , de unde
5

z1  z2  z15  z25  5 z1 z2 z13  z23  10 z12 z22 z1  z2 . Folosind inegalităţile din enunţul
5

problemei, avem z1  z2  2  5 1 2  10 1 z1  z 2  12  10 z1  z2 . Notând r  z1  z 2 , am


5

 
obţinut r 5  10r  12  0 sau r  2 r 4  2r 3  4r 2  8r  6  0 . Deoarece r  0 , rezultă
imediat că r  2 .

Subiectul II
Comparând x cu y , avem unul din cazurile x  y sau x  y .
Fie x  y , avem log 7 x  4  log 7  y  4 şi folosind primele două ecuaţii ale sistemului,
obţinem log 2  y  5  log 2  z  5, de unde  y  z , adică y  z şi atunci
log 7  y  4  log 7 z  4. Folosind ultimele două ecuaţii ale sistemului, relaţia de mai înainte
devine log2  z  5  log2  x  5 , adică z  x , de unde log 7 z  4  log 7 x  4 şi folosind
ecuaţia a treia şi prima a sistemului se obţine log 7  x  4  log 7  y  4 , adică x  y . Aşadar
x  y şi analog x  z .

Pentru x  y , analog se obţine x  y  z .


Deci sistemul se reduce la ecuaţia log 7 x  4  log 2  x  5 care are soluţia x  3 care este
unică pentru că funcţia din dreapta este strict crescătoare, iar cea din stânga este strict
descrescătoare. În concluzie sistemul are soluţia x  y  z  3

Subiectul III
1 1 y 2  z 2 1 2 yz
Folosind inegalitatea mediilor, avem y  z   2 yz   2 yz , deci
2 2
 
2 2 x 2x x
z x
2 2
1 2 xz x  y 2 2
1 2 xy
. Analog   ,   şi atunci
y 2y y z 2z z
y 2  z 2 z 2  x 2 x 2  y 2 1  1 1 1   yz xz xy 
        2   . Folosind din nou
x y z 2  x y z   x y z 

206
yz xz xy yz xz xy
inegalitatea mediilor, avem    3 3    3 şi atunci
x y z x y z
y2  z 2 z 2  x2 x2  y2 1  1 1 1 
        6 .
x y z 2 x y z

Clasa a XI-a

Subiectul I
an1 a
Relaţia de recurenţă se scrie  n n  1, n  1, ceea ce înseamnă că şirul
n  1  2 n  2
n1

 an  a
 n 
este o progresie aritmetică cu raţia egală cu 1. Cum 1 1  1, imediat se obţine
 n  2  n1 1 2
nn1
an
 n , adică a  n 2
 2 n
, n  1 . Atunci a  a    a  n!2
 2 2
şi deci
n  2n n 1 22 n

lnn!2 nn1 nn1 1


n2 a1a2 an  e n2
2 2n2 . Deoarece lim 2 2n2
 2 2  2 , atunci
n
lnn!2 lnn!
lim 2 lim
lim a1a2 an  e
n2 n  n 2
 2  2 e n  n2
. Folosind teorema Stoltz – Cesaro, avem
n
ln n! ln n!  ln n  1! ln n
lim  lim  lim  0 şi atunci lim n2 a1a2 an  2  e 0  2
n  n 2 n  n  n  1
2 2 n  2n  1 n

Subiectul II
Prin inducţie se arată că f n  x   2 e
n x 
sin x 

n 
4 
 
n 
 şi g n  x  2 e x cos x 

n 
4 
 , pentru


orice n . Atunci f n  x  g n  x  
2
  
2 2n  
 2 e 2 x  sin 2  x 
n  
  cos2  x 
n  
   2 n e 2 x .
  4   4 
Subiectul III
Deoarece A  B  A2  B2  AB  BA, atunci din A  B  A2  B2 se obţine
2 2

AB  BA  On (1)
De asemenea A  B  A  B
4
   A
2 2 2
 B2 
2
 A4  B4  A2 B2  B2 A2 şi cum
A  B4  A4  B4 , atunci se obţine
A2 B2  B2 A2  On (2)
Înmulţind (1) la stânga cu A şi la dreapta cu B , rezultă
A2 B 2   AB2  On (3)
Înmulţind (1) la stânga cu B şi la dreapta cu A , rezultă
BA2  B2 A2  On (4)
207
Adunând (3) cu (4) obţinem  AB  BA  A2 B 2  B 2 A2  On şi ţinând cont de (2), avem
2 2

AB2  BA2  On (5)


BA  AB , deci (5) devine 2 AB  On , adică  AB  On
2 2
Din (1), avem

Clasa a XII-a

Subiectul I
Folosim inegalitatea Cauchy – Buniakovski – Schwarz şi vom avea
 b 
2
 b 2  1   b 2x  b
2

 ln 2     2 dx    2x dx 


   2 1  . Deoarece
dx
 a  1  a x  1  a 
 a x  1
2 2 
  
2
 b2  1  4 3 3
   
b b b
4 3 4 3 1
a 4x dx  3 x a  3 b  a , atunci  ln 2   b  a dx , adică
2 3

 a  1 3 a x 2
1 2

2
 b 1
 
b 2
1
4 b3  a 3   dx  3 ln 2  .
a 
x 12

 a  1
2

 
Inegalitatea este strictă, deoarece funcţia 2x x 2  1 nu este constantă pe intervalul a, b

Subiectul II
Deoarece 1 K şi K este corp, avem  K .a 
 a  b   a  b 
2
În cazul trivial K  , putem lua d  1 .
Să considerăm cazul netrivial când K  .
Fixăm un a  K  , prin urmare există b  K  astfel încât a  b  şi ab . Avem

a  b  K  (căci a  b  ar duce la a 
 a  b   a  b  ). Deoarece
2
 a  b   a  b  4ab  , putem scrie a  b  q 2 d cu q   şi d  , d  2 , liber de
2 2 2

pătrate.
 a  b   a  b 
Rezultă a  b  q d , apoi d
a b
q
K  , a
2
 d  , deci
a  d   K (1)
Arătăm acum incluziunea reciprocă. Fie x  K arbitrar. Dacă x , avem evident x   d .
Dacă x  K  , cu un raţionament similar celui anterior, găsim un d   , d   2 , d  liber de
pătrate şi depinzând de x , astfel încât:
x  d   K
Arătăm că d  d  . Pentru aceasta considerăm elementul   d  d   K . Nu putem avea
  (    dd   d  d  şi apoi    2 d   d  , fals), deci
 K  .

208
Din ipoteză, există   K  astfel încât     e  şi   f  . Atunci


f


f
d  d
 
 g d  d  cu g  şi din     e obţinem

 
d  d   g d  d   e , de unde d   u  v d cu u, v , v  0 . Dacă u  0 , ridicând
d   u2  dv2
la pătrat rezultă că d   , fals. Prin urmare u  0 şi deci d   v d şi dacă
2uv
înmulţim cu d , obţinem dd   , deci d  d  .
Din x   d   , obţinem x   d  şi cum x a fost ales arbitrar, rezultă incluziunea
K   d  (2)
Din (1) şi (2) rezultă egalitatea dorită K   d 

Subiectul III
Considerăm trei din relaţiile lui Viete:
S1  x1  x2  x3  x4  x5  4
S 4   x1 x2 x3 x4  2
S5  x1 x2 x3 x4 x5  c
Dacă admitem că x1  x2  x3  x4  x5  r , trecând la modul în relaţia cu S 5 , rezultă
r 5  c . De asemenea trecând la conjugat în prima relaţie a lui Viete, avem
S r2
4   x1   x1  r 2 
1 2
 r 2 4 , deci r 2  4 , adică c   sau trecând la modul
x1 S5 c 2
r2 r2 1
c  . Deoarece r 5  c , obţinem r 5  , adică r 3  .
2 2 2
4
Din prima relaţie a lui Viete, avem 4  x1  x2  x3  x4  x5  5r , deci r  , adică
5
1 3 64
r  prin urmare 125  128, ceea ce este absurd.
2 125

Ediţia a XVIII-a, 23-25 martie 2012

Clasa a VII-a
Subiectul I
a) Fie d  a, b , rezultă a  d  n , b  d  p cu n, p  N . Deoarece a  b , atunci n  p şi cum
n, p sunt numere naturale, avem p  n  1. Prin urmare b  a  d  p  n  d  a, b .

209
ba
b) Deoarece pentru a, b  N  , a  b  a, ba, b, avem b  a  a, b 
ab 1
şi deci 
a, b a, b ab
1 1 1
, adică   . Folosind această ultimă relaţie pentru numerele a1 , a2 ,, a2011, obţinem
a, b a b
1 1 1 1 1 1 1 1 1
          , adică
a1 , a2  a2 , a3  a2010, a2011 a1 a2 a2 a3 a2010 a2011
1 1 1 1 1
    
a1 , a2  a2 , a3  a2010, a2011 a1 a2011
.

Subiectul II
În triunghiul ABC , notăm AB  AC  b  N , BC  a  N , ha  AD , D  BC, AD  BC ,
hb  BE , E  AC, BE  AC înălţimile din A, respectiv B.
2
a
1. ha  2012 . Aplicând teorema lui Pitagora: AB  AD  BD , avem b  h    , adică
2 2 2 2 2
a
 2
4b  a  4  2102. Evident a este număr par, a  2x , x şi atunci obţinem ecuaţia
2 2

b  xb  x  4 503. Cum 503 este număr prim, b  x, b  x au aceeaşi paritate (amândouă
pare) şi b  x  b  x , avem b  x  2 şi b  x  2 503. Se obţine b  504 şi
a  2x  2  502  1004.
a h a b  hb  a2 
2. hb  2012 . Cum S   , avem a 2  ha2  b2  hb2 , adică a 2  b 2    2012b 2 .
2 2  4
 
Deci avem ecuaţia a 2 4b 2  a 2  4  2012b 2 . Imediat a , este număr par, a  2x , x  N şi deci
   
avem ecuaţia x 2 b 2  x 2  503b 2 sau x 4  b 2 x 2  503 . Notăm x2  y  şi ecuaţia devine
y  b  y  503 . Avem b y , adică b y , y  b  z , z  N şi atunci obţinem ecuaţia
2 2 2 2

z  1
z 2  bz  503 , adică zb  z  503. Cum 503 este număr prim, avem  sau
b  z  503
z  503 z  1 z  503
 , adică  sau  . Atunci y  b  z  504 sau y  b  z  503 504 şi deci
b  z  1 b  504 b  504
x 2  y  504, 503 504 de unde x . Deci în acest caz nu există triunghi.
În concluzie singurul triunghi care verifică ipoteza este cel cu laturile a  1004 , b  c  504 .

Subiectul III
Fie BF ED , unde F  DC .

210
Avem AM  ED şi cum BF ED , rezultă că AM  BF . Cum şi BM  AC ( BD  AC ),

rezultă că M este ortocentrul triunghiului ABF şi atunci FM  AB şi deci FM BC .

Deoarece FM BC şi M este mijlocul lui BD, rezultă că FM este linie mijlocie în triunghiul

DBC , deci F este mijlocul lui CD.


Cum BF ED şi F este mijlocul lui CD, rezultă că BF este linie mijlocie în triunghiul CDE

şi deci B este mijlocul lui EC . Atunci în triunghiul AEC , AB este şi înălţime şi mediană,
deci rezultă că triunghiul AEC este isoscel.

Clasa a VIII-a
Subiectul I.
 
Fie n  p , p  N  , atunci p  n  p  1, sau p 2  n   p  1 . Deoarece n 
2 
, rezultă
 
n  p 2 , p 2  1, p 2  2, p 2  2 p  1, p 2  2 p .

1. Dacă n  p , p  1, p  2, p
2 2 2 2
 2 p  1, atunci  n    
n  1  p şi atunci
n  2010 n  2011
,  N  , deci p n  2010 şi p n  2011, prin urmare p n  2011  n  2010 ,
p p
deci p  1 şi atunci n 1,2, valori care înlocuite verifică cerinţa.
   
2. Dacă n  p 2  2 p , atunci n  p , iar n  1  p  1 şi avem
n  2010 p 2  2 p  2010  p  12  2009 n  2011 p 2  2 p  2011 p p  2  2011
 n 1 
p 1

p 1
,
n  
p

p
.

Prin urmare p  1 2009 şi p 2011, ceea ce nu este posibil pentru că unul din numerele p şi p  1
este număr par, iar 2009 şi 2011 sunt impare.

Subiectul II

211
Avem condiţiile: x  3  0 , 7  x  0 , x  2  0 de unde x  2,7 . Calculăm diagonala
paralelipipedului: d 2  x  3  7  x  x  2
2 2 2
 
sau d 2  3x 2  24x  62  3 x 2  8x  62 ,
adică d  3x  4  14  14. Deci valoarea minimă este 14 pentru x  4 .
2 2

Subiectul III
 a  b a  b   ab a  b
Cum x  a,  , y , b , avem x  y  a  ,b  şi atunci
 2   2   2 2 
 ab a  b a b a b a b
a  2 , b  2   a, b , adică a  a  2 şi b  2  b , deci 0  2  0 . Prin
urmare a  b , cu b  0 .
 a  b
Avem I   b, b. Pentru x  a, 
a  b 
  b,0, y   
, b  0, b , avem x  y   b 2 ,0 şi
 2   2 
prin urmare  b  b . Cum b  0 , obţinem b  1 .
2

Aşadar I   b, b cu b  0,1 . Într-adevăr, dacă x   b,0 şi y 0, b, atunci x  y  b, b


 
şi x  y   b 2 ,0   b, b.

Clasa a IX-a
Subiectul I
Se observă că n  0 , verifică ecuaţia.
Presupunem în continuare că n  1.
n  2 n2
Deoarece a  a,  a  , n   n    1  n   1  2n 1  3 n
 3  3
Ridicând la pătrat, se obţine n  4n 13 1  0 . Deci n  3 . Se observă că 1 şi 2 verifică ecuaţia,
iar 3 nu o verifică.
În concluzie mulţimea soluţiilor este S= 0,1,2,

Subiectul II
AB  AC
a) Fie BC  , deci BC  a , AC  b  a  y şi AB  c  a  y , a  y  0 .
2

212
3a 3ar ah
Atunci semiperimetrul triunghiului este p  . Avem S  pr  , S  a şi imediat
2 2 2
ha  3r . Fie A, D, E  BC astfel încât AA este bisectoare, AD înălţime, iar D punctul de
contact al cercului înscris în triunghiul ABC . Din asemănarea triunghiurilor AID şi AAE ,
IA ID r 1
avem    şi atunci IG BC .
AA AE ha 3
AI 2
b) Folosind punctul de mai înainte ( IG BC ) avem  , adică
AA 3
bcp p  a 
2 2 2 bc
AI  ia   , deci bc  3AI 2 .
3 3 bc 3
abc bc 3AI 2 AI 2
Avem R     , deci AI 2  2Rr . Atunci din relaţia lui Euler
4S 2ha 2ha 2r
OI  R  2Rr , obţinem OI  AI 2  OA2 , deci mAIO  900 , adică AI  OI
2 2 2

Subiectul III
Fie AD , D BC bisectoarea unghiului BAC şi I   AD  PQ .

Din teorema transversalei în triunghiul ABC şi transversala P , I , D , avem


DI QC BP
 BC  BD   DC  , adică
IA QA PA
DI c b
 a  BD   DC  (1)
IA a a
Deoarece I este centrul cercului înscris în triunghiul ABC , atunci BI este bisectoare în
triunghiul BAD şi atunci conform teoremei bisectoarei avem
DI BD BD
  (2)
IA BA a
BD AB c
Aplicând teorema bisectoarei pentru AD , avem   şi atunci
DC AC b
b
DC  BD  (3)
c

213
BD c b b
Înlocuind relaţiile (2) şi (3) în relaţia (1), obţinem  a  BD   BD   , de unde
c a c a
a c b2
  , adică a 2  c 2  b 2 ceea ce înseamnă că triunghiul ABC este dreptunghic în A
c a ac
Clasa a X-a
Subiectul I
Fie punctele A, B, C de afixe z1 , z 2 , z3 şi a  BC  z 2  z3 , b  AC  z1  z3 ,
c  AB  z1  z2 .
1
zz z z 1 1
Deoarece z1  1 , adică z1  z1  1, avem 1 2  1 2   şi atunci
z1  z 2 1  1 z 2  z1 z1  z 2
z1 z 2
z1 z 2 1 1
   2 . În mod analog se obţin celelalte două relaţii şi atunci relaţia din
z1  z2 2
z1  z 2
2
c

enunţ devine  2 1 . Folosind inegalitatea a 2   2  9 (se demonstrează folosind de


1 1
a a
două ori inegalitatea mediilor; egalitate pentru a  b  c ), avem  a 2  9 . Din
z1  z2  z3  1 , rezultă că raza cercului circumscris triunghiului ABC este 1 şi atunci
folosind inegalitatea cunoscută a 2
 9R 2 , obţinem a 2
 9 şi prin urmare a 2
 9 , deci

a   a
1
2
2
 9 şi atunci a  b  c

Subiectul II
Evident z  i verifică cele două inegalităţi. Considerăm acum că z  1  i  1 şi z 2  1  i  1.

Avem cercul de centru punctul de afix 1  i şi rază 1: CP i  1,1  M z  z  1  i  1 , 

respectiv discul de centru punctul de afix 1  i şi rază 1 DP i  1,1  M z  z  1  i  1 
(vezi figura).

214
 
 
Se observă că dacă M z DP i  1,1, atunci argz    ,   . Cum M z 2  DP i  1,1 ,
2 
    
  
  
atunci şi arg z 2   ,   . Dar cum 2 argz   arg z 2   ,   , avem argz    ,  şi prin
2  2  4 2
 
urmare argz   . Singurul punct de pe disc cu argz   este z  i
2 2

Subiectul III
Fie patrulaterul ABCD cu laturile AB  a , BC  b , CD  c , DA d circumscris unui cerc şi
  mBAD şi   mBCD . Cum patrulaterul ABCD este circumscriptibil atunci
ac bd .

Trei laturi sunt suficiente pentru că a patra se găseşte din relaţia de mai înainte.
Avem S  S ABCD  S ABD  S BCD şi deci S 
1
ad sin   bc sin    1 ad  bcsin  .
2 2
Aplicând teorema cosinusului în triunghiurile ABD şi CBD , avem
BD2  a 2  d 2  2ad cos  b 2  c 2  2bc cos  . Deci avem relaţiile
ad sin   bc sin   2S

ad cos  bc cos   2 a  d  b  c 
 1 2 2 2 2

Ridicând la pătrat aceste egalităţi şi adunându-le, obţinem


a 2 d 2  2abcd cos     b 2 c 2  4S 2 
1 2
4

a  d 2  b2  c 2
2

Ţinând cont că a  d  b  c , avem a  d 2  b 2  c 2  2ad  bc , atunci relaţia de mai înainte
2

devine a 2 d 2  2abcd cos     b2c 2  4S 2  ad  bc sau 4S 2  2abcd  2abcd cos   
2

 
şi imediat S 2  abcd sin 2  abcd , deci S  abcd , adică S max  abcd şi se atinge
2
 
pentru sin  1 care înseamnă      , adică patrulaterul este inscriptibil.
2

Clasa a XI-a

215
Subiectul I
În rezolvare vom folosi două rezultate cunoscute: pentru C, D  M 2   , avem trCD  trDC
şi detC  xD  det C  xtrC  trD  trCD  x det D , sau făcând x  1:
2

detC  D  det C  det D  trC  trD  trCD (1)


Dacă A  O2 sau B  O2 , evident detA  B  det An  Bn .
n
 
Presupunem în continuare că A, B  O2 .
Din AB  O2 , rezultă imediat că A p B q  O2 , p, q  1 , 0  trAB  trBA şi de asemenea
det A  det B  0 (dacă det A  0 , atunci B  A1 AB  O2 , dar A, B  O2 ).
Deoarece A p B q  O2 , atunci A  B  An  BAn1  B2 An2   Bn2 A2  Bn1 A  Bn , adică
n

A  Bn  An  Bn  BCA, unde C  An2  An3 B  An4 B 2   ABn3  B n2 .


Folosind (1), avem
      
det An  B n  BCA  det An  B n  det BCA tr An  B n trBCA  tr An  B n BCA .  
Dar detBCA  0 , trBCA  trCAB  trCO2   0 ,
         
tr An  B n BCA  tr An BCA  B n1CA  tr O2  B n1CA  tr B n1CA  tr ABn1C  trO2  0
   
şi atunci det An  B n  BCA  det An  B n , deci
detA  B  detA  B  BCA  detA  B  .
n n n n n

Subiectul II
a  3b 1
Demonstrăm întâi că  . Inegalitatea de mai înainte este echivalentă cu
a  b3a  b 2a
a  3b
2a 2  3ab  3a 2  ab  3ab  b 2 , adică 0  a  b2 . Deci
1
 şi în mod analog
a  b3a  b 2a
a  3c 1
se demonstrează că  . Prin adunarea celor două inegalităţi se obţine
a  c3a  c 2a
inegalitatea din enunţ.
Observaţie. Cei mai mulţi din concurenţi au dat această soluţie care este mult mai simplă decât
soluţia autorului care foloseşte minimul unei funcţii derivabile.

Subiectul III
Se consideră funcţia g : 1, a  R , gx  Fx1  ln Fx  a , care evident este continuă.

Avem g1  1  a  0 , ga  a ln a  0 , deci g1ga  0 şi atunci există c  1, a astfel încât

gc  0 , adică Fc 1  ln Fc  a

Clasa a XII-a
Subiectul I
Fie x, y  G oarecare.
Cum funcţia f este endomorfism, avem f  y  x  f  y f x , adică

216
yxn  y n  x n (1)
n1
 e ,  yxn n1  e de unde
2
n1
Deoarece ordG  n 2  n  1 , avem x n  e , yn
2 2

x n  x n1 , y n  y n1 (2)


2 2

yxn n  y  x
2
(3)

        x  y . Ţinând cont de
De asemenea, deoarece f x n  y n  f x n  f y n , avem x n y n
n n2 n2

relaţiile (2), ultima relaţie se mai scrie x y x y  x y  x  y sau compunând la stânga


n n n n n2 n n n1 n1

cu x , iar la dreapta cu y , avem y x y  x  x  y , adică y x   x  y . Folosind


n n n n n n2 n n n n1

relaţia (1), ultima relaţie devine  yx


n2 n
 x  y . In final, datorită relaţiei (3), relaţia de mai
înainte devine y  x  x  y şi deci G,  este abelian.

Subiectul II
 x 2n1
1
x 2n1  1
x 2n1
1. Avem I n1  I n     dx  1 (1  x 2 ) n1 dx  0 , deci şirul este
2 n1
1  (1  x ) (1  x 2 ) n 
2 2
1
x 1 5 1 8
descrescător. Şirul fiind descrescător, avem 0  I n  I1   1 x
1
2
dx   ln 2  ln   ln .
2 4 2 5
2

1 
Prin urmare şirul este descrescător şi mărginit, deci este convergent. Pentru x   ,1 , avem
2 
x 2n1 x 2n1
n
 4
    x 2n1 , deci
2 n
(1  x )  5 
2 n

1 1   1   1 x 2n1 x 2n1  4  1   1  
2n n1 n 2n
 4
1

2n 2n   2   1 2n
 1    dx   dx     x dx    
2n1
1   şi
1 1  x  5 1  5  2n   2  
2 n

2 2 2

1   1    
2n n 2n
imediat 1   dx  nI n  1  4  1   1   , de unde lim nI n  0 .
2  2n   2  
 2  5    2  

n

2. Pentru k  1, avem
k 1  k 1 k 1
1
x 2k 1 1 1 2x  x 2  1 1  x2   x2  1  x2 
I k  I k 1  dx   
2 2  2
 dx    2   2  dx   2 
1 (1  x 2 k 1
) 2 1 (1  x )  x  1  2 1  x  1  x  1 2k  x  1  1
2 2 2 2
k k
1 1 1 1 1 n  1 1  n
, adică I k  I k 1       . Atunci  k  k   I k  I k 1   I1  I n1 ,
2k  2  2k  5  2 k 1  k 2 k 5  k 1
n
 1 1 
deci lim  k  k   2 lim I1  I n1   2I1  ln
8
k 1  k 2 k5 
n n 5

217
Subiectul III

Vom folosi în rezolvare următoarea proprietate:


b b
(p) Dacă funcţia f : a, b  este continuă, atunci  f xdx   f a  b  xdx .
a a
Relaţia se demonstrează uşor folosind schimbarea de variabilă t  a  b  x .

2
cos2 x  sin x2012
Fie I   dx . Folosind proprietatea (p), avem
0 1  sin x  cos x2012
2012

   
2012
  
cos   x    sin  x  
2
2
2   2  2
sin 2 x  cos x2012
I  dx  0 1  cos x2012  sin x2012 dx  J şi atunci
   
2012 2012
 
1   sin  x     cos  x  
0

 2   2 
 
cos x  sin x  cos x  cos x
2 2 2012 2 2012 2
 
2I  I  J   dx  1dx  , deci I 
0 1  sin x2012  cos x2012 0 2 4

Ediţia a XIX-a, 22-24 martie 2013


Clasa a VII – a

Subiectul I
Pentru două laturi de lungimi x, y cel mare segment cu care nu se poate forma un triunghi este
cel de lungime x  y . Astfel un şir de lungimi de laturi cu care nu se poate forma un triunghi va
fi: x , y , x  y , x  2 y , 2x  3y , 3x  5 y , . Pentru a avea valori minime în şir putem lua
x  y  1 şi se obţine şirul 1,1,2,3,5,8, (şirul lui Fibonacci). În acest şir, al 18-lea termen este
2589, care îl depăşeşte pe 2012. Deci conform principiului cutiei, oricum am alege 23 de numere
din intervalul 1,2012, există 3 care pot fi lungimile laturilor unui triunghi.
Notă. Soluţia permite diminuarea numărului de numere de la 23 la 18.

Subiectul II
a a
Dacă b 2  c 2  a 2 , atunci ma  a 2   , ceea ce este adevărat.
2 2

218
2
a  a
Reciproc, să presupunem că ma  b  c  . Rezultă că  ma    b 2  c 2 , adică
2 2
2  2
ma2  ama 

4b c a
2 2

2
. Deoarece ma2 
2b c a
2
2 2

, atunci ultima relaţie devine
4 4

ama 
   
4 b2  c 2  a 2 2 b2  c 2  a 2 b2  c 2
  sau prin ridicare la pătrat a ma 
2 2  
b2  c 2
2

4 4 2 4
şi ţinând cont de lungimea medianei, avem a 2
2b c a
2 2
 2

b c
2
2 2
. Deci

4 4
     
b2  c 2  2a 2 b2  c 2  a 4  0 , adică b2  c 2  a 2  0 şi prin urmare b 2  c 2  a 2 ceea ce
2 2

înseamnă că mA  900

Subiectul III

Arătăm că pentru orice număr natural n  1 are loc relaţia:


 1
2 n 1  n 
n

 2 n  n  1 (1)  

Avem 2 n  1  n   
2 n 1  n n 1  n
n 1  n
 
2
n 1  n
 
1
n
şi analog


2 n  n 1   2
n  n 1

1
n
.

1 1 1
Fie E  1    . Atunci folosind relaţia (1), avem
2 3 2005
        
E  2 2  1  3  2  4  3   2006  2005  2 2006 1 . Pentru că 
 
2006  44 , atunci E  2 2006 1  244 1  86 .
Folosind din nou relaţia (1), avem
E  1  2 2  1  3  2   4  3   2005  2004  1  2 2005 1. Deoarece
2005  45 , avem E  1  2 2005 1  1  245 1  89 . Aşadar 86  E  89 .

Clasa a VIII – a

Subiectul I
a 2  b2  a  b 
2
Este adevărată următoarea inegalitate   , inegalitate care este echivalentă (după
2  2 
efectuarea calculelor) cu a b  0 . Şi atunci pentru a  b , a, b  0 , avem
2

a 2  b2 ab ab k 2  k  12 k  k  1


2
    şi deci  , adică
2  2  2 2 2

219
2k 2  2k  12 1
 k  (1)
2 2
Unde k  1, n . Însumând inegalităţile (1) cu k  1, n , avem
5 13 25 2n 2  2n  1 1
S     1  2  3    n  n  , adică
2 2 2 2 2
nn  1 n nn  2
S  
2 2 2

Subiectul II
 x x  x  x x  x
a) Folosind definiţia părţii întregi, avem        1,        1,
2 2 2 3 3 3
 x x  x  x  x  x
 6   6   6   1. Adunând cele trei relaţii şi ţinând cont că  2    3    6   2013, obţinem
2013  x  2016 , adică x 2013,2016
x  2013 2016  671 672   x
Avem   ,  ,  şi deci    335
6  6 6   2 2  6
 x x x  x
Notăm    p  , p  335 şi          0,1 . Avem  p   , adică
x
6 6  6  6  6
x  6 p  6  2010  6 . Înlocuim în ecuaţia iniţială si obţinem
3p  3 2 p  2 p  2013. Deoarece x  n  x n , pentru orice n şi x , relaţia
de mai înainte devine 6 p  3   2   2013 şi cum p  335 , obţinem 2   3   3, unde
2   1
  0,1 . Cum pentru   0,1 , 2   1, 3   2 şi 2   3   3, atunci  , adică
3   2
   ,1   ,1   ,1 . Aşadar x  6 p  6  2010 6 2014,2016 .
1 2 2
2  3  3 
 x x
b) Asemănător ca la punctul precedent, notând    p  ,      0,1 avem x  6 p  6
6 6 
 x  x  x
şi atunci f x           6 p  3   2 
2 3 6

 1 1 1  1 2 
Pentru   0,  , f x  6 p , pentru    ,  , f x  6 p 1, pentru    ,  ,
 3 3 2  2 3 
2 
f x  6 p  2 , iar pentru    ,1 , f x  6 p  3 , deci
3 
f x  6 p,6 p  1,6 p  2,6 p  3 p  Z şi cum 2014  6  335  4  6n  4 , rezultă că
f x  2014, x 

Subiectul III
Notăm AB  a , BC  b , EC  c .

220
Din congruenţa triunghiurilor EBC şi CBA se obţine c  EC  CB  b , EB  CA ,
b  CB  BA  a , mECB  mCBA  900 . Deci EC  CB , a  b  c . Aşadar ABCD este
pătrat şi deci EB  CA  a 2 .
Din CB  EC şi CB  DC , rezultă CB  DCE şi deci E se află în planul care conţine pe
DC şi este perpendicular pe planul ABC .
Din AC  DB şi AC  EB , rezultă AC  EBD , deci E se află în planul care conţine pe DB
şi este perpendicular pe planul ABC . Cele două plane în care se află E fiind perpendiculare pe
ABC şi conţinând pe DC şi respectiv DB se intersectează după dreapta d  ABC care
trece prin D . Deci E  d şi prin urmare triunghiul EBD este dreptunghic în D şi isoscel (
DB  DE  a 2 ) şi prin urmare triunghiul EBD este degenerat în segmentul DB şi D  E ,
DE  0

Clasa a IX – a

Subiectul I
Presupunem că n9n  2012  k 2 , k  
. Înmulţind relaţia cu 9 şi adunând în ambii membri
1006 , obţinem 9n  2  9n 1006 10062  3k 2  10062 sau, echivalent
2 2

9n 10062  3k 2  10062 , adică 9n  1006  3k 9n  1006  3k   22  5032 . Ţinând cont că
termenii din membru stâng au aceeaşi paritate şi că 503 este număr prim, atunci avem
9n  1006  3k  2
 . Prin adunarea celor două ecuaţii, obţinem 9n  1006  1  5032 ,
9n  1006  3k  2  503
2

adică 9n  1  5032  2  503 sau 9n  503 1  5022 , ceea ce este imposibil pentru că 5022
2

nu se divide prin 9.

Subiectul II
a) Fie M , N , P picioarele înălţimilor din A, B, C ( M mijlocul lui BC ).

221
Dacă H ar fi mijlocul înălţimii BN , în triunghiul ABN bisectoarea AH ar fi mediană, deci
triunghiul ABN ar fi isoscel cu AB  AN şi astfel AC  AN , absurd. Analog, H nu poate fi
mijlocul înălţimii din C . Prin urmare H este mijlocul înălţimii din A .
Fie Q mijlocul lui BP , deci QM este linie mijlocie în triunghiul BPC , iar PH devine linie
AB
mijlocie în triunghiul AQM . Aşadar AP  PQ  QB  . Avem MQ  AB şi aplicând
3
BC2 AB
teorema catetei în triunghiul AMB , avem MB  BQ  AB , adică
2
  AB şi imediat
4 3
BC AB AC
  .
2 3 3
b) Din teorema bisectoarei în triunghiul ABM (evident I  AM ), avem
AI AB 2 AB
   3  1 , deci AI  IM , adică pe înălţimea AM , avem ordonarea
IM BM BC
AI AI 3 3 3
A  H  I  M . Atunci din  3 , avem   , deci
IM AI  IM 1  3 2

AI 
3 3
2
   
AM  3  3 AH . Rezultă că HI  AI  AH  2  3 AH . Cum vectorii HI şi

 
AH sunt coliniari şi au acelaşi sens, rezultă imediat HI  2  3 AH .

Subiectul III
Lemă Fie triunghiul dreptunghic ABC cu mA  900 . Dacă D BC , atunci sunt adevărate
inegalităţile: minBD, DC  AD  maxBD, DC
Demonstraţie. Presupunem prin absurd că AD BD şi AD  DC .

Atunci mDBA  mDAB şi mDCA  mDAC . Prin adunarea celor două inegalităţi,
avem 900  mBAC  mDAB  mDAC  900 , ceea ce este o contradicţie. Analog dacă

222
presupunem prin absurd că AD  BD şi AD  DC . Deci pentru mA  900 , avem
minBD, DC  AD  maxBD, DC.
Rezolvăm acum problema ţinând cont de lema de mai înainte şi eliminând cazul triunghiului
dreptunghic.
a) mA  900 . Distingem situaţiile:
1. mB  900 . Construim perpendiculara în A pe AB care intersectează pe BC în E (vezi
figura 2).

Deoarece mA  900 , rezultă C  BE , deci DC  DE . Folosind şi lema pentru triunghiul
dreptunghic ABE, avem minBD, DC  minBD, DE  AD .
2. mB  900 (vezi figura 3).

În acest caz AD  BD  minBD, DC.


3. mB  900 . Construim perpendiculara în A pe AB care intersectează pe BC în E (vezi
figura 4). În acest caz, deoarece mEBD  900 , avem AD  BD  minBD, DC.
b) mA  900 . Construim perpendiculara în A pe AB care intersectează pe BC în E .
Distingem următoarele situaţii:
1. D BE (vezi figura 5).

Rezultă că DE  DC şi atunci AD  maxBD, DE  maxBD, DC.


2. D  E (vezi figura 6). În acest caz AD  BD  maxBD, DC .

223
3. D CE (vezi figura 7) rezultă că AE  BE , de unde AE  DE  BE  DE , deci
AD  AE  DE  BD şi atunci AD  maxBD, DC

Clasa a X – a

Subiectul I
Evident numerele a şi c au aceeaşi paritate şi atunci a  c sunt numere pare. Relaţia din enunţ
se mai scrie 2a 2  4b  a 2  c 2 , de unde
a 2  c 2 a  c2  a  c2  a  c   a  c 
2 2
a  2b 
2
    
2 4  2   2 

Subiectul II
Fie z1  r1 cos1  i sin 1  , z2  r2 cos2  i sin 2  ,
z3  r3 cos3  i sin 3  . Ipoteza se scrie:
ar1 cos1  br2 cos 2  cr3 cos 3  0

ar1 sin 1  br2 sin  2  cr3 sin  3  0
De aici rezultă ar1 cos1  br2 cos 2   ar1 sin 1  br2 sin  2   c 2 r32 şi mai departe
2 2

c 2 r32  a 2 r12  b 2 r22


cos1   2   .
2abr1r2
Atunci z1  z2  z3  Re2 z1  z 2  z3   Im2 z1  z2  z3  , de unde
2

 2 c 2 r32  a 2 r12  b 2 r22 


z1  z 2  z3   r  2 r1r2 cos1   2    r1   sau
2 2
1
 ab 
 2
b   a a a2
  r12  r32  r12  r22    r12 1     , adică
c a
z1  z 2  z3
2

 ab b a   bc b c 
z1  z2  z3
2

a  bc  ab  ac 2
2
r1 
a  ba  c r 2 şi deci relaţia din enunţ este
1
bc bc
demonstrată.

Subiectul III
Evident z  0  1 z  A

224
 Dacă A i , atunci i  0 1 i  i  A , iar z  A  i implică z  a  bi , cu
a, b , deci z 2  a2  b2  2abi  i   A .
 Presupunem că i  A şi z 2  A . Deoarece i  A , există u, v , v  0 ,astfel încât
i  u  vz , de unde:
i u
z (1)
v
Deoarece z 2  A , există a, b , astfel încât z 2  a  bz , deci z verifică ecuaţia de gradul doi
cu coeficienţi întregi: z 2  bz  a  0 .
i u
Dar şi z   verifică această ecuaţie şi atunci din relaţiile lui Viete, avem z  z  b  şi
v
   
2 2
z  z  a  , prin urmare z  z  z  z  4z  z  a2  4b  , adică pe baza lui (1):
 i u i u 
2
4
 v  v    v2  . Rezultă v  1 sau v  4 .
2 2

 
u2 1
Dacă v 2  4 , atunci z  z   , ceea ce este absurd pentru că u 2  1 nu se divide prin 4.
4
prin urmare singura posibilitate rămâne v 2  1 , adică v  1 . Atunci din (1) avem z  i  u .

Rezultă A  a  b i  u  a, b    a u   bi a, b    a  bi a, b    i
Clasa a XI – a

Subiectul I
Trecând la „urmă”, avem trA  trAB  BA  trAB  trBA  0 şi atunci din ecuaţia
caracteristică, rezultă A2  d  A , unde d  det A .
Vom arăta că d  0 . Din ipoteză avem:
AB  BA  A  ABA  BA2  A2  d  B  d  I 2 (1)
BA  AB  A  ABA  A2 B  A2  d  B  d  I 2 (2)
Din (1) şi (2) rezultă  d  I 2  d  I 2 , adică d  0 .
Revenind în (1), avem ABA O2 . Prin inducţie se demonstrează că ABn A  O2 , pentru price
n   . Pentru n  1, am arătat mai înainte, iar dacă presupunem ABn A  O2 , atunci

O2  ABn A   AB  BAB n A  ABn1 A  B ABn A  ABn1 A 
Subiectul II
n n n
Deoarece 1   1 2  1 2 , pentru k  1, n , atunci
n n 2
n k n 1
n n n n
 n  n
 n   n   n   n 
1  2   1  2   1   , adică 1  2   an  1  2  .
 n n k 1  n  k   n2  1  n n  n  1
n n2 n n2
 n  lim 2  n  lim 2
Deoarece lim 1  2   e n n n  e şi lim1  2   e n n 1  e , atunci folosind
n
 n n n
 n  1
criteriul cleştelui, obţinem lim an  e
n

225
Subiectul III
Calculăm limita pe un caz general: L  lim
 
ln 1  x n  ln n 1  x
. Fie L  lim
 
ln 1  x n  x n
şi
x n1 x n1
1
x0 x0

x n  ln n 1  x
L2  lim . Pentru L1 folosim regula lui L’Hospital:
x0 x n1
nxn1 1
L1  lim
 
ln 1  x n  x n
 lim 1  x n
 nxn1

n
lim 1  x n
1
 0 , pentru n  2 . De
x0 x n1 x0 n  1x n n  1 x0 x

x  ln1  x  x n1k ln k 1  x


n1

x  ln 1  x  n1  ln 1  x  
k
 

k 0
asemenea L2  lim  lim   .
x0 x n1 x0 x2  k 0  x  
1
1
 ln 1  x  x  ln 1  x 1  x  1 , atunci L  n . Deoarece
Deoarece lim   1 şi lim  lim
 x 
2
x0 x 0 x2 x0 2x 2 2
n n
L1  0 şi L2  , atunci L  L1  L2 
2 2
Clasa a XII – a

Subiectul I
Fie e elementul neutru din G , iar a inversul (simetricul) elementului a . Fie
 
H  h1 , h2 ,, hp un subgrup al lui G, iar b1 , b2 ,, br elementele lui H care coincid cu
inversele lor (elemente de ordin  2 ), adică bj  b j , j  1, r .
Deoarece într-un grup finit comutativ produsul elementelor grupului coincide cu produsul
elementelor de ordin  2 , atunci produsul elementelor subgrupului H este a  b1b2 br . Dar
atunci a 2  b1b2 br   b12b22 br2  e . Aşadar pentru orice subgrup al lui G produsul
2

elementelor sale este un element a  G cu proprietatea că


a 2  e (1)
Reciproc, fie a  G care verifică (1). Atunci H  e, a este un subgrup al lui G având produsul
elementelor egal cu a .
În concluzie mulţimea elementelor din G cu proprietatea (A) este AG  a  G / a 2  e .  
Cum G este comutativ, rezultă uşor că AG este un subgrup al lui G

Subiectul II
Soluţia 1 (soluţia autorului). Avem

1
1
1 1
1 2 2x  1
I0   2 dx   dx  arctg 
0 x  x 1
2
1   3 
2
0  3 3 0 3 3
x    
 2  2 

226
1
x 1 1  2x  1 1  
I1   dx    2  2 dx şi imediat I1  ln 3  , iar pentru
0 x  x 1
2
2 0  x  x  1 x  x  1 6 3
n  2 , recurenţa
1
1
I n  I n1  I n2   x n2 dx  (1).
0 n 1
Prin inducţie se arată că există an , bn cn  astfel încât

I n  an  bn ln 3  cn (2)
3
1
Folosind această relaţie şi relaţia (1), pentru n  2 , rezultă an  an1  an2  ,
n 1
1 1 1
bn  bn1  bn2  0 , cn  cn1  cn2  0 şi de aici b3k  0 , b3k 1  , b3k 2   , c3k  ,
2 2 3
1 1
c3k 1   , c3k 2   . Aşadar, nu putem avea simultan bn  cn  0 şi deci
6 6

I n  an  bn ln 3  cn care este iraţional.
3
Soluţia 2 (Marcel Ţena) Conform teoremei împărţirii cu rest, avem
 
x n  Pn x x 2  x  1  an x  bn , unde Pn este un polinom cu coeficienţi raţionali, iar an , bn  ,
an  0 sau bn  0 (pentru că x nu se divide prin x  x  1 ) Atunci
n 2

1
 an x  bn  1
I n    Pn x  2 dx , sau, notând cn   Pn xdx  Q , avem I n  cn  an I1  bn I 0 .
0 x  x  1 0
 1 
Deoarece I 0  şi I1  ln 3  (se calculează), avem I n  cn  an I1  bn I 0 , adică
3 3 2 6 3
ln 3 
I n  cn  an  2bn  an  . Dacă prin absurd, există n cu In  , atunci
2 6
an  2bn  an  0 , adică an  bn  0 , ceea ce nu este posibil. Deci I n este iraţional
Soluţia 3 (Diana Drăjneanu, premiul „Fl. Cioacă” pentru această soluţie). Avem
1
I n 3  I n  

x n x3 1  1
  1 1
0 x  x 1
2
dx , adică I n 3  I n  
0
x n1  x n dx . Prin urmare I n3  I n  
n  2 n 1
1
, adică I n3  I n  , n  0 . Deci I n este iraţional dacă şi numai dacă I n3 este
n  1n  2
iraţional, n  3 . Şi atunci pentru a demonstra că I n este număr iraţional pentru orice n  0 este
de ajuns să demonstrăm că I 0 , I1 şi I 2 sunt numere iraţionale. Se calculează
 1 1 2x  1
1
1
1 2 2x  1 1
I0   dx  arctg , I0  , I1   2 dx  I 0 ,
0 
2
1 3 3 3 0 3 3 2 0 x  x 1 2
x   
 2 4

227
1  1
1 
I1  ln 3  . Deoarece I 2  I1  I 0  1dx  1 , obţinem I 2  1  ln 3  . Evident I 0 ,
2 6 3 0 2 6 3
I1 şi I 2 sunt numere iraţionale şi deci In 

Subiectul III
 
1) G,   R , verifică (P), deoarece pentru n par Hn  x   
/ xn  1  1,1 , pentru n
impar Hn  x  
/ xn  1  1 , iar H  1,1.
2) Este uşor de verificat că H n şi H sunt părţi stabile ale lui G , deci subgrupuri ale lui G
(deoarece sunt mulţimi finite). Să luăm m  H . Deoarece H este un grup de ordin m , avem
x m  e , x  H , deci H  H m . Dar H  H n , deci H m  H . Rezultă H  H m .
nN 

3) Fie K un subgrup de ordinul d al lui G . Atunci x d  e , x  K , deci K  H d . Dar K are


d elemente, iar H d cel mult d elemente, prin urmare K  H d . Cum H d este subgrup al lui H
, din teorema lui Lagrange rezultă d m . Aşadar K  H d cu d divizor al lui m .
Observaţie (Marian Andronache) Numărul m nu este unic determinat. De exemplu, dacă G
este grupul lui Klein, avem H  G şi putem lua m  H  4 , dar avem şi H  H 2 , aşa că putem
la fel de bine lua m  2 . Totuşi grupul lui Klein nu verifică (P), deoarece H 2  4  2

Ediţia a XX-a, 28-30 martie 2014

Clasa a VII-a

Subiectul I.

F
A 45o
45o
E
x y
x 90o-2x y
B D C

A  BF   m DAE  m EAF   45o.


 
Fie F astfel încât
   
 
Deci E este pe bisectoarea interioară a lui ABD și pe bisectoarea exterioară a lui BAD  E este pe

bisectoarea exterioară a lui ADB  m ADE  m EDC. .
 
  

   
 
  

Fie m ABE  m EBC  x  m BDA  90o  2x
 

     
228
y 

180o  90o  2x 90o  2x  45o  x .
2 2
În BED  m BED  180o  x  90o  2x  45o  x  180o  135o  45o .
 

 

Subiectul II.
z
D C
v y
t u

A x B
Fie x, y, z, t, u, v lungimile segmentelor AB, BC, CD, DA, AC, respectiv BD. Din ipoteză, dacă p este
perimetrul triunghiului ABC, atunci
x+y+u=p (1)
y+z+v=p (2)
z+t+u=p (3)
t+x+v=p (4)
Adunând aceste relații, obținem 2( x + y + z + t + u + v ) = 4p, deci
x + y + z + t + u + v = 2p (5)
Din (1) și (5), obținem z + t + v = p (6)
Din (3) și (6), obținem u = v (7)
Din (1), (2) și (7), obținem x = z (8)
Din (1), (4) și (7), obținem y = t (9)
Din (8) și (9), rezultă că ABCD este paralelogram (10)
Din (7) și (10), rezultă că ABCD este dreptunghi.

Subiectul III.
Fie x  N  astfel încât 3n  22n  x2 .
Pentru n  0 , avem x  N , iar pentru n  1 , avem y  5 . Presupunem în continuare că n  2 şi
observăm că x este număr natural impar.
Vom folosi în rezolvare două rezultate cunoscute:
1. Dacă a  N este număr impar ( a  2 p  1 ), atunci a 2  4 p p  1  1  M8  1 ( M 8 înseamnă
multiplu de 8).
2. a  b  M a  b n
n

Vom demonstra că n este număr natural par. Dacă n  3 este impar ( n  2 p 1), atunci
3n  32 p  3   M8 1  3  M8  3 , 22n  M8 şi atunci 3n  22n  M8  3 , iar x2  M8 1 şi prin
urmare 3n  22n  x2 .
În continuare dăm două soluţii în cazul ăn care n este număr natural nenul par.
Soluţia 1. Deci n  2 p , p  N  şi atunci ecuaţia devine 32 p  222 p  x2 . Vom demonstra că
 
32 p  222 p nu poate fi pătrat perfect. Evident 222 p  32 p  22 p , de asemenea
2

229
22 1
2
p
 222 p  2  22 p 1  222 p  9 p  222 p  32 p . Aşadar 32 p  222 p este situat între două

   
2 2
pătrate consecutive ( 22 p şi 22 p 1 , deci 32 p  222 p  x2

Soluţia 2. Deci n  2 p , p  N  şi atunci ecuaţia devine 32 p  222 p  x2 sau x2  222 p  32 p ,


 x  22  3
p t


adică x  22 p  
x  22 p  32 p , de unde 
 x  22  3
p 2 p t
, cu t  2 p  t . Prin scădere, obţinem

 
2  22 p  3t 32 p2t 1 , de unde t  0 . Deci avem ecuaţia 2  22 p  32 p 1, adică 2  22 p 1  9 p ,
ecuaţie care nu are soluţii pentru că 2  22 p 1  22 p  9 p .

În concluzie 3n  22n este pătrat perfect doar pentru n  1 .

Clasa a VIII-a
Subiectul I.
D`
C` A` O` C`
O`
A`
B` M l
M
N l
D N C 2
A C
l 2
A B

AB`M   A`MC`  AB`M   ACC`A`  AM .


B`O`  AA`C`C   T .3  
   
 B`N  AM și deci m  AB`M ,  A`MC`  m B`NO` .
O`N  AM , N  AM     
l l 2 3l
Fie l latura cubului  AC  l 2 , MC  , AM  2l 2   .
2 4 2
ArieO`AM   ArieACC`A`  ArieACM  ArieMC`O`  ArieAA`O` 
l2 2 l2 2 l2 2 2 5l 2 2 3l 2 2
 l2 2    l 2  .
4 8 4 8 8
AM  O`N
ArieO`AM  
2
3l
 O`N
3l 2 2 2 2  3l 2 2 l 2
  O`N   .
8 2 12l 2

230
l 2
  O`B`  2  1  m B`NO
În B`O`N dreptunghic în O`  tg B`NO` 
 
`  45o .

  O`N l 2  
2

Subiectul II.
Fie x ℝ astfel încât x  0  x 0,1 .

Dacă x  1  x 
x  x  1 , absurd.
x
Dacă x  1,0  x  1 și x  x  x  x 1.
x 1 1
Trebuie ca xx .
1 2
Dacă x  1  x  1  1 
x  x , absurd.
x
1
Singura soluție este x   .
2

Subiectul III.
 
2
 
2
 
Avem   k 2 1  42k 1 . Dar observăm că, pentru k  4 , k 2 1    k 2 , deci  nu
2

poate fi pătrat perfect. Rămân de rezolvat cazurile k 0,1,2,3.


Cazul k  0    3  0 nu convine.
Cazul k  1  x 2  1  0  x  1 și ecuația are rădăcina naturală x  1.
Cazul k  2    21, nu convine, nefiind pătrat perfect. La fel și în cazul k  3 , când se obține
  84 .

În concluzie, ecuația dată admite numai soluția x  1, când k  1 .

Clasa a IX-a

Subiectul I.
Din teorema lui Ptolemeu avem:
AC  BD  AB  CD  AD  BC (1)
Evident AC  2R , BD  2R și de aici, prin înmulțire:
AC  BD  4R 2 (2)
Din inegalitatea mediilor avem:
AB  CD  AD  BC  2 AB  BC  CD  DA , de unde
 AB  CD  AD  BC 
2
AB  BC  CD  DA    (3)
 2 
Combinând inegalitățile (1), (2), (3), avem:
2
 4R 2 
AB  BC  CD  DA     4R 4 .
 2 

Subiectul II.

231
Notând N  1 2  3  2  3  4  ... nn 1n  2 , avem

k k  1k  2k  3  k  1k k  1k  2  1 nn  1n  2n  3


n
1 n
N   k k  1k  2  
k 1 4 k 1 4

Presupunând prin absurd că N este pătrat perfect, N  m2 , m  * , rezultă că 
nn  1n  2n  3  4m2 , ceea ce revine la
    
2

4m2  n2  3n n2  3n  2  n2  3n  2 n2  3n  n2  3n  1 1  
2

 
2
 
De aici rezultă că n2  3n  4m2  n2  3n  1 , deci n 2  3n  2m  n 2  3n  1 . Absurd!
2

Subiectul III.
Deoarece m, n, p  3 , deducem că
cos A cos B cosC cos A cos B cosC
E      .
sin m A sin n B sin p C sin 3 A sin 3 B sin 3 C
Dar
cos A 8R 3 b 2  c 2  a 2 4R 3 b 2  c 2  a 2 4R 3  b 2 c 2  R 2  b 2 c 2 
         1      1 ,
sin 3 A a 3 2bc abc a2 4RS  a 2 a 2  S  a 2 a 2 
R2  b2 c2 c 2 a 2 a 2 b2 
deci E    2  2  2  2  2  2  3 .
S a a b b c c 
b2 a 2 3R 2
Cum 2  2  2 și analoagele, avem E  .
a b S
absin C 3
Dar S   2R 2 sin Asin B sin C , deci E  .
2 2 sin Asin B sin C

Clasa a X-a

Subiectul I.
 Presupunem că ecuația are o rădăcină de modul 1, fie aceasta z0 .
Avem: z0n1  bz0n  z0  b  z0n z0  b  z0  b (1)
Trecând în (1) la module, obținem z0  b  z0  b , deci z0  b  z0  b 
2 2

 
 z0  bz0  b  z0  bz0  b  z0  b z0  b  z0  b z0  b   
 1  1
z0  bz0  b z0  b 2  z0  bz0  b z0  b 2  b z0    b z0   
2 2

 z0   z0 

 z0   0  z02  1  z0   i, i.
1
z0
Analizând cazurile n  4k , n  4k  1, n  4k  2 , n  4k  3 , se constată ușor că egalitatea (1)
este posibilă, când z 0  i , doar pentru n  4k  1 și b  1.
 Presupunem b  1, n  4k  1, deci ecuația dată se scrie
z 4k 2  z 4k 1  z 1 .
Constatăm ușor că z 0  i este o rădăcină de modul 1 a ecuației.

Subiectul II.

232
Dacă tgx  0 , atunci ctgx  0 , deci 2tgx  2ctgx  1 1  2 , deci x nu este soluție. Prin urmare,
dacă x este soluție, avem tgx  0 și ctgx  0 , inegalitățile fiind de fapt stricte.
Folosind inegalitatea mediilor, avem:
2tgx  2ctgx  2 2tgxctgx  2 22  4 .
1
Așadar , trebuie să avem peste tot egalități, deci tgx  ctgx  tgx   tg 2 x  1  tgx  1
tgx
( deoarece tgx  0 ). Mulțimea soluțiilor ecuației este:
 
S    k | k   .
4 

Subiectul III.
Avem
 
1  2z  3z 2  ... nz n1  z  2z 2  3z 3  ... n  1z n1  nz n  n  1 1  z  z 2  ... z n1 
  
1  z  z 2  ... z n1  nz n  n  1 1  z  z 2  ... z n1 
Înmulțim relația cu 1  z și obținem
   
1  z  z 2  ... z n1 1  z   nz n 1  z   n  1 1  z  z 2  ... z n1 1  z  
 
1  z n  nz n  nz n1  n  1 1  z n 
1  z n  nz n  nz n1  n  nz n  1  z n  nz n1  n  z n1  1 , de unde rezultă soluțiile
2k 2k
zk  cos  i sin , unde k 1,2,3,...,n, deoarece z k  1 .
n 1 n 1

Clasa a XI-a

Subiectul I.
an  an1  an  an1 an  an1  an  an1
a) Avem: xn  , yn  .
2 2
Rezultă imediat că lim xn  lim yn  lim an .
n n n

b) Reciproca este falsă. Contraexemplu : an   1 .


n

Subiectul II.
t 2 1
a) Vom arăta că  1, t  0,1.
2t
t 2 1
Considerăm funcția f : 0,1  ℝ, f t   t .
2
Derivata sa este f `: 0,1 ℝ, f `t  
   
2t  2t  t 2  1 2t ln 2 2t  t 2  1 ln 2 gt 
  t , unde
22t 2t 2
g : 0,1  ℝ, gt   t  1ln 2  2t .
2

233
Derivata funcției g este g`: 0,1  ℝ, g`t   2t ln 2  2  2 t ln 2  1 și are rădăcina

 1 , deci g`t   0, t 0,1. Atunci g este crescătoare pe 0,1, ceea ce ne conduce la


1
t0 
ln 2
un tablou de variație pentru funcția f de tipul:

t 0 t1 1
f `t  - - - - - - - - 0 + + + + + +
f t  1 m 1

( Am notat cu t1 rădăcina ecuației f `t   0 din intervalul [0,1], adică rădăcina din [0,1] a
1  1  ln 2 2
ecuației gt   0 , mai exact t1  ).
ln 2
t 2 1
Din tabel, rezultă f t   1, t 0,1, adică  1, t  0,1.
2t
Egalitatea are loc dacă și numai dacă t 0,1.
b) E necesar ca sin x  0 și cos x  0 ( căci, dacă, de exemplu, sin x  0 , atunci 2sin x  1 , deci
2cosx  3  2sin x  3  1  2 , deci cos x  1 , absurd ).
Folosind inegalitatea de la a) în care luăm, pe rând, t  sin x , respectiv t  cos x , obținem
inegalitățile 2sin x  1  sin 2 x și 2cosx  1  cos2 x . Prin adunare, rezultă 2sin x  2cosx  3 , prin
urmare este necesar să avem egalitate în fiecare dintre inegalitățile folosite. Rezultă sin x 0,1
și cos x 0,1. Convin doar posibilitățile sin x  0 , cos x  1 și sin x  1 , cos x  0 . Mulțimea
soluțiilor este:
 
S  2k | k  Z    2k | k  Z  .
2 

Subiectul III.
a b
Fie a  r cos și b  r sin  , deci r  a 2  b 2 ; cos  ; sin   ( am admis cazul netrivial
r r
r  0 ). Atunci
 cos 0  sin  0   cos 2 0  sin 2 0 
   
 0 cos 0  sin   2 0 cos 2 0  sin 2 
M  r , M r 
2
sin  0 cos 0  sin 2 0 cos 2 0 
   
 0 sin  0 cos   0 sin 2 0 cos 2 
Prin recurență avem
 cos n 0  sin n 0 
 
n 0 cos n 0  sin n 
M r 
n
0 
(1)
sin n 0 cos n
 
 0 sin n 0 cos n 
Considerând numărul complex z  a  bi  rcos  i sin   , avem
z n  a  bin  r n cosn  i sin n 

234
z  a  bin  r n cosn  i sin n 
n

Deci r cos n 
n zn  z

a  bin  a  bin , r n sin n  z n  z  a  bin  a  bin .
n n

2 2 2i 2i
Aceste rezultate se înlocuiesc în (1).

Clasa a XII-a

Subiectul I.
f `x 1
 2013 2013
Ecuația se scrie
f x   x x 1  . Integrăm:

f `x x 2012
 f x    x 2013 x 2013  1 dx  (notăm t  x ) 
 
2013
dx 2013

dt 1 1  t x 2013
    dt  ln  C1  ln 2013  C1 .
t t  1  t t  1  t 1 x 1
 Cx 2013
 2013
Așadar ln f x  ln 2013   f x  2013 , C  0 .
Cx
 x  1 x 1
x 2013
Condiția f 1  dă C  1 și astfel, unica funcție căutată este f x  2013 .
1
2 x 1

Subiectul II.
Ne bazăm pe echivalența: U k  U p  k | p .
De asemenea, ne bazăm pe faptul cunoscut că subgrupurile finite ale grupului (ℂ*, ·) sunt
Uk , k  * .
a) Deoarece G  U n , G  U m , rezultă că G este finit, deci G  U k , pentru un anumit k  * .
Deci U k  U n ,U k  U m , adică k | n, k | m și k trebuie să fie maxim, deci k  n, m . Rezultă
G  Un,m .
b) Căutăm H finit, deci H  U p , pentru un anumit p  * . Din U n  U p ,U m  U p , deducem
că p este multiplu de n și m , dar p trebuie să fie minim, prin urmare p  n, m. Rezultă
H  Un,m .

Subiectul III.
Folosind metoda integrării prin părţi, avem
 1
1
 xn1  xn1 1 1 xn1
In     arctg  
nx dx  arctg  
nx   ndx , adică
0 n  1  n  1 0
n  1 0 n2 2
x  1
1 n n n2
In  arctgn J şi atunci nI n  arctgn J . Deoarece
n 1 n 1 n n 1 n  1 n
xn1
0  2 2  2 , x 0,1 (inegalitatea din dreapta este echivalentă cu inegalitatea
1
n x 1 n 1
n x  xn1 1   xn1 1  0 care este adevărată pentru x0,1 ), atunci
2 2

235
1
1 1 n2 n2 n2
0  Jn      Jn  0 .
0 n 2
 1
dx
n 2
 1
, deci 0
n  1
J n
n  1n 2
 1 şi prin urmare lim
n n  1

 n n2  
Rezultă atunci că lim nI n  lim  arctgn  J n   .
n n  1 n 1  2
n

Ediția a XXI-a, 27-29 martie 2015

Clasa a VII-a

Subiectul I.
Notând numărul cu p , avem succesiv:
p  2015a  2015b  c  20150 a20150 b  c  201502  20150a  b  ab  c .
Folosind ipoteza, obținem imediat:
p  201502  2015010  25  201502  2  20150 5  52  20150 52  201552 , ceea ce
încheie rezolvarea.

Subiectul II.
 
a) Deoarece 16n 12n  22n 4n  3n , atunci 16n 12n este pătrat perfect dacă 4n  3n este pătrat
perfect. Pentru n0,1 numărul 4n  3n este pătrat perfect.
Presupunem în continuare că n  2 și fie x  
, astfel încât 4n  3n  x2 , de unde
2  x  3
n k

 n
 n

2  x 2  x  3 . Deoarece 2  x  2  x , atunci  n
n n n
n k
cu k  n  k . Prin
2  x  3
 
adunarea celor două relați obținem 3nk  3k  2n1 sau 3k 3n2k 1  2n1 și imediat 3k  1 și
3n2k 1  2n1 , adică k  0 și 3n 1  2n1 . Dacă n este par, atunci 3n  M8 1 ( M8 înseamnă
multiplu de 8) și deci 3n 1  M8  2  2n1  M8 . Dacă n  2 p 1, atunci
3n  32 p1  9 p  3  3 M8 1  M8  3 și atunci 3n 1  M8  4  2n1  M8 . Așadar 16n 12n
este pătrat perfect pentru n0,1

b) Prin înlocuire, se arată că n  0 și n  1 nu verifică, iar pentru n  2 , avem


162 122  112  121 25 16  112  52  42
  4 n    4 3  37
Pentru n  3 , avem 16n 12n  12n    1  12n    1  12n   11n , și cum
 3    3   27
   
11n  11n  5n  4n , avem 16n 12n  11n  5n  4n .
În concluzie singura valoare convenabilă este n  2 .

236
Subiectul III.
E

D C

A B
BD fiind mediatoarea segmentului AE, rezultă DA  DE și BA  BE, relație care
   
     
conduce la DEA  DAE și BEA  BAE . De aici obținem că m BED  m BAD și, mai
   
 
  

departe, că m BED  m BCD , ceea ce demonstrează că patrulaterul BCED este inscriptibil,
   
de unde concluzia.

Clasa a VIII-a

Subiectul I.
   
Fie x  N , astfel încât p n  8  x 3 , adică x  2 x 2  2x  4  p n . Fie d  x  2, x 2  2x  4
c.m.m.d.c. Deoarece x  2x  4  x  2x  4  12 , atunci d 12 şi prin urmare
2

d 1,2,3,4,6,12. Pentru că d 2 x  2x 2  2x  4  p n , rezultă că d  pk , k  0 şi cum p este


prim, atunci d 1,2,3,4.
 
- Dacă d  1 , atunci deoarece x  2  x 2  2x  4 şi p este prim, din x  2 x 2  2x  4  p n
x  2  1
rezultă  . Obţinem x  3 , p n  19 şi deci x  3 , n  1, p  19 .
x  2 x  4  p
2 n

x  2  2
- Dacă d  2 , atunci p  2 şi din x  2x 2  2x  4  2n rezultă  2 n1
. Obţinem
x  2 x  4  2
x  4 , 2n1  24 şi deci n  N .
x  2  3
- Dacă d  3 , atunci p  3 şi din x  2x 2  2x  4  3n rezultă  2 n1
. Obţinem
x  2 x  4  3
x  5 , 3n1  39 şi deci n  N .
x  2  4
- Dacă d  4 , atunci p  2 şi din x  2x 2  2x  4  2n rezultă  2 n2
. Obţinem
x  2 x  4  2
x  6 , 2n2  52 şi deci n  N .
În concluzie n  1, p  19 .

Subiectul II.

237
Numărul prim p  2 nu convine, deci p este prim impar. Atunci  p 1! este par,  p 1!1

este impar,
 p 1!1 este impar, deci q 6  39 este impar. Rezultă că numărul q 6 este par, deci
p
q  2.
 
Ipoteza devine  p  1!1  q 6  39 p sau, echivalent  p 1!1  103p (1)
Ecuația (1) este verificată doar de numărul prim p  7 .
Într-adevăr, pentru p 3,5 avem  p 1!1  25  103p , iar pentru p  11 vom arăta
inegalitatea de sens contrar  p 1!1  103p . Pentru aceasta, să observăm că putem scrie:
 p 1!1   p 1!  p  3! p  2 p 1  8! p  2 p 1  103 p  2 p 1  103p , deoarece
 p  2 p 1  p ( ultima inegalitate fiind echivalentă cu  p  22  2 , evidentă când p  11)
Așadar, soluția unică a problemei este  p  7, q  2 .

Subiectul III.
A
M P

B N
D

C
R
Q

Este evident că Q  MN  BC și R  PN  CD .


Cu teorema lui Menelaus aplicată în triunghiurile ABC și ACD pentru transversalele
M  N  Q , respectiv P  N  R , se obține imediat că C este mijlocul segmentelor BQ și RD.
Rezultă că BDQReste paralelogram și BR || DQ .
d   ABR   ADQ 

BR   ABR, DQ   ADQ  d || BR || DQ

BR || DQ 
Cum BR  BCD , rezultă d || BCD .

Clasa a IX-a

Subiectul I.
Notând x  p, p   , ecuația devine x 2  5 p 2  2x  p  0 , echivalentă cu
2

2 p  13 p
3x 2  4 px  3 p 2  0 , cu rădăcinile x1,2  .
3
Pentru p  0 obținem soluția banală x  0 . Studiem cazurile p  0 :

238
2  13
1) Dacă p  0 , avem p  1 x  1, iar rădăcinile ecuației sunt x1, 2  p , dintre care
3
2  13
reținem x  p.
3
2  13
Dar p  x  p  1  p  p  p  1 ’.
3
 
Inegalitatea din stânga este adevărată, iar cea din dreapta este echivalentă cu p 13 1  3 , de
13  1  5  2  13
unde obținem p   1,  . Rezultă p  1 și obținem rădăcina x  .
4  4 3
2  13
2) Dacă p  0 , avem p  1 și x  0 , iar rădăcinile ecuației fiind x1, 2  p , reținem
3
2  13
x p , în celălalt caz x și p fiind de semne contrare.
3
2  13
În inegalitatea dublă p  p  p  1 , inegalitatea din stânga este falsă pentru p  0 , ea
3
 
revenind la p 13 1  0 .
2  13
În concluzie, soluțiile ecuației date sunt x1  0 și x2  .
3

Subiectul II.
A B C
Arătăm că cos  cos  cos și analoagele.
2 2 2
A BC B C C B C B
Într-adevăr: cos  sin  sin cos  sin cos  cos  cos .
2 2 2 2 2 2 2 2

Subiectul III.

B Q C

D
Fie P mijlocul segmentului AH și Q mijlocul segmentului BC .
În BAH , BP este mediană  BH  BA  2BP (1)

239
În CAH , CP este mediană  CH  CA  2CP (2)
   
Însumând cele două relații: BH  BA  CH  CA  2BP  2CP  2 BP  CP  2 2QP  4QP .
Fie D punctul diametral opus lui A pe cercul circumscris triunghiului ABC . Demonstrez că
patrulaterul BDCH este paralelogram:
AD  diametru  m DAC  90o  DC  AC și BH  înălțime  BH  AC , de unde rezultă

 
că DC || BH (3)
Analog BD || CH (4)
Din (3) și (4)  BDCH  paralelogram  BC  HD  Q, Q fiind mijloc pentru segmentele
BC și HD .
AD
În AHD, PQ este linie mijlocie  PQ|| AD și PQ  .
2
AD=2R ( unde R este raza cercului circumscris triunghiului ABC )  PQ  R și ipoteza

devine 4 PQ  4R  BC  R .

 2R  sin A   m A  30o .


BC R 1 
Din teorema sinusurilor,  2R , deci
sin A sin A 2  

Clasa a X-a

Subiectul I.
Folosind cunoscuta proprietate a logb c  c logb a (care se obține ușor, prin logaritmare în bază b ),
ecuația se scrie 2log5 x  1  5  4log5 x  2 și dacă notăm y  2log5 x , cu
1  y  5  1  x  5log2 5 , aceasta devine y 1  5  y 2  2 (1)
Rezolvăm ecuația irațională (1). Aceasta se scrie 5  y 2  2  y  1 și, ridicând la pătrat,
obținem 5  y 2  4  4 y  1  y  1 sau 4 y  1  y 2  y  2 sau încă 4 y  1   y  1 y  2
(2)
O soluție a ecuației (2) este y1  1, iar pentru y  1, ecuația (2) devine:
 y  2 y 1  4 (3)
Ecuația (3) are soluție unică y2  2 , deoarece funcția f : 1,   R , f  y    y  2 y  1 este
strict crescătoare.
Ambele soluții y1  1 și y2  2 verifică ecuația inițială (1).
Corespunzător, avem soluțiile:
1) log 5 x  log 2 y1  log 2 1  0 , deci x1  1.
2) log 5 x  log 2 y2  log 2 2  1 , deci x2  5 .
Ambele soluții x1  1 și x2  5 verifică ecuația.

Subiectul II.

240
r
Avem AI  și analoagele. Egalitatea de demonstrat devine:
A
sin
2
A B C A B C
sin 2 sin 2 sin 2 2r sin sin sin
2 2 2 2 2 2  1 sau
2 2 2 3
r r r r r2
A B C A B C
sin 2  sin 2  sin 2  2r sin sin sin  1.
2 2 2 2 2 2
A 1  cos A
Ținând seama că sin 2  și analoagele, ultima egalitate de demonstrat revine la
2 2
1  cos A 1  cos B 1  cosC A B C
   2 sin sin sin  1 sau încă:
2 2 2 2 2 2
A B C
cos A  cos B  cosC  4 sin sin sin  1 .
2 2 2
Ultima egalitate este bine-cunoscută într-un triunghi.

Subiectul III.
1
Fie Pk  z k  ; k  1, n . Rezultă
zk
1  1
2 2
1  z z 1 z2  z 1
Pk  z k    z k   z k    z k z k  k  k   z k  k 2k  2 
2 2

zk  z k  zk  zk zk zk zk zk zk

 tk  
1 2 Re z k2  
; t k  z k , k  1, n
2

tk tk
tk  Pk tk  1  Re zk2  0
2 2
 
  
  0  Pk4  4 1  2 Re zk2  0

Pk4  4 1  2 Re zk2 
4

 
n n n
   Pk4  4n  8 Re z k2 .
1
zk 
k 1 zk k 1 k 1

Clasa a XI-a

Subiectul I.

241
Putem scrie determinantul în forma
4S 2 c
1 a 1 a
ab abc 1 a c
1 b
4S 2
bc
 4S 1 b
2 a

4S 2
abc abc
1 b a
4S 2
abc

b  ab  c  c  a2  
4S 2 b 1 c b
1 c 1 c
ca abc


abc

4S 2 2
a  b 2  c 2  bc  ab  ac  
4S 2
2abc

b  c2  c  a2  a  b2  

2S 2
abc
b  c2  c  a2  a  b2 
2S 2S 2S
și, evident, det A  0 . Am folosit ha  , hb  , hc  .
a b c
Avem det A  0 dacă și numai dacă a  b  c , adică ABC este echilateral.

Subiectul II.
3xn2  xn  2
Se observă că xn  0 pentru orice n  1 și xn1  xn   xn 
2xn2  xn  3

 

 2xn3  4xn2  4xn  2 21  xn  xn2  xn  1 
, n  1.
2xn2  xn  3 2xn2  xn  3
3x 2  xn1  2 1  xn21
Dar 1  xn  1  n21  2 , deci sgn1  xn   sgn1  xn1  , n  2 și prin
2xn1  xn1  3 2xn1  xn1  3
1  x02
recurență sgn1  xn   sgn1  x1  . Cum 1  x1  2 , rezultă că
2x0  x0  3
 
sgnxn1  xn   sgn1  xn   sgn 1  x02 , n  1.
Prin urmare: xn1  xn  0 și 1  xn  0 , dacă 1  x02  0
xn1  xn  0 și 1  xn  0 , dacă 1  x02  0
adică xn1  xn (șir crescător) și xn  1 , n  1, dacă x0  1
xn1  xn (șir descrescător) și xn  1 , n  1, dacă x0  1 .
În ambele cazuri șirul este convergent.
3l 2  l  2
Notând lim xn  l , din relația de definiție avem prin trecere la limită l  . Rezultă
n 2l 2  l  3
ecuația l 3  2l 2  2l  1  0 , echivalentă cu l  1l 2  l  1  0 . Obținem l  1 .

Subiectul III.
Considerăm intervalul I  Im f , care este nedegenerat (în caz contrar, f ar fi constantă, deci
f ` 0 , contrar cu (ii) ).
Din (ii) avem f `t   1, t  I , prin urmare f t   t  a , cu a ℝ, pentru t  I . Din (i) rezultă
a  0 , deci f t   t , pentru orice t  I .

242
Vom arăta că I  ℝ și atunci singura funcție „bună” este funcția identică f : ℝ ℝ, f x  x ,
pentru orice x ℝ.
Notăm p  inf I , q  sup I , p  q şi arătăm că p   , q   . Presupunem prin absurd
că p ℝ. Din continuitate avem: f  p  lim f t   lim t  p , deci f  p  p .
tp tp
t p t p

f t   f  p tp
Din derivabilitate avem: f ` p  lim  lim  1 , deci f ` p  1 (1).
tp
t p
tp tp t  p
t p

Dar f  p  p  inf f x , deci p este un punct de minim al funcției f , în interiorul


xR

intervalului  , . Conform teoremei lui Fermat: f ` p  0 (2).


Egalitățile (1) şi (2) se contrazic, prin urmare p   . Analog q   şi soluția se încheie.

Clasa a XII-a

Subiectul I.
2
Avem : tgx  ctgx  , prin urmare:
sin 2x
 sin x lntgx  ctgxdx  sin xln 2  ln sin 2xdx   ln 2cos x   sin x ln sin 2xdx (1)
  sin xln sin 2xdx  cos x`ln sin 2xdx  cos x ln sin 2x  2 cos x sin 2x cos 2xdx 
1

1  2 sin 2 x 1
 cos x ln sin 2x   dx  cos x ln sin 2x  2 sin xdx   dx 
sin x sin x
dx
 cos x ln sin 2x  2 cos x   (2)
sin x
dx sin x dt 1 1 t
 sin x dx   1  cos2 x dx   1  t 2 ( cu substituția cos x  t )  2 ln 1  t 
1 1  cos x 1  x 
2
x x
 ln  ln tg   ln tg  ln tg (3)
2 1  cos x 2  2  2 2
Din (1), (2) și (3) rezultă că primitiva cerută este:
 ln 2cos2x  cos x ln sin 2x  2 cos x  ln tg x  C  2  ln 2cos x  cos x ln sin 2x  ln tg x  C
2 2
Subiectul II.
a) Probăm egalitatea prin dublă incluziune.
„ ” . Fie xU A , deci există x 1  A cu xx1  x 1 x  1 . Dacă, prin absurd, am avea x 2  0 ,
înmulțind la stânga cu x egalitatea xx1  1, ar rezulta x 2 x 1  x , adică 0  x , ceea ce ar
însemna că 0 este inversabil, absurd. Prin urmare, x 2  1.
„ ” . Dacă x  A , cu x 2  1, înseamnă că x este inversabil, iar x 1  x .
b) Observăm că este adevărată implicația: x U A 1  x U A .
Într-adevăr, x U  A  x 2  0 și atunci 1  x1  x  1  x1  x  1  x 2  1 , deci 1  x este
inversabil, având ca invers pe 1  x .

243
Să arătăm comutativitatea inelului A . Fie x, y  A și să dovedim că xy  yx . Avem patru
cazuri:
1) x, y U A . Grupul U A este abelian, căci x 2  1, xU A și astfel xy  yx .
2) xU A , y U A . Atunci x , 1  y U A , deci x1  y  1  yx  xy  yx .
3) xU A , y U A . Atunci 1  x , y U A , deci 1  xy  y1  x  xy  yx .
4) xU A , y U A . Atunci 1  x , 1  y U A , deci 1  x1  y  1  y1  x  xy  yx
.

Subiectul III.

 
1
Fie I n   ln x n  e x dx .
0
1
  xn  1 1
 xn  1 1
 xn  1
I n   ln e x 1  x  dx   ln e x dx   ln1  x dx   xdx   ln1  x dx   An (1)
0   e  0 0  e  0 0  e  2
1
Vom arăta că lim An  0 și, folosind (1), va rezulta că limita propusă este .
n 2
 x  xn n
Deoarece ln1  y  y pentru orice y  0,1, avem ln1  x   x , prin urmare
 e  e
1 1
xn 1
0  An   dx   x n dx  , deci lim An  0 .
0 e x
0 n  1 n

Ediția a XXII-a, 1-3 aprilie 2016

Clasa a V-a

Subiectul I.
a) Fie d ℕ* astfel încât d | 4n  3, d | 5n  1  d | 20n  15, d | 20n  4  d | 19 .
d  19  19 | 4n  3  există x ℕ* astfel încât
4n  3  19x  4n  3  20x  x  x  3  20x  4n  x  34
Fie k ℕ* astfel încât x  3  4k  4n  3  194k  3  n  19k 15
b) Fie n ℕ* astfel încât n  2016c  r , r  2016 , unde c și r sunt câtul, respectiv restul
împărțirii lui n la 2016.
Atunci r  2c  n  2016c  2c  2018c , 2c  2016  c  1008.
Fie S  2018 1  2018  2   2018 1007  20181  2   1007 ,
1007 1008
S  2018   1009 1007 1008
2

Subiectul II.
a) 125  53  52  102  125 este cub bipătratic 9  32  13  23  9 este pătrat bicubic

244
   5n   10n 
b) 125n6  53 n6  52 n6  102 n6 , 125n6  5n2
3 3 2 3 2
125n6 este cub bipătratic,
n ℕ*.
   n   2n 
9n6  32 n6  13 n6  23 n6 , 9n6  3n3
2 2 3 2 3
 9n6 este pătrat bicubic, n ℕ*

Subiectul III.
nn  1
a) Pe primele n linii avem primele 1  2  3  ... n  numere pare și nenule, deci ultimul
2
nn  1
număr din linia n este 2   nn  1 .
2
Astfel avem că 2  1 2,6  2  3,12  3  4,20  4  5,... deci linia n are ca ultim număr pe nn  1 .
Linia 2015 se termină în 2015 2016  linia 2016 începe cu 2015 2016  2  4062242.
b) Dacă n este numărul liniei pe care se află 2016, atunci nn 1  2016  nn 1 ,
2016  25  32  7  2  3  7  24  3  42  48
1980  44  45  2016  2070  45  46  n  45
c) Linia n este n 1 n  2, n 1 n  4, n 1 n  6,...,n 1 n  2n .
Suma elementelor liniei n este n  1n 2  21  2  3  ... n  n  1n 2  nn  1  n3  n
n3  n  8020  8000  20  203  20  n  20 .

Clasa a VI-a

Subiectul I.
a b c b ac ac
Din     b  . Cum a  c  112  2b și
k k  1 k  2 k  1 2k  2 2
a  c  2b  a  c  112  2b  a  c  112 și b=56. Cum unul dintre numerele a și c este
a 56 56k
natural și a  c  112, rezultă că a ℕ, c ℕ.  a  ℕ   k  1 | 56  k şi
k k 1 k 1
k, k 1  1  k 1| 56 .
Dar k este impar, k  3  k 3,7,13,27,55. Obținem soluțiile
a, b, c, k 42,56,70,3, 49,56,63,7, 52,56,60,13, 54,56,58,27, 55,56,57,55.
Subiectul II.
A

E
D N
M
P

B
C

245
a) Avem
m AMP  m ANP  180o  m BAC  m MPN   180o  m BAC  m DPB
     

           
Din DPB ,
   
m DPB  180o   m PBD  m PDB   180o   m EBA  m ABD  m ADB  m ADC  
      

               
   
 180o   m ABD  m ADB    m EBA  m ADC  .
   

         
         
Avem: m DPB  m BAD   m EBA  m ADC 
   (1)
        
1
 
 

Din congruența triunghiurilor DAC și BAE rezultă că EBA  ADC m DPB  m BAD , și
 

   
 
      
cum m BAD  m BAC  m BAC  m DPB 
  
       
m BAC  m MPN   180o  m AMP  m ANP  180o .
   

       
b) Ughiurile AMP și ANP sunt congruente și suplementare
 m AMP  m AMP  90o  BN, CM sunt înălțimi în ABC  AP este înălțime
 

   
 AP  BC .

Subiectul III.
2 3 4 3 1
S  1  2  3  11   3 .
2 2 2 4 2
Am demonstrat că S  3 (1)
1
Înmulțind egalitatea din enunț cu și scăzând cele două egalități, obținem:
2
1 1 1 1 2016
S  S  1   2  ... 2015  2016 (2)
2 2 2 2 2
1 1 1
Fie M  1   2  ... 2015 . Rezultă:
2 2 2
1 1 1 2  1 1 2016
M  M  1  2016  M  2  2015  S  2  2015  2016 
2 2 2 2 2 2
1 2016 1
 S  4  2014  2015  S  4 3  S  4 .
2 2

Clasa a VII-a

Subiectul I.
Avem, succesiv, 2n  1  2n 1  8n3  12n2  6n  1  8n3  12n2  6n  1  24n2  2 
3 3

 16n2  2n 12  2n  12 .

246
Subiectul II.

D C G

E O F

H A B

Notăm FB  DC  G, AB  CE  H, DF  CE  I .


Din ABCD paralelogram rezultă AB || CD, AB  CD . Cum AC  AB , deducem AC  DC ,
 
    

deci m DCA  90o , adică m DCI   m ICA  90o .
    
 
  
       
În DCE , avem m DIC  90o , m CDI   m DCI   90o  CDI  ICA , adică GDF  ACH
     
(1).
 
  
  

Din m CAB  m ABG  m ACG  90o  CGBA dreptunghi, de unde m CGB  90o ,
 

       
 
GB=CA (2), CG=AB (3). Din AB || CD, DA secantă, rezultă DAH  ADC ( alt.int.)
AC
Arătăm că AEH  DEC (U.L.U), de unde HA=DC. Cum DC=AB, deducem HA  AB 
2
și DG  2 AB  AC (4).
AC
Arătăm că DGF  CAH ( cf. C.U.), de unde AH  GF . Cum AH  AB  , deducem
2
AC GB
GF  și GF  .
2 2
În trapezul ABGD avem EF linie mijlocie, de unde EF|| GD. În ACD avem E mijlocul lui
AD , EO||DC, unde CA EF  O EO linie mijlocie, deci O este mijlocul lui AC .
Subiectul III.
Vom folosi în rezolvare două rezultate cunoscute:
1. Dacă a  N este număr impar ( a  2 p  1 ), atunci a 2  4 p p  1  1  M8  1 ( M 8 înseamnă
multiplu de 8).
2. a  b  M a  b n .
n

a) Avem 53  125  7 16 1  M7 1 și atunci 53k  M7   1 . De asemenea 33  27  M7 1,


k

deci 33k  M7   1 .


k

247
Fie an  5n  3n . Avem a3k  53k  33k  M7  2   1  M7 ,
k

a3k 1  53k  5  33k  3  M7   1 5  3  M7   1  M7 ,


k k

a3k 2  53k  25  33k  9  M7   1  25  9  M7  6  1  M7 , deci an  5n  3n nu se divide


k k

prin 7.

b) Presupunem că există y  astfel încât 5n  3n  y2 . Evident y este număr natural par.


Pentru n număr natural par, avem 5n  M8 1 , 3n  M8 1 , 5n  3n  M8  2 , iar y2  M4 şi
prin urmare 5n  3n  y2 . Deci n  2z 1, z  N şi atunci ecuaţia devine 52 z 1  32 z 1  y2 .

Deoarece 52 z 1  32 z 1  5  3 52 z  52 z 1  3  52 z 2  32  
 5  32 z 1  32 z  8  p şi p este număr
natural impar (suma unui număr impar de numere impare, rezultă că 52 z1  32 z1  M16  8 .
Deoarece y2  M16  4 sau y2  M16 , rezultă că ecuaţia 5n  3n  y2 nu are soluţii în mulţimea
numerelor naturale.

Clasa a VIII-a

Subiectul I.
a) Fie z  AQMN . Se arată că AQMD  AQNC  zt .

Însă
z MN a  b k  1
   , unde a  AB , b  CD , a  b . Rezultă z
k  1 t
, deci
t CD 2b 2 2


AMNCD  z  t 
2 k  32  t .
 4
A 3a  b 3k  1
Avem și ABNM   , deci
AMNCD a  3b k  3

AABCD  AABNM  AMNCD 


k  33k  1  t  k  32  t  k  1k  3  t
4 4
b) Răspunsul este afirmativ: k 1k  3 t  2016  6  8  42 , de unde k  5, t  42 .

Subiectul II.
a) Prin ridicare la pătrat se obține imediat 3x  y  0 .
2

 
b) Cu teorema cosinusului în ABC , cu m BAC  60o , obținem:

 

60o

B D

C
248
AB  AC
BC  AB2  AC2  AB  AC  , conform punctului a) și încă două inegalități
2
analoage, care, adunate, conduc la BC  BD  CD  AB  AC  AD . Dar condiția a doua din
ipoteză afirmă contrariul, deci inegalitatea se transformă în egalitate, deci tetraedrul este regulat.

Subiectul III.

D` C`

A` B`

F
P
D C
E
O
A B

a) Notăm AB  CD  O. Din proprietățile cubului avem A`C  C`BD ; punctul P este
A`C 2 C`O 2
centrul de greutate în C`BD și CP  , A`P  A`C , PO  , PC` C`O . Dacă
3 3 3 3
notăm AB  a , rezultă A`C  a 3 și A`C` AD` a 2 .
Construim PE  AC, E  AC . În A`AC avem PE || AA` CPE ~ CA`A , de unde obținem
a a 2 2a 2
PE  și CE  . Avem AE  .
3 3 3
În AEP, cu teorema lui Pitagora obținem AP  a .
  
În A`D`C , cu m A`D`C   90o , avem A`D` a , D`C  a 2 , A`C  a 3 . Construim
 
a 3 a 6
D`F  A`C, F  A`C . Cu teorema catetei și cea a înălțimii, obținem A`F  , D`F  .
3 3
2a 3 a 3 a 3
Avem A`P  și împreună cu A`F  duce la FP  . Din D`FP obținem
3 3 3
D`P  a , iar din D`PA cu PA  PD` a și AD` a 2  m APD`  90o , m PAD`  45o .
 

   
Comparând ABC cu APD`, rezultă congruența lor din cazul C.C.
AADE AE a2 a2
b) Din  , obținem AADE  , iar cu AAPD"  , obținem
AADC AC 3 2
  2
cos m  APD`,  ADE   ’

   3

249
Clasa a IX-a

Subiectul I.
Deoarece x  x  x, ecuația se mai scrie x  22x  10x  x  9 sau
2

x2 12x 9  10x (1)


Cum 0  x  1 , deducem că membrul stâng din (1) este un număr din intervalul 9,22 , în timp
ce membrul drept din (1) este un multiplu (întreg) de 10. multipli lui 10 din intervalul 9,22
sunt 10 și 20, prin urmare x  1 sau x  2 . Avem două cazuri:
I. x  1. Ecuația (1) devine x  12x1  0 și are soluția situată în intervalul 0,1 :
2

x  6  37 . Rezultă x1  1  6  37  5  37 .


II. x  2 . Ecuația (1) devine x  12x11  0 și are soluția situată în intervalul 0,1 :
2

x  6  47 . Rezultă x2  2   6  47  4  47 .


Mulțimea soluțiilor este așadar:

S   5  37,4  47 . 
Subiectul II.
1)

M N
c b
ha

B Q D a P C
Fie MNPQ pătratul căutat, unde M  AB , N  AC , P, Q  BC. Fie AD înălțimea din A
și ha lungimea acesteia. Notând cu l a latura acestui pătrat, pentru a construi pătratul este

 și construim poziția lui M pe AB ,


AM MN la
suficient să determinăm l a , căci atunci 
AB BC a
apoi pe a punctului N pe AC . Din asemănări de triunghiuri, avem:
AM l a BM la
 ,  ,
c a c ha
1 1 1 aha 2S
care, prin adunare, dau: 1  la    , de unde la    , unde am notat
a ha  1 1 a  ha a  ha

a ha
cu S aria triunghiului.

250
2S 2S 2aS
2). Continuînd exprimarea lui l a , dacă înlocuim ha  , avem: la   2 . Ținând
a 2S a  2S
a
a
seama de exprimările analoage pentru l b , l c , putem scrie:

l a  lb 
2aS

2bS

 a
 2  

b  2S ab2  2aS  a 2b  2bS


a 2  2S b 2  2S
2S
 a  2S b  2S 
2
 
a 2  2S b 2  2S 
2S a  b2S  ab 2S a  bab sin C  ab 2aba  bsin C  1S
 2
a  2S b2  2S   a 2  2S b2  2S   a 2  2S b2  2S   0 , deci la  lb .
Analog lb  lc și soluția se încheie.

Subiectul III.
Soluţia I. Probăm prin inducție inegalitatea „mai tare”:
1 1 1 4 1
P(n) :   ...   , n ℕ*.
n 1 n  2 3n 3 3n
I. P1 :      1 este adevărată.
1 1 4 1 5
2 3 3 3 6
II. Pn  Pn 1, n ℕ*. Notăm Sn 
1 1 1
  ... .
n 1 n  2 3n
4 1 4 1
Presupunem Sn   și demonstrăm că Sn1   .
3 3n 3 3n  1
1 1 1 1 1 1 2
Avem: Sn1  Sn      Sn    
3n  1 3n  2 3n  3 n  1 3n  1 3n  2 3n  3
4 1 1 1 2 4 1 1 1 1 1
           .
3 3n 3n  1 3n  2 3n  3 3 3n  1 3n  1 3n  2 3n 3n  3
1 1 1 1 1 1
Este suficient să arătăm că    0  , care
3n  1 3n  2 3n 3n  3 3n  13n  3 3n3n  1
este evidentă.
Soluţia a II-a. Membrul stâng al inegalității cerute este o sumă cu 2n termeni, pe care îi grupăm
în n perechi astfel: primul cu ultimul, al doilea cu penultimul ș.a.m.d. Prin urmare:
 1 1  1 1  1 1 
Sn        ...    .
 n  1 3n   n  2 3n  1   2n 2n  1 
1 1 1 1
Suma din prima grupare este cea mai mare, căci putem arăta că    ,
n  1 3n n  k 3n  k  1
k 1,2,...,n.
4n  1 4n  1
Într-adevăr, această inegalitate este echivalentă cu  sau cu
3nn  1 n  k 3n  k  1
n  k 3n  k 1  3nn 1  2n  k k 1  0 , evidentă.
 1 1  4n  1 4n  4 4
Atunci: S n  n     și soluția se încheie.
 n  1 3n  3n  3 3n  3 3

Clasa a X-a

251
Subiectul I.
Un calcul simplu arată că: 29 2  41   
5
2  1 ; 29 2  41   5
2  1 . Egalitatea din enunț

 2  1   2 1
5 5
devine: n n 2 (1)
Funcția g : ℝ  0,  , g x   2  1 x
este strict crescătoare, funcția h : ℝ  0,  ,
 
hx  2  1 este strict descrescătoare, prin urmare funcția f : ℝ  0,  ,
x

f x  gx  hx este strict crescătoare, deci injectivă.


5
Egalitatea (1) se scrie f    2  f 1 , prin urmare  1, adică n  5 .
5
n n

Subiectul II.
Pentru n  0 ipoteza devine f 2 0  f 0, deci f 00,1.
Cazul f 0  1 . Pentru n  1, avem f 0 f 1  f 1 f 0  2 f 1  2 f 1  2 f 1 , deci f 1
este un număr real arbitrar. Să notăm f 1  a ℝ.
Pentru n  2 , avem
a2
f 0 f 2  f 1 f 1  f 2 f 0  4 f 2  2 f 2  a 2  4 f 2  f 2  .
2
Pentru n  3 , avem, f 0 f 3  f 1 f 2  f 2 f 1  f 3 f 0  8 f 3 
a3 a3
 2 f 3  2   8 f 3  f 3  .
2 6
an
Demonstrăm prin inducție că f n  , n ℕ*.
n!
Presupunem egalitatea adevărată pentru k 1,2,...,n 1 și o demonstrăm pentru k  n . Ipoteza
n1
din enunț se scrie: f 0 f n  f n f 0   f n f n  k   2 n f n 
k 1

   
n1 k n k n n1
  2n  2 f n     2n  2 f n 
a a a n!

k 1 k! n  k ! n! k 1 k!n  k !
     
n n1 n
an
  Cnk  2n  2 f n  2  2  2n  2 f n  f n 
a a n
și demonstrația prin
n! k 1 n! n!
inducție se încheie.
Cazul f 0  0 . Pentru n  1, avem f 0 f 1  f 1 f 0  2 f 1  2 f 1  2 f 1 
0  2 f 1  f 1  0 .
Demonstrăm prin inducție că f n  0, n ℕ*.
Presupunem f 1  f 2  ...  f n 1  0 și demonstrăm că f n  0 .
n
Ipoteza  f n f n  k   2 f n devine 0  2 f n  f n  0 și demonstrația prin inducție
k 1
n n

se încheie.
an
Concluzie. Funcțiile căutate sunt f n  , cu a ℝ* arbitrar, funcția constantă nulă f n  0 ,
n!
precum și funcția care se obține pentru a  0 în cazul f 0  1 , și anume:

252
1, n  0
f n  
0, n  1

Subiectul III.
  0,  , f t    a 1 , g t   at  at 1 și vom ține cont în continuare că
t
Fie funcțiile f , g :
funcția f este crescătoare pentru a  0,  , iar funcția g , este descrescătoare pentru a 0,1
 f  x  g  y 

și crescătoare pentru a 1,  . Sistemul se mai scrie  f  y   g  z  .

 f  z   g  x
I. a  1 , avem 2x  2y  2z  2 , de unde x  y  1 .
II. a  1 ; f și g crescătoare
Demonstrăm întâi că x  y .
Presupunem că x  y , atunci x  y sau y  x .
Dacă x  y , cum a  1 atunci f  x   f  y  , adică g  y   g  z  , deci y  z şi imediat
f  y   f  z  , adică g  z   g  x  , deci z  x şi prin urmare x  y  z  x ceea este o
contradicţie. Asemănător se analizează cazul y  x .
Deci x  y şi atunci prima ecuaţie a sistemului devine a  1  a x  a x1 cu soluţia x  1.
x

(ecuaţia se mai scrie a  1  a x1 a  1 sau a  1  a x1 ) Înlocuind în ecuaţia a treia a


x x1

sistemului se obţine z  1. Deci soluţia sistemului este x  y  z  1 .


III. a 0,1 ; f crescătoare și g descrescătoare
Avem x  1 sau x  1 și observăm că f 1  g 1 .
Dacă x  1, atunci f  z   g  x   g 1  f 1 , deci z  1. Avem f  y   g  z   g 1  f 1 ,
deci y  1. Avem f  x   g  y   g 1  f 1 , deci x  1 și prin urmare x  1 . Atunci imediat
x  y  z  1.
Analog dacă x  1

Clasa a XI-a

Subiectul I.
1 1
Se știe cășirul de termen general cn  1   ...  ln n este convergent către constanta lui
2 n
Euler c  0,577 .
De asemenea, utilizând criteriul Cauchy - d`Alembert ( criteriul radical ), avem:
n  1!
lim
n
n! n!
 lim n n  lim
n  1n1  n  1
 lim
n

  .
n n  1
n n n n n n!   e
nn

253
n n
n! n! 1 n n! 1 1 1
Atunci, limita din enunț devine: L  lim  lim  lim   c   c1 .
n 1
1 ...
2
1
n
n ecn ln n n e cn n e e e
e

Subiectul II.
Dacă înmulțim la stânga și la dreapta cu A ipoteza A2015  B 2015  C 2015  O2 , obținem
A2017  ABB 2014A  AC 2014CA  O2 , adică A2017  O2 .
Trecând la determinanți, rezultă detA  0 și folosind ecuația caracteristică deducem că
A2  tr A  A . De aici, prin inducție, rezultă ușor: An  trAn1  A , prin urmare:
A2017  trA2016  A  O2 .
Atunci trA  0 sau A  O2 , ambele posibilități ducând la A2  O2 .
Analog B 2  C 2  O2 .
 0 1
Un exemplu de matrice nenule care verifică ipotezele îl constituie A  B  C    .
 0 0 

Subiectul III.
ex
Considerăm funcția f : 0,   ℝ, f x   x.
1 ex
e x 1  e x   e 2 x e2x  e x  1
Derivata acestei funcții este: f `x  1    0, x  0,  . Așadar
1  e x 2 1  e x 2
funcția f este descrescătoare, prin urmare f x  f 0, x 0,  , adică
ex
  x, x  0,  .
1
1 e x
2
Dacă scriem această inegalitate pentru x  x1 , x2 ,...,xn  și sumăm inegalitățile obținute, rezultă
inegalitatea din enunț.
Cazul de egalitate are loc pentru x1  x2  ...  xn  0 .

Clasa a XII-a

Subiectul I.
Folosind formulele lui Viète, avem:
2
 4 
x  x  x  x    xk   2  xk xl  32  2  5  1  0 , ceea ce arată că nu toate
2
1
2
2
2
3
2
4
 k 1  1k l 4
rădăcinile sunt reale.
Notând cu f polinomul din membrul stâng al ecuației, avem f 0  2  0 și lim f x   ,
x

prin urmare avem o rădăcină reală x1  0,  . Deoarece numărul rădăcinilor complexe nereale
este par, înseamnă că ecuația are două rădăcini complexe conjugate, să zicem x3 , x4  ℂ\ℝ și
două rădăcini reale x1 , x2 cu x1  0 .
 
Dar x3  x4 și cum x1 x2 x3 x4  x1 x2 x3 x3  x1 x2 x3  2 , rezultă x2  0 . În planul complex
2

putem avea una dintre situațiile


254
M3(x3) M3(x3)

M2(x2) M1(x1) M2(x2) M1(x1)

M4(x4) M4(x4)

Semiplanul determinat de axa imaginară, care conține trei dintre punctele M k xk , k  1,4 , este
un semiplan deschis, ceea ce revine la a spune că punctele M 3 x3 , M 4 x4  nu aparțin axei
imaginare, adică Rex3   Rex4   0 . Într-adevăr, dacă am presupune prin absurd că
Rex3   Rex4   0 , atunci x3  i ,   0 și, scriind că f x3   0 , obținem
 4  3 3i  5 2  mi  2  0 , deci  3 3  m  0  3 2  m  0  m  3 2  0 ,
contradicție cu ipoteza m  0 .

Subiectul II.
1

 x
1 n

 f  n dx  n f t dt , prin urmare, notând cu L


x
Prin schimbarea de variabilă t  , avem
n 0 0
1
n
1
n  f t dt
limita de calculat, avem: L  lim n  f t dt  lim
3 0
3
(1)
n n
1
 
0

n
x

 f t dt f x  f `x
Calculăm atunci : L1  lim 0
3
 (l`Hospital)  lim 2
 (l`Hospital)  lim 
x0 x x0 3x x 0 6x
f `x  f `0 1
 f ”0
1
 lim (2)
6 x0 x0 6
1
Luând șirul xn   0 și folosind definiția cu șiruri a limitelor de funcții, rezultă, din (1) și (2):
n
f ”0.
1
L  L1 
6

Subiectul III.

Fie f  X    an X n o serie formală cu proprietatea din enunț.
n0

 n 

n

Ipoteza se scrie   n   n   an 2 X 
 n
a X a X (1)
 n0  n0  n0

255
Coeficientul lui X n în membrul stâng din (1) este a a
i  j n
i j , iar coeficientul lui X n în membrul

drept din (1) este 2 an . Așadar, trebuie să determinăm șirul de numere complexe an n0 care
n

verifică recurența: a a
i  j n
i j  2 n an , n ℕ (2)

Pentru n  0 în (2), obținem a02  a0 , deci a0  1 sau a0  0 .


Cazul a0  1 . Pentru n  1 în (2), obținem a0 a1  a1a0  2a1  2a1  2a1 , deci a1 nu poate fi
determinat. Notăm a1   ℂ.
2
Pentru n  2 în (2), avem a0 a2  a1a1  a2 a0  4a2  2a2  2  4a2  a2  .
2
Pentru n  3 în (2), avem
3 3
a0 a3  a1a2  a2 a1  a3 a0  8a3  2a3  2   8a3  a3  .
2 6
n
Demonstrăm prin inducție că an  , n ℕ*. (3)
n!
Presupunem (3) adevărat până la n  1 și o probăm pentru n . Scriem (2) astfel:
i ni
 
n1 n1
a0 an  an a0  ai ani  2n an     2n  2 an 
i 1 i 1 i! n  i!
n  n1 n n
       
n1 n
  2n  2 an   Cni  2n  2 an 
n!
 2  2  2n  2 an 
n! i1 i!n  i ! n! i1 n!
n 
n
 an  . Seria căutată este f  X    n!  X n
,  ℂ*, arbitrar. (4)
n! n0
Pentru   0 avem seria constantă f X   1.
Cazul a0  0 . Recursiv, se obține, pe baza lui (2), a1  a2  ...  an  0 , deci seria căutată este
seria formală nulă f X   0 .

Editia a XXIII-a, 1 aprilie 2017

Clasa a V-a

Subiectul I
Iniţial avem 2017 numere şi anume 1, 2, 3,..., 2017 a căror sumă este
S  1 2  3  ...  2017 .

După pasul 1 avem 2016 numere a căror sumă este S1 iar S  S1  x1  y1  r1 unde x1 şi y1
sunt numerele şterse iar r1 este restul împărţirii lui x1  y1 la 251. Atunci S  S1  M 251 .

La pasul 2 avem 2015 numere a căror sumă este S2 iar S1  S2  x2  y2  r2 unde x2 şi y2


sunt numerele şterse iar r2 este restul împărţirii lui x2  y2 la 251. Atunci S1  S2  M251 .

256
Ȋn final la pasul 2015 avem două numere a căror sumă este S2015 iar S2014  S2015  M251 . Cele
două numere rămase sunt 1007 şi  .

Deoarece 1007 nu poate fi restul împărţirii unui număr natural la 251 atunci după pasul
2014 avem numerele 1007, a, b şi restul împărţirii lui a+b la 251 este  , deci
 0,1,2,...,250 .

Din S  S1  M 251 , S1  S2  M251 , ..., S2014  S2015  M251 prin adunare obţinem
S  S2015  M 251 .
2017  2018
S  1  2  3  ...  2017   2017 1009  203515381408  25  45
2
1007  4  251  3
8 1408  25  45   4  251  3     M 251
  42

Subiectul II
D = Ȋ C + R , R < Ȋ
2  3201751017   20  22 10  32015  51015  20  22  32015  51015 10  32015  51015
Câtul este 20, restul este 10  32015  51015  22  32015  51015 .

Subiectul III


xabcde  4  abcdex, x00000  abcde  4 10abcde  x , 
x 100000  4  abcde  40 1 , 99996x  39abcde , 99996  39  2564, 2564x  abcde
x  4  abcde  2564  4  10256, x  5  abcde  2564  5  12820
x  6  abcde  2564  6  15384, x  7  abcde  2564  7  17948
x  8  abcde  2564  8  20512, x  9  abcde  2564  9  23076

Clasa a VI-a

Subiectul I
2n2 1 2n2 1

n5  n n  n 1 n 1  n2 1
(1) Numărul n  n 1 n  1 este divizibil cu 3 şi deci numitorul fracţiei este divizibil cu 3.
 
(2) Prin calcul, pentru n  3k ,3k 1,3k  2 / k  N  se constată că numărătorul fracţiei, 2n2 1,
nu este divizibil cu 3.
 
(3) La fel pentru n  5k ,5k 1,5k  2,5k  3,5k  4 / k  N  avem că n  n 1 n  1 n2  1  
este divizibil cu 5.

257
(4) Ultima cifră a lui 2n2 1 este diferită de 0 şi 5 deci 2n2 1 nu este divizibil cu 5. Din (1),
(2), (3), (4)  2n2 1 nu este divizibil cu 15 şi n5  n este divizibil cu 15
2n2 1
 se scrie ca o fracţie zecimală periodică mixtă.
n5  n

Subiectul II
1122  2  31117
Din 6c  17b 11a , obţinem 17  c  b  11 c  a   17/c-a şi 11/c-b  1117 / n (1)
Dintre numerele a, b, c cel puţin două dintre ele au aceeaşi paritate şi atunci diferenţa lor este
număr par  2/n (2)
Dacă două din numerele a, b, c dau acelaşi rest la împărţirea cu 3, atunci diferenţa lor este
divizibilă cu 3  3/n (3)
Dacă a, b, c dau resturi diferite la împărţirea cu 3
a  3k, b  3 p 1, c  3l  2, k, p, l  N atunci a + b + c este un număr divizibil cu 3
Deci şi în acest caz 3/n (4)
Din (3), (4) 3/ n  din (1),(2) 2  3 1117 / n 1122 / n

Subiectul III

  
Putem considera că m ABC  m ACB . 
Construim MN  AB, N  AB şi MP  AC, P  AC .
Cum M aparţine mediatoarei laturii BC şi bisectoarei unghiului BAC  MB  MC şi
MN  MP
 MNB  MCP (cazul IC)  MBN  MCP  MCA  ABM  180 (1)
Folosind ABM şi ACM avem că :
  
ABM  BMA  BAM  MAC  ACM  AMC  180  180  360 
1
ABM  ACM  BAC  BMC  360 180  BAC  BMC  360  BAC  BMC  180

Clasa a VII-a

Subiectul I

258
Notăm: a = BC, b = CA, c = AB
x = BA’, y = A’C
Din ipoteză avem : x + c = y + b (1)
Dar, evident : x + y = a (2)
Rezolvând sistemul dat de (1) şi (2), obţinem:
a bc a b  c
x  p  c, y   p b
2 2
a bc
unde p  . Atunci:
2
A' B x p  c
  (3)
A' C y p  b
B 'C p  a
Analog vom avea :  (4)
B' A p c
C ' A p b
 (5)
C'B pa
A' B B ' C C ' A
Ȋnmulţind egalităţile (3), (4), (5) obţinem:   1
A' C B ' A C ' B
De unde conform reciprocei teoremei lui Ceva, deducem că dreptele AA’, BB’, CC’ sunt
concurente.
Observaţie: Punctele A’, B’, C’ sunt izotomicele punctelor de contact ale cercului înscris cu
laturile triunghiului.

Subiectul II
1. Vom demonstra că n1,2 .
Prin înlocuire, pentru n  1 avem 3  7  8  18  4  5  9 , iar pentru n  2 , avem
32  72  82  122  42  52  92
Demonstrăm în continuare că 4n  5n  9n  3n  7n  8n , n  3 . Pentru n  3 , avem
43  53  93  64 125  729  918  882  27  343  512  33  73  83 .
Pentru n  4 , avem 4n  5n  3n și vom arăta că 9n  8n  7n , n  4 . Avem
9n  8n  8   9  
n n
9  7 8 
n n n
 1       1  1 . Deoarece pentru n  4 avem
 7    8  
n
7 
 8   9    8   9    8   9  
n n 4 4 4 4

 7    8  1   7    8  1 atunci este de ajuns să arătăm că  7    8  1  1 . Ultima


              
9 8
4 4
relație este echivalentă cu  1. Dar
74
 
94  84  9  89  8 92  82  17 145  2465  2401  74 . Prin urmare din 9n  8n  7n și
4n  5n  3n , rezultă că 4n  5n  9n  3n  7n  8n , n  4 .
2. Vom folosi în rezolvare următoarele două rezultate cunoscute:
(1)  a  b  M a  bn
n

(2) orice pătrat perfect este de forma M 5 sau M5  1 sau M5  4

259
Avem a  4n  5n  9n  5 1  5n  10 1  M5  2  1 .
n n n

Dacă n este par, atunci a  M5  2 , iar n este par, atunci a  M5  2  M5  3 , deci a nu este
pătrat perfect.

Subiectul III

Construim DT  AC, T  AC şi CS  AB, S  AB


     
CSB : m BSC  90 , m SBC  80  m SCB  10
BCD echilateral  DC  BC si m  DCB   60

 m  DCT   10

Considerăm DCT si BSC


DC  BC 
 IU
DTC  BSC  90   DCT  BSC  SC  TC

DCT  SCB  10 

Ȋn

    AC AC
T 30
ASC : m ASC  90 , m SAC  30  SC   TC 
2 2

    
Ȋn DAC :  DT  mediana şi înălţime  m DCA  m DAC  10 şi m DAB  20 .

Ȋn EBC : m  ECB   60 , m  EBC   80  m  BEC   40

BEC este exterior AED  m  BEC   m  EAD  m  EDA  m  EAD  m  EDA  20


 EAD este isoscel  ED  EA

Clasa a VIII-a

Subiectul I
Vom folosi în rezolvare următorul rezultat cunoscut:  a  b  M a  bn , unde M a înseamnă
n

multiplu de a.
Avem 2017n   2000  17  M 20  17n , şi atunci
n

2017n 17 M20 17n 17


  p

17 17n1 1
, deci

2017n 17
 
17n1 1
 , Deoarece
20 20 20 20 20
n1 2017n 17 17n1 1
17n1 1  16 1 1  M4 pentru orice n   , atunci    .
20 5
n1 2017n 17 2n1 1
Deoarece 17n1 1  15 1 1  M5  2n1 1, atunci   
20 5

260
2n1 1
Pentru n  1  4k , avem 2n1  24k  16k  M5  1 şi atunci  .
5
2n1 1 M5  1
Pentru n  1  4k  1, 2n1  2  24k  2 16k  2M5  1 şi atunci   .
5 5
2n1 1 M5  3
Pentru n  1  4k  2 , 2n1  4  24k  4 16k  4M5  1 şi atunci   .
5 5
2n1 1 M5  2
Pentru n  1  4k  3 , 2n1  8  24k  8 16k  3M5  1 şi atunci   .
5 5
Aşadar A  n  4k  1 k  N  4  1 .

Subiectul II

Notăm cu a latura cubului şi cu x distanţa de la punctul C la planul (AC’M).


Pentru a determina valoarea lui x scriem volumul tetraedrului cu varfurile C, A, C’ M în două
moduri : prima oară îl gândim cu varful C’ şi baza triunghiul MAC, a doua oară îl gândim cu
vârful C şi baza triunghiul MAC’.

Evident avem D 'T  DB  M  si


MB BT 1 MB 1 BD a 2
     MB   .
MD DD ' 3 MB  BD 3 2 2
Atunci aria triunghiului ACM este:
AC  MO AC  MB  BO a 2  a 2
SACM     a2  36  a  6
2 2 2
S  CC ' a2  a a3
Volumul tetraedrului C’MAC este: V  ACM   (1)
3 3 3
Calculăm aria triunghiului AMC’ cu formula lui Heron, deci vom calcula mai întâi laturile
acestuia.
2
 a 2  a 10
 
2
Avem AC '  a 3; MA  MO  OA 2 2
a 2    
 2  2
2
 a 10  a 14
MC '  MC  CC '  MA  CC '  
2 2 2
  a 
2 2

 2  2

261
a2 26
Folosind formula lui Heron obţinem: S AMC '  , prin urmare volumul tetraedrului CAMC’
4
S AMC '  x a2 x 26
este : V   (2)
3 12
4a 24
Din (1) şi (2) rezultă x   .
26 26

Subiectul III
Din inegalitatea dintre media geometrică şi cea armonică, avem:
ab ab  a2  ab  b2  1 2ab  a2  ab  b2  2ab
  2   2 2 şi analoagele:
a  ab  b
2 2
a  ab  b
2 2
a  ab  b ab  a  ab  b
2 2 2
a b
bc 2bc ca 2ca
 2 2; 2  2 2 . Ȋnsumând cele trei relaţii obţinem inegalitatea:
b  bc  c b  c
2 2
c  ca  a c  a
2

ab bc ca 2ab 2bc 2ca


 2  2  2 2 2 2 2 2
a  ab  b
2 2
b  bc  c 2
c  ca  a a  b b  c c  a
2

 a  b 2   b  c  2   c  a  2  3  4  a  b  c  2 2  a2  b2  c2   4  ab  bc  ca  ab  bc  ca
 3   3  4 1
a 2  b2 b2  c 2 c2  a2 2a  b  c
2 2 2
 a 2
b c
2 2
 a 2  b2  c 2
de unde concluzia.
Clasa a IX-a

Subiectul I
Cu inegalitatea Cauchy- Buniakovski- Schwarz

 x z  1 yz 
   x  y    x  z  z  y   z  x  y 
2
2 2 2 2 2 2 2 2 2
2 (1)
 
Analog
 x  y  z
2 2 2
 x2   x2  y  z  (2)
2

 y z  x
2 2 2
 y2   y2  x  z  (3)
2

Ȋnmulţind membru cu membru relaţiile (1), (2), (3) obţinem


x  y 2   y 2  z 2   z 2  x2   x2 y 2 z 2  x  y   y  z   z  x 
2 2 2 2 2 2 2

De unde rezultă inegalitatea cerută.

Subiectul II

Aria triunghiului A’B’C’ este minimă dacă şi


numai dacă suma ariilor triunghiurilor AB’C’,
BA’C’, CA’B’ este maximă.

Notam cu S (x) suma ariilor acestor


triunghiuri şi atunci avem:

262
x b  x  sin A x c  x  sin B x  a  x  sin C
S  x    
2 2 2

ax b  x  bx  c  x  cx  a  x    a  b  c  x2   ab  bc  ca  x
  
4R 4R 4R 4R
S(x) este o funcţie de gradul II în variabila x.
ab  bc  ca
Punctul de maxim al acestei funcţii este: x0  (1)
2 a  b  c
Ȋn contextul problemei trebuie să avem 0  x  min a, b, c . Ţinând seama de (1), deducem că
problema are soluţie dacă şi numai dacă x0  min a, b, c
Unde x0 este dat de (1). Ȋn acest caz soluţia căutată este x0 .

Subiectul III
Folosind inegalitatea dintre media aritmetică şi cea geometrică , putem scrie:
1 y 1 z  2 1 y 1 z  (egalitate pentru y  z )
x 1
Ţinând cont de relaţia de condiţionare, putem scrie  2 (egalitate pentru y  z )
x  yz
y 1 z 1
Analog obţinem:  2 (egalitate pentru z  x ),  2 (egalitate pentru x  y )
y  zx z  xy
Adunând membru cu membru ultimele trei inegalităţi , obţinem inegalitatea din enunţul
1
problemei. Avem egalitate dacă şi numai dacă x  y  z  .
3

Clasa a X-a

Subiectul I
a) Fie r  z1  z2  z1  z2 , precum şi punctele în plan A z1  , B  z2  , C  z1  z2  , care
aparţin cercului de centru O (originea planului) şi raza r.

Vectorial avem , corespunzător:


OA  OB  OC
Rezultă că patrulaterul OACB
este un paralelogram în care
OA= OB, prin urmare este un
romb.

Atunci OAC şi OBC sunt


echilaterale, ceea ce înseamnă
că:
  2
AOB   
3 3 3

263
 2 2 
Prin urmare z2  z1  cos  i sin  , de unde z13  z23
 3 3 
b) Conform cu a) avem z33    z4  , de unde , ridicând la pătrat : z36  z46 .
3

Subiectul II
x2  5x
5 x

 1  3x 4 x  x2  5x  3x  3x 4 x  x2  4x  3x  x
2 2 2
Ecuaţia se poate scrie: 3x x
3
Definim funcţia f : R  R, f  x   3x  x .
Fiind sumă de două funcţii strict crescătoare, funcţia f este la rândul ei strict crescătoare, deci
este injectivă.
 
Cum ecuaţia este de fapt f x2  4x  f  x  , din injectivitate rezultă ecuaţia: x2  4x  x cu
soluţiile 0 şi 5.
Deci x0,5 .

Subiectul III
a). Calcul simplu
b). Luăm a  sin2 x, b  cos2 x şi notăm sn  an  bn , p  ab .
Relaţia de la punctul a) devine sn2  sn1  psn . Din aproape în aproape avem:
s1  1; s2  1 2 p; s3  s2  ps1  1 3 p; s4  s3  ps2  1 4 p  2 p2
s5  s4  ps3  1 5 p  5 p2 ; s6  s5  ps4  1 6 p  9 p2  2 p3
s7  s6  ps5  1 7 p 14 p2  7 p3  1 7 p 1  p  .
2

 
2
Ultima egalitate se scrie echivalent sin14 x  cos14 x  1  7sin 2 x cos2 x 1  sin 2 x cos2 x , ceea ce
trebuia demonstrat.
q
c). Notând q  sin 2 2x , avem p  sin2 x cos2 x  şi cos2 2x  1  q2 .
4
Ecuaţia dată devine :
2
7q  q  169
1 1   1 q3  64  7q  4  q2  169 1 q3 
4  4 64
64 112q  56q  7q3  169  507q  507q2 169q3 
2

162q3  451q2  395q 105  0   2q 1 81q2 185q 105  0 (1)


1 185  205 185  205
Din (1) rezultă q1  , q2  , q3  . Avem q  sin2 2x  1 . Deoarece
2 162 162
1
q3  q2  1 , reţinem doar q1  .
2
 
Aşadar sin2 2x  1  cos4x  0  4x   2k 1 , k   x   2k 1 , k  Z . Cum
2 8
   3 
x   0,  , rezultă x   ,  .
 2 8 8 

264
Clasa a XI-a

Subiectul I
5 pn  2  xn  pn  2xn  3 pn
Notând  xn   pn  N , obţinem xn1  
3 pn  4  xn  pn  4xn  pn
.

Observăm că:
1) Dacă x1   0,1 , avem p1  0 şi x2  .
1
2
1 1
Rezultă p2  0 şi x3  . Prin inducţie obţinem xn  , n  2 .
2 2
1 7 pn 
2) Dacă x1  1  p1  1 şi vom scrie termenul general sub forma xn1  1  
2 4xn  pn 
Observăm că pn1  xn  pn 1  3 pn  4xn  pn  3 pn şi atunci
7 pn 7 pn 7 5 pn  2 5
  ;  xn1  .
3 pn  4 4xn  pn 3 3 pn  4 3
2  pn 1 5
Ultima inegalitate se poate scrie sub forma 1+  xn1 
3 pn  4 3
2xn  3 1  7 
Ceea ce implică pn1  1, n  N   xn1   1   (1)
4xn 1 2  4xn 1 
Considerăm subşirurile  x2n  şi  x2n1  . Din (1)
sgn  x2n2  x2n   sgn  x2n1  x2n1   sgn  x2n  x2n2   ...  sgn  x4  x2   sgn  x1  x3 
2x2  3 16x1  3
Deoarece x3   , rezultă că
4x2 1 4x1  13
 3  57 
sgn  x1  x3   sgn  4x12  3x1  3  sgn  x1   ( semn constant dat de compararea
 8 
3  57
valorilor lui x1 şi .
8
Deducem că şirurile  x2n1  şi  x2n  , n  N  sunt monotone, de monotonie diferită. Ele sunt
şi mărginite deoarece pn1  1 şi anume xn1 1,2 .
Prin urmare ele sunt convergente , cu limitele l1  lim x2n1 şi l2  lim x2n .
n n
2l2  3 2l  3
Trecând la limită în (1) obţinem l1  şi l2  1
4l2 1 4l1 1
4l1l2  l1  2l2  3 def
Adică avem sistemul:   l1  l2  l  4l 2  3l  3  0
4l1l2  l2  2l1  3
3  57
Ultima ecuaţie are rădăcina pozitivă l   lim xn .
8 n

Subiectul II

265
 
a) Prin inducţie obţinem an  cos n1
, bn  sin , deci lim an  1,lim bn  0 .
2 2n1 n n

Altfel: an2  bn2  1, deci  an n1 şi  bn n1 sunt şiruri mărginite.


Prin inducţie rezultă că  an n1 este crescător iar  bn n1 este descrescător deci şirurile
1 a 1 a
sunt convergente. Notând lim an  a,lim bn  b rezultă că a  ,b  , de unde
n n 2 2
a  1, b  0 .
b) Luând k  kn  2n2 avem :
2n2
   
 cos n1 sin n1 
2 2  cos 2 sin 2 
Ank       I2 .
  sin  cos     sin 2 cos 2 
 
 2n1 2n1 

 sin
c) Avem kn  2n2 , prin urmare: lim knbn  lim 2n2 sin  lim 2n1  2  2 .
n n 2 n1 n 
2n1
Subiectul III

y
f

M(x0,f(x0))
O
N(x0 -1,0) x

Ecuaţia tangentei în M la graficul funcţiei este :


y  f  x0   f '  x0  x  x0  (1)
Intersecţia dreptei (1) cu axa Ox se obţine pentru y = 0, deci:
 f  x0   xf '  x0   x0 f '  x0  (2)
f '  x0 
Din ipotezâ, x  x0 1 verifică (2), adică:  f  x0   f  x0  x0 1  x0 f  x0  
'
1'
f  x0 
Cum x0 este arbitrar, vom avea :

266
f '  x
 1, x  R   ln f  x   x' , x  R  a  R cu
'

f  x
ln f  x   x  a, x  R  f  x   exa , x  R
Notând k  ea , obţinem funcţiile căutate: f  x   kex , k  0 .

Clasa a XII-a
Subiectul I
1
Fie In   n
1 x  ln  2  x  dx .
0
1 1
Pentru orice x 0,1  ln  x  2dx  In  2 ln  2  x  dx .
 n

0 0
1
Conform criteriului cleştelui , rezultă: lim In  ln  x  2dx

n
0
Folosind o schimbare de variabilă şi apoi o integrare prin părţi, avem:
1 3
3 31
0 ln  x  2dx  2 ln tdt  t ln t 2  2 t  tdt  3ln3  2ln 2 1 . Aşadar lim
n
In  3ln3  2ln 2 1 .

Subiectul II
Procedăm prin dublă incluziune
"  " Fie  K . Luăm h  X    K  X  , deci h  0 si h    0 .
"  " Fie   L, asfel incat h  K  X , h  0, cu h    0 .
Atunci există un factor ireductibil g  K  x al lui h, pentru care g    0 . Conform ipotezei,
polinomul g rămâne ireductibil şi în inelul L  X  .
Ȋnsă , în inelul L  X  polinomul g se divide cu X   şi fiind ireductibil, vom avea g X  .
Cum g  K  X  , deducem că   K .

Subiectul III
 
Folosim inegalitatea sint  t , valabilă pentru orice t  0,  .
 2
Notând cu I integrala din membrul stâng, avem:
 

2
    2
  2 2
I    sin   sin x   sin  cos x  dx     sin x  cos x dx    cos x  sin x  2  .
0 2   0 2  4
0
4

EDITIA a XXIV-a, 4-6 mai 2018


Clasa a V-a
Subiectul I
dcba  2  abcd  3068 este număr par  a este cifră pară, a  0 (1)

267
Dacă a  4  2abcd  3086  2  4000  3068  11068  dcba (2)
Din (1) şi (2)  a  2
dcb2  2  2bcd  3068 10dcb  2  4000  2bcd  3086
10dcb  2bcd  7066
Atunci ultima cifră a numărului 2bcd este 4  d  2 sau d  7
1) Dacă d  2 12  2cb  2  bc2  7066  2000 10cb  20bc  4  7066
10cb  20bc  5070 imposibil !
2) Dacă d  7 10  7cb  2  bc7  7066  7000 10cb  20bc 14  7066
10cb  20bc  80  cb  2bc  8 10c  b  20b  2c  8
8c  19b  8  b  0, c  1
deci abcd  2017

Subiectul II
Presupunem că cel mult 9 dintre numerele considerate sunt egale. Atunci avem cel mult 9
de 1, cel mult 9 de 2,..., cel mult 9 de 100.
100 101
a1  a2  ...  a900  9 1 9  2  ...  9 100  9   9  50 101  45450
2
Contradicţie cu a1  a2  ...  a900  45449 .

Subiectul III

 n  8c1  5  n  3  8c1  8  n3 8


  
 n  9c2  6   n  3  9c2  9   n  3 9  n  3 8,9,10  360
n  10c  7 n  3  10c  10 n  3 10
 3  3 
282
n  3  360k, n  360k  3, k  N  , 360k  3  99999  360k  100002  k  277 ,
360
k maxim este 277  n  360  277  3  99717

Clasa a VI-a
Subiectul I
Dacă două din cele trei puteri sunt egale sau dacă dau acelaşi rest la împărţirea cu 5, atunci
diferenţa lor este divizibilă cu 5.
Dacă cele trei puteri sunt distincte şi dau resturi diferite la împărţirea la 5, atunci ele aparţin
mulţimii
5k 1,5k  2,5k  3,5k  4 / k  N .
Observăm că oricum am alege două numere din această mulţime, există două a căror sumă este
divizibilă cu 5.

Subiectul II
m q
Din egalitatea   m  m 12  q  n 1 (1)
n 1 m 12
Dacă m este impar, din (1) rezultă q  n  1 impar şi deci n = 2.

268
Obţinem m  m 12  3q  3/ m sau 3/ m 12  3/ m
Cum m este prim, avem deci m = 3. Atunci din (1) obţinem q = 15 care nu este prim.. Deci m
este par şi prim adică m = 2.
Relaţia (1) devine 22  7  q  n 1  7 / q sau 7 / n 1 .
Dacă 7/n+1, cum 22 şi q sunt prime între ele obţinem că 22 / n  1 şi deci q = 1 care nu este
număr prim. Avem deci 7/q, adică q=7 şi n=3.
Imediat obţinem p = 5.
Deci m = 2, n = 3, p = 5, q = 7 şi aceste numere verifică condiţiile problemei.

Subiectul III
a) Se vede imediat că G este ortocentrul triunghiului FCE de unde rezulta FG  CE .
ABE  DBE  ABD  90  BDC  ABD 
b)   ABE  ADC (1)
ADC  ADB  BDC  90  ABD  BDC 

AEB  AEC  BDC  90  ACE  BEC 


  AEB  ACD (2)
ACD  DCE  ACE  90  BEC  ACE 
Din (1) , (2) si CD  BE  ACD  AEB  AC  AE si AB  AD .
Obţinem că ABD si ACE sunt dreptunghice isoscele
 ABD  45 si ACE  45  CBH  135  ABC si BCH  135  ACB (3)

 
(3)
Ȋn HBC, BHC  CBH  HCB  180  BHC  270  ABC  ACB  180

 
 BHC  270  180  BAC  180  BHC  BAC  90

Clasa a VII-a
Subiectul I

269
Pentru x  0 , avem y 2  2 , deci y  N , iar pentru x  1, y  7 . Presupunem în continuare că
x  2 şi observăm că y este număr natural impar. Vom demonstra că ecuaţia nu are soluţii
pentru x  2 .
Distingem două cazuri după paritatea lui x .
a) Dacă x este număr natural impar, x  3 x  2z  1 , z  N  şi atunci ecuaţia devine
292 x1  202 z 1  y 2 . Deoarece 292 x  1(mod8) , atunci 292 x 1  5mod8 . Deoarece
202 z 1  0mod8 , avem 292 x1  202 z 1  5mod8 , iar y 2  1mod8 . (Sau: deoarece 292𝑥 =
𝑀8 + 1, atunci 292𝑥+1 = 𝑀8 + 5. Cum 202𝑧+1 = 𝑀8 , avem 292𝑥+1 + 202𝑧+1 = 𝑀8 + 5, iar
𝑦 2 = 𝑀8 + 1.)
b) Dacă x este număr natural par, x  2z , z  N  , atunci ecuaţia devine 292 x  202 z  y 2 , adică
y  20 y  20   29 .
z z 2z

Fie d  y  20 , y  20  c.m.m.d.c. care evident este număr impar. Din d y  20  şi


z z z

d y  20 , rezultă d y  20  y  20  , adică d 2  20 . Din d y  20 , d y  20  şi


z z z z z z

y  20 y  20   29 , rezultă că d 29 , adică d 29 . Din d 2  20 , d 29 rezultă că


z z 2z 2 2z z z z

d  1 şi atunci din y  20 y  20   29 avem y  20  1 şi y  20  29 , de unde prin


z z 2z z z 2z

scădere, obţinem ecuaţia 2  20z  1  292 z , care nu are soluţii pentru că


292 z  29  29z  29  20z  2  20z  1.
În concluzie ecuaţia are doar soluţia x  1, y  7

Subiectul II

p1  p ABD
1  bc 2 ac
  

 c  
c
2  b  c a  c  2 b  c 
a  2 1 b  c    
1 1 bc 2 2 bc2
S1  aria  ABD  AD  AB  sin 45   c 
2 2 bc 2 2 b  c 

Deci
1 p1 a 
 
 
2 1 b  c
r1 S1 bc

Deci
1 1 p1 a 
  
 
2 1 b  c a 

 
2 1 c  b

2 b  c  1 1
 2  .
r1 r2 S1 bc bc bc c b

270
Subiectul III
Prima soluţie:
  
Ecuaţia se scrie : x2 1 y2 1  4xy  4 x y , adică x 1  2
 y 1  2 x  2 y .
2

Deoarece x  1  2 x şi y 1  2 y trebuie să avem x  1  2 x şi y 1  2 y , adică


2 2 2 2

x  y  1.
Deoarece x  y  1, soluţiile posibile sunt x  1, y  1 şi x  1, y  1.
A doua soluţie:
     
2 2
Ecuaţia se scrie : y2 1 x2  4xy  y2 1  0 ,   4 y2  y2 1   y2 1
Pentru a avea soluţii reale trebuie ca y 2 1  0 , y  1  ecuaţia devine x2 1  2x  x  1
y  1  ecuaţia devine x2 1  2x  x  1
Soluţiile sunt : x  1, y  1 şi x  1, y  1.

Clasa a VIII-a

Subiectul I

Fie a 
 x  y
2

. Din  x  y   4xy şi xy  0 , rezultă a  4 .


2

xy
1 65
Inegalitatea de demonstrat se scrie succesiv:  4a   4 16a2  65a
a 4
 16a  a  64a  4  0  a 16a 1  4 16a 1  0   a  416a 1  0 , care este
2

adevarată pentru a  4 .
Egalitatea se atinge pentru a  4 , echivalent pentru x  y .

Subiectul II
Putem presupune că x  y  z , deci x  y  1, y  z  1 şi x  z  2 .
Aşadar  x  y    y  z    x  z   11 4  x2  y 2  z 2  xy  yz  zx  3 .
2 2 2

Ȋn concluzie x2  y2  z 2  3  xy  yz  zx  3  26  29 .

Subiectul III

Ȋn triunghiul isoscel VBC, mediana VM este şi înălţime ,


deci VM  BC .
Aşadar , triunghiul BMV este dreptunghic isoscel.
Notând cu 2a latura triunghiului echilateral ABC, avem
AB  2a, BM  MV  a, AM  a 3, VA  VB  a 2 .
Din reciproca teoremei lui Pitagora , rezultă atunci că
triunghiul AVM este dreptunghic în V, adică
m  AVM   90 .
Dar AVM este unghiul format de dreapta AV şi planul
(VBC) , prin urmare acest unghi căutat este un unghi drept.

271
Altfel : tot datorită reciprocei teoremei lui Pitagora, triunghiurile AVB şi AVC sunt dreptunghice
în V , adică AV  VB şi AV  VC  AV  VBC  , deci unghiul căutat este un unghi drept.
Observaţie: Tetraedrul VABC este unul „tridreptunghic”

Clasa a IX-a

Subiectul I

Notând  x  m,  y  n,  z  p , avem m, n, p  Z , iar sistemul se scrie :


 x yz n 1

 x  y  3z  3 p  2
2x  y  z  2m 3

Scăzând (1) din (3) obţinem x  2m  n  Z , deci x  0 şi atunci din ecuaţia a treia (forma
iniţială ) avem y = z si din (3) avem x = m .
n  3p
Din (1) si (2) avem : x  n, yz
4
n  3p
Din  y  y  n  , deci n  p .
4
n  3p
Din  z  z  p  , deci p  n
4
Rezultă n = p şi y = z = n .
Ȋn concluzie , x = y = z = n, adică mulţimea căutată este :  n, n, n / n  Z .
Subiectul II

Notăm A' B  x, B ' C  y, C ' A  z şi cu  aria, ipoteza


  AB ' C '    BC ' A '   CA ' B ' devine
x  c  z  sin B  y  a  x  sin C  z b  y  sin A
b
şi cum sin B  etc., rezultă :
2R
def
bx  c  z   cy  a  x   az b  y   k ,
(1)
k k ck  a  x 
Din (1) avem y  ,z   (2)
c  a  x a b  y  a  abc  k  bcx 
bcx  k
Dar tot din (1) avem z  (3)
bx
bcx  k ck  a  x 
Din (2) şi (3) avem    abc  k  bcx2  abcx  ak   0 (4)
bx a  abc  k  bcx 
Cum x  a, c  z  c , avem bx  c  z   abc , adică , conform (1) , k  abc .

272
Rezultă că (4) este echivalentă cu : bcx2  abcx  ak  0 (5)
Cum k  bx  c  z  , (5) devine bcx2  abcx  abx  c  z   0
x z 1 x y z def
sau cx  az  0 adică      (6)
a c a b c
  AB ' C ' x c  z
Dar     1   şi notând S    ABC  , avem :
  ABC a c
 2
1 S
  A ' B ' C '  S 1  3 1     S 3 2  3 1  3S        
1
 2 12  4
1
Egalitate avem când   , deci cand A’, B’, C’ sunt mijloacele laturilor triunghiului ABC.
2

Subiectul III

Presupunem prin absurd că ecuaţia are o rădăcină raţională ( deci din formulele lui Viete, pe
amândouă).
Atunci   b2  4ac este un pătrat perfect, adică   k 2 , k  N  (1)
Din ipoteză avem a  2017   2b  4c  şi atunci (1) se scrie:
b2  4  2017  2b  4c  c  k 2  b2  8bc 16c2  k 2  4  2017  b  4c   k 2  4  2017 
2

b  4c  k b  4c  k   4  2017
Numerele b  4c  k şi b  4c  k au aceeaşi paritate (căci diferenţa lor este număr par) şi
b  4c  k < b  4c  k iar singurul mod de a scrie numărul 4  2017 sub forma AB cu A < B şi A,
B de aceeaşi paritate ( de fapt pare) este acela când A 2 şi B  2  2017 , deducem:
 b  4c  k  2
 (2)
b  4c  k  2  2017
Adunând egalităţile din (2), obţinem b  4c  2018 . Atunci 2017  a  2b  4c  b  4c  2018
Contradicţie!

Clasa a X-a
Subiectul I

Fie funcțiile 𝑓, 𝑔: (0, ∞) → 𝑅, 𝑓(𝑡) = 𝑙𝑜𝑔𝑎+1 (𝑡 + 1), 𝑔(𝑡) = 𝑙𝑜𝑔𝑎 𝑡 și vom ține cont în
continuare că funcția f este crescătoare pentru 𝑎 ∈ (0, ∞), iar funcția g este descrescătoare
 f  x  g  y 

pentru a  0,1 și crescătoare pentru a 1,  . Sistemul se mai scrie  f  y   g  z  .

 f  z   g  x
I. a  1 ; f și g crescătoare
Demonstrăm întâi că x  y .
Presupunem că x  y , atunci x  y sau y  x .

273
Dacă x  y , cum a  1 ( f și g crescătoare), atunci f  x   f  y  , adică g  y   g  z  , deci
y  z şi imediat f  y   f  z  , adică g  z   g  x  , deci z  x şi prin urmare x  y  z  x ceea
este o contradicţie. Asemănător se analizează cazul y  x .
Deci x  y şi atunci prima ecuaţie a sistemului devine loga 1x  1  loga x . Avem
loga 1x  1  loga x  t , adică x  1  a  1t și x  at , de unde at  1  a  1t sau
t t
 a   1 
     1 , ecuație care ținând cont de monotonia fucției exponențiala de bază
 a  1  a  1
subunitară are soluția unică 𝑡 = 1, deci 𝑥 = 𝑦 = 𝑎.
Înlocuind în ecuaţia a treia a sistemului se obţine loga 1z  1  1 , adică 𝑧 = 𝑎. Așadar soluţia
sistemului este 𝑥 = 𝑦 = 𝑧 = 𝑎.

II. a  0,1 ; f crescătoare și g descrescătoare


Avem 𝑥 ≤ 𝑎 sau 𝑥 ≥ 𝑎 și observăm că 𝑓(𝑎) = 𝑔(𝑎) = 1.
Dacă 𝑥 ≤ 𝑎, atunci 𝑓(𝑧) = 𝑔(𝑥) ≥ 𝑔(𝑎) = 𝑓(𝑎), deci 𝑧 ≥ 𝑎. Avem 𝑓(𝑦) = 𝑔(𝑧) ≤ 𝑔(𝑎) =
𝑓(𝑎), deci 𝑦 ≤ 𝑎. Avem 𝑓(𝑥) = 𝑔(𝑦) ≥ 𝑔(𝑎) = 𝑓(𝑎), deci 𝑥 ≥ 𝑎 și prin urmare 𝑥 = 𝑎. Atunci
imediat 𝑥 = 𝑦 = 𝑧 = 𝑎.
Analog dacă ≥ 𝑎

Subiectul II
Alegem arbitrar z  C, z  1 . Dacă  zz
1i n
i  n , problema e rezolvată.
Ȋn caz contrar, adică dacă
 zz
1i n
i n (1)

observăm că z  zi  z  zi  2 , pentru fiecare i 1,2,3,..., n .


Prin sumare, rezultă că  zz   zz
1i n
i
1i n
i  2n , apoi folosind (1) se ajunge la

 zz
1i n
i  n , deci  z  C adică  z  1, verifică proprietatea cerută.

Subiectul III

1
Notând n
2018  2017   , rezultă că n
2018  2017  ,

1
deci numărul din enunţ este   .

1
Presupunem prin absurd că   Q .

 11  1   1 
Deoarece  k 1      k  k    k 1  k 1  , k  N  , rezultă imediat prin
   
k 1
    
1
inducţie după k, faptul că  k  k  Q , pentru orice k  N  .

274
1
Luând k = n, rezultă că  n   2 2018  Q , ceea ce este o contradicţie (2018 nu este pătrat
n
perfect).

Clasa a XI-a

Subiectul I
Notăm cu 1, 2  C valorile proprii ale matricei A adică rădăcinile ecuaţiei caracteristice
 2  trA   det A  0, deci trA  1  2 ,det A  12 .
Conform teoremei Hamilton – Cayley, avem:
A2  trA A   det A I2  O2 (1)
Mulţimea M fiind finită, există n, m  N  , n  m , cu An  Am (2)
Atunci 1n  1m , 2n  2m , deci i 0,1 , i  1,2 .

 
Mulţimea P   det A / n  N  este finită şi cum det A  Z , rezultă det A1,0,1 .
n

De asemenea trA  1  2  1  2  2 şi cum trA  Z , avem trA2, 1,0,1,2 .


Cazul detA = 0 .
Din (1) avem A2  trA A şi de aici An  trA
n1
A, n  N  .

 
Mulţimea M fiind finită, rezultă că mulţimea Q  trA / n  N  este finită, deci
n

trA1,0,1 .
1
1) trA  1 A2  A  O2  A2   A  A3   A2  A  M  A, A2  M  2  
1
2) trA  0  A2  O2  M   A, O2   M  2 .
1
3) trA  1 A2  A  O2  A2  A  M   A  M  1 .
Cazul detA = -1 .
Nu se poate ca 1 , 2 să fie complexe conjugate, căci ar rezulta det A  12  1 1  1  1.
2

Prin urmare 1 , 2 sunt reale , de modul 1 şi au produsul 12  det A  1, deci 1  1 , 2  1
.
Atunci trA  1  2  0 şi (1) devine A2  I2  O2 , deci A2  I2 . Rezultă M   A, I 2  şi
M  2.
Cazul detA = 1 .
Distingem trei subcazuri:
1). 1, 2  R, 1  0, 2  0, deci 1  2  trA  2 şi (1) devine
A2  2A  I2  O2  A2  2A  I2 . Prin inducţie A2  2 A  I2  O2  Ak  kA   k 1 I 2 , k  N  .
Cum A este inversabilă , din (2) rezultă
Anm  I 2   n  m A   n  m 1 I 2  I 2   n  m A   n  m I 2 
A  I 2  M  I 2   M  1

275
2). 1, 2  R, 1  0, 2  0, deci 1  2  1  trA  2 şi din (2) ave
A2  2 A  I2  O2  A2  2 A  I2  A3  2 A2  A  2  2 A  I2   A  3A  2I 2
Prin inducţie Ak   1 kA   k 1 I2  şi cum Anm  I2 , avem :
k 1

 1nm1  n  m A   n  m 1 I2   I2 . Dacă n -m este par obţinem


  n  m A   n  m 1 I 2  I 2    n  m A   n  m I 2  A  I 2
 M  I 2 , I 2  M  2
 2 
Dacă n - m este impar   n  m A   n  m 1 I 2  I 2  A   1
n  m  2
I

Şi cum A  M 2  Z  , e necesar ca n – m =1 şi A  I 2 , dar atunci trA  2 , contradicţie !
Situaţia n – m impar se elimină.
1). 1, 2  C  R, 2  1 . Cum 1  2  1, vom avea 1  a  bi, 2  a  bi , cu
trA  2a   2,2  trA1,0,1
a2  b2  1, b  0 , deci a  1,1 .Atunci .
1
Dacă trA  1 A  A  I2  O2  A  I2  O2  A  I2  M   A, A2 , I 2  M  3
2 3 3

1
Dacă trA  0  A  I2  O2  A  I2  A  I2
2 2 4

 M   A, A2 , A3 , I2  M  4
1
Dacă trA  1 A  A  I2  O2  A  I2  O2  A  I2  A  I 2
2 3 3 6

 M   A, A2 , A3 , A4 , A5 , I 2  M  6
.
Observaţie: Niciodată nu avem M  5 .
Exemple:
1) A  O2 sau A  I2  M  1.
 1 1
2) A  I 2 sau A     M  2.
 1 1
 1 1
3) A     M  3.
1 0
1 2
4) A    M 4
 1 1
1 1
5) A    M 6
1 0 

Subiectul II
Vom arăta că  n 1  xn  n3 , n  N  . Proprietatea este adevarată pentru n = 1.
3

Presupunem că proprietatea este adevarată pentru un n  N  .


Atunci xn1  n2 1  n  n3  n2   n  1 si xn1  n2 1  n 1  n3 .
3

276
n 3 x
n1 n 1 3 x
n1
Atunci   şi cum limitele laterale sunt egale cu e obţinem lim e.
n
n! n
n! n
n! n  n
n!
1 3 xn xn1 xn1  n 1 
3
1  3 xn
Cum  3 şi 1  3    , n  N 
rezultă că lim  1.
n n n  n  n n

Subiectul III
n
1  1 
an    2
1 1 1
 2   1  2 şi atunci
k 1 2  k  k  1 k  k  1  2  n  n  1 
 1  1
arctg 1  2ak   arctg 1 1  2   arctg 2
 k  k 1  k  k 1
1 1 1
Dar arctg 2  arctg  arctg , prin urmare:
k  k 1 k k 1
n n
 1   1  
lim  arctg 1  2ak   lim  arctg  arctg
1
  lim  arctg1  arctg 
n
k 1
n
k 1  k k  1  n  n  1  4

Clasa a XII-a

Subiectul I
 
n
2
sin x 2
cosn x
I  a dx  b  dx .
0  sin x  cos x  n2
0  sin x  cos x  n2

 
 2 n
cos x 2
sin n t
Cu schimbarea de variabilă x 
2
 t , obţinem 0 sin x  cos xn2 0 sin t  cos t n2 dx
dx 


2
sin n x
Deci I   a  b   dx
0  sin x  cos x  n2

 1
Cu schimbarea de variabilă x   t , sin x  sin t  cos t  , sin x  cos x  2 cos t
4 2
n
 1 
 2 sin t  cos t 
 

 dt  a  b tgt  1n 1 dt  a  b tgt 1


n1 
4

4
a b
I   a  b 
2n1 
4 
 
n2 
 2 cos t cos2 t 2n1 n 1 
4 n 1
 
4 4

Subiectul II
a). Fie F : Z p  X   Z p  X  un endomorfism al inelului Z p  X  .

 
Din F 0  0 şi F 1  1 rezultă F  a   a, a  Z p .
Pentru un polinom arbitrar f  Z p  X  , f  a0  a1 X  ...  an X n putem scrie :

277
F  f   F  a0   F  a1  F  X   ...  F  an  F  X   a0  a1F  X   ...  an F  X 
n n

 a0  a1u  ...  anun  f u   Du  f  , unde am notat cu u  F  x   Z p  X  .


Rezultă F  Du . Se arată uşor că Du este endomorfism al lui Z p  X  :
Du  f  g    f  g u   f u   g u   Du  f   Du  g 
Du  fg    fg  u   f  u  g u   Du  f  Du  g 


Du 1  1 u   1
b). Fie F  Du un automorfism al inelului Z p  X  . Deoarece F este este surjectiv, există
f  Z p  X  astfel încât F  x   X  Du  f   X  f u   X .
Trecând la grade, rezultă grad f  grad u  1 , de unde grad u (= grad f) = 1.
c). Pentru u  X p arătăm că Su  Du  Su  Du  u  f   f u   f p  f  X p  1
Scriind că f  a0  a1 X  ...  an X n , egalitatea de demonstrat (1) este echivalentă cu
a  a1 X  ...  an X n   a0  a1 X p  ...  an X pn , care este adevarată, deoarece a  a p
p
0

( teorema lui Fermat), iar  x  y   x p  y p , în caracteristică p.


p

Endomorfismul Sn , pentru u  X p , adică Su  f   f p


se numeşte endomorfismul lui Frobenius.

Subiectul III
1 2
t 2018 2 22018 1
  x 1 dx   t dt  
2017
Observăm că 2017
.
0 1 2018 1 2018
Ipoteza se scrie aşadar :

  dx  0
1 1 2

 f  x dx   x 1 dx   f  x    x 1
2017 2017
(1)
0 0 1

Conform teoremei de medie, există a  0,1 astfel încât

  f  x   x 1  dx  f a  a 1


1
2017 2017
(2)
0

Din (1) şi (2) rezultă f  a    a 1


2017
.

278
CEI MAI BUNI DINTRE CEI BUNI

Prezentăm o listă cu elevii care au obținut de cel puțin două ori premiul I la Concursul
”Gheorghe Mihoc”

Au obținut de două ori premiul I următorii:

Ablai Ainur , C.N. “ Mircea cel Bătrân”, Constanţa;


Andrei Alexandru, Şcoala „Mihai Eminscu”, Brăila, C.N. “Bălcescu”, Brăila;
Bâra Andrei, Liceul Internațional de Informatică Constanţa
Bejgu Andreea, C.N. „ Mihai Viteazul” Slobozia
Blănaru Adina, Gr. Şc. „Şt. Procopiu” Vaslui
Bonifate Tudor, C.N. „I.L. Caragiale” Ploiești
Bratu Florin, C.N. “Nicolae Bălcescu”, Brăila
Bratu Ovidiu, C.N. „N. Balcescu” Brăila
Ceacîreanu Filip, C.N. „Mihai Viteazul” Ploieşti
Custură Mihai, Şc. Nr. 3 Slobozia, C.N. „Mihai Viteazul” Slobozia
Donescu Irina, C.N. ”N. Bălcescu” Brăila
Dragomir Horia Alexandru, C.N. „I. C. Brătianu” Piteşti
Ionescu Dan, Şc. Gimn. "Sf. Andrei" Slobozia, C.N. „M. Viteazul” Slobozia
Marin Bogdan-Adrian. C.N. „Mihai Viteazul” Slobozia
Marin Violeta, C.N. “Mihai Viteazul”, Slobozia
Mincu Diana, Lic. de Artă Slobozia;
Nicolae Iulian-Mihai, C.N. ”I.L. Caragiale” Ploieşti
Nicorici Ileana, C.N. „Mircea cel Bătrân”, Constanţa
Ovreiu Auraş, Şc. Gimnazială „A. Vlaicu” Feteşti, Lic. Teoretic ”Carol I” Fetești
Pantelimon Ioana, Şcoala Nr. 2, Slobozia, Liceul Internaţional de Informatică, Constanţa
Păunescu Liviu, C.N. “Mircea cel Bătrân”, Constanţa
Radu Oana, Şc. nr.3 Slobozia
Spirea Andreea, C.N. “Mircea cel Bătrân”, Constanţa
Stănescu Andrei, Şc. Gimnazială „Sf. Andrei” Slobozia
Tănase Gheorghe Narcis, Şcoala Generala 7, Feteşti, Liceul Teoretic Fetești;

279
Au obținut de trei ori premiul I următorii:

Dumitru Cătălin, C. N. „B.P. Hașdeu” Buzău,


Mihai Bogdan, Gr. Şc. Industrial „Ştefan Procopiu” Vaslui
Toader Sebastian, Șc. Gimn. nr. 3 Slobozia

Au obținut de patru ori premiul I următorii:

Zecheru Daniela, C.N. „Al. I. Cuza” Ploieşti


Voiteanu Mircea, Şcoala Nr. 2, Slobozia, C.N. „Mihai Viteazul”, Slobozia

Au obținut de cinci ori premiul I următorii:

Cioacă Florin Daniel, Lic. “M. Viteazul”, Slobozia


Drăjneanu Diana, Şc. Nr.3 Slobozia, C.N. „M. Viteazul” Slobozia

A obținut de șase ori premiul I:

Drăgoi Octav – Şc. Gen. Nr.3 Slobozia, Lic. Internaţional de Informatică, Bucureşti

Distribuția premiilor I pe județe

IALOMIȚA - 68 Premii I
(Colegiul Național ”Mihai Viteazul” Slobozia – 25, Școala Gimnazială nr. 3 Slobozia – 20,
Școala Gimnazială nr. 2 ”Sf. Andrei” Slobozia – 8, Liceul de Arte Slobozia – 5, Colegiul
Național ”Gr. Moisil” Urziceni – 3, Școala Gimnazială nr. 7 ”A. Vlaicu” Fetești – 3, Liceul
Teoretic ”Carol I” Fetești – 2, Școala Gimnazială ”I.H. Rădulescu” Urziceni – 2)
PRAHOVA - 46
CONSTANȚA - 25
BRĂILA - 15
BUZĂU - 9
BUCUREȘTI - 5
VASLUI - 5
ARGEȘ - 3
CĂLĂRAȘI - 3
GALAȚI – 3
TULCEA -1

280
Indice de autori*
Acu Dumitru (1)
Andronache Marian (1)
Albu Toma (1)
Alexandrescu Petruş (1)
Anghel Costel (1)
Apostol Constantin (2)
Băban Dorina (1)
Băban Vladimir (1)
Băietu Ioan (1)
Bălăucă Artur (1)
Bătineţu-Giurgiu Dumitru (5)
Berghea Vasile (1)
Berindeanu Mihaela (1)
Bişboacă Nicolae (1)
Blăjuţ E. (1)
Bourbăcuţ Nicolae (1)
Bunget Mihai (1)
Burcă Ion (1)
Chichirim Nelu (1)
Chirciu Marin (1)
Chiriţă Marcel (1)
Chiţescu Ion (1)
Ciocan Antonia (1)
Cristea Cătălin (5)
Cucoaneş Marian (1)
Cunţan Irina (1)
Damian Marius (1)
Diaconu Ilie (1)
Dinu Sorana (1)
Dinu Răzvan (1)
Doboşan Aurel (7)
Dragomir Lucian (3)
Drăghicescu Inocenţiu (25)
Drăgoi Octav (2)
Eckstein Alfred (1)
Frăsilă Mihail (1)
Frecuş Viorica (1)
Ghenghiu Radu (1)
Ghiţă Gh. (1)
Ghiţă Ioan (3)
Ghiţă Romanţa (3)
Haivas Mihai (1)
Ionescu Sorana (2)
Ioniţă Bogdan (1)
Iordache Bianca Teodora (1)
Iosifescu Marius (1)

281
Iurea Elena (1)
Lăutaru Alexandru (1)
Lupu Cezar (1)
Macsut Camelia (1)
Maftei I.V. (1)
Marin Verona (1)
Marinescu Rodica (1)
Marinescu Victor (1)
Mărgărit Marian (10)
Miculescu Radu (2)
Mihăilă Adriana (1)
Militaru Cristina (1)
Molea Gheorghe (2)
Moanță Cristian (1)
Neacşu Ilie (3)
Neață Ion (2)
Nedeianu Dan (4)
Nedelcu Ion (2)
Negrii Costin (1)
Nicolaescu Florin (1)
Nicolau Constantin (1)
Nicula Valentin (1)
Niculescu Benedict (2)
Olteanu Marius (1)
Oprişan Liviu (1)
Ozunu Cezar (1)
Panaitopol Lurențiu (1)
Pană Cătălin (1)
Papacu Nicolae (23)
Păunescu Lidia (1)
Petrea Constantin (3)
Popa Dumitru (6)
Popa Vasile (1)
Popa Vasile Mircea (2)
Popescu Marcel (9)
Preda Traian (2)
Raicu Julieta (1)
Rotaru Florin (5)
Rusu Constantin (2)
Săceanu Victor (1)
Smarandache Liviu (2)
Sitaru Daniel (4)
Stanciu Neculai (4)
Stănescu Florin (1)
Stoica Gheorghe (3)
Stroe Marin (1)
Șteopoaie Vasile (1)
Tarciniu Vasile (1)
Tămâian Traian (5)
282
Tilincă Daniela (1)
Toderiuc Liliana (1)
Todor Petru (1)
Tonţ Adriana (1)
Tudor Ion (1)
Tudoran Viorel (1)
Tutulan Gh. (1)
Tuţă Luca (1)
Tuțescu Lucian (2)
Ţena Marcel (25)
Vasile Emil (2)
Vernescu Andrei (1)
Vlăduţiu Iulian (1)
Voicu Ioan (2)
Zvonaru Titu (1)

(*) În paranteză sunt trecute numărul problemelor propuse la concurs.

283
BIBLIOGRAFIE

[1] C. Dumitru și colaboratori, Concursul Interjudețean de Matematică ”Gheorghe Mihoc”, Ed.


Cetatea de Scaun, Târgoviște, 2009

[2] N. Papacu și colaboratori, Concursul Național de Matematică ”Gheorghe Mihoc”, Ed.


Metamorfosi, Slobozia, 2014

[2] Colecția revistei Gazeta Matematică, 1995-2018

[3] Colecția Revistei de Matematică din Ialomița, 2011-2018

284
CUPRINS

PARTEA I – ENUNȚURI ȘI REZULTATE


Ediția I…………………………………………………………………….8
Ediția a II-a……………………………………………………………….10
Ediția a III-a………………………………………………………………14
Ediția a IV-a………………………………………………………………17
Ediția a V-a……………………………………………………………….19
Ediția a VI-a………………………………………………………………23
Ediția a VII-a……………………………………………………………...26
Ediția a VIII-a……………………………………………………………..29
Ediția a X-a………………………………………………………………..33
Ediția a XI-a……………………………………………………………….36
Ediția a XII-a………………………………………………………………41
Ediția a XIII-a……………………………………………………………..45
Ediția a XIV-a…………………………………………………………..…49
Ediția a XV-a…………………………………………………………...….53
Ediția a XVI-a…………………………………………………………..…57
Ediția a XVII-a………………………………………………………….…61
Ediția a XVIII-a……………………………………………………….…..65
Ediția a XIX-a……………………………………………………………..70
Ediția a XX-a……………………………………………………………...74
Ediția a XXI-a……………………………………………………………..78
Ediția a XXII-a…………………………………………………………….81
Ediția a XXIII-a……………………………………………………………86
Ediția a XXIV-a…………………………………………………………....91

PARTEA a II-a – REZOLVĂRI


Ediția a II-a………………………………………………………………..95
Ediția a III-a………………………………………………………………103
Ediția a IV-a………………………………………………………………111
Ediția a V-a……………………………………………………………….120
Ediția a VI-a………………………………………………………………129
Ediția a VII-a…………………………………………………………...…135
Ediția a VIII-a……………………………………………………………..143
Ediția a X-a………………………………………………………………..148
Ediția a XI-a……………………………………………………………….154
Ediția a XII-a………………………………………………………………159
Ediția a XIII-a……………………………………………………………...167
Ediția a XIV-a………………………………………………………………174
Ediția a XV-a……………………………………………………………….184
Ediția a XVI-a………………………………………………………………192

285
Ediția a XVII-a………………………………………………………………201
Ediția a XVIII-a……………………………………………………………..209
Ediția a XIX-a……………………………………………………………….218
Ediția a XX-a……………………………………………………………..…228
Ediția a XXI-a……………………………………………………………….236
Ediția a XXII-a………………………………………………………………244
Ediția a XXIII-a……………………………………………………………...256
Ediția a XXIV-a……………………………………………………………...267

Cei mai buni dintre cei buni…………………………………………………279


Indice de autori………………………………………………………………281
BIBLIOGRAFIE……………………………………………………………..284

286

S-ar putea să vă placă și